Vous êtes sur la page 1sur 324

BCPST 1

MÉTHODES A. BÉGYN, R. LEROY, G. (ON NAN

ET EXERCICES
Conception et création de couverture : Atelier 3+

Le piclogramme qui figure ci-cantre d'enseignement supérieur, provoquant une


mérite une explication. San objet est baisse brutale des achats de livres el de
d'alerter le lecteur sur la menace que revues, au pointque la possibilité même pour

particulièrement dans le domaine DANGER


de l'édition technique et universi·
taire, le développement massif du
photocopillage.
®
représente pour l'avenir de l'écrit, ,-------. les auteurs de créer des ceuvres
nouvelles et de les faire édite< car-
redementestaujourd'hui menacée.
Nous rappelons donc que toute
reproduction, partielle ou totale,
Le Cade de la propriété intellec- de la présente publication est
tvelle du 1•r juillet 1992 interdit LE Pt«J1'0CailLII interdite sons autorisation de
en effet expressément lo photaco· TUE LE LIVRE l'auteur, de son éditeur ou du
pie à usage collectif sans autori- Centre français d'exploitation du
sation des ayants droit. Or, cette pratique droit de copie (CF(, 20, rue des
s'est généralisée dons les établissements Grands-Augustins, 75006 Paris).

© Dunod, 2015

u 5 rue Laromiguière, 75005 Paris


0 www.dunod.com
C:
::J
0
lfl ISBN 978-2-10-072985-2
,-i
0
N Le Code de la propriété intellectuelle n'autorisant, aux ter mes de l'article
@ L. 122-5, 2° et 3° a) , d'une port, que les « copies ou reproductions strictement
...., réservées à l'usage privé du copiste et non destinées à une util isation collective»
.c
Ol et, d'autre par i, que les analyses et les courtes citations dons un but d'exemple e l
·;:: d'illustration, « toute représentation ou reproduction intégra le ou pa rtielle laite
>-
a. sons le consentement de l'auteur ou de ses ayants droit ou ayants couse est
0 illicite » (art. L. 122-4).
u
Cette représentation ou reproduction, par quelque procédé que ce soit, constitue-
rait donc une contrefaçon sanctionnée par les o r ticles L. 335-2 et suivants du
Code de la propriété intellectuelle.

www.bibliomath.com
CHAPITRE l LOGIQUE, ENSEMBLES, SIGNES L ET IT 1
Méthodes à retenir 2

Énoncés des exercices 5

Du mal à démarrer ? 8

Corrigés des exercices 9

CHAPITRE 2 NOMBRES COMPLEXES ET TRIGONOMÉTRIE 17


Méthodes à retenir 18

Énoncés des exercices 21

Du mal à démarrer ? 25

Corrigés des exercices 26

CHAPITRE 3 SUITES RÉELLES 38


Méthodes à retenir 39

Énoncés des exercices 42

Du mal à démarrer ? 47
u
0
C:
Corrigés des exercices 48
::J
0
lfl
,-i
0
N CHAPITRE 4 SYSTÈMES LINÉAIRES ET CALCUL MATRICIEL 62
@
...., Méthodes à retenir 63
.c
Ol
·;::
>- Énoncés des exercices 65
a.
0
u Du mal à démarrer ? 70

Corrigés des exercices 71

www.bibliomath.com
CHAPITRE 5 ESPACES VECTORIELS ET APPLICATIONS LINÉAIRES 86
Méthodes à retenir 87

Énoncés des exercices 92

Du mal à démarrer ? 98
Corrigés des exercices 99

FONCTIONS, POLYNÔMES, CONTINUITÉ 125


Méthodes à retenir 126

Énoncés des exercices 131

Du mal à démarrer ? 136

Corrigés des exercices 137

CHAPITRE 7 DÉRIVABILITÉ, DÉVELOPPEMENTS LIMITÉS 153


Méthodes à retenir 154

Énoncés des exercices 157

Du mal à démarrer ? 162

Corrigés des exercices 163

INTÉGRATION, ÉQUATIONS DIFFÉRENTIELLES 182


Méthodes à retenir 183

Énoncés des exercices 186

Du mal à démarrer ? 191


u
0 Corrigés des exercices 192
C:
::J
0
lfl
,-i
0
N CHAPITRE 9 DÉNOMBREMENT, PROBABILITÉS 213
@
...., Méthodes à retenir 214
.c
Ol
·;::
>- Énoncés des exercices 217
a.
0
u Du mal à démarrer ? 223

Corrigés des exercices 224

Il

www.bibliomath.com
CHAPITRE 10 VARIABLES ALÉATOIRES 240
Méthodes à retenir 241
Énoncés des exercices 242
Du mal à démarrer ? 246
Corrigés des exercices 247

CHAPITRE 11 VECTEURS ALÉATOIRES 257


Méthodes à retenir 258
Énoncés des exercices 260
Du mal à démarrer ? 265
Corrigés des exercices 266

CHAPITRE 12 GÉOMÉTRIE 283


Méthodes à retenir 284
Énoncés des exercices 289
Du mal à démarrer ? 294
Corrigés des exercices 295

CHAPITRE 13 STATISTIQUES 310


Méthodes à retenir 311

Énoncés des exercices 312


Du mal à démarrer ? 315
Corrigés des exercices 316

u
0
C:
::J
0
lfl
,-i
0
N
@
....,
.c
Ol
·;::
>-
a.
0
u

11 1

www.bibliomath.com
Copyright© 2015 Dunod.

www.bibliomath.com
CHAPITRE 1J..

Logique, théorie des ense1nbles et


1nanipulations des signes L, et fl

'Ilièmes abordés dans {es e~ercices


- Raisonnements mathématiques
- Opérations sur les ensembles
- Propriétés générales des applications
- Manipulation des symboles Let n.

Points essentiels du cours pour {a réso{ution


des e~ercices
- Démonstration d'une implication, d'une équivalence.
- Raisonnement par contraposée.
- Raisonnement par l' absurde.
- Raisonnement par récurrence.
u - Démonstration d'une inclusion, d'une égalité entre ensembles.
0
C:
::J - Règles de calcul pour les opérations sur les ensembles.
0
lfl
- Image directe ou réciproque d'une partie par une application.
,-i
0 - Injectivité, surjectivité ou bijectivité d'une application.
N
@ - Théorème d' inversibilité pour la loi de composition.
....,
.c - Théorème de la bijection pour les fonctions numériques .
Ol
ï::::
>- - Règles de calcul avec les symboles L et n.
a.
0 - Règles de calcul sur les coefficients binomiaux.
u
- Sommes usuelles : sommes arithmétiques, sommes géométriques, formule du binôme.

www.bibliomath.com
Chapitre 1 Logique, ensem bles, signes Let n

Les méthodes à retenir

- Pour démontrer que A ==> B on suppose que la propriété A est


verifiée et on doit démontrer que la propriété B l'est aussi.
<---+ Exercice 1.4

Pour démontrer l'implication A ==> B, on peut raisonner par


contraposée, c'est-à-dire démontrer l'implication non(B) ==>
Pour démontrer une implication ou une non(A): on suppose que B n'est pas verifiée et on démontre
équivalence qu'alors A ne l'est pas non plus.
<---+ Exercice 1.1

Pour démontrer une équivalence A ~ B on raisonne par double-


implication: on démontre l'implication A==> B ainsi que sa réci-
proque B ==> A.
<---+ Exercice 1.6

- Pour démontrer que A est vérifiée : on suppose que A n'est pas


vérifiée et on en déduit une contradiction évidente du type 1 = 0,
Pour raisonner par l'absurde 3 :S 2 etc .. .
<---+ Exercices 1.3 et 1.6

Pour démontrer que Vx e: E, P(x) : on se fixe un x e: E quelconque


et on doit alors démontrer que P(x) est vérifiée pour ce x fixé.
<---+ Exercice 1.1
u Pour démontrer que 3x e: E/ P(x) : on doit donner (au moins) un
0
C:
::J
exemple de x e: E qui vérifie la propriété P(x) . Lorsque P (x) est une
0 équation alors x est l'inconnue et on doit trouver (au moins) une
lfl
,-i
Pour démontrer une proposition solution.
0 logique dépendant de quantificateurs
N <---+ Exercice 1.5
@
...., Pour démontrer que 3!x e: E : on démontre comme précédem-
.c
Ol
ï::::
ment que 3x e: E/ P (x) et, de plus, qu'il ne peut y avoir deux va-
>-
a. leurs distinctes de x pour lesquelles P(x) est vraie (ceci à l'aide
0 d'un raisonnement par l'absurde).
u
<---+ Exercice 1.5

www.bibliomath.com
Logique, ensembles, signes L, et n Chap,itre 1

Si la propriété à démontrer, pour tout entier naturel n, vérifie une


relation donnée entre le rang n et le rang n + 1 on utilise alors le
principe de récurrence.
c.......+ Exercices 1.2, 1.11 et 1.13

- Si la propriété à démontrer, pour tout entier naturel n, vérifie une


relation donnée entre les rangs n, n + 1 et n + 2 on utilise alors le
Pour raisonner par récurrence principe de récurrence à deux pas.
c.......+ Exercice 1.2

- Si la propriété à démontrer, pour tout entier naturel n, vérifie une


relation donnée entre tous les rangs k tel que k ~ n, on utilise alors
le principe de récurrence forte.
c.......+ Exercice 1.2

Pour démontrer l'inclusion E c Fon démontre l'implication


xe: E ~ xe: F.
c.......+ Exercices 1.3, 1.6 et 1.8

Pour démontrer l'égalité E =Fon raisonne par double-inclusion:


Pour démontrer une inclusion ou une on démontre l'inclusion E c F et l'inclusion réciproque F c E.
égalité entre deux ensembles <.......+ Exercices 1.3, 1.6, 1.7 et 1.8

Dans les deux cas, on peut aussi utiliser les opérations sur les en-
sembles.
<.......+ Exercices 1.6 et 1.7

"O
0
C:
::J Pour démontrer que f : E - F est injective sur E : on se d!onne
0 (x1 , x2 ) e: E2 tel que f (xi) = f (x2 ), et on doit alors montrer que
lfl
,-i XI= Xz.
0
N c.......+ Exercices 1.4 et 1.5
@ Pour démontrer qu'une application est
...., Pour démontrer que f : E - F est surjective de E sur F : on se
.c injective ou surjective
Ol
ï::::
donne y e: F fixé quelconque , et on doit alors donner (au moins)
>-
a. un x e: Etel que y= f(x), par exemple en démontrant que l'équa-
0 tion y= f(x) d'inconnue x a (au moins) une solution dans E.
u
c.......+ Exercices 1.4 et 1.5

www.bibliomath.com
Chapitre 1 Logique, ensembles, signes Let n

On revient à la définition en démontrant qu'elle est à la fois injec-


tive sur E, et surjective de E sur F.
c...... Exercice 1.4

On démontre les deux en même temps : on se donne y E F fixé


quelconque, et on doit alors montrer que 3!x E E/ y= f(x), par
exemple en démontrant que l'équation y= f(x) d'inconnue x a
une unique solution dans E.
Pour démontrer qu'une application est c...... Exercice 1.5
bijective
On utilise le théorème d'inversibilité pour la loi de composition:
on détermine une application g: F - E telle que f o g = idF et
gof = idE.
c...... Exercice 1.15

Dans le cas d'une fonction numérique, on peut utiliser le théo-


rème de la bijection.
c...... Exercice 1.5

Pour y E F fixé quelconque, f- 1 (y) est l'unique solution de l'équa-


tion y= f(x) d'inconnue x E E.
c...... Exercice 1.5
Pour déterminer l'application
réciproque d'une bijection - Si on a trouvé g: F - E telle que f o g = idF et go f = idE, alors
r1 =g.
c...... Exercice 1.15

On met en facteur les termes ne dépendant pas de l'indice de som-


mation, on utilise ensuite les règles de calcul sur les symboles :r,
et on conclut en faisant apparaître les sommes usuelles à l'aide de
changements d'indice.
Pour calculer une somme formelle c...... Exercices 1.9, 1.12, 1.13 et 1.14

- Si le résultat final est donné dans l'énoncé, on peut aussi démon-


trer la formule par récurrence.
c...... Exercices 1.11 et 1.12

u
0
C:
::J
0
lfl
,-i
0
N
@
....,
.c
Ol
ï::::
>-
a.
0
u

www.bibliomath.com
Logique, ensem bles, s ignes L, et n Chap,itre 1

Énoncés des exercices

---- - Exemple de démonstration d'une implication par contraposée


n 2 pair =

---- -
Etablir que : 'ri n e: 1\1, n pair.

Exemples de démonstration par récurrence

n
a) Etablir que, pour tout ne: 1\1: I: k2 = n(n+ 1~2n+ ll.
k=O
b) On définit une suite réelle (Un) ne:N par : uo = u1 = 3 et 'v' n e: 1\1, Un+2 = Un+ 1 + 2un.
Etablir que, pour tout ne:N: un= zn+l + (-1) 11 •
c) On définit une suite réelle (unlne:N par: uo = 0, u1 = 3 et

2 n
'v'ne:N*, Un+l =- L Uk·
n k =O

---- -
Montrer que, pour tout ne: N: un= 3n.

Autour de l'image directe d'une application


Soit f : E - + F une application. Soient A et B deux parties de E.
a) Montrerque:/(AuB)=/(A)u/(B).
b) Montrer que: .f (An B) c .f(Al n .f (B).
c) Montrer que si f est injective: /(An B) = /(A) n f (B).
d) Montrer que si f est injective : f (:i\) c f(A) .

e) Tvlontrer que si f est surjective: f (A) cf (°A).

---- - Injectivité, surjectivité, bijectivité et composition


Soient f
vantes:
: E - + F et g : F -

a) go .f injective sur E
E deux applications. Démontrer les implications sui-

= .f injective sur E
b) go/surjectivedeEsurE = gsurjectivedeFsurE
c) go f surjective de E sur E et g injective sur F = f surjective de E sur F
u d) go f bijective de E sur E et f o g bijective de F sur F
0
= .f bijective de E sur F et g bijective de F sur E

---- -
C:
::J
0
lfl
,-i Exemple de fonctions numériques bijectives
0
N
@ a) On considère l'application .f : Ili - Ili définie par : 'v' x e: Ili, .f(x) = x 2 - 5. .f est-
....,
.c elle injective, surjective, bijective? Montrer que la restriction de f à l'intervalle
Ol
·;:: 10, +ool induit une bijection dont on déterminera la réciproque.
>-
a. b) Montrer que l'application g: Ili - lll1 définie par: 'v' x e: IR!, g(x) = sin x + lx, est
0
u bijective. Montrer que l'équation g(x) = 2 admet une unique solution réelle, et
que cette solution est strictement positive.

www.bibliomath.com
Chapitre 1 Logique, ensembles, signes Let n

---- - Exemples de démonstration d'une équivalence ou d'une égalité entre en-


sembles
Soient E un ensemble et (A, B, Cl e: 8i1'(E)3 .
a) Tvlontrer que: Ac B <==>Au B = E.
b) Etablir que: (A\B)\(A\C) = (A\B)nC = (AnC)\B.

AuB Aue
c) Démontm quec { <==>B=C.
AnB AnC

AuB AnC
d) Démontœc quec { <===> A = B = C.
AnB = AuC

---- - Différence symétrique de deux ensembles


Soient E un ensemble et (A, B, C) e: 8i1' (E)3. On définit la différence symétrique de A et
B, notée At.B, par: At.B = (Au B) \(An B).
a) Montrer que: At.B = (A\B) u (B\A).
b) On suppose que At.B = MC. Établir que B = C.

---- - Image directe d'une image réciproque et inversement


Soient f: E - F une application, A une partie de E et B une partie de F.
a) Etablir que: Ac J- 1 (f (A)). Montrer qu'on a l'égalité dans le cas où f est injective
deEsurF.
b) Etablir que: 1u- (B))1 C B. Montrer qu'on a l'égalité dans le cas où f est surjec-
tive sur E.

---- - Calculs classiques de sommes


Pour tout entier naturel n non nul, calcuJer les sommes suivantes:

a)
Il
I: l,
k= l
n

i= lj = l
n
I:I:l, (.t i) + ( .t
1=1 ; =l
j ),
n
I: I:
n

i= lj = l
(i + j), I:
1-5,j-5,i-5,n
1.

Il n-1 n n n n
b) I: (k)' I: 31k (Z), I: k(k), [ kl1(k), I: k2(,l
k= l k=O k=O k=O k=O
n
u
0
c) f: 1n(1+~J
k=l
I: k(1+ 1).
k= l

---- -
C:
::J
0
lfl
,-i
0
Calculs classiques de produits
N
Pour tout entier naturel n non nuJ, calculer les produits suivants:
@
...., a) Le produit des entiers entre 1 et n .
.c
Ol
·;::
b) Le produit des entiers pairs entre 1 et 2n.
>-
a. c) Le produit des entiers impairs entre 1 et 2n + 1.
0
u n k
d) 11 k+l.
k= l

www.bibliomath.com
Logique, ensem bles, signes L, et n Chap,itre 1

---- - Som.me des termes d'une ligne dans le triangle de Pascal


Soit ne: N. Etablir que :

'v'pe: ŒO,n], Ln (k) = (n+ I)


· .
k=p p p+l

---- - Calcul de somme et coefficients binômiaux


Soient net p deux entiers naturels tels que n ~ p. Déterminer
p
.r c:H;=:l-
.

----
1= 0

- Calculs de sommes doubles

a)

bl
n n j
_I: .r.
1.
j = lt= J
I: ij.
n k 2 = n(n+l~C2 n + ll et I:
Soit n E N *. On admettra que: I:
k=O
n k3
k =O
=( ~ )2 .Calculer:

----
l :5j<i:5n

- Sommes de coefficients binômiaux "de deux en deux"


Soit ne: N. On considère les sommes

An = n L (n)
k , L
Bn = n ( - 1) k(n)k , Sn= L (n), Tn= L ( n )·
k=O k =O 0:52k:5n Zk 0:52k + l:5n 2k + 1

a) Calculer An et 8 11 en fonction den.


b) En déduire Sn et T n en fonction den.
, . n 2n
c) Determmer L (2k).

---- -
k =O

Exemple d'application fonctionnelle: l'application shift


On pose E = lllllli (ensemble des fonctions numériques de IJll dans llll). Pour 0 E IJll on
note q>e l'application de E dans E qui associe à .f E Ela fonction g q>e (.f) définie par =
g e: E et :
'v'x e: llll, g(x) = .f(x+9)

a) Etablir que pour tout (81. 82) E 111? 2 • on a : (/)91 0(/)92 = q>9 1 +0 2 •

u b) En déduire que, pour tout 0 e: llll, l'application (/)0 est bijective de E sur E et donner
0 sa réciproque.
C:
::J
0
lfl
,-i
0
N
@
....,
.c
Ol
·;::
>-
a.
0
u

www.bibliomath.com
Chapitre 1 Logique, ensembles, signes Let n

Du mal à démarrer ?

..

Ill n est impair éq uivaut à: 3p e: N/ n = 2p + 1. a) Faire apparaître la somme ar ithmétiq ue


n
I, k.
Ill a) Par récurrence sur n. b) Faire apparaître la formu le du binôme.
k=I

b) Par récurrence à deux pas sur n. c) Recon naître des sommes té lescop iques.
c) Par récurrence forte sur n.
ID a), b) etc) Faire apparaître des factoriels.
Ill a) Par double-inclusion en raisonnant sur les éléments. d) Ecrire le produit sous forme développée.
b) Raisonne r sur les éléments.
c) Par une application injective un antécédent est un ique ...
all Raison ner par récurrence sur n.
d) et e) Raisonnement par l'absurde.
ll9 Simpl ifier le te rm e général de la somme.
Ill a) et b) Uti liser les définitions.
1111 a) Permu ter les ~ et faire apparaître la somme arithmétique
c) Uti liser b).
d) Utiliser les défi nitions. n
I. k.
Ill Etudier les var iations des fonctions. k=I
b) Utiliser une somme double et faire apparaître une somme arith -
1111 a), b) etc) Raison ner sur les éléments. d) Pour= : commen- métique.
cer par montrer que C c A lm a) Utiliser la formule du binôme.
Ill a) Faire le calcul en util isant les règles de calcu l avec l'union b) Exprimer Sn et T en fonction de An et Bn.
c) Uti liser b).
11

et l' intersection.
b) Raisonner sur les éléments. 111!1 a) Utiliser la définition de <p9.
1111 Uti liser les définitions. b) Utiliser le théorème d 'inversibilité pour la loi de composition.

u
0
C:
::J
0
lfl
,-i
0
N
@
....,
.c
Ol
·;::
>-
a.
0
u

www.bibliomath.com
Logique, ensembles, signes L, et n Chap,itre 1

Corrigés des exercices


- Soit n e: N fixé quelconque. On raisonne par Donc:
contraposée, c'est-à-dire qu'on va démontrer l'implica- n
tion : n impair = n2 impair. I: k2 1 ( (n + 1)3 - 3 n(n + 1) - (n + 1))
3 2
On suppose donc que n est impair. Il existe donc p EN tel k=O
que n =2p + 1. En élevant au carré, on obtient: n+l
= - - (2(n 2 + 2n+ l) - 3n - 2)
6
n 2 = (2p+ 1) 2 =4p 2 +4p+ 1 = 2(2p 2 +2p)+ 1 = 2q+ 1 n+J
--n(2n+l)
6
où q =2p2 + 2p e: N, ce qui montre que 112 est impair.
if8 li.
a) On vérifie la formule par récurrence sur l'entier
Cette méthode se généralise au calcul des sommes d'Euler
n
L kP, en fonction de p e: N (il suffit de développer (k + l)P ) .
n k=O
• Pour n = 0, [ k 2 = 0 = n(n+ 1 2n+ ll ,
~< la formule est donc n
k=O On pourra essayer de l'appliquer pour calculer L k3 (réponse:
vraie. k=O
• Supposons la formule vraie à un rang n E N fixé . On a ( n(,~+ ll)\
alors: b) On dispose d'une relation de récurrence du second
n.+ 1 n ordre puisqu'elle relie Un+ 2, Un +I et u 11 • On va donc dé-
I: k2 I: k2 + (n+1)2 montrer la formule en raisonnnant par récurrence à deux
k=O k=O pas sur l'entier n.
n(n+ 1)(2n+ 1) 2 • Pour n = 0, Un= 3 = 2+ 1 = 2n+l +(-J)n, et pour n= 1,
- - - - - - + (n + 1)
hyp. rec. 6 un = 3 = 4 - 1 = 2 11+ 1 + (-1) n, la propriété est donc vraie
2n3 + 9n 2 + 13n + 6
aux rangs O et 1.
6 • Supposons la formule vraie aux rangs n et n + 1 pour un
n E N fixé quelconque.
Et d'autre part:
Ona:
(n + l)(n + 2) (2n + 3) 211 3 + 9n2 + 1311 + 6
6 = 6 Un.+2 Un+l + 2un
2n+2 + (- l)n+l + 2(2n+l + (-l)n)
La formule est donc vraie au rang (n + 1). hyp. rec.
D'après le principe de récurrence, la formule est donc 2n+ l (2 + 2) + (-l)n (-1 + 2)
vraie pour tout n e: N.
2n+3 + (-1) n.
On peut se demander comment obtenir la formule sans
qu'elle soit donnée dans l'énoncé? et (-1) 2 = 1 donc (-1) 11 +2 = (- l)n, ce qui donne :
L'astuce consiste à remarquer que l'identité (k + 1)3 = k3 +
3k2 +3k+ 1 donne k2 = !((k+ 1)3 -P-3k-1). Un+ 2 = 2n+3 + (- l)n.+2
"O En additionnant ces inégalités pour k E [o, nil, on obtient:
0 La formule est donc vraie au rang n + 2.
C:
::J
0 D'après Je principe de récurrence, la formule est donc
lfl vraie pour tout n E N.
,-i
0
La première somme est une somme« télescopique » : c) On dispose d'une relation de récurrence forte puis-
N
qu'elle relie Un+ l , Un, ... , u1 et uo. On va donc démontrer
@ n.
...., la formule en raisonnnant par récurrence forte sur l'entier
.c I: [ck+ 1)3-k3 J n.
Ol k=O
·;::
>- • Pour n = 0, Un = 0 = 3n, la propriété est donc vraie au
a. [(n+1)3 - / ] + [ J I - ~ + · · · + [ /- 03 ]
0 rang O.
u (n + 1)3 -03 • Supposons la formule vraie à tous les rangs k tels que
(n + 1)3 k E [o, nil, pour un ne: N fixé quelconque.

www.bibliomath.com
Chapitre 1 Logique, ensem bles, signes Let n
Ona : Mais comme f est surjective, il existe x e: E tel que : y =
f(x).
2 n
Un+l = -n k.L=Ouk Par l'absurde: si x E A, alors y= f(x) E f (A). Ceci est ab-
surde, donc x ( A.
2 n Alors : y= f(x) e: J(A).
hyp. rec.
- .L 3k
n k =O Ceci prouve que /(A) c J(A).
~
11
[,k - . a) On suppose go .f injective. Montrons que f est
k =O m1ect1ve.
6 n(n+ 1)
Soit (x1,x2) e: E2 tel que : /(x1) = f(x2). En composant par
som. arithm. n 2 g: go f(x1) =go J'(x2), et donc x 1 = x2 puisque go f est
= 3(n + 1) injective.
Ceci prouve que f est injective.
La formule est donc vraie au rang n + 1.
b) On suppose go f surjective. Montrons que g est surjec-
D'après le principe de récurrence, la formule est donc tive.
vraie pour tout n e: N.
Soit y e: E. Puisque go f est surjective, il existez e: Etel que :
y = go f( z ). On a donc: y = g(x), avec x = .f(z) E F.
- a) On raisonne par double-inclusion.
Ceci prouve que g est surjective.
0 Soit y e: f (Au B). Par définition, il existe x e: Au B tel
c) On suppose go f surjective et g injective.
que: y = .f(x).
D'après le b), g est aussi surjective et donc bijective. On
• Si X E A, alors y = .f(x) E .f(A), et donc y E .f (A) u f (B).
peut donc considérer son application réciproque g- 1 qui
• Si x ( A, alors x E B car x E A u B. On en déduit que est aussi bijective et vérifie: go g - 1 = g - 1 o g = idE. On a
y = f(x) e: f (B), et donc y e: /(A) u /(B). alors: f = g- 1o(go.f). Or g- 1 et go f sont surjectives, donc
Dans tous les cas : y e: /(A) u f (B). f est aussi surjective comme composée de surjections.
@]Soit y E /(A) U /(B) . d) On suppose go f et .f o g bijectives.
• D'après a) f est injective et d'après b) g est surjective.
• Si y E .f(A), alors y E /(Au B).
• Le problème est symétrique: en échangeant Je rôle de f
• Si y ( .f(A), alors y e: f(B), car y e: /(Al u .f (B). Et donc:
et g, on obtient les mêmes hypothèses. On peut donc dire
ye:f(A u B).
que g est injective et f surjective.
Dans tous les cas : y e: f(AuB). En regroupant: f et g sont bijectives.
b) Soit y e: f (An B). Par definition, il existe x e: AnB tel que :
y = f (x). Comme x e: AnB, on ax e: A et x e: B. On en déduit - a) On obtient facilement le tableau de variations:
que: y= f(x) E f (A) et y E f (B). Et donc: y E /(A) n f (B). X - oo 0 +oo
c) On raisonne par double-inclusion. +oo +oo

~-5~
Variations de
0C'est le résultat du b). f
@J Soit y e: f (A) n f (B). On a donc: y e: f (A) et y e: /(B).
Par définition, il existe donc x E A et z E B tels que: y = On en déduit que f n'est pas injective sur Oll, en effet: -1 t l
'O .f (x) = .f (z). Or f est injective, donc on a: x = z, et donc: et/(-1) = /(1) =-4.
0
C: x e: An B. Ceci prouve que y= f(x) e: /(A n B). De plus f n'est pas surjective de IJll sur Oll, en effet : ri x e:
::J
0 d) Soit y e: J(A). Par définition, il existe x e: A tel que : Oll, f(x) :2:: -5, donc -10 n'a pas d'antécédent par f.
lfl
,-i
y= f(x). A fortiori f n'est donc pas bijective de IJll sur Oll.
0
N On raisonne par l'absurde: supposons que y E f (A) . Par contre la restriction de f à l'intervalle I = [O, +ool. notée fi1
@ JI existe donc z E A tel que: y= f( z). On a donc: f(x) = est strictement croissante et continue (car polynômiale) sur 1.
...., D'après le théorème de la bijection, elle induit donc une bijec-
.c f (z), et donc : x = z, puisque f est injective. Ceci donne
Ol
·;:: que x E Aet x E A, ce qui est clairement absurde. tion de I sur J = f (I) = (- 5, +oo[.
>-
a. On en déduit que y ( f (A), c'est-à-dire que y e: /(A). fi'j
Déterminons 1 . Soit y E J fixé. On résout l'équation d'incon-
0 nue x e: I : y= f(x).
u Ceci prouve que J(A) cf(A).
e) Soit y e: f (A). Par définition y ( f (A). y= f(x) = y=x2-5 =x= ± Jy 2 +5 =x=
x::co
Jy2 +5

10

www.bibliomath.com
Logique, e nsem bles, signes L, e t n Chap,itre 1
Donc: c) On raisonne par double-implication.

Jjï l : (- 5, +oo[ [0,+oo[ ==> !On suppose queAuB = AuC et que AnB = AnC. Pour
1

montrer que B = C, on procède par double-inclusion.


y Jjï l (y) = J
y2 + 5 • @]Soit x e: B.
b) La fonction g est dérivable sur llll (comme somme de fonc- • • Six e: A, alors x e: An B =An C, et donc: x e: C.
tions dérivables) et: 'ef x e: llll, g' (x) = cosx + 2 > O. g est donc •• Six fl A, alors x e: Au B = AuC, puisque x e: B. Mais x fl A,
strictement croissante sur llll et on dresse facilement son ta- donc x e: AuC donne: x e: C.
bleau de variations : Dans tous les cas: x e: C. Ceci prouve que : B c C.
X -oo a +oo • @] Les hypothèses du problème étant symétriques on ob-
tient de même que: Cc B.
Variations de
f
- oo -----2 -----+oo
On en déduit que : B = C.
1<= 1Il est évident que, si B = C, alors Au B = Au C et An B =
AnC.
La fonction g est donc continue (car dérivable) et strictement
croissante sur 1 = llll. Elle induit donc une bijection de I sur d) On raisonne par double-implication.
J= /(1) = llll. ==> !On suppose queAuB = AnC et que AnB = AuC. Pour
1

Comme 2 e: J. on en déduit qu'il existe un unique a e: I = llll tel montrer que B = C, on procède par double-inclusion.
que: g(a) = 2. De plus, on a g(O) = 0 < 2 = g(a). On a donc: • Montrons d'abord que : Ac C.
a > 0, puisque g est strictement croissante sur llll.
Soit x e: A. On a alors x e: Au B =An C, donc x e: C.
- a) On raisonne par double-implication. • On a donc: AnC =A, donc: AuB =A, ce qui donne: B cA.

1 ==> 1 On suppose que Ac B. Pour montrer que Au B = E, on • Mais alors : AnB = AuC, donne: B = C.
procède par double-inclusion. Puisque B cAc C, on a bien: A= B = C.
• @] Comme A, B sont deux parties de E, on a : Au B c E. 1 =
1n est évident que, si A= B = C, alors Au B =An C = A et
AnB =AUC =A.
• @] Soit x e: E.
• • Si x e: A, alors x e: Au B. - a) On utilise les règles de calculs sur les ensembles.
• • Si x fl A, alors x e: A et à fortiori x e: B puisque Ac B. Donc :
xe:AuB. (A\B) u (B\A)
Dans tous les cas: x e: Au B. Ceci prouve que E c Au B. (AnB)u (BnA)
On en déduit que E =Au B. (AuB) n (Au A) n (Bu B) n (B uA)
1 = !OnsupposequeE=AuB. distributivité
(AuB) n (Bu A)
Soit x e: A. Raisonnons par l'absurde: supposons que x fl B. AUA=E
On a alors x e: An B = AuB, d'après les lois de Morgan. Or (AuB) n (AnB)
AuB = E, donc on a x fl E, ce qui est absurde puisque x e: Ac E. lois Morgan
On en déduit que: x e: B. Ceci prouve que: Ac B. (AuB)\(An B)
b) On utilise les règles de calculs sur les ensembles. MB
u
0
C:
::J
(A\B)\(A\C) = (AnB)\(Anc)
b) Pour montrer que B = C, on raisonne par double-inclusion.
0
lfl
(AnB) n (AnC) @]Soit xe: B.
,--i
0 (AnB)n(AuC) • Six e: A, alors x e: An B. Ainsi, d'après la définition de la dif-
N lois Morgan férence symétrique, x fl MB, et donc : x fl A6.C. Ceci donne:
@
...., ( (An B) nA) u ( (An B) ne) xft AuB ou x e: AnC.
.c distributivité
Ol Comme x e: B, à fortiori: x e: Au B. On a donc x e: An C, ce qui
ï::::
>- 121u((AnB) n c) donne bien : x e: C.
a. AnA=0
0 • Six fl A, alors x e: B\A, et donc: x e: A6.B. On a donc x e: MC,
u (AnB) ne= (A\B) ne c'est-à-dire: x e: A\C ou x e: C\A. Comme x fl A, ceci donne
-
commutativité
(AnC) nB = (AnC)\B x e: C\A, et à fortiori: x e: C.

11

www.bibliomath.com
Chapitre 1 Logique, ensembles, signes Let n
Dans tous les cas, on a: x E C. On a donc établi que: B c C. • On commence par calculer la somme de droite pour i E
BLes hypothèses du problème étant symétriques on obtient [1, n] fixé. On a:
de même que: Cc B.
n n n n(n + 1)
On en déduit que: B = C. .L (i + j) Li + .[ j =Card([l,nil) xi+ - -
j =l j =l j =l 2
- a)SoitxEA. . n(n + 1)
On aalorsf(x) E B = f(A), etdoncxE r 1 (B) =r 1 (f(A)). = nz + - - -
2
Ceci prouve que: Ac 1(f(A)). r car, dans la première somme, le terme général ne dépend pas
On suppose désormais que f est injective sur E. Pour montrer de l'indice, et, la seconde somme est aritlunétique.
r
que A = 1 (.{(A)), il reste à établir que: .r 1 (.{(A)) c A.
Ceci donne, en mettant en facteur les termes constants par
Soit XE r 1 {f(A)). rapport à l'indice de sonunation :
On a alors f(x) E f(A), c'est-à-dire qu'il existe un z E A tel que:
f (x) = f(z). Mais f est injective, donc x = z , ce qui donne n n 11 11
n ( . n(n + 1)) • n(n + 1)
bien: XEA. .[ Eu +jl .[ m+
2
=n:[z+
2
.[1
i =l j=l i=l i =l i=l
Ceci prouve que r
l {f (A)) C A, et donc, par double- n(n + 1) n(n + 1) 2
r
inclusion: A= 1 (f(A)). nx
2
+n x
2
= n (n + 1).
b) Soit y E f(r 1 (B)).
Il existe alors X E r
J (8) tel que: y = f(x) . Et X E l (8) r • Formellement: I:
n i nn
= I: I: = I: I: . On obtient :
donne: y= f (x) E B. ls.js.is.n i=lj=l j=li=j
Ceci prouve que: f (r l (B)) C B.
Il i
On suppose désormais que f est surjective de E sur F. Pour E 1 = :[:[1
montrer que B = f(f- 1 (B)), il reste à établir que: B c ls.js.is.n i =lj=l
f(.r 1 (BJ).
Soit yE B. Dans la sonune double, on conunence par calculer la somme
Comme f est surjective, il existe un x E E tel que: y = f (x). de droite. Pour tout i e: [1, n]. on a:
On a f(x) = y E B, et donc x e: A = 1- 1 (B), ce qui donne:
i
Y= f(x) E f(A) =f(r 1 (B)). .[ 1 = Card( [1, iill x 1 = i
Ceci prouve que B c f(f- 1 (B)), et donc, par double- j=l
inclusion: B = f(r 1(B)).
ce qui donne :
- a)• Il s'agit d'une somme dont le terme général ne
dépend pas de l'indice, donc: [, 1 = [, i = n(n + 1)
l s.js.is.n i= l 2
n
L 1 = Card( [1. nill x 1 = n x 1 = n
k=l b) • On fait apparaître la formule du binôme:
• Dans une somme double, on commence par calculer la
u somme de droite, pour toute valeur possible du premier in-
0
C:
dice.
::J Il
0 On vient de voir que pour tout i E [1, nil : I: 1 = n, résultat
lfl j =l
,--i indépendantdei,donc
0
N
n n n
@
...., L L 1 = .L n = Card( [l, nil) x n = n 2
.c i =l j =l i =l • On fait apparaître la formule du binôme :
Ol
ï:::: n

0
>-
a. • On connaît les sommes arithmétiques : I: k =
k=l
ne;+ 1). d'où : f (n) lk 1n-k_(n)~3 1o
k =O k 3 n
u
~ .
L,
i= l
z + =
~ .
L. J
j=l
n(n + 1)
2
+
n(n + 1)
2
=n n + (
1) (i+}r-(ir=(~r-(ir
12

www.bibliomath.com
Logique, e nsem bles, signes L, e t n Chap,itre 1
• On utilise la formule de factorisation. Pour tout k E [1, nil, obtient:
km= n(t::n . On obtient:
f
k=O
k(k - l)(n)
k
= o(n) + o(n) +
0 1
f
k=2
k(k - l)(n)
k
f k(n) = o(n) + f k(n) = f n(n - 1)
k=O k 0 k= 1 k k= 1 k- 1
Ln n(n-1) (n
k -- 2)
2
= n f (n-1)
k=l k-1
k=2

n(n-1) [.n (n - ,2)


k =2 k-2
On effectue alors le changement d'indice j = k- 1
On effectue alors le changement d'indice j = k - 2

n ntl (n ~ l) = nntl (n ~ 1) 11n-1- j j


t'on k(k-1) k
(n) n(n-1) n-2
L (n ~ 2)
j=O 1 j=O 1 j=O J
n(l+l)n- 1 =n2n- l
n(n-1) nL-2(n -2) . 2.
. 111n- -1
Ce calcul est important en probabilités: il donnera l'espérance j=O J
d'une loi binomiale. n(n- 1)(1 + 1) 11- 2 = n(n - 1)2n- 2
• De même la formule de factorisation donne, pour tout k E
[o. nil: kl1 m
= nl1 (i:D. On obtient :
On a donc

f k2 (n) = n(n - 1)211- 2 + n2n-l =n2 11


-
2 ((n - 1) + 2)
11
E 1- (n)
- E1- + 1) - -
11
1E + 1) (n 11
(n k=O k
k+1 k
k=O k=O n + 1 k + 1 n + 1 k=O k + 1 n(n+ 1)2n-2

On effectue alors le changement d'indjce j = k + l : Ce calcul est important en probabilités: il donnera la variance
d'une loi binomiale.
c) • Pour tout k e: [l; nil, on a:

ln ( 1 + i) = ln ( k; l) = ln(k + 1) - ln(k).
_l
n +1
(rf (n~
j =O J
1)-(n + 1))
0 On a donc, par télescopage:

_l_
n +l
(nf (n~
j=O J
1) 1j 1n+l-j - l) 1)
[n ln ( l+- = En (tn(k+l)-ln(k))=ln(n+l)-ln{l).
k=l k k=l
_1_ ((1 + l)n+l -1) Donc:
n+l
= _1_ (211+1 - 1) f
k =l
ln{l + .!.) = ln(n + l).
k
n+l
• De même, pour tout k e: [l;n], on a:
• Ce calcul classique repose sur l'astuce suivante:
'O
0 k 2 = k(k - l) + k. Ona: 1
C: ----
::J k(k+ 1) k k+l
0
lfl
,--i
k=O
f k2(n)
k
= f
k=O
(k(k - 1) + k] (n)
k
On a donc, par télescopage :
0 11
n (1 1 ) 1
N
@
...., = f
k"':o
k(k - l)(n) +
k
f k(n)
k"':o k
1
kz;I k(k + l) - t'i k k+l - l
1
n+l .
.c
Ol Donc:
·;:: n 1 1
0
u
>-
a.
La deuxième somme a été calculée précédemment et est égale
à n2n- 1. t' 1
k(k + 1) =1 - n + 1·
Pour la première somme on utilise deux fois la formule de fac- Comment vérifier ces calculs avec Python? On peut utiliser le
torisation: pour tout k E [2, nil, k(k- l)(Z) = n(n- l)(i:~). On script suivant :

13

www.bibliomath.com
Chapitre 1 Logique, ensem bles, signes Let n
- Il suffit de vérifier la formule donnée par récurrence
def f(k) :
return 1./(k"(k+l))
sur l'entier n. La difficulté réside dans le fait qu'à n fixé, la
#en Python 2. 7 on utilise 1 . pour que / soit formu le doit être vraie pour tout p E llü, nn.
Il ne faut pas
#la division des flottants oublier de l'inclure dans l'hypothèse de récurrence.

def somme(n) :
• Pour n =0, p e: Uo, n] donne p =0 et alors:
s=f(l) # premier terme de la somme
for k in ran ge(2, n+1)
S+=f(k)
return s

Dans la console, on vérifie que somme ( 1 0) et 1 - 1 . / 11 Or:

-
donnent le même résultat.
a) On nous demande de calculer
n
n k. On a:
(;:~) =U) =l
k=l
• Supposons que pour un rang n EN fixé quelconque, on ait:
n k=lx2x3x .. ·xkx .. ·xn=n!
I!

k=l
n
f
k=p p
(k)-(" +
p+ 1
1)
b) On doit calculer n (2k). On a:
k=l
On doit démontrer que la propriété est vraie au rang n + l,
n n
n =
k=l
(2k) 2X 4X6 X"· X 2k X"· X 211 = k=ln =
21! X k 2"11!
c'est-à -dire que:

c) On nous demande de calculer O (2k + 1). On a :


n

k=O
'v'p E [o. n+ Ill, t~l (k)
k=p p p+l
= (n + 2)

nn (2k +1) = 1
k=O
X 3 X 5 X "· X (2k + 1) X .. · X (211 +1)
Soit p E ll0, n + 1] fixé quelconque.
.. Si p E llO, nil :
Pour ce calcul classique, l'astuce consiste à faire apparaître le
produit des nombres pairs: f
k=p p
(k) + (n+ 1)
p
n 1 x · · · x 2k x (2k + 1) x · · · x 2n x (2n + 1)
n (2k+ 1) =
k =O
(2n + 1)!
2X 4 X 6 X •·• X 2k X ··• X 2n
-
hyp. rec. (;:~)+(";l)
znn!
- n+2)
d) Ona: form. Pascal ( p+l

n k
n-
k=l k+ 1
• • Si p
mais :
=n + l, l'hypothèse de récurrence ne s'applique pas
'O
0 = n+l n+l (k) n+l ( k) (n + l)
C:
::J
k"f,p p =k=~+ 1 n+1 = n+1 = 1
0 Pour vérifier avec Python :
lfl Or :
,--i def f(k) :
(n+2) = (n+2) = 1
0
N return float (k) / (k+ l)
#en Python 2 . 7 on utilise float() pour que / soi~ p+ 1 n+2
@
...., #la division des f lottants
.c La formule est donc vraie pour tout p e: [ 0, n + 1] .
Ol
·;:: def produi t(n) :
>- Ceci prouve que la propriété de récmrence est vraie au rang
a. p =f( l ) # premier terme du produit
0 n+ 1.
for k in range(2,n+l)
u p "=f (k) D'après le principe de récurrence, la formu le est donc vraie
return p pour tout ne: N.

14

www.bibliomath.com
Logique, ensembles, signes L, et n Chap,itre 1
i n n
Pour une preuve directe, remarquer que, pour k E ŒP, n], (;) = Mais, formellement : L =
Il
L L r r.
( k+l) k)
p+ 1 - ( p+ 1 (pour k = p on adopte la conventwn p+. (
p) = 0).
1
l sjsisn i=lj=l j = li= j

En additionnant ces égalités on trouve par télescopage: On peut donc permuter les signes L:

n n · n

ktJ (;: ~)-(p: 1) l


i ·
I:I:4=I:I:4
j=li=j I i=lj=l z

(;:~)-(p:1) On commence par calculer la somme de droite, pour i


fixé quelconque:
E Œl, n]

= (;: ~)-o t ! _~ [ j _ ~ i(i + 1) _ i + 1


j=lz zj=l z 2 2

(;: ~) On a donc:
n n j
I: I:-: n i+l 1(n.
I:-=- n)
I: z+ I:l
j = li=j 1 2
i= l 2 i=l i=l
- Puisque nous ne reconnaissons pas de somme
2
usuelle, simplifions le terme général. On a, pour tout i E = ! (n(n+ 1) + n) =n +3n =n(n+3)
ŒO,p]: 2 2 4 4

(n)i (np -- i)i = itJ.n.--ij!'


n! JA.--i)T n! b) La condition reliant les deux indices est j < i, c'est-à-dire:
n i-1 n-1 n
(p- i)!(n- p)! = i!(p- i)!(n- p)! j !', i - 1. Formellement : L = L L = L L .
lSj<iSn i =2 j = l j = l i =}+l
I.:astuce consiste alors à multiplier en haut et en bas par Les deux formules amènent au résultat, on choisit ici d'utiliser
pl, pour faire apparaître le produit de deux nouveaux coef- la première:
ficients binomiaux: n i-1

n! p!
I: iJ=I:I:iJ
lS}<iSn i =2} =1
------x-
i!(p - i)!(n-p)! p! On commence par calculer la sorrune de droite, pour i E Œ2, 11]
p! n! fixé quelconque:
--- x ---
i!(p - i)! p!(n - p)!
i- 1.. . i- 1 . . (i - l)i i2(i - 1)
I:11 = zI:1 = z- - = - - -
(7)(;) J=l J=l 2 2

On a donc:
Comme le coefficient binomial de droite ne dépend pas
de i, on va pouvoir le mettre en facteur dans la somme à
calculer: I:
lSj<iSn
iJ ! f.. i
2 i=2
2
(i -1)

"O
[ (~)(n=~)
i=Oz p 1 = ! (f.. f..
2 i =2
i3 -
i =2
i2)

0
C: 2
::J
!(n (n+I)2 _ n(n+l)(2n+l) )
0 2 4 6
lfl
,-i n(n + 1) ( n(n + l) _ 2n + l)
0
N 4 2 3
@ n(n+ l)(n -1) (3n+2)
....,
.c 24
Ol
·;::

0
>-
a. 16@8 a) Grâce à la formule du binôme :
u - a) A première vue, on ne va pas savoir simplifier
n .
.2:.f ·
1=1

15

www.bibliomath.com
Chapitre 1 Logique, ensembles, signes Let n
et: qui est une égalité de fonctions définies sur E, c'est-à-dire
que:
Bn = [
k=O k
(n)(-lll n- k = (-1 + l)n =On =O

Soit f e: E.
b) Dans la somme An on distingue les indices k pairs, de la
On pose g = <Pe2 (f), on a donc: V x e: ~. g(x) = f(x + 82) .
forme k =2p, et les indices k impairs, de la forme k =2p + 1 :
Ceci donne, pour tout x e: Ili:

An
·
L (2pn) + Os;2p+Is;n
=Os;2ps;n L (2pn+ 1 )=Sn + T n <Pa1 (g)(x)
g(x + 01)
De même:
f(x +01 +82)

= [ (2n )(- 1)2p + [ ( n )(- 1)2p+1


<Pei +9z (f)(x)
Bn
Os;2ps;n P Os;2p+ Is;n 2p + 1
Comme l'égalité précédente est vraie pour tout x e: Ili, on en
déduit l'égalité dans E:
Ü
$~$J2:)-0$2~191 (2/+ 1)
= Sn - Tn

On en déduit que : et comme celle-ci est vraie pour tout f E E:

1 1 n n-I
Sn = -(An+Bn) = -(2 +0) = 2
2 2
et: Ceci prouve le résultat demandé.
Tn = ~(An -Bn) = ~(2n-0) = zn- I b) Soit f e: E. On a, pour tout x e: Ili:
2 2
n
c) Formellement : [ = [ . On a donc : <Po(f)(x) =f(x + 0) =f (x)
k=O Os;2ks;2n
Ceci prouve que: Vf e: E, <Po(/}= f. Donc: <Po= IdE.
f (2n)-
k=O2k -
L
Os;2ks;2n
(2n)-
2k -
S _2
211 -
211-1 On a donc: <Pa o<P- a =<Pa- a= <Po= Ide .
Et de même: <P- 0 o<Pe ==<Po= ldE.
- 2
a) Soit (81,82) e: lli . On doit établir que: Ceci prouve que <Pa est bijective de E sur E et que:

u
0
C:
::J
0
lfl
,--i
0
N
@
....,
.c
Ol
·;::
>-
a.
0
u

16

www.bibliomath.com
CHAPITRE

Nonibres coniplexes
et trigononiétrie

'Thèmes abordés dans {es e~ercices


- Calculs dans C
- Fonctions trigonométriques
- Equations du second degré à coefficients réels
- Applications de C dans C

Points essentiefs du cours pour {a réso{ution


des e~ercices
- Forme algébrique d'un nombre complexe : parties réelles et imaginaires.
- Forme trigonométrique d'un nombre complexe non nul: module et argument.
- Équations et inéquations trigonométriques.
- Formules d'Euler et de De Moivre.
15
C:
- Le nombre complexe j = ei~ =-!+if.
6 - Formules de résolution dans C des équations du second degré à coefficients réels.
lfl
,-i
0
N
@
....,
.c
Ol
·;::
>-
a.
0
u

17

www.bibliomath.com
Chapitre2 Nombres complexes et trigonométrie

Les méthodes à retenir

- S'il s'agit d'un produit de formes algébriques, on développe.


~ Exercice 2.1

- S'il s'agit d'un quotient de formes algébriques, on multiplie en


haut et en bas du trait de fraction par le complexe conjugué du
Pour déterminer la forme algébrique dénominateur.
d'un nombre complexe
~ Exercice 2.1

- Si un complexe est sous forme trigonométrique: z = rei 6 , on le


met sous forme algébrique: z = rcose + irsin8.
~ Exercice 2.1

- A partir d'une forme algébrique, on calcule le module puis on le


met en facteur, ce qui donne alors sinus et cosinus de l'argument
principal.
~ Exercice 2.1

- S'il s'agit d'une somme ou d'une différence de formes trigonomé-


triques de module 1: eix + eiY ou eix - eiY, on factorise alors par
·~
Pour déterminer la forme e' 2 pour faire apparaître les formules d'Euler.
trigonométrique d'un nombre ~ Exercices 2.1 et 2.3
complexe
- S'il s'agit d'un produit: z =z1z2, on a les formules: lzl =lz1 llz2I et
Arg(z) =Arg(zi) + Arg(z2) [2rr].
~ Exercice 2.1

- S'il s'agit d'un quotient: z = ~ ' on a les formules: lzl = ::: et


Arg(z) =Arg(zi) -Arg(z2) [2rr].
~ Exercice 2.1

"O
0
C:
::J - Pour résoudre une équation, on se ramène aux équations de réfé-
0 rences:
lfl
,-i 1) cos a= cosb ~a= b [2rr] ou a= -b [2rr),
0
N 2) sin a= sinb ~a= b [2rr] ou a= rr - b [2rr ],
@ Pour résoudre équations et inéquations 3) tan a= tanb ~ a= b [rr].
....,
.c trigonométriques ~ Exercices2.6et2.12
Ol
·;::
>-
a. - Pour résoudre une inéquation, le mieux est d'utiliser le cercle tri-
0 gonométrique pour visualiser les intervalles solutions.
u
~ Exercice 2.6

18

www.bibliomath.com
N ombres comp lexes et trigonométrie Chap,itre 2

Pour linéariser une expression trigonométrique (=transformer


produits et puissances en additions), on commence par rempla-
cer chaque terme par son équivalent dans la formule d'Euler, puis
on développe.
~ Exercice 2.2

Pour transformer des expressions - Pour calculer une somme d'expressions trigonométriques, on
trigonométriques utilise la forme algébrique de l'exponentielle complexe : é =
é i;P,e(zl( cos(.9'm(z)) + isin (.9'm(z))) .
~ Exercices 2.3 et 2.6

- Toute expression du type acosx + bsinx peut se mettre sous la


v
forme A cos(x + <p) en factorisant par a 2 + b 2.
~ Exercice 2.6

- S'il s'agit d'une équation du second degré à coefficients réels (E):


az2 + bz + c =0 (a t 0), on calcule le discriminant t::. = b2 - 4ac. Si
t::. > O alors (E) a deux racines réelles distinctes : z = - bia..fi,., si t::. = O
alors (E) a une unique racine réelle (appelée racine double} : z =
- 2~, si t::. < 0 alors (E) a deux racines complexes pures conjuguées :
- -b+i .,Ï-K
Z - 2a ·
~ Exercices 2.4 et 2.6
Pour résoudre une équation Les racines complexes de l'équation (E) : z 2 - sz + p = 0 coïn-
polynomiale complexe cident avec les solutions du système d'équations à deux incon-

nues: CS) { x+y = s


xy = p
~ Exercice 2.7

On peut aussi chercher les solutions sous forme trigonométrique,


en procédant par équivalences successives.
~ Exercice 2.10
"O
0
C:
::J
0
lfl
,-i
0
N - On dispose de deux formules concernant le module: lzl 2 =
@ ~e(z) 2 + .9'm(z)2 =zz.
....,
.c ~ Exercices 2.8 et 2.13
Ol Pour faire des calculs sur les nombres
·;::
>- complexes Parties réelles et imaginaires vérifient : 2~e(z) = z + z, 2i .9' m(z) =
a.
0
u z - z, ~e(iz) =- .9'm(z) et .9'm(iz) =~e(z).
~ Exercice2.8 et2.l3

19

www.bibliomath.com
Chapitre2 Nombres complexes et trigonométrie

On a les caractérisations suivantes :


z
1) z e: ~ ç:::::> 5 m (z) = O ç:::::> z = ç:::::> z = O ou Arg(z) = O [n ),
2) z e: i~ ç:::::> ~e(z) = 0 ~ z = -z ~ z = 0 ou Arg(z) = ~ [n ].
<.......+ Exercice 2.13

La forme algébrique est unique. La forme trigonométrique l'est


aussi en se rappelent que l'argument est défini modulo 2n .
<.......+ Exercices 2.4 et 2.9
• 2rr
Le nombre complexe w = e1 ïi , pour ne: N*, vérifie wn = 1.
Pour n = 3, on a w = j et alors : j 3 = l, j2 =] et 1 + j + j2 = O.
<.......+ Exercices 2.1, 2.7, 2.11, 2.14 et 2.15

Mettrez sous forme algébrique: z = a+ ib, donne une nouvelle


expression de f(z) en fonction de a= ~e(z) et b = Jm(z) .
Pour étudier une application définie sur
<.......+ Exercice 2.16
un sous-ensemble de C, à valeurs
complexes Mettre z sous forme trigonométrique: z = reiO, donne une nou-
velle expression de f(z) en fonction der= lzl et 8 = Arg(z) [2n J.
<.......+ Exercice 2.16

'O
0
C:
::J
0
lfl
,-i
0
N
@
....,
.c
Ol
ï::::
>-
a.
0
u

20

www.bibliomath.com
Nombres comp lexes et trigonométrie Chap,itre 2

Énoncés des exercices

---- - Calculs de formes algébriques et trigonométriques

a) Déterminer les parties réelle et imaginaire de z


nométrique.
= (!~g!. Donner sa forme trigo-

b) Soit z = J2- J3- i J2 + \1'3. Calculer z2 , puis déterminer module et argument


dez.
c) Soit f) € (0, 21t). Déterminer module et argument de ei0 + 1 et e16 - 1.
d) On pose .i ei ?f- = =-! +if. = =
Montrer que _;3 1, J _;2 et 1 + .i + _;2 O. =
Utiliser ces formules pour simplifier les nombres complexes (1 + J) 5 et ). j )4 puis
0
déterminer leurs formes algébrique et trigonométrique.

---- - Exemples de linéarisations


Linéariser les expressions trigonométriques suivantes.
a) cos 4 (x).
b) cos(2x)sin3 (x).
c) cos 2 (x) sin(2x) cos(3x).

---- - Calculs de sommes d'expressions trigonométriques

a) Calculer : S = cos ( 1 1)+ cos (3 f1 ) + cos (5 f1 ) + cos (7 fi ) + cos (9 fr).


n n
b) Soit n € N. Calculer: Sn(x) = [ cos(kx) et T n(x) = [ sin(kx), en fonction de
k =O k=O
X€~.
n n
c) Soit n € N. Calculer: S 11 (x) = [ (Z) cos(kx) et T n(x) = [ (Z) sin (kx), en fonction
k =O k =O
de X€~-
n n
d) Soit n € N. Calculer : Sn(x) =[ cosk(x) et In(x) =[ sink(x), en fonction de
k=O k =O
XE:lli.

---- -
u
0
C:
::J
0 Equations polynomiales complexes
lfl
,-i Résoudre dans C les équations suivantes.
0
N a) z 2 + z + 1 =O.
@ b) z 2 -2z+5=0 .
....,

---- -
.c
Ol
·;::
>-
a.
0
Exemple d'équation non polynomiale
u Résoudre dans C : z 2 + z2 = o.
Représenter graphiquement les solutions obtenues.

21

www.bibliomath.com
Chapitre2 Nombres complexes et trigonométrie

---- - Equations et inéquations trigonométriques


Résoudre dans~ les équations ou inéquations trigonométriques suivantes.
a) 2cos(2x+ ~) =
b) sin(x)s-!.
v'3.

c) sin2 (x) + 3cos(x) - 1 = O.


d) cos(2x) - v'3sin(2x) = 1.
=
e) sin 2 (2x + ~) cos2 (x+ ~).
f) sin(x) +sin(2x) +sin(3x) = O.

---- - Exemple de systèmes somme-produit

a) Résoudre dans C : (S) { x + y = 2


xy = 2

b) On pose: w = ei ~, u = w + w2 + w4 et v = w3 + w5 + w6 . Calculer u + v, uv et en

---- -
déduire la valeur de u et v.

Deux identités remarquables

a) Etablir que :

b) Soit (z 1, z2, u) E c 3 tel que: z1z2 = u 2 . Montrer que:

---- - Surjectivité de l'exponentielle complexe


Soit Z E C*. Résoudre dans iC l'équation d'inconnue z: ez =Z. Que peut-on en dé-

---- -
duire sur la fonction exponentielle complexe?

Exemples d'équations complexes

u
0 a) Montrerquel'onal'équivalence :z3 = 1Q zE {l, j,J},
C:
::J 2hr
0 où l'on a noté j =e T.
lfl
,-i b) En déduire les solutions complexes de l'équation : z3 = -(2 + i)3 .
0
N c) Résoudre dans C : z 6 - 2z3 + 2 =O.
@
...., d) Résoudre dans C: z=
j z2 .
.c

0
u
Ol
·;::
>-
a.
---- - Exemple d'équation non polynomiale
Résoudre dans C : 1 + u~i)
+( !~ i) 2
= O.
On donnera les solutions sous forme algébrique.

22

www.bibliomath.com
Nombres comp lexes et trigonométrie Chap,itre 2

---- - Exemple d'équation trigonométrique

a) Soit ne: N* . Etablir que:

n- 1 sin 2 (nx)
'v' x e:JO, n[, L
k=O
sin((2k+l)x) = - . - -
sm(x)

b) En déduire les solutions dans JO, n[ de l'équation:

(E) sin(x) + sin(3x) - sin(4x) + sin(5x) + sin(7x) = 0

---- - Détermination de sous-ensembles de C


Déterminer l'ensemble des nombres complexes z tels que:
a) lzl = lz-6+5il;
b) z(2z + l) = l;
c) z+4i e: ~ -
5z-3 '
d) .0"ee(z-1)=0·
z+l '
e) Arg( z+q
Z-1
= - li
4
[ni

---- - Sommes de coefficients binomiaux "de trois en trois"


Soit ne: N. On considère les sommes :

An= f (~), Bn = f (~)/. Cn = f (~)t.


k=O k=O k =O

et les sommes

u
0
Sn= L
Os:3ks:n 3 k
(n) , Tn = L ( n
Os:3k+ls:n 3 k+ 1
), Un= L ( n
Os:3k+2s:n 3 k + Z

C:
::J
0 a) Calculer An, 8 11 et C 11 en fonction den.
lfl b) En déduire Sn, T n et U 11 en fonction de n.
,-i
0
N
@
....,

---- -
.c
Ol
·;::
>-
a.
0
Un calcul de somme
u . 2n n.- 1 k
Soit ne: N*. On pose: w = e1 ïi . calculer: I: (1 + w )n.
k=O

23

www.bibliomath.com
Chapitre2 Nombres complexes et trigonométrie

---- - Exemple d'application de C dans C


On considère l'application:

f: C\{- i} C
z- i
z z+ i

On pose .01> ={z e: C/ Jm(z) > O}, CC = {z e: Cl lzl =1 et z;: - i l et 99 =fz e: Cl lzl < 1).
a) Déterminer l'image réciproque de la droite réelle par f.
b) Déterminer l'image directe de CC par f.
c) Etablir que f induit une bijection de .01> sur~. et déterminer son application réci-
proque.

u
0
C:
::J
0
lfl
,-i
0
N
@
....,
.c
Ol
·;::
>-
a.
0
u

24

www.bibliomath.com
Nombres complexes et trigonométrie Ch ap,itre 2

Du mal à démarrer ?

. . a) Pour la forme algébrique, mu ltiplier z par (1+1)


Cl +!>!. Pour la le résu ltat du a).
forme trigonométrique, mettre le module en facteur.
. . Mettre Z sous forme trigonométrique puis chercher z sous
b) Calculer z 2 .
,0 forme a lgébrique.
1
c) Mettre en facteur e 1.
d) Effectuer les calculs sous forme trigonométrique. GD a) Chercher les solutions sous forme trigonométrique.
. . a), b) etc) Utiliser les formules de De Moivre et d'Euler, puis b) Se ramener aux racines cubiques de l' unité.
celle du binôme... c) Util iser le changement de variable: Z= z3.
d) Chercher les solut ions sous forme trigonométr ique.
Ill a) et b) Utiliser la formule de De Moivre et reconnaître une
somme géomét rique. 1111 Utiliser le changemen t de variable: Z= ; : ~.
c) Utiliser la formu le du binôme et reconnaître une somme géomé-
trique. fl9 a) Utiliser les formules de De Moivre et d' Eu ler.
d) Reconnaître une somme géométrique. b) Uti liser le a) pour une valeur particulière de n.
Ill a) et b) Calculer le discriminant. 1111 a) et b) Poser x=~e(z) et y=Jlm(z).
Ill Chercher les solutions sous forme algébrique. c) Utiliser z E: ~=
=
z = z.
d) Utiliser ~e(z) 0 <==> z =-z.
1!11 a) Déterminer un a E: ~ tel que : cos a=J.}. e) Remarquer que Arg(l - i) = -f
[2rr) .
b) Utiliser le cercle trigonométrique.
=
c) Uti liser le changement de variable : z cosx.
fJII a) Utiliser la formule du binôme.
d) Mettre le membre de gauche sous la forme : Acos(x + <j>). b) Uti liser le fait que l + j +} =O.
e) Utiliser les formules: cos0 = sin(! - 0), puis: - sin(0)
sin(-0). 811 Faire apparaître une somme double en développant le terme
f) Simplifier le membre de gauche. général à l'aide de la formu le du binôme, puis permuter les signes
L.
Ill a) Résoudre l'équation du second degré associée.
. . a) Utiliser z E: r
1(~) <==> /(z) f(z).=
b) Remarquer que w7 = 1.
b) Mettre z sous forme trigonométrique.
. . a) Utiliser la formule: lzl 2 = zz. c) Commencer par montrer que f (9) c @. Etablir ensuite que tout
b) Elever le membre de droite au carré puis utiliser judicieusement élément de@ admet un unique antécédent dans 9 par f.

"O
0
C:
::J
0
lfl
,-i
0
N
@
....,
.c
Ol
·;::
>-
a.
0
u

25

www.bibliomath.com
Chapitre2 Nombres complexes et trigonométrie

Corrigés des exercices

- a)Ona : On en déduit que lz 21= 2J /32 + 1 = 2v'4 = 4 et alors:


2

(1 + i)2 (1 + i) 2 (l + i) 3 (1 + i)5
z =-- = - - - - = ---
(1 - il3 (1 - i)3 (1 + il3 (Il - il2l3 Z2 (-v'3 .
= 4 - -+z - 1 ) = 4 ( COS ( 71!) + !Slll
2 2 6
. . ( 71! )) = 4e1· 11!
6 6

Or Il - il 2 = 2 et, d'après la formule du binôme :


On a donc lzl = M = 2 et Arg(z2 ) = 2Arg(z) [21!1, ce qui
(1 + i)s =[
k=O
(5)k l donne:
1
Pour déterminer les (k), on utilise le triangle de Pascal: Arg(z) - Arg(z2) [Tl)
2
71!
1 - [Tl )
12
1 1 71! 191!
- [2n ] ou - [2n]
1 2 1 12 12
1 3 3 Mais ~e(z) > 0 et .!I m(z) < 0, donc :
4 6 4 191!
Arg(z) = (21!)
5 10 10 5 12
Donc: c) • On a:
(1 + i)s lio + 5i 1 + 10i2 + 10i 3 + 5i4 + 1i5
e 1·O(·O
2 e'ï. +e- 1·O)
2
1 +5i - 10 - 10i + 5+ i

D'où :
-4(1 + i) -
form. Euler
2cos (28) e 1,0
2

- 4(1 + i) 1 .
z= 23 =-2(l+z) Ce qui donne :

c'est-à-dire: ~e(z) = .!I m(z) = -! .


2cos(~) si 8 e: (0, Tl [
D'autre part lzl = ! 11 +il = f = ~.et :
0 si 8 = Tl
- 2cos(~) si 8 e:Jn, 2rr)
z

car cos(x) > 0 pour x e: [o,; [, cos(x) = 0 pour x = ; et


cos(x) < 0 pour x e: ] ~,Tl ] .
l -~ Donc, pour 8 -:fi n :
- e' 4

u v12
0
C:
::J
d'où: Arg(z) = 5 [2n). J' Arg(ei6 + 1) = { ~
l'2.! + Tl
[2n ) si 8 e: (0, Tl [
0 b) Ona: [2n ) si8e:]TI,2n]
lfl
,--i
0 2 car : Arg(- z) = Arg(z) + n [2n]. pour tout z e: C*.
N (J2 - /3r +i (J2+/3r
@ • De même:
....,
.c
Ol
- 2i ( )2 - 13){ J2+ 13) .9( .9
e'ï. e1 2 -e- 1 2 .9)
·;::

0
>-
a. = _2
( 2 - 13) - ( 2 + 13) - 2i v ~22__ _13
- ..
2 zsm (e)2 jl!.
e2
u -2V3-2iv'4-3 form. Euler

-2(13 + i) 2sinrn)/(~+~)

26

www.bibliomath.com
No m bres complexes et trigo nométrie Chap,itre 2

• lt
car: i = e 1 2 binôme:
Ce qui donne:
cos 4 (x) = ( eix ~e-ix)4
Si 0 E]Ü,27I [ = ~ (@ei4x + (1)ei3x e-ix + (i)ei2xe-i2x + (;)eixe-i3x
si 0 = 0 ou 0 = 2n +(!)e- i4x)

car sin(x) > 0 pour x E ]O,n [ et sin(x) = 0 pour x = 0 ou


= ~ (ei4x + 4 ei2x + 6 + 4e- i2x + e - i4x)
X= 1T. = ~(2cos(4x)+8cos(2x)+6)
Donc, pour 0 'I. {O, 2n) : = !(cos(4x)+4cos(2x)+3)

Arg(e16 - 1) = ~+ ~ [2n) • De même:

•211)3 = e 1·21T = 1.
cos(2x) sin 3 (x) = ( ei2x -i;e - i2x) ( eix 2 - ix)3 f
d) • j 3 = ( e1 T
= ~(ei2x + e- i2x>(@ei3x -(r)ei2xe- ix +@eïx e- i2x
· 4n 1·( 4n ) 1· 2n -
Demêmej2 =e' T =e T - 2n =e- T =j. -@e- i3x)
Pour la troisième formule on reconnaît une somme en pro- = ~(ei2x + e- i2x)(ei3x _ 3eix + 3e- ix - e- i3x)
gression géométrique:
= ~ (eiSx _ 3ei3x + 4 eix _ 4e-ix + 3e-i3x _ e-i5x)
. ·2 1- j3 0 = ~ (2isin(5x) -6isin(3x) + 8isin(x))
l + J+J = - - . = - -.=0
· 1-J l-J
= -! (sin(Sx) - 3 sin (3x) + 4 sin (x))
• Ona:
• De la même manière :
(l + j)s -
l+j=-j2 cos2 (x)sin(2x) cos(3x) = ( eix+{ix )2 (ei2x 2 (2x) ( ei3x~e-i3x)
· 10
-1 = ) i (ei2x + e-i2x + Z) ( ei2x _ e-i2x)( ei3x + e-i3x)
-
j9=(j3)3=l
-j = !.(ei7 x + zei5x _ eix + e - ix - ze- i5x _ e - i7 x)
2 /.

l
- -i -
J3 = ) i (2i sin(7 x) + 4i sin(Sx) - 2i sin(x))
2 2 = k(sin(7x) + 2sin(5x) -sin(x))
Et donc 1 (1 + j) 5 I = 1- j 1= lj l = 1 et:
- a)Ona:
Arg( - j) = Arg(j) + 1T
21T 1T • 3,r )
3 +7I = -3 [Zn ) S = f!le ( e'ïî
• 1t )
+f!le ( e'ïî +f!le ( e'îî
· Sn ) +f!le ( e'ïî
· 7n )

+f!le e'ïî ( ·9n)


'O
0
C:
::J
• De même: · n ·b ·~ ·h
= [!le ( e 1 TI + e' ïî + e' ïî + e 1 ïî + e 1 ïî
·b)
0 1 1 1 1 .
- - = - - = - = - =]
lfl
,--i
(1 + j)4 (- j2)4 j8 j2 On reconnaît une somme géométrique de 5 termes, de rai-
0 ·~ · lt
N son e1 ïî i l, de premier terme e 1 TI. On a donc :
@ Donc 10 .;jl4 I = lj l = 1 et:
...., ·" · 3n · 511 · 711
e 1 ïî + e 1 ïî + e 1 TI + e 1 TI + e 1 TI
· 911
.c
Ol )5
·;::
>-
a.
Arg ( (1 :j)4) = Arg(j) = 2; [27I) . 2L
e' 11
1- ( e' îî
, 2n

. 2n
0 1 - e 1 TI
u i !On
· 71 1 - e n
- • D'après les formules de De Moivre, d'Euler et du e 1 ïî . 2 ,,
1- e1 îî

27

www.bibliomath.com
Chapitre2 Nombres complexes et trigonométrie

Pour déterminer la partie réelle de ce nombre complexe, On a donc :


le plus simple est de le mettre d'abord sous forme trigono-
. Sn
métrique. Pour cela on factorise le numérateur pare' IT et cos( IJ,J-) sin(~)
• 11 Sn(X) = ~e(Un(x)) = . (x)
le dénominateur par e1 IT : Slll
2

. 1011 . 511 . Sn . 511 <Yf' • ( Sn ) et:


· n
1
1-e·1 11 · n e1 IT e- 1 IT -e1 IT ·fur7.c.tSlll
1
TI
e IT . 2n = e'·IT - . n- . n . n = e 11 ----'----'-
1 - e' IT e'TI e- 1 rr - e'TI ~sin( 1 1) Tn(X) =Jf m(U n(X)) =
sin(9f)sin(~)
. (x)
sm 2
Ceci donne:
c) De même qu'au b), on pose: U 11 (x) = f. (i)eilcx. On
~e sin0f) e 1·fui) sinOf) . ( ·fur) k=O
s ( sin ( fi)
11 = ~e e' 11 utilise alors la formule du binôme :
sin ( fi)

sin ( t'f) cos ( t'f)


sin( i\)
Encore une fois, on commence par mettre ce complexe
En utilisant la formule classique sin(2x) = 2sin(x) cos(x). sous forme trigonométrique, avant de calculer sa forme al-
on obtient: gébrique. En utilisant le résultat de l'exercice Le): 1+ eix =
·X
sin ( lOn) 2cos(!)e'ï.Cecidonne:
s= 11
2sin(fr) ·X)ll = 211 cos11 (X) e'T
·IIX
Un(x) = (2cos ( X) e'ï
et puisque sin(n-x) = sin(x), on a: 2 2
On obtient:
sin (TC - 1!..) 1
S= 11 =- 11 11
;'7)
11
2sin ( f1 ) 2 Sn(X) = ~e(U 11 (x)) = 2 cos ( 1)cos(

11 'k
et:
b) On pose: U 11 (x) = I: e1 ·x. On alors : S 11 (x) = T 11 (x) = Jf m(U 11 (x)) = 2 11 cos11 ( 1) sinC x)
k=O 2
~e(Un(x)) et T 11 (x) = Jfm(U 11 (x)). Pour calculer U 11 (x),
Il . k
d) • Sn (x) est une somme géométrique de (n + 1) termes,
on utilise la formule de De Moivre: Un (x) = I: (e'x) , de raison cos(x) et de premier terme 1. On a donc :
k=O
ce qui fait apparaître une somme géométrique de n + 1
termes, de raison eix et de premier terme 1.
• Si eix = l, c'est-à-dire x = 0 [2n). alors U 11 (x) = n + l, ce
Sn (X) =
{ n+l
1- cos»+1(x)
1-cos(x)
si cos(x) = 1
si cos(x) i: 1
qui donne:

S 11 (x) = ~e(Un(x)) = n+l et Tn(x) = JfmCUn(x)) = O { n+l


l - cos 11 + 1 (x)
1-cos(x)
six=0 [2n )
six i: 0 [2n )

'O
• Si eix i: l, c'est-à-dire x f. O [2n]. alors: • De même T 11 (x) est une somme géométrique de (n + 1)
0
C: termes, de raison sin(x) et de premier terme 1. On a donc:
::J
0 1- (eixt+l 1-ei(n+J)x
lfl Un(X) = 1 x . n+l six = ; [2n]
,--i 1- e1x 1- eix Î,i(X)
0 { l - sin11+ 1 {x)
N 1-sin(x) si xi: ; [2n]
@ Pour déterminer la forme algébrique de ce nombre com-
....,
.c
Ol
·;::
plexe, le plus simple est de le mettre d'abord sous forme
trigonométrique. Pour cela on factorise le numérateur par WJI a) On calcule le discriminant : 6. = 12 - 4 x 1 x 1 =
>-
· (n+ llx
e' - 2- et le dénominateur par e 1 ï
·X
:
-3 < O. {;équation admet donc deux solutions complexes
a. pures conjuguées :
0
u i (11+l ) x -i i
e 2 e
(n+l )x
- e
(11+llx ~sin((n+l)x)
2 2 - l ± iv'3
Un(X) = - --x- ·x ·x = ei fil2 - - - - - , - -2. - -1 z= - - -
e1 ï 1
e- 2 - e 1 ï ~sin(!) 2

28

www.bibliomath.com
Nombres complexes et trigonométrie Chap,itre 2

b) De même, le discriminant vaut ici 6. = -16 < O. Léqua- c) On commence par faire disparaître sin(x) de l'équation, à
tion admet donc deux solutions complexes pures conju- 1' aide de la formule : sin2 (x) + cos2 (x) = 1. On obtient :
guées :
z = 1 + i ou z = 1 - i . sin2(x) + 3cos(x) -1 = 0 <==> -cos2 (x) +3cos(x) =0
= cos(x)( - cos(x) + 3) =0
- Dans cet exercice on munit Je plan d'un repère or-
thonormé. = cos(x) = 0 ou cos(x) = 3
cos(x) = 0
Si z E<C, on écrit z = x + i y, avec x , y E ~- On a alors :
n
x= [ni
z2 +z2 =0 <==> (x+iy) 2 +(x-iy) 2 =0 2
= x2-i=o car: cos(x) -t- 3, pour tout x E ~-
<==> (x - y)(x+ y) = 0 Lensemble des solutions est donc:
<==> y = xouy =- x.

Graphiquement, il s'agit de la réunion des deux droites


d'équation respective y = x et y = - x.
d) Le membre de gauche de l'équation est de la forme
- a) En remarquant que cos rn) = {}, on a: acos(x) + bsin(x) . On va le simplifier en mettant,./ a2 + b2 =
J12 + (-J3)2 =A= 2 en facteur. Pour tout x E ~. on a:
2cos(2x+ ~) = V3

cos(2x+ ~)=cos(~)
cos(2x) - v'3sin(2x) i
= 2 ( cos(2x) - {j. sin(2x))
n n n n = 2( cos (-J) cos(2x)
2x+ = [2n] ou 2x + = - (2n]
3 6 3 6 + sin (-J) sin(2x))
n n
x =- - [nl ou x =- - [ni
12 4 = 2cos (2x + J)
r.: ensemble des solutions est donc: grâce à la formule: cos(a - b) = cos(a) cos(b) + sin(a)sin(b),
pour tout (a, b) E ~ 2 .
St' = {xE~/ x =- ~ [ni ou x =- ~ [nl}
On peut alors résoudre l'équation:
b) Pour résoudre cette inéquation on s'aide du cercle tri-
gonométrique. cos(2x) - V3sin(2x) = l = 2cos(2x+~) = 1

cos (2x + ~) = cos ( ~)


n
2x+ -
3
= ± n-3 [2nl
n
= x=-
3
[ni ou x = 0 [n i

"O
0
Lensemble des solutions est donc:
C:
::J
0
lfl
,-i Sur la figure (donc dans l'intervalle [O, 2n)), les x solutions sont
0
N entre n + ~ = 76rr et 2n - ~ = 1 ~rr. On en déduit les solutions e) Comme l'argument du sinus et du cosinus sont différents,
@ dans~: on ne peut pas utiliser la même méthode qu'au c). Pour ob-
...., tenir une équation avec seulement cosinus, on utilise la for-
.c
Ol
ï::::
mule: cos(x) = sin rn -
x), pour tout X € ~- On a ainsi:
>-
a.
0 Lensemble des solutions est donc: 2 2
u sin ( 2x + ~) = cos ( x + ~)

.9' = LJ [ -7n + 2kn, -lln +2kn ] <==>


2 2
sin (2x+~)=sin rn-x)
k€ l 6 6

29

www.bibliomath.com
Chapitre2 Nombres complexes et trigonométrie

D'autre part: a 2 = b2
sin(- a), ce qui donne:
= a = ±b et 'ri a e: [ij, -sin(a) = Cette dernière formule permet de résoudre l'équation, pour
X ;é O (27T] :

2 sin(x) + sin(2x) + sin(3x) = 0


sin ( 2x + ~) = cos2 (x + ~)
sin(2x) sin {3:{)
sin(2x+ ~) =sinrn-x) ou sin(x- ~) ------ = O
sin ( f)
7T
= 2x+ -
6
=-7T6 - x [2nl 3
sin(2x) =Oou sin{ ;) =0
ou 2x + ~ = 1T-rn-x) (27T] 3x
2x = 0 [n] ou - = 0 (n]
7T 7T 2
ou 2x + - = x- - [2n]

ou 2x +
6
i =n - (x-i)
6
(2n]
X =Ü [il 2
OU X = Ü [ ; l
2 2 En regroupant les deux cas, on obtient l'ensemble des solu-
= x =O [ ;] oux = ; [2n] oux =- ~ [2n] tions:

~
2
ou x= [ ; ]

= x =O rn1 - a) Par théorème :

L'ensemble des solutions est donc : (x, y) solution de (S) = x et y racines de (E) : z2 -2z+2 = 0

Pour résoudre cette équation du second degré, on calcule son


discriminant : 11 = (-2) 2 - 4 x 1 x 2 = - 4 < O. On obtient deux
f) On va simplifier le membre de gauche grâce aux formules racines complexes pures conjuguées:
de De Tvloivre et d'Euler. On a, pour tout x e: IR?:
-(-2)±iv'4 .
J m( eix + ei2x + ei3x) z= = l±z
sin(x) + sin(2x) + sin(3x) = 2
Jm(eix +(eix)2 + (eix)3)
L'ensemble des solutions de (S) est donc:

• Si eix = l, c'est-à-dire six= 0 (2n ], alors sin(x) + sin(2x) + .:/ = {(1 + i, 1 - i); (1 - i, 1 + i))
sin(3x) = O. x est donc solution de l'équation.
• Si eix ;é l, c'est-à-dire six ;é O[2n], on a: b) Remarquons tout d'abors que w7 = ei211 = 1. On a:

Jm ix 3)
/x 1-(e.) u+v w+w 2 +w 3 + w4 + w5 + w6
sin(x) + sin(2x) +sin(3x)
( l - e1X l-w 6
-
w;i l
w- -
1-w
. _
Jm e1x 1- _ei3x)
.-
= ( 1 - e1x w - w7
u
= 1-w
0 Pour déterminer la partie imaginaire de ce nombre complexe, w- 1
C:
::J
0
on le met d'abord sous forme trigonométrique. Pour cela, on 1-w
.~ •X
lfl factorise le numérateur par e1 T, et le dénominateur par e1 ï . -1
,-i
0 On obtient:
N D'autre part:
@ .
. 1 - e13 x . __ · 3x · 3x
e' T e - i. T - e' T = e12x · 3x
. _ _......;..---'-
,:,,:' · ( 3x )
7"' 1 sm T
...., e'x _ _ _ = e'x uv (w+w 2 +w 4)(w3 +w 5 +w 6)
.c 1- eix eif e-if -ei1 ,,.lfsin(f)
Ol
ï:::: w4 +ws +w6 +3w7 +wa + w9 +wlO
>-
a. On a donc: w4 +w 5 + w6 +3+w+w 2 +w 3
0
u = 3 +(u+v)
sin(2x) sin ( ~)
sin(x) +sin(2x) +sin(3x) = . (x)
Stn 2
= 2

30

www.bibliomath.com
Nombres complexes et trigonométrie Chap,itre 2

On en déduit que u et v sont racines de l'équation: (E) z 2 + z+ D'autre part:


2 = O. On calcule son discriminant: l:i = 12 - 4 x 1 x 2 = - 7 < O.
On obtient deux racines complexes pures conjuguées : 2 l( -z1+z2)2
2- -z1z2 1

- l±iV7
z =---
2
2:i jzr +z~ +2z1z2 -4z122I
On a donc deux possibilités pour le choix des valeurs de u et = 2 41 z 12 +z22 - 2z1z2
1 1

v. On détermine les bonnes valeurs de la façon suivante:


= ~ l(z1 - z2J2I
.!/m(u) .!/m(ei') +.!/m(ei'.!f) +.!/m(ei') 1 2
2 4 8 21z1-z2 I
sin ( ; ) +sin ( ; ) +sin ( ; )
En injectant ces deux formules dans notre premier calcul:
2 4
= sin ( ; ) + sin ( ; ) - sin ( ~)
(1 z1;z2 _ u l + 1z1;z2 + u l/

car : sin(n + x) = - sin(x), pour tout x E llll. !


= (121 + 221 2 + 121 - z2 12) + 2121 z2 I

Or .f e:JO. n [, donc sin { 4.f) > O. De plus sin est strictement


4
On utilise alors à nouveau le résultat du a) :
croissante sur [0, ~], donc : sin ( 27rr) > sin (;). Ceci prouve
que: .!/m(u) > O.
(1 z1 ;z2 _ ul + 1z1;z2 + ul)2 lzi1 2 + lz21 2 + 21z1 z2 I

On a donc: 121 12 + lz21 2 +2lz1 1122 I


- l +i V7 - l-iV7 2
u= etv = - - - (lz1 1+ lz21)
2 2
Ce qui donne, en passant à la racine carrée:
2
- a) Soit (z1,z2) E iC . On a:

lz1 +z21 2 + lz1 -z21 2


= (z1 + z2Hz1 + z2) + (z1 - z2Hz1 - z2) - Puisque Z t 0, on peut mettre Z sous forme trigono-
métrique: Z = IZlei Arg(Z). On a alors:
= (z1 + 22)(Z} + z2) + (21 - 22)(Z} - z2)
=21z1 + ~ +_.?rt2 +22z2 +z121 - ~-_.?rt2 +222 é =z = e~e(z) ei -~ m(z) = IZI eiArg(Z)

= 2(1z1 12 + 122'2) Donc, par unicité du module et de l'argument principal:


b) r:astuce est d'élever au carré, car les modules sont alors
plus faciles à manipuler. On part du membre de droite de
é =Z = ef!lee(z) = IZI et .!/ m(2) = Arg(Z) [27T)

l'égalité: = ~e(z) = ln (IZI) et .!/ m(z) = Arg(Z) [27T)


= 3kEZ/ z = ln(IZl) +i(Arg(Z) +2k7T)
(1z, ;z2 - ul + 1z1;z2 + ul)2
u r.:ensemble des solutions de l'équation est donc:
0 = 1z1;z2 - ul2 + 1z1z:z2 + ul2 +2 1z1z:z2 - ul Iz1z:z2 + ul
C:
::J = 1z1;z2 - ul2 + 1z1;z2 + ul2 + 2 1(z1;z2)2 - u21 .:/ = {ln(IZI) + i(Arg(Z)+2k7T)/ ke: Z}
0
lfl = 1z1;z2 -ul2 + l z1;z2 +ul2 +2l( z1;z2)2-u21 On en déduit que la fonction exponentielle complexe est sur-
,-i
0 jective de iC sur iC*, mais non injective.
N
@
....,
.c
Or, d'après le résultat du a) :

2 2
fju• a) On a :

Ol
·;::
z1 +z2
-
l 2
- - u 1 + 1z1
-- +z2
2
+ u1 2 1z1 ;z2 12 +21ul2 z3 =1 = z3 -1= 0
0
>-
a. 2 2
2~ 121 +22 1 +2l u 1
= (Z - 1) (1 + Z + z2 ) = 0
u 4 = z =1 ou 1 + 2 + z2 =0
1
2
2
lz1 +z2I +2lz122I = Z = 1 OU 2 E {j,J).

31

www.bibliomath.com
Chapitre2 Nombres complexes et trigonométrie

où la dernière affirmation provient de la question a) de En conclusion, l'ensemble des solutions de z 6 - 2z3 +2 = 0 est
l'exercice 2.4, avec j = -!
+ i {f = ei~. donc .:/= .:/1 u .:/2, c'est-à-dire:
b) On va résoudre: z3 = -(2 + i) 3 . On remarque que: zo =
-(2 + i) est solution évidente. On a alors:

z3 = - (2 + i)3 = z3 = z3

=
zo;,10
(~r
z
=l
-
d) • z = 0 est solution de l'équation.
• Si z cl 0, alors on peut mettrez sous forme trigonométrique:
z =rei6 , où r > 0 et 8 e:J - 1t, 1t). On a:

= ;, il
-zo e: fl, ..
= ei ~ (rei6 ) 2
= z e: {zo,jzo,jzo)
z= jz2 = rei0
re- i6 = r2ei(26+~)
En conclusion, l'ensemble des solutions est:
=
l r r2

.9 = {-(2+ i);-(2 + i)j;-(2+ i)J}


= -8 28 + 211
3 [27T]

l
r 1
c) On pose : Z = z 3 . On a alors :
=
r;,!O e - 9
211
[ tl
2
z 6 - 2z3 +2 = 0 <==> z 2
- 2Z + 2 = 0
;( 2n + 2k1l)
3kEZlz=e - 9 T
Pour résoudre cette dernière équation, on calcule son discri-
minant: t::,. = (- 2) 2 - 4 x 1 x 2 = - 4. Elle a donc deux solutions
On obtient en tout 3 solutions distinctes: pour k = o. l ou 2.
complexes pures conjuguées:
En conclusion, l'ensemble des solutions est:
2± iv,Ï . .
Z = - - - = l - t ou Z = 1 + t · 2n · 4n · !On }
2 y,= { o.e- 's;e's;e'T
On obtient donc:

z6 -2z3 + 2 = 0 = z3 = 1- i ou z3 = 1 + i fJ•i t:équation n'est pas définie pour z = 1.

• Résolvons: z3 = 1 + i = /2e 1~. On remarque que:


J • 1(
zo = Pour z cl l, on pose: Z = !~ i. On a Z ~ 1 et:
2 ii e' ïï est solution évidente. On a alors :
2
z +-1 ) + ( -
z +-1 ) =0 1 +z+ z =
2
o
z3 = l+i = z zJ 3= l+ ( -

(~r
z-1 z-1
z
3 -1
=
zo;,!0
=l <=>
Ul
-- = O
Z-1
z -
= -zo e: fl,.i,.il = z3 =1
<=> Z = {l,j,/},
u
= zE {zo,jzo,7zol
0 d'après la question a) de l'exercice 2.10.
C: I.:ensemble des solutions est donc:
0
::J
Le cas Z = 1 est exclus. Traitons d'abord le cas Z = .i. On a :
lfl
,-i
0
N •Résolvons: z3 = 1- i.
Z=j = z+l
--= 1
z -1
.

@ j+l
...., On remarque que : Z= - -
.c j- 1
Ol
ï::::
>-
z 3 =l-i <==> z3=1-i <==> z3 =I+i D'après la question c) de l'exercice 2.1, on a :
a.
0 I.:ensemble des solutions de z3 = 1 - i est donc:
u
j + 1 e~ + 1 ,icosG)/f . 1 i
-. - = 2in = ;%=-z - (11)=-r-; ·
1- l eT -1 ,iisinG)Y3 tan 3 v3
32

www.bibliomath.com
Nombres complexes et trigonométrie Chap,itre 2

De même, on obtient : On a donc sur JO, n i :


sin(x) + sin(3x) -sin(4x) +sin(Sx) +sin(7x) = 0
Z=/ <==> Z=Î
sin 2(4x)
<==> Z=j <==> = sin(4x)
sin(x)
z+ 1 .
= 2 - 1 =1 <==>
Sin(x) ~O
sin2 (4x) = sin(x)sin(4x)
-Z=- - i <==> sin(4x) = Oou sin(4x) = sin(x)
<==>
v3 <==> 4x = O[n] ou 4x = x[2n) ou 4x = n - x [2n]
<==> z = v13·
i
<==> X =Û [ ~l
2
OU X = Û [ 7T
3
l OU X = ~ [ ;
2
l
7T 7T 7T 37T 27T 37T }
Finalement, l'ensemble des solutions est : <==>
X€]Ü,1T( XE { 5'4'2'5'3'4
I.:ensemble des solutions est donc:
.:/ _ { ~ ~ ~ 37T 27T 37T }
- s'4'2' s ' 3' 4
- a) Soient n E 1\.1* et x E)O, n[. Pour calculer cette
somme de sinus, l'astuce est d'utiliser l'ex'J)onentielle
fJFI Dans cet exercice on munit le plan d'un repère or-
thonormé.
complexe:
a) Ona:
n- 1
L sin((2k+ l)x) = n-L1.9"m(ei(2k+l)x) = ..?"m (n-Ll ei(2k+l)x ) lzl = lz - 6+5il
k=O k =O k =O <==> lzl 2 = lz-6+5ïl 2
tout est ;::o
On remarque alors que: ei(2 k+l)x = eix(ei2 x)k et comme <==> x 2 + y2 = (x -6) 2 + (y +5)2
x =&ee(z)
ei 2x t l (car x e:JO, n[), on se ramène à une somme géomé- y=5m(z )
trique:
= / +I =/-12x+36+ / + 1oy+2s
n- 1 11.-l 1 (ei2x)'1 . l-ei2nx <==> 12x-10y - 61 = 0
L ei(2k+l)x = eix L (ei2x)k = eix - . = e1x _ __
<==> ZE@
k =O k =O 1 - e12x 1 - ei2x
où@ est la droite d'équation: l2x- lOy- 61 = O.
n -1 ( . i211x) On a donc: {z EC/ lzl = lz-6+5il} =@.
On a donc: :[ sin((2k + l)x) = J m e1 x 1-e ;zx .
k=O 1- e
b) On a:
Pour calculer cette partie imaginaire, on factorise le numéra-
teur par einx et Je dénominateur par eix : z(2z +l) = l <==> (x +iy)(2(x - iy)+l)=l
x=&ee(z)
y =5m(z)
2
<==> zx +x+zy2 + i ~ + y - ~ = 1
u
0

0
C:
::J On obtient:
<==> l 2x2+x+ 2~ =
0
lfl
,--i
0
N
n- l
[ sin((2k + l)x) = J m e 1 x
k =O
( . l - ei2nx) sin2 (nx)
.
1 - e 12x
= -.--
sm(x)
<==> {2x'+x-; 0
0
@
...., A la place de cette preuve "directe", on aurait pu aussi vérifier J x = - 1 ou!
.c
Ol
·;::
>-
la formule par récurrence (mais c'est un peu plus long). 1 y=O
a. b) On applique la formule précédente pour n = 4: 1
0 <==> z = - 1 ou -
u 2
sin 2 (4x)
Vx e:JO, n[,
. . . .
sm(x) + sm(3x) + sm(Sx) + sm(7 x) = - .- -
sm(x)
On a donc: {zEC/ z(2z+ 1) = l} = {-1, H·
33

www.bibliomath.com
Chapitre2 Nombres complexes et trigonométrie

c) La condition est définie pour z -:J ~. On a, pour z -:J ~ : a :

z+.i) = -TI [TI]


Arg ( - i)
z +. = Arg(l + i) [TI)
Arg ( -
z +4i
z-1 4 z-z
-- € ~
5z - 3 z+ i )
Arg ( (Z - i)(l + i) = O [1l]
z+ 4i=( z +4i) z +i
5z - 3 5z - 3 ----€~
(z- i)(l + i)
z +4i z- 4i
Sz -3 = Sz -3 z +i ( z +i )
(z - i) (1 + i) - (z - i)(l + i)
(z +4i)(Sz -3) = (z -4i)(Sz -3)
z+i z- i
..56-3z + 20iz -12i =..56-3z -20iz + 12i (z - i) ( 1 + i)
(z + i )( 1 - i)

-3(z - z) + 20i(z + z) = 24i <==> (z+ i)(z+ i)(l- i)

=
x =~e(z)
- 6iy +40ix = 24i = (z - i) (1 + i)(z - i)

y = J m(z) En développant et en utilisant les formules z + z = 2E€ e(z) et


= 20x - 3 y - 12 = 0 zz= lzl2 , on obtient :
= ZE @
z +i)
Arg ( - . = n- [n]
z- 1 4
= 2E€ e(z) = lzl 2 - 1.

où 9JJ' est la droite d'équation : 20x - 3 y - 12 = O. En posant alors x = E€e(z) E ~ et y= ~ m(z) E ~ , on a :

On a donc: { z E C/ i;!i E ~} = 9JJ' . Arg ( -z +i)


. = -n [n) <==> 2x =x2 + y2 - l
z-z 4
<==> (x - 1)2 + y2 = 2
d) La condition est définie pour z -:J - 1. On a, pour z -:J - 1 : = lz -11 =2
2

= lz -ll=V2
z - 1 ·~ Cette dernière condition est vérifiée pour z = i et z = -i.
E€e( z - l ) = o
z+ l = --e:z
z+l r.:ensemble des solutions est donc le cercle de centre n d'af-
fixe 1 et de rayon Jï, privé des points d'affixe i et - i.
= : : ~ =-( : :~)
fj@• a) On utilise la formule du binôme, trois fois. On a :
z -1 1- z
= z +l z + 1
=
z;,! - 1
(z-l)(z+l) = (z +l)(l-z)

= zz +/,-1,-1 = - zz +j,-1, + 1
u
0
= 2zz = 2
Bn= Ln (n) / = Ln (n) /1 11 -k=(l+j)\
C:
::J = lzl = 1 k=O k k =O k
0
= z e: <(s et:
lfl
,-i
0
N
C11 = f (n)t
k
k= O
= f (n)tin
kk=O
-k= (1 + ])
11

@
...., où <If est le cercle de centre O et de rayon 1. b) • Si k e: N alors le reste de la division euclidienne de k par 3
.c est égal à 0, 1 ou 2. C'est-à -dire qu'il existe un unique p e: N tel
Ol
ï:::: que: k = 3p, k = 3p + 1 ou k = 3p + 2. Dans la somme définis-
>-
a. On a donc: {ze:C/ E€e( ; ~i) =o} = <IJ\ {-1}. sant An, on distingue les ks'écrivant3p, 3p+ 1 ou3p + 2 :
0
u
e) La condition est définie pour z -:J i et z -:J - i. Dans ce cas, on L (n) L ( n)
An= 0:.3p:rn 3p + 0:.3p + l :.n 3 p + l + 0:.3p+2:. n 3P+ 2
L ( n)
34

www.bibliomath.com
Nombres comp lexes et trigonométrie Chap,itre 2

On a donc : An = Sn + T n +Un . On obtient donc:


On procède de même pour Bn. On obtient:
Sn = ~ (2n + 2(- 1) n cos ( 4 ;n ) ) .
•• On effectue l'opération (L1) + J(L2) + j(L3), ce qui donne:

An+JBn +jCn = (l+}+.i)Sn +(l+ J j+j])În+(l+y2 +/)Un = 3Tn,

car jJ = ljl 2 = 1 et 7 = j2.


On a donc:
Or, si p e: N: j3P = (j3)P = lP = 1, j3p+l = j3P j = j et
1 -
j3P+2 = j3P j2 = j2 = j. Tn
3(An + jBn + jCn)
Ona donc: Bn = Sn+ jTn + ]Un.
~ (zn + J(l + j)n + j(l + j)n)
PourCn:
~(zn+ j(l+ j)n+ j(l+ j)n)
C n= °" (n )-:3p
i...., 1 +
· Os:3ps:n 3p
°" ( n )-:3p+1
i....,
Os:3p+ ls:n 3p+l
I
= ~(2n + 2~e(}c1 + .i)n))

+ °" (
i...., n )-:3p+2
J . D'après les calculs précédents:
Os:3p+2s:n 3p + 2 - 2in 4inn 2t'(2n- 1)n
j(l + j)n = e-T (- l)ne-3- = (- 1) 11 e _3_ ,
2
Et si p e: N: }3P = j3P = Î = l, }3p+l = 3P = Jet 1 3P+ =
1 1 ce qui donne: ~e(7o + j)n) = (-l)n cos ( 2<2n; l)JT ).
13Py2 = y2 = j2 = j=j.
On obtient donc :
Donc: Cn = Sn + ] În + jUn.
Tn = 1 ( zn + 2(- l)ncos ( 2(2n - l)n )) .
An= Sn +Tn + Un (L1) 3 3
• On a donc obtenu : Bn =Sn+ ~T n + ~Un (L2)
{ •• On effectue l'opération (L1) + j(L2) + 1(L3), ce qui donne:
Cn=Sn + JTn+JUn (L3)
Pour en déduire Sn, Tn etUn,onvautiliserl'égalité: l + j +J = An+ jBn + JCn = (1 + j +])Sn+ (1 + y2 + j2)Tn
o. + (l +jJ + Jj)Un
•• On effectue l'opération (L1) + (L2) + (L3), ce qui donne: = 3Un.

An + Bn +Cn = 3Sn +(l + j + j)În + (1 + J + j)Un = 3Sn. On a donc:


1 -
On a donc: Un - (An + jBn + jCn)
3
1
(An+Bn+Cn) 31 (2n+j(l+j)n + -j(I + -j)n)
3
'O
0
C: ~ (zn + (1 + j)n +(l + j)n ) ~ (zn + j(l + j)n + j(l + j)n)
::J
0 1
lfl ~ (zn+ (1 + j)n +(l + j)n) = 3 (2n + 2~e(j(l + j)n))
,-i
0 1
N - (2n +2~e((l + j)n)) D'après les calculs précédents:
@ 3
2hr 4inn 2i(2n+l)n
...., j(l + .i)n = eT (-l)ne - 3- = (-l)ne- 3- ,
.c Mais 1 + j = - } = - j2, donc :
Ol
ce qui donne: ~e(j(l + j)n) = (- l)n cos ( 2<2n3+ l)n ).
·;::
>-
a. 2in 2 4inn
0 (l+j)n=(-j 2)n=(-l)nj2n=(-l)n(eT) n=(-l)ne-3-, On obtient donc :
u
ce qui donne: ~e((l + j)n) = (-l)n cos(~). Un= 1 ( zn+2(-l)ncos (2(2n + l)n)) .
3 3

35

www.bibliomath.com
Chapitre2 Nombres complexes et trigonom étrie

- Pour k e: ŒO, n] fixé : - a) On a, pour z i' -i :

z E rl(~) <=> f(z) E IR!


<=> f(z)=f(z)
Ceci donne: <=> :::=(::!)
z -i z+i
L
n- 1(l+wk)n =nL- 1 ( Ln ( ' . ) wjk ) . <=>
z+i z-i
k=O k=O j=O 1
<=> (z - i) (z - i) = (z + i) (z + i)
On permute alors les signes ! : <=> ,7:Z- i(z+zvî = ,z'z+ i(z+ zv[

n- l
<=> =2i~e(z)
- 2i~e(z)
L_ (l + wk)n <=> ~e(z) =0
k=O <=> Z E i~

On a donc: 1- 1 (~) =ilR!.


Etw 0 = w11 = l, donc: b) Soit z e: CC. Alors z =eie avec-;< 0 <'et:

Donc d'après les formu les d'Euler:


n-1 n-1
et comme [ 1k =[ 1= n : ,{• sm. (02 - 41T)
k=O k=O
f (z) =
f-l
(
. ( e
) = 1.tan - - - = 1.g(0),
1T ) •

j,cos ~ - f 2 4
L (I+wk)n =2n + n-Ll(n)(n-1
n- 1 . )
. L (wll .
k=O J= l 1 k=O
oùg:0 - tan(~ -i)·
Pour je: Œl, n-1] fixé, on a wi fc l, donc :
Comme g est continue et strictement croissante sur ]- ;, 3f [.
n-1 . k 1- (wjt 1-wjn 1- (wn)j
r, (wl) = . g (] - ;, 32n [) = ] lim ng(x), lim g(x) [ =) - =, +=[ = ~.
k=O 1 - wl 1 - wj 1 - wj ' x-- x-,2

u
0
et w11 =l, donc (w =11 =l, ce qui donne:
11
/
On a donc: f (CC)= i!R!.

C: c) • On considère la restriction fig, de f à [!JJ, que l'on note


::J
0 encore f. Montrons d'abord qu'elle prend ses valeurs dans@.
lfl Pour tout z e: [!1>, on a :
,--i
0
N
On a donc:
@
....,
1z-i1 = -lz-il.12
1/(z)I 2 = - .
2 2

z+ z 1z+i
.c
Ol
ï::::
L (n). (n-L,l (wl). k) = nL- l (n). xO = nL_-1O= O,
n- 1

>- j =l 1 k=O J=l l J=l


a. On pose x = ~e(z) et y= J m(z). On a y> 0 (car z E [!JJ) et
0
u et ainsi:
n-1 2
L (1 + wk)n = 2n + O= 2n. l/(z)l2 = x2 + (y - 1) .
x2 + (y+ 1)2
k=O
36

www.bibliomath.com
Nombres comp lexes et trigonométrie Chap,itre 2

Ceci donne: On a donc trouvé une unique solution dans C. Il reste à mon-
trer que cette solution est dans f!/J, c'est-à-dire que e: f!P if~~
f(z) e:@ <=> 1/(z)I < 1 lorsque w E ~-
<=> 1/(z)l 2 < 1 On a :
x2 + (y - 1)2
<=> <l l+w l+w 1- w
x2 + (y+ 1)2
i --
1-w
= i -----
l-w 1-w
<=> _/ + (y-1)2 </+(y+ 1)2
x2 +(y+l)2>0 . (1 + w)(l - w)
1-----
11 - w12
<==> l-2r +I <I +2r +I
l+w-w-ww
<=> 0<4y = i------
ll-wl2
<=> O<y
. l- lwl 2 +2iJm(w)
= !-------
ll-wl2
et comme cette dernière condition est vérifiée, en remontant
les équivalences on obtient que f(z) e:@ pour tout z e: f!/J. 2Jm(w) . l - lwl2
On a donc:
= - ll - wl2
+ z-~-2
ll-wl
f: f!j'J @
2
z z- i
z+i
0 na donc .. ..,.,m
n: ( · l+w)
z 1_w =ll-wl
l - lwl
2•
Doncs1w
· e:=u,
r,, al
ors I w I <l
n:
d onc..,.,m (·l+w)
1 1 _ w > O,1.e.1
· ·l+w ms
• Pour étudier la bijectivité de f, on fixe w e: @ et on résout 1 _ wE.:r.
l'équation f (z) = w d'inconnue z e: f!P : On en déduit que l'équation f(z) = w a une unique solution
z- i
dans f!J>, pour tout w E ~ -
f(z) = w <==> --
z+i
=w Ceci prouve que f est une bijection de f!P sur @. De plus
<=> z-i=w(z+i) f(z) = w <==> z = f - 1(w), on a donc :
#-i
<==> z(l- w) = i(l + w)
ri: @
.l+w
<=> z = z- - w i l+w
w;,!l 1- w 1-w

u
0
C:
::J
0
lfl
,--i
0
N
@
....,
.c
Ol
·;::
>-
a.
0
u

37

www.bibliomath.com
CHAPITRE ~

Suites réelles

'Tlièmes a6ordés dans {es e~ercices


- Étude de la monotonie d'une suite
- Étude de la nature d'une suite (convergence ou divergence?)
- Calculs de limite
- Études de suites récurrentes
- Études de suites définies par une relation implicite
- Calculs du terme général d'une suite
- Calculs et utilisations d'équivalents

Points essentiefs du cours pour {a réso{ution


des e~ercices
- Suites monotones, majorées, minorées, bornées
u - Suites convergentes, divergentes de première ou seconde espèce
0
C:
::J - Opérations sur les limites, sous-suites
0
lfl - Croissances comparées
,-i
0
N - Passage à la limite dans une inégalité, théorème des gendarmes
@ - Théorème de la limite monotone, suites adjacentes
....,
.c - Suites récurrentes
Ol
·;::
>- - Suites arithmétiques, géométriques, arithmético-géométriques, récurrentes linéaires d'ordre 2
a.
0
u - Suites équivalentes

38

www.bibliomath.com
Suites réelles Chap,itre3

Les méthodes à retenir

Pour étudier la monotonie de (un), on détermine le signe de


Un+l - Un en fonction des valeurs den. Celui-ci doit être constant
à partir d'un certain rang.
<.......+ Exercices 3.2, 3.9 et 3.10

- Si la suite (u11 ) est de signe constant au sens strict à partir d'un


Pour étudier la monotonie d'une suite certain rang, on peut aussi comparer le rapport u~;
1
à l.
<.......+ Exercices 3.2 et 3.4

- S'il existe une fonction f définie (au moins) sur un intervalle de ~


et vérifiant Un= f(n) à partir d'un certain rang, alors la monoto-
nie de la suite (un) est la même que celle de la fonction f.
<.......+ Exercice 3.2

On commence par analyser si on peut utiliser les opérations sur


les limites et/ou les croissances comparées. Ce n'est pas toujours
possible car on peut tomber sur une forme indéterminée. Très
souvent, on lève l'indétermination en mettant en facteur au nu-
mérateur et au dénominateur le terme prépondérant.
<.......+ Exercices 3.3, 3.4, 3.7, 3.8 et 3.11

En encadrant la suite, le théorème des gendarmes peut donner


l'existence et le calcul de sa limite éventuelle.
<.......+ Exercices 3.1, 3.3, 3.5, 3.7, 3.9 et3.13

- Si la suite est monotone, sa limite existe d'après le théorème de


la limite monotone. Il suffit alors d'étudier si elle est majorée ou
minorée.
Pour prouver l'existence et/ou calculer <.......+ Exercices 3.4, 3.9 et 3.10
la limite d'une suite
- Si on dispose deux suites, on peut essayer de démontrer qu'elles
sont adjacentes.
u <.......+ Exercices 3.12 et 3.15
0
C:
::J . existe n . , . l d
0 Pour prouver 11m u 11 = {., 11 est eqmva ent e prouver
lfl
,-i
. u 2n existe
1rm . u2n+1 = cn Les sous-smtes
= 11m . ( ) (
u2n et u2n+1
)
0
N peuvent aussi être utilisées pour prouver que la limite de (un)
@ n'existe pas .
....,
.c <.......+ Exercices 3.7 et 3.9
Ol
·;::
>-
a. - Un équivalent simple donne souvent l'existence et la valeur de
0 l'éventuelle limite.
u
<.......+ Exercices 3.10 et 3.12

39

www.bibliomath.com
Cha pitre 3 Suites rée lles

On doit d'abord vérifer qu'elle est bien définie.


~ Exercices 3.2, 3.3, 3.11, 3.14, 3.16 et 3.18

- S'il s'agit d'une formule de récurrence du type Un+l = f(un), où


f: ~ - ~. il est utile de dresser le tabelau de variations de f, de
rechercher ses points fixes et des intervalle stables par f .
~ Exercice 3.1 4

On étudie la monotonie de (Un) en calculant Un+ 1 - Un à l'aide de


la formule de récurrence. Il faut éventuellement discuter en fonc-
tion des valeurs de UQ .
~ Exercices 3.2, 3.14, 3.15 et 3.16

Pour étudie r une suite récurrente Le théorème de la limite monotone permet de prouver l'existence
d'une limite finie ou infine.
~ Exercices 3.14, 3.16 et 3.18

Passer à la limite dans la formule de récurrence (ou dans un de ses


encadrements) donne les valeurs possibles de l'éventuelle limite.
~ Exercices 3.3, 3.14, 3.16 et 3.18

- Si on a plusieurs suites avec des relations de récurrence "cou-


plées", on peut essayer de montrer qu'elles sont adjacentes, ou
plus généralement trouver une nouvelle relation de récurrence en
combinant celles qui sont données dans l'énoncé.
~ Exercices 3.15 et 3.18

Pour définir (un) par la relation fnCun) = 0, il faut étudier la fonc-


tion fn et utiliser le théorème de la bijection.
~ Exercices 3.17 et 3.19

- Si (u 11 ) est définie par la relation fnCun) = 0, et si fn est monotone


alors comparer Un à un réel a reviens à comparer fn(u 11 ) et fn(a) .
En particulier on obtient la monotonie de la suite en comparant
u
0 fn(Un) et fn(Un+d (ou aussi fn(Un) et fn(Un-d).
C:
::J Pour étudier une suite définie par une ~ Exercices 3.17 et 3.19
0
lfl relation implicite
,-i Le théorème de la limite monotone permet de prouver l'existence
0
N d'une limite finie ou infine.
@ ~ Exercices 3.17 et 3.19
....,
.c
Ol
ï::::
Lorsqu'on sait que la limite existe, passer à la limite dans la rela-
>-
a. tion implicite fn (un) =0 (ou dans un de ses encadrements) d!onne
0 les valeurs possibles de cette limite.
u
~ Exercices 3.17 et 3.19

40

www.bibliomath.com
Suites réelles Chap,itre3

On conjecture une formule générale à l'aide des premiers termes.


Cette formule doit ensuite être vérifiée par récurrence.
~ Exercice 3.6

On essaye de reconnaître un des exemples du cours: suite arith-


métique, géométrique, arithmético-géométrique, récurrente li-
Pour calculer le terme général d'une néaire d'ordre 2.
suite ~ Exercices 3.6, 3.10 et 3.11

On effectue un changement de suite, pour obtenir une suite dont


on sait calculer le terme général. On inverse ensuite les formules
pour revenir à la suite de départ.
~ Exercices 3.6, 3.10, 3.11 et 3.18

- Une suite convergente de limite non nulle est équivalente à sa li-


mite.
~ Exercice 3.8

On utilise les équivalents usuels et les opérations sur les équiva-


lents.
~ Exercices 3.8, 3.10 et 3.12

On conjecture que Un - Vn et on le démontre en établissant que :


lim uv" = 1. Pour cela, on peut faire appel à un encadrement et
n-+oo n
au théorème des gendarmes.
Pour déterminer un équivalent simple
~ Exercice 3.13

On ne peut pas additionner des équivalents mais en prenant un


équivalent de chaque terme de la somme on peut deviner l'équi-
valent final. On le vérifie alors "à la main".
~ Exercice 3.8

Il n'y a pas de résultat général permettant de composer un équi-


valent par une fonction, mais on peut supposer que sur l'exemple
considéré le résultat est valide. Il suffit de le vérifier" à la main".
~ Exercice 3.8

"O
0
C:
::J
0
lfl
,-i
0
N
@
....,
.c
Ol
·;::
>-
a.
0
u

41

www.bibliomath.com
Chapitre 3 Suites rée lles

Énoncés des exercices

---- - Exemples d'utilisations des théorèmes sur les limites


Soient (u 11 ) et (v 11 ) deux suites réelles.
a) On suppose que lim un
diverge vers +oo.
n-+oo
= +oo et que (Vn) est bornée. Montrer que (un+ Vn)
b) On suppose que lim u 11
n-+oo
= 0 et que (vnl est bornée. Montrer que (u 11 v 11 )
converge vers O.
c) On suppose que lim Un= +oo. Tvlontrer que (u 11 ) n'est pas bornée.
n-+oo
d) On suppose que (un) et (v 11 ) sont bornées. Montrer que (un+ v 11 ) et (un v 11 ) sont
bornées.

---- - Étude de la monotonie


Étudier la monotonie des suites définies par :

a) \;/ne:N, un = I: 'lc+i;
n - kz

k=O
b) \Jn
V ~ "'' u n -- ~
c.1"', n! ,·

c) \;/ne: 1\1 • u = ln(n) .


' n n ,

d) uoe:im+ et\;/n e:N, Un+l = ln(2 - cos(unl) +7un .

---- - Calculs de limites


Étudier la convergence des suites :

a) \;/ne: N' u n -- l+(-n l)" '·

b) \;/ne:N, u n -- z»
n2 + cos(n•e-
·
11 )·
,
u
0
C:

0
::J c) \;/ne: 1\1, Un = en - n 5 ( cos n + (- l)n};
lfl
,--i
0 d) \;/n EN, Un = 2"-3"
2,,+3";
N
@ = n+sin(2n) .
...., e) \;/ne: 1\1 ' u n n-ln(n 2 ) '
.c
Ol
ï::::
>- f) \;/ne:N, Un= n 2 e- ../n;
a.
0
u
g) uo e: lm et\;/ n e: 1\1, Un+ 1 =cosu,
V,!+î.
n+l

42

www.bibliomath.com
Suites réel les Chap,itre3

---- - Étude d'une suite réelle


Pour tout n e: N, on pose : Un =
a) Montrer que (un) est convergente.
b) Pour tout ne: N, on pose: Vn
4
}1~~;) 2 .

= (n + 1) u~. Etablir que (v 11 ) est convergente.


c) En déduire le calcul de la limite de (un).

---- - Limite d'une somme de n termes


Étudier la convergence de la suite (Sn) nEN * définie par:

Vne:N*, Sn=
n n2
L ~k2 .
k= l n +

---- - Calcul du terme général d'une suite définie par une relation de récurrence
Déterminer en fonction de n le terme général des suites réelles suivantes, et en dé-
duire leur limite (si elle existe) :
a) uo = 1 et Vn e: N, Un+ 1 = ! Un + 1.
b) uo = l, u1 = 2 et Vn e: N*, Un+l =2un + 3Un-l·
c) uo = l, UJ = 0 et Vn e: N, Un+ 2 = 4Un+ l -4un.
d) uo = 1, u1 = 1 et Vn e: N, Un+2 = Un+l - Un.
e) a1 = l, b1 = 0 et Vne: N*, an+ 1 = 2bn et bn+ 1 = an+ bn.
f) uo e: ~ et Vn e: N, Un+ 1 = (urif.

---- - Calcul de limite en fonction d'un paramètre


Discuter en fonction de la valeur de a e: ~ l'existence et la valeur de
fonction de a.
lim an" en
n-+oo

u
0

0
C:
::J

lfl
,-i
---- - Calculs de limites en utilisant un équivalent
A l'aide des équivalents usuels, déterminer la limite de la suite (u 11 ) définie par:

a)
u
v n e: "'*
,,.. , _ (1-coscfï )cos *.,
Un -
0 e-;;'[ -1
N
@ b) Vne:N*, u 11 =nsin(ti);
....,
.c 11
Ol
·;:: C) Vne:N* ' u n = (1 + l)
n ,•
>-
a.
0 sin(i)-sin(f.) .
u d)
u
v
._,.
ne:'" , Un -
_
1 ,
nn -1

43

www.bibliomath.com
Chapitre 3 Suites rée lles

e) rln e: N, Un= vn+Î - .,/n;

ri n E: N * , Un = ( 4 n-l
4 n )2n+l.
f) ,

g) rlne:N*,un=(e-(i+i))'\

h) r/ne: N, Un= ln (sin(-k)) (trouver d'abord un équivalent de u 11 ).

---- - Suite harmonique


Pour tout ne: N, on pose: Hn =

a) Etablir que (Hn) ne:N est monotone.


:[I!
k=l
1
k.

b) Montrer que, pour tout ne: N* : H2 11 - Hn ;;:: !.


c) En déduire que: lim Hn = +oo.

---- -
n- +oo

Calcul du terme général d'une suite par changement de suite


On pose, pour tout n;;,: 2 : Un = IT
k=2
cos ( ffe).
a) Montrer que la suite (un) est monotone et convergente.
b) On pose, pour tout n ;;,: 2 : Vn = Un sin ( 2~,). Montrer que (vn) est une suite géo-
métrique.
c) En déduire Vn puis un en fonction de n.

---- -
d) Calculez la limite de (un) .

Un autre exemple de calcul du terme général d'une suite par changement


de suite
On considère la suite (u 11 ) n définie par uo = 0 et Un+1 = 5:,;i;}, pour tout ne: 1\1.
a) Montrer que la suite (un) ne:N est bien définie et que Un > 1 pour n;;:: 3.
b) En déduire que la suite (vn) neN définie par Vn = ~~=Î est bien définie. Montrer
qu'elle est géométrique.
c) Donner Vn, puis Un en fonction de n .
d) En déduire l'existence et la valeur de lim un.

---- -
n-+oo

u
0 Approximation de n par des suites adjacentes
C:
::J Pour tout n;;:: 2, on pose:
0
lfl
,-i
0
N
@
...., On rappelle les formules de trigonométrie suivante:
.c
Ol
·;:: 2tan(x)
>- sin(2x) = 2sin(x) cos(x) et tan(2x) = - - ~ -
a. l - tan2 (x)
0
u Tvlontrer que ces suites sont adjacentes. Déterminer la valeur de leur limite commune.

44

www.bibliomath.com
Suites réel les Chap,itre3

---- - Équivalent d'une somme de n termes

a) Montrer que pour tout entier n ~ l, on a :

2( v'n+i - rn) ~ )n ~ 2(rn- vn-1).


b) En déduire la limite, puis un équivalent de la suite (un) n~ 1 de terme général :
Un = 1 + _L
v'2 + · ·· + _J_.
,/n

---- - Étude de la suite des radicaux du nombre 2


Étudier la convergence de la suite (u,i) définie par:

'v'nEN, Un = JJ 2+ 2 + · · · + V2 (n+ 1 radicaux).

---- - Étude de suites récurrentes couplées


On considère les suites (un) et (Vn) définies par

uo =0
vo = 2
u ,. , 3un+vn
{ v n E '"• Un+l = --
4-
'v'n e: N, Vn+l = U11~3v11

Montrer que ces deux suites sont adjacentes et déterminer leur limite commune.

---- - Étude complète de suites récurrentes (d'après Escp-Eap 1999 et 2002)


Étudier la monotonie et la limite éventuelle des suites (un) définies par:

a) UJ = 1 et'v'n~ l, Un+l =
l+uf+u~+··+u~,
n

b) uo>Oet'v'ne:N,un+ l =~ -
u
0

---- -
C:
::J
0
lfl Étude d'une suite définie par une relation implicite (d'après Agro-Véto)
,-i
0
N Pour tout entier n ~ 2, on note (En) l'équation (En): xn -x = n.
@ a) Montrer que, pour tout n ~ 2, (En) admet une unique solution Un dans ~+, et
....,
.c prouver que Un > 1.
Ol 2
·;::
>- b) Établir que : 'v' n ~ 2, n ïi ~ n. En déduire la limite de (un).
a.
0 c) On pose, pour tout n ~ 2, Vn = Un - 1.
u
Montrer que: 'v'n ~ 2, n ln(v 11 + 1) =ln(v11 + 1 + n). En déduire que v 11 -
1~ n.
1
n~+oo

45
Chapitre 3 Suites rée lles

---- - Un autre exemple de suites récurrentes couplées (d'après Agro-Véto)


On considère les suites (un) et (vn) définies par

vo > uo >0
- u~
'rln EN, Un+l - -u +v

1 'rlne:N, Vn+I
"il.
=u;;fv,;
17

a) Montrer que (un) nEN et (v11)nEN sont bien définies et qu'elles sont strictement
positives.
b) Montrer que (u 11 ) et (v 11 ) sont convergentes.
c) Calculer leur limite respective.
d) Calculer un et v 11 en fonction de n, uo et a = ~~.

---- - Un autre exemple d'une suite définie par une relation implicite
Pour tout entier n ~ l, on note (En) l'équation (En) : x 11 + xn- l + ·· · + x2 + x = 1.
a) Montrer que, pour tout
prouver que x 11 e: [ ! ,1].
n ~ l, (E 11 ) admet une unique solution Xn dans ~+, et

b) Démontrer que (Xn) est convergente.


c) Calculer sa limite.

u
0
C:
::J
0
lfl
,-i
0
N
@
....,
.c
Ol
·;::
>-
a.
0
u

46
Suites réel les Chap,itre3

Du mal à démarrer ?

Ill a) et b) Ut iliser le t héorème des gendarmes.


1111
c) Raisonner par l'absurde.
a) Raisonner par récurrence.
d) Utiliser les opérations su r les inégali tés.
b) Exprimer Vn +1 e n fonction de V,i.
l9 a) et b) Calculer Un+l - Un,
c) Exprimer Un e n fonction de Vn .
c) Ut iliser une é tud e de fonctions.
d) Uti liser les opérations sur les lim ites.
d) Vérifier d'abord que la su ite est bien définie et positive.

Ill a) et b) Ut iliser le t héorème des gendarmes.


119
c) d) et e) Mettre e n facteur le terme prépondérant q ua nd n - +oo. La m onotonie des suites s'obtie nt à l'aide des formu les données. Pour
f) Faire apparaître les croissances comparées. les calculs de limites, u tiliser l'équ ivalent usuel de la fonction sinus.
g) Utiliser le t héorème des gendarmes.

Ill
a) On demande de prouver la convergence sans déterm iner la lim ite:
a) Utilise r la quantité conjuguée.

..
il faut t rès probablement fai re appel au théorèm e de la lim ite mo- b) Add itionner membre à membre les doubles inégal ités obtenues à
notone. l' aide de la formule précéde nte pour différentes valeu rs de n.
b) Calculer Vn+ l.
V>r

c) Passer à la li mi te dans la relation entre Vn et Un.


En fait Un+l = ,,/2+ un. Ét udier la fonc t ion associée, déterminer ses
Ill Procéder par encadrement et conclure avec le théorème des points fixes et trouver u n intervalle stable qui contient u-0. Pour la mo-
notonie de (Un) uti liser une étude de fonction.
gendarmes.
11111 Étudier la su ite (un - vnl-
1111
a) à d) Reconnaître des suites correspondant à des modèles du cours.
e) Faire apparaître u ne su ite récurrente linéaire d'ordre 2.
11111
f) Conject urer une formule générale. a) Vérifier que, pour tout ne: N : Un ;;: n.
b) Trouver une relation de récurrence simp le entre Un+ 1 et Un .
Ill
Distingue r différents cas suivant les valeurs de a e n s' inspiran t de ceux
connus pour la limite de la su ite géom étrique (a"lru,N·

1111 a) et b) Ut iliser les opérat ions sur les équivalents.


a) Étud ier ln : x - xn - x.
2
c) Passer à la forme exponent ielle. b) Calculer ln ( n ïi }.
d) Ne pas additionner les équivalents mais ut il iser la trigonomé-
c) Déterminer la li m ite de ln(~~:n).
t r ie. 11
e) Faire apparaître l'équ ivalent usuel approprié...
f) e t g) Passer à la forme exponentielle et uti liser l'équivalent usuel
du logari th me.
a) Procéder par récurrence.

..
h) Les t héorèmes généraux ne permetten t pas de passer au loga-
rithme dans un équivalent, mais vérifier que sur cet exemple cela b) Ut iliser le théorème de la limite monotone.
donne le bon équivalent pour u 11 •
c) Étud ier la sui te (un - Vn) , puis passer à la lim ite .
u
0 d) Ét udier la suite ( ~;; ).
C:
::J a) Calculer Hn+1 - Hn.
0
lfl b) Pour minorer une somme, on commence par minorer son terme gé-
,-i
néral. ..
0 a) Étud ier la fonction x- x" + · · · + x2 + x.
N c) Uti liser le théorème de la lim ite monotone. 1 (Il
@ b) Remarquer que pour tout réel posi tif différent de 1 ln (x) = x 1-~x .
...., L'idée est m ai ntenant de calculer ln(Xn+tl et de le comparer à 1 ce
.c
Ol =
qui nous permettra de comparer x 11 et Xn+ 1 pu isq ue ln (x 11.) 1 et
ï:::: a) Vérifier que la suite est à termes str ictem ent positif et étudiez
11 1
~; • q ue ln est croissante sur IR+ .
0
u
>-
a. b) Faire de même avec (vn) .
d) Util iser le l'équivalent usuel de la fonction sinus.
c) Utiliser fn(Xn) = 1 = Xn t~: pour en tirer une nouvelle re lat ion
sans oubl ier que lxn l < 1 pour n ~ 2.

47
Chapitre 3 Suites rée lles

Corrigés des exercices


- a) Par hypothèse (vn) est bornée, c'est-à-dire qu'il ou encore:
existe une constante M ? 0 telle que : 'v'ne:N, - M~un~M

De plus lim un = +oo. Le théorème des gendarmes (ver-


'v'ne:N, lvnl ~ M 11- +oo
sion limites infinies) donne alors que lim M existe et est
n-+oo
ou encore : égale à +oo. C'est absurde car lim M = M (suite station-
'v'ne:N, -M ~ Vn ~ M n- +oo
naire).
En additionnant Un à chaque membre de l'inégalité: La suite (un) n'est donc pas bornée.
d) Puisque (un ) est bornée, i1 existe M1? 0 telle que:
'v'ne:N, - M+un~Un + Vn~M+un

Par somme de limites: lim ( - M +un)= - M + oo = +oo.


n-+oo
D'après le théorème des gendarmes (version limites infi- De même (vn) est bornée, donc il existe M2 ? 0 telle que:
nies), lim (Un + vn) existe et:
n- +oo

lim (Un + Vn)


n-+oo
= +oo
• D'après l'inégalité triangulaire:
b) De même (vn) est bornée, donc il existe une constante
M ? 0 telle que : 'v'n e: N, lun + Vnl ~ lun l + lvn l

Donc, par somme d'inégalités :

En multipliant par Iuni qui est un réel positif:

Comme M1 + M2 ne dépend pas de n, on peut donc en


déduire que (un+ v 11 ) est bornée.
et comme lunvnl =lunllvnl, on a: •Ona :

Donc, par produit d'inégalités (tous les termes sont posi-


Par opérations sur les limites: lim (Tvllun 1) = M x O= O. tifs) :
n-+oo
D'après le théorème des gendarmes (version limites fi-
nies), lim IUn v11 1existe et:
n- +oo Comme M1M2 ne dépend pas den, on peut donc en dé-
duire que (un vn) est bornée.
lim IUn Vn 1= 0
n-+oo

"O De plus: . urtoutne:N;


0
C:
::J
0
lfl
Un+l - Un
,-i Donc, toujours d'après le théorème des gendarmes (ver-
0
N sion limites finies), lim UnVn existe et:
n- +oo
@
....,
.c lim UnVn
n- +oo
=0
Ol
·;::
>-
a. c) Par l'absurde, on suppose que (u 11 ) est bornée, c'est-à- =
0 dire qu'il existe M? 0 telle que:
u car : 'v'xe:llll, ex ?O.
Ceci prouve que (un) ne:N est croissante.

48
Suites réelles Chap,itre3

b) On remarque que: 'rln E N, Un > 0 (produits et quo-


tients de nombres strictement positifs). De plus, pour tout def u(n) :
return log(n)/n
nEI\I:
x=range(l, 11)
Un+l (n+ l)! 22n
y =[u(n) for n in x]
Un z2n+2 ~ import matplotlib.pyplot as plt
plt. plot(x, y, '• ', markersize=S)
n+l _n_t2 2 f
= 4 "jinl,;
plt.ylim([ - 0.1 ,0 .4])
plt . xlim([ - 0 . 5,10.5])
n+l pH.show()
4
donne la figure :
Or 1
11 1
;;:: 1, pour n ;;:: 3.
Donc: o.,~ - - - - - - - - - - - ~

0 .3

Comme Un > 0, on a donc :


0 .2

rln;;:: 3, Un+l ;;:: Un


0.1

Ceci prouve que (un) est croissante à partir du rang 3.


0 .0
c) On pose:
- 0.1......,,__ __ _ _,__ __,_ _...__ ___,,,_,
,o

f: llll*+ llll
X f(x) = l':::x d) Tout d'abord, on remarque que: 'ri x E llll, 2- cos(x) ): 1 > O.
On voit donc que si Un existe à un rang n alors Un + 1 existe
f est dérivable sur llll: (comme quotient de fonctions déri- aussi, ce qui prouve que la suite (unlne:N est bien définie.
vables) et:
De plus, une récurrence immédiate donne: 'ri n E 1\1, un ;;:: 0
(car ln(x) ;;:: 0 pour x;;:: 1).
'r/ X > 0, f
,
(X) = ±x-X21nx =-1 --X2lnx- , D'autre part, pour tout n E 1\1 :

donc f' (x) est du signe de 1- ln x. On obtient le tableau de Un+l -un= ln (2-cos(un)) + 7un - Un= ln (2-cos(un)) +6un
variations :
Donc: 'ri n E 1\1, Un+l - Un ~ 6un ;;:: 0 (car 2- cos(un) ~ 1).

X 0 e +oo Le suite (unlne:N est donc croissante.


Signe de
/'(x)
+ @
'
-
Mil
"O
Variations de
f
-oo
~~~ 0
a) Vous aurez remarqué que pour tout entier naturel n on a
(-1J2n = 1 et (-1J2n+l = -1.
0
C: Ainsi
::J 2
0 0~ Un~ -
lfl Ainsi f est décroissante sur [e, +oo[. n
,-i
0
N
D'autre part, pour tout n E 1\1* : un =f (n) , donc: Or lim -
2
=
0 donc, d'après le théorème des gendarmes
n-+oo n
@ (version limites finies) lim Un existe et:
...., Un+l - Un= f(n + 1)- f(n)
n-+oo
.c
Ol
·;:: Pour n;;:: 3, on a (n, n + 1) E [e,+0012, donc n ~ n + 1 donne lim Un = 0
>-
a. n-+oo
f(n);;:: f(n + 1).
0
u Ainsi (un) est décroissante à partir du rang 3. b) Pour utiliser les croissances comparées, il suffit de remar-
2n enln2
Visualisons ce résultat avec Python. Le script: quer que 'rln E 1\1*, -n2 -- -n2- .

49
Chapitre 3 Suites rée lles

eax
On sait que pour tout a> 0 et be: IR : lim - b = O. f) D'après les croissances comparées : lim x 4 e - x = O.
x - +oo x x -+oo
zn Il suffit de remarquer que Vn e: l\l, u 11 = (v'n) 4 e- vri et que
ln2 > 0 donc: lim 2
n- +oo n
= +oo. 1im .,/ïï = +oo pour en déduire que lim un = 0 (par
n- +oo n- +oo
2"
Il suffit alors de minorer Je cosinus par - 1 : Un ;;:: :;-i - 1. composition de limites, en posant x = ../ïï).
n
D'après Je théorème des gendarmes (version limites infi- g) Ona:
nies), on en déduit que lim Un existe et que lim Un =
n-+oo n- +oo
+oo. 1 1
Vne:1\1, - Vn+l ~ Un+I ~ Vn+l
c) D'après les croissances comparées, le terme prépondérant
quand n __, +oo est en. On le met donc en facteur:
donc (en posant n' = n + 1) :
Vne:1\1, un = e 11
(1 - en (cosn+(- l)nl)
n
5

* 1 1
V ne: l\l , - '- ~ Un ~ '-
ns v3 n 3
vn
Or 1im -;-
n- +oo e'1
=0 (croissances comparées) et, pour tout
ne:1\1: 1
ns ns
- 2 - ~ - (cosn + (-l)n) ~ 2 -
ns Or
rn
lim -
n-+oo
= 0, donc, d'après Je théorème des
en en en gendarmes (version limites finies), lim un existe et
x-+oo
ns
donc lim n(cosn+ (- l)nl
n-+oo e
=O d'après le théorème des lim Un = O.
n-+oo
gendarmes et, par opérations sur les limites, on obtient fi-
nalement que: 1im Un = +oo(l - 0) = +oo.
11-+oo
d) Cette fois -ci le terme prépondérant semble être 311 • On le
met en facteur :
Mil
a) Étudions le sens de variation de la suite qui est à termes
3n(- 1+(~r) -1+(ir strictement positifs (donc en particulier non nuls). Pour
Vne:N,un= { { )n) = { ) 11
3n l + ~ 1 + 32 tout entier naturel n :

Or 1 ~ 1 < 1 donc 11~:VOO rn r = 0 (suite géométrique).


Un+I (2n+2)! 411 (n!)
2
11
- - - - - -2 x - - -
On en déduit à la fois que n.!!1roo (-1 + ( ~) ) = -1 et Un 4n+1 ( (n + 1)!) (2n)!
(n!) 2
lim (1 +
n- +oo
(~)n)
3
=1 (somme de limites). =
(2n + 2)!
---X
(2n)! ((n+l)!)2
X--
4n
4n+l
Finalement, lim u 11 = -1 (quotient de limites).
n- +oo 1 1
e) D'après les croissances comparées, le terme dominant se- (2n + 1)(211 + 2) X --- X -
(n+ 1) 2 4
rait n. On le met en facteur : 2(n + 1)(2n + 1)
'O
0
4(n + 1) 2
* n{1+~) 1+~
0
C:
::J Vne:1\1, Un = (
n 1-2Lnn
) = 1 - 2..!l.ll:
1
2n + 1
- - <l
n n 2n+2
lfl
,-i
0 * 1 sin(2n) 1 . 1
N Or Vn e: 1\1 , - - ~ - - - ~ - et hm -
n n n n -+oo n
O. = On en déduit que la suite (un) est strictement décroissante.
@ D'après le théorème des gendarmes (version limites finies), Or eHe est à termes strictement positifs donc eHe est mino-
...., sin(2n) rée par O.
.c lim - - - existe et vaut O.
Ol
·;:: n- +oo n Étant décroissante et minorée, eHes est convergente
>-
0
a. D'autre part, lim 1n n
n- +oo n
=0 (croissances comparées). d'après le théorème de 1a limite monotone.
u On en déduit que lim un = 1 (sommes et quotient de li-
n-+oo b) Étudions également le sens de variation de la suite (v 11 ),
mites). également à termes strictement positifs :

50
Suites réelles Chap,itre3

··~-----------~
Vn+l (n+2)u~+l
03
Vn (n+l)u~
0,2
n + 2(un+1)2
= n+l Un
0.1
n+2 (2n+l)2
-- x (en utilisant le a))
n + l (2n+2)2 0 .0

4n3 +l2n 2 +9n + 2


(en développant) -0.1~ - - - - ~ - - - - - - ~
4n3 + 12n2 + 12n + 4 ,o
3n+2
l ---------
4n3 + 12n2 + 12n + 4
>0 MJ# On va procéder par encadrement. Pour encadrer
< une somme, il faut encadrer le terme général puis addi-
tionner les inégalités.
La suite (vn) est donc elle aussi décroissante et minorée Soit n e: I\J* . Pour tout k e: [1, nil, on a: 0 < n3 + 1 s n3 + k s
par O : elle est donc convergente d'après le théorème de la n3 +n. Donc
limite monotone.
c) Les deux suites (un) et (vn) étant convergentes, on peut n2 n2 n2
Vke:[1,nil, --<--<--
appeler eet e' leurs limites respectives. n3 + n - n3 + k - n3 + 1
D'après la définition de la suite (vn), on a :
On additionne alors ces inégalités:
2 1
'tin e: I\J, Un = - - vn
n+l n n2 n n2

1
L - 3- - SSnS I: - 3-
k= l n + n k=l n + 1
Or lim - - = 0, lim Vn = e' et lim u~l = e2 .
n-+oo n+ 1 n -+oo n-+oo
On obtient donc (produit de limites finies) : c'est-à-dire (le terme général des sommes à gauche et à droite
ne dépendant pas de k) :
l!2 = 0xl!' = O

On en déduite= 0, i.e. que lim un = O.


n-+oo
Visualisons ces résultats avec Python. Le script:
ceci pour tout ne: 1\1*.
def fact(n) :
p=l n3 1 n3
for k in range(n) Or lim - - lim - - l et lim - -
n -+oo n3 + n n -+oo 1 + ...!.. n -+oo 113 +1
p*=k+l n2
return p . 1
lim - - = l.
n-+oo 1 + ...!..
113
'O
0
def u (n) :
C: return float(fact(2*n))/(4**n*fact(n)**2) On peut donc conclure grâce au théorème des gendarmes
::J
0 #en Python 2. 7 on utilise float() pour que / (version limites finies) : lim Sn existe et est égale à 1.
#soit la division des flottants n -+oo
lfl Vérifions à l'aide d'un script Python.
,-i
0
N x=range(31)
y=[u(n) for n in x] def S(n) :
@ sornme=O
...., import matplotlib.pyplot as plt
.c plt.plot(x, y, ' *' ,markersize=S) for k in range(n+l) :
Ol sonune+=float(n* • 2) / (n *• 3+k)
·;:: plt.ylim([ - 0 .1 ,1 .1 ))
>- plt.xlim((-0.5,30.5)) return somme
a.
0
u pH.show()
L:appel S(lOOO) donne 1. 0009994995. Il paratt donc
donne la figure: plausible que la suite converge vers l.

51
Chapitre 3 Suites rée lles

-
a) C'est une suite arithmético-géométrique de point fixe
que:€= 1e+1 donc€= f
On considère donc la suite (vn) définie par:
etel
Or, connaissant les conditions initiales, on obtient que À et
µ sont solutions du système :

À =
1
{ !À + 4 µ= 1
On obtient À = 1 et µ = I}.
Donc :
3
'ifnE I\J, Vn =Un - 2
'vnEI\J, Un = cos(';n) + ~ sin(n3n)
On a, pour tout n E I\J :
e) À partir de la deuxième relation (en posant n' = n + 1) on
Vn+ 1 = Un+ 1 - ~ = ~Un+ l - ~ = ~ ( Un - ~) = ~ Vn obtient
'if n EN, bn+2 = an+ 1 + bn+ 1

Donc (vn)n€N est géométrique de raison 1· On en déduit Il suffit alors de remplacer a 11+1 par son expression en
fonction de bn donnée par la première relation :
que:
l)n V - -1
'ifnEI\J, V
n - (3
-
O - 2x 3n 'vn E I\J*, bn+2 = 2bn + bn+ J
d'où: Cette relation permet d'obtenir l'expression du terme gé-
néral de (b11 ) comme cela a été vu dans les exemples précé-
'vnEI\J, Un =- ~(~r+;
dents, en faisant bien attention au fait qu'elle n'est valable
b) En posant n' = n - 1 on obtient: 'if n E I\J, Un+2 = 2u11 +1 + qu'à partir de n = 1 (les premiers termes à utiliser seront
3un. C'est donc une suite récurrente linéaire d'ordre 2. donc b 1 et b2).
{;équation caractéristique est r 2 - 2r - 3 = 0 de racines -1 [;équation caractéristique est r 2 - r - 2 = 0 de racines - 1 et
et 3. Donc 2.
Ainsi

Or, connaissant les conditions initiales, on obtient que À et


µ sont solutions du système : Or b1 = 0 et b2 = a1 + b1 = 1 donc À et µ sont solutions du
système:
À+ µ = l
{ - À+ 3µ =2
On obtient À = ! et µ = i. -À+ 2µ =0 J
On obtient À = 3 et µ = i.
{ À+ 4µ =1
Donc :
(- l)n + n+l Alors d'une part,
'if n EN, Un = - - - 3- -
4
* 2(- 1) 11 +2n (- l)n +211- l
c) C'est une suite récurrente linéaire d'ordre 2. {;équation ca- 'vne:N , bn = - - - - -
ractéristique est r 2 - 4r + 4 = 0 de racine double 2. On en 6 3
déduit que D'autre part, 'vn?:2, an =2bn-l (enposantn1 = n-1 dans
la première relation). Donc:
3(À, µ) E ~ 2• 'vn E I\J, Un = (À+ µn)2n
2(-l)n- l +2n- l
u Or, connaissant les conditions initiales, on obtient que À et 'if n?: 2, an= - - - - - -
0 3
C: µ sont solutions du système :
::J et on vérifie ensuite que cette formu le est valable pour
0 1 aussi pour n = 1.
À = On obtient À= 1 et µ = - 1.
lfl
,-i
0
{ 2À + 2µ = O f) On regarde les premiers termes : u1 = u6,
u2 = uf = ut,
N Donc : u3 -- u22 -- u80' ...
@ 'vn EN, Un= (l- n)2 11 On peut donc conjecturer que, pour tout n E I\J: Un= uf .
....,
.c Vérifions cette formu le par récurrence. Elle est vraie pour
Ol d) C'est une suite récurrente linéaire d'ordre 2. {;équation ca-
ï::::
>- ractéristique est r 2 - r + l = Ode racines complexes conju- n = 0 car u6° = u5 = uo. De plus si elle est vraie à un rang
0
u
a.
-f
guées 1 3 = e-ii et l+~v'3 = eilf . On en déduit que "'' alOrSUn+I = u 2 = ( u 2")2 = uo2x2" = uo211+1 .
nE,w, 11 0
3(À, µ) E (ij 2 ' 'vn E I\J, Un= ÀCOS (mr)
3 + µsin (mr)
3 On a donc bien: 'vn E I\J, Un= uf.

52
Suites réelles Chap,itre3

• Si -1 < a < 1 : lim an = 0 donc par opérations c) Lorsque on a une forme du type atn on ne peut surtout pas
n- +oo
an lim b
- sur les limites : lim - = O. direque llin a!"= ( lim an)n-+oo n_Ilfautsystéma-
n-+oo n n-+oo n-+oo
tiquement repasser à la définition d'une puissance réelle.
an
• Si a = 1 : pour tout n e: N, a 11 1 donc: 1im - Ici on a: Un= enln(I+i).
n- +oo n
. 1 On sait que si (x11 ) converge vers O alors ln(l + x 11 ) -
Jun - = O. n-+oo
n.-+oo n
Xn donc: ln - fi
(1 + -k) n-+oo et par produit d'équivalents:
• Si a = -1 : pour tout n e: "'' n ::: n.
'"': _ln <_ a" 1 0 n a d one
d'après le théorème des gendarmes (version limites finies): n1n(1 + l) - 11. = I.
n n-+oo n
an
lim - = O.
n-+oo n On a donc: lim nln (1 + .!..) = 1 et par continuité de la
an en.ln a n.-+oo n
• Si a> 1: pour tout n e: l'\J: - = - - donc, comme lna > 0, fonction exponentielle en 1 : lim Un = e 1 = e.
n n n-+oo
an . . , Remarquez que le raisonnement naïf consistant à dire que
on a: lim - = +oo (par crmssances comparees).
n-+oo n la limite serait 1+oo = 1 donne un résultat faux ...
a2n 1 (a2)n d) On sait que si (x,i) converge vers O alors sin Xn. - Xn,
• Si a < -1 : lim - lim - - - = +oo car a 2 > 1 n.-+oo
n-+oo2 n
n-+oo 2n cosxn. - 1 et ex" - 1 - Xn- Mais attention pas de
a2n.+l n-+oo n-+oo
(et on utilise Je cas précédent), et de même lim - - =
n-+oo 2n + l
sommes d'équivalents pour sin i -sin i !
n (a 2)n. 1 Sachant que sinp-sinq = 2sin(2yl)cos(2i1), on ob-
lim a - - - - = a - (+oo) = -oo car a< 0 (et on uti-
n.-+oo 2n+ 1 n 2 tient:
lise la limite précédente). Comme ces deux limites sont dif-
férentes et puisqu'e11es correspondent aux limites des deux
sous-suites de rangs pairs et impairs, on sait par théorème que 2sin (in)cos(~)
an Un. = ln 11
lim - n'existe pas. en-1
n-+oo n
• Bilan: Par produits et quotient d'équivalents, on en déduit que:

0 si - 1::: a::: 1
1
llin -a• = { +oo si a>l et donc lim Un = lim - · = 0
n.-+oo n Inn n-+oo n-+oolnn
n'existe pas si a<-1
e) On sait que si (Xn.) converge vers O alors JI + Xn. - 1 -
n-+oo
- t. On factorise donc par ../n pour utiliser cet équivalent:
a) On sait que si (xn) est une suite convergente vers O alors, ( )
1-COSXn. - X~i eteXn -1 - Xn.Onobtient: Un. = vn Jl + ~ - l
n-oo 2 n- +oo

2~.2 X 1 ll2
Donc par produit d'équivalents:
"O
0
C: Un +-;, n2
1 2n2 2 l l . lim -1- =O
::J Un - vn- = - - et 11m Un=
0 +oo 2n 2../n n.-+oo n.-+oo 2../n
lfl
,-i
c'est-à-dire:
0 f) Corrune au c) on passe à la forme exponentie11e : un =
N
1 e<2n+ 1) ln( 4 ~~ 1 ).
@ lim Un = -
...., n-+oo 2 4n
.c On a 4n 4n - 1 donc lim - - = l. On va donc
Ol b) Si (xn) converge vers O alors sinxn Xn, donc n-+oo n-+oo4n - l
ï::::
>-
a. sin -k -
n- +oo
-k et par produit d'équivalents: nsin -k -
écrire 4 !~ 1 = 1 + Xn où (xn) est une suite convergente vers
n.-+oo n.-+oo 0, pour pouvoir utiliser ln(l + x 11 ) - x 11 • Plus concrète-
0
u
*= 1.
On a donc : lim Un = 1.
ment:
4n 4n
- - =l+ - - -l=l+ - -
n-+oo
1
n- +oo 4n - l 4n - l 4n - l

53
Chapitre 3 Suites rée lles

1
donc : Mais comme sin
n-+oo
i - tz, on a lim nsin - = l. Par
n-+oo n
opérations sur les limites :
ln ( -4n- )
4n- l
=ln (1 + - 1- ) - -1- - - 1
4n -1 n-+oo 4n- l n-+oo 4n
ln(nsinf,:) 0
lim =-=0
D'autre part 2n + 1 - 2n donc, par produit d'équiva- n -+oo - ln n -oo
n-+oo
lents:
ln(sinfi:) _ J.
(2n+l)ln -4n)
- - -2n = -1 On a donc: lim
n-+oo -Inn
0, d'où:
( 4n-l n-+oo 4n 2
ln{sin tz)
4 llin = l , ce qui donne bien :
On en déduit que Jim (2n+ l)ln( -. n ) = !,et par n-+oo -Inn
n-+oo 4n-l 2
l
continuité de l'exponentielle: lim Un= e ï = ,/ë.
n- +oo
Un = ln (sin 2.) - ln 2. = - ln
n n-+oo n
n

g) De même qu'au c) et au f) on passe à la forme exponen-


On en déduit: lim Un = -oo (remarquez qu'on obtient
tielle: Un= enln(e- 1- ~) . n-+oo
ce résultat beaucoup plus rapidement en utilisant directe-
On a lim e - 1 -
n-+oo
2.n = e - 1 donc pour se ramener à ment les opération sur les limites).
ln(l + x 11 )
n-+oo
Xn avec ln(l + x 11 )
e - 1 en facteur. Concrètement on a :
n -+oo
x 11 , on va mettre
Mil
a) ri n EN*, H 11+ 1 - Hn = n~ 1 > 0 donc la suite (Hn) nEN* est
strictement croissante.
Un= (e- l)n (1- __
l_ )n = (e- l)nen1n(1- Ce-\)11)
(e - l)n b) Pour tout n E N* :

Comme ln ( 1 - (e-\)n) n-+oo - (e-\)n on a, par produit


, . al ents: n ln (1 - (e-1l)n )
d' eqmv _-:;_ -
11 1 ·
(e-l)n = - e-1
11 00

La continuité de l'exponentielle sur ~ (donc en - e~ l )


1 2n
donne alors : lim (1 - - -- ) n = e- .!.1. I:
n-+oo (e-l)n k =n+I k
1
Finalement : Un - (e - l)n x e - e- 1 et comme e - 1 > 1
on a : llin Un = +oo.
n-+oo n
De plus pour tout k E ll + l , 2nil, Î ~ /n.Si on additionne
n-+oo ces inégalités on obtient:
1
h) On a sin -n -
n-+oo
tz mais attention on ne peut pas, en géné- [ 2_ ~ [ _2._ = 2n- (n + l) + l = ~
rai, composer par la fonction ln. Pourtant ce résultat faux k =n+ I k k =n+I 2n 2n 2
en général s'avère souvent vérifié en pratique, on va voir
comment le démontrer sur cet exemple. (en remarquant que Je terme général de la somme de
droite ne dépend pas de l'indice k).
On va donc établir que: ln {sin i ) n-+oo
- ln tz = - ln n, et c) D'après le théorème de la limite monotone, la suite (H11)
u ln {sin tz) étant croissante :
0
C: pour cela vérifier que: Jim = 1. Jusqu'ici rien soit elle est majorée et alors elle converge vers une li-
::J n-+oo - 1nn
0 de bien compliqué, sauf qu'un quotient de ln ne se simpli- mite e;
lfl fie pas, l'astuce à retenir est qu'il faut en fait verifier que: soit elle n'est pas majorée et alors elle diverge vers +oo.
,--i
0
N Raisonnons par l'absurde: supposons que Je premier cas
@ ln (sin ! ) soit vrai. On aurait lim H2n = lim Hn = e(limite de la
...., lim n - 1= 0 n-+oo n-+oo
.c n-+oo -ln n suite extraite des rangs pairs) et alors, en passant à la limite
Ol
ï:::: dans l'inégalité du b), on aurait e- e ? ou encore O ~ ! !.
>-
a. C'est parti : Ceci est impossible.
0
u C'est donc Je deuxième cas qu'il faut retenir :
ln (sin tz) 1n (sin tz) + ln n ln ( n sin fi)
----- 1=
-Lnn -Inn
=- -Inn
--- lim Hn = +oo
n-+oo

54
Suites réelles Chap,itre3

-
a) Tout d'abord 'v' k ~ 2, 0 <

On en déduit que 'v'k~ 2,


2
1. ~ f
4- ~cos(;)< l.
-
a) Calculons les premiers termes de la suite:
- 7 u = 37
• 3 5 ·
Démontrons que la propriété Un existe et un. > 1 est vraie
pour tout entier supérieur à 3.
u1 = - 1, u2 =

Ainsi, la suite (un) est à termes strictement positifs. On


peut donc étudier le rapport de deux termes consécutifs : • Elle est vraie pour n = 3.
• Supposons qu'elle est vraie à un rang n ;::: 3. Vérifions
Un+l
'v'n~2. - - =cos ( -7T- ) < 1 alors qu'elle est aussi vraie au rang n + 1.
Un. 2n+ 1
1
Pui que Un > on a donc Un + 2-:/ 0et donc Un+1 existe.
La suite (u11 ) est donc strictement décroissante or elle est D'autre part:
minorée par O : on en déduit qu'elle converge d'après le
théorème de la limite monotone. Un+I > 1 = 5Un - 2
--->l
b) Par analogie au a), 'v' n ~ 2, 0 < sin ( ;, ) ~ Ainsi, la suite f. =
Un.+2
5un - 2>un + 2
(vn) est elle aussi à termes strictement positifs. On peut U11 +2>3>0
donc étudier le rapport de deux termes consécutifs : <==> 4un >4
= Un>l

Un+I sin(?) Or cette dernière inégalité est vérifiée (c'est notre hy-
'v'n ~ 2, Vn+I
pothèse de récurrence), donc, en remontant ces équiva-
Vn --;;;; sin ( 2~,)
lences : Un+ 1 > 1. La propriété est donc vraie au rang n + 1.
n ) sin L,~+1) D'après le principe de récurrence, la suite est donc bien dé-
= cos ( - -
2n+l sin(;,) finie et 'v'n 2:: 3, un> 1.
b) On en déduit de même que (vn.) est bien définie.
!sin(2#,:r)
Su 11 -2
sin(r,) u,;+2 - 2 3un-6 3(un - 2) 3
1 Vn+l = = --- = = -vr,
Su11-2 _ 1 4Un - 4 4(u 11 - 1) 4
U11+2
2
La suite (v 11 ) est donc géométrique de raison i et de pre-
La suite (Vn) est donc géométrique de raison ! et de pre- mier terme vo = 2.
mier terme V2 = COS rn)
sin ( = =!· i) }i }i c) Onendéduitque'v'ne:N, v11 =2x(i)".
2 1 Or, pour tout n e: N :
c) Onendéduitque'v'n~2. Vn=!x(!)"- =(!)"- .

Ainsi, Un - 2
Vn=-- <==> (un - l)vn =Un - 2
Vn Un -1
'v'n ~ 2, Un = . ( 11
Stn 211
) =2n-lsin(r,) <==> Un(Vn-l)=Vn-2
u Vn - 2
0 <==> Un= - -
C: 7T Vn - 1
::J d) lim -;:- = 0 , donc: sin (z1T ) 11
-
11
zlî •
0 n-+oo 2·1 n-+oo
lfl
Donc
Par produit d'équivalents, on obtient :
,--i
0 2x(J)"-2
N 'v'n e:N, Un=--'---'-,.,--
2 2x (i)"-l
@ Un -
...., n-+oo 2n- l :lJ,- 7T
.c
~4
11
Ol
·;:: d) lim (~) = 0 car 1 1 < 1. Par opérations sur les limites,
>- Conclusion : n-+oo 4
a. on en déduit que (un) converge et:
0 2
u lim Un.= -
n- +oo 7T
-2
llin Un= -
n-+oo - 1
=2

55
Chapitre 3 Suites rée lles

- • Ona,pourtoutll~2: • D'aprèscequiprécède, liman= lim 2nsin(!!-)=1t,et


11-+oo n-+oo zn
donc lim bn =JT.
n-+oo
2n+I sin (-1T-)-2n sin(~)

''''
211+ l 211

= 2 11 +1sin(~) 11
2n+l - 2 sin(2x ~
2n+l)
a) Il suffit de remarquer que

2n [ 2sin(z: 1 ) - 2cos(z: 1 )sin(z: 1 ) n+l-n l l


Jn+i." - Jn = = ~ ---
11 1
Jn+î + rn vn+T + rn rn + rn
= 2 + sin(~)
211+1 (1-cos(~))
211+1 pour en déduire que

De plus 'vn;;,: 2, 0 < 2,~+ 1 < ~ donc O < sinC,~+ 1) et O < 2( Jn+i - rn) ~ )n
cos( 2;+ 1 ) < 1. On obtient l'autre inégalité de manière analogue.
On en déduit que, pour tout n ~ 2 : an+ 1 - an > 0, et donc que b) La double inégalité précédente permet d'écrire :
la suite (an) nû est (strictement) croissante.
• On applique la même méthode pour (bn), Pour tout n ~ 2,
on a O < 2,::1 < ~ donc : 2( Vl+l -v'ï) ~ 1
JI ~ 2 ( v'ï - v'i-=-i)
2( V2+1 - v"2) ~ 1
/2 ~ 2( Jz - h=I)
bn+l - bn 2n+ 1 tan (- JT- ) - 2n tan (2 x ~ )
211+1 2n+l
2( Jn+T-rn) ~ 1
./n. ~ 2( Jn - v'n-=I)
,n(2 tan (-1T- ) -
2
2tan( 2,:r.1) )
-----'-----'--
2n+ l 1- tan 2 ( ,1+
1l )
En additionnant membre à membre ces doubles inégalités
2 1

zn+l tan(- JT- ) (1- -- 1--) on obtient :

211+1 l-tan2(2,~1)
2 ( vn+l - Jî) ~ Un ~ 2 ( V,Ï- vo)
2
(. 2,~+1)) On en déduit que lim u 11 = +oo d'après le théorème des
= 2 ll+l tan ( -1T- )(-tan 11-+oo
2n+1 1- tan2( 211+11l ) gendarmes (version limites infinies).
Pour l'équivalent, on «sent» que ça doit être 2../n. On va
donc diviser chaque membre de la double inégalité par
;+ î
Or, \:/ li ;;,: 2, 0 < 2 1 < donne O < tan ( 2 n11+1 ) < 1. 2 ../n et essayer de déterminer la limite de fJn.
On a donc: \:/ li ;;,: 2, bn+ J - bn < O. La suite (bn) nû est donc
On obtient successivement, pour tout entier naturel non
(strictement) décroissante.
nul:
• Il reste à étudier la convergence de la suite (bn - an), Pour
tout n ~ 2:
2(Jn+î - 1) ~ Un ~2V,Ï
1
'O b11 - a 11 = 211 tan(~) - 211 sin(~) = 211 sin(~) ( - 1) 2(vn+T-1)
0 2n 211 211 cos( 2~,) ~
Un
C: ~l
::J 2../n ~ 2./n.
0
lfl
,-i
0
N
Comme
~, et
lim JTn = 0 on a d'une part que : sin ( 2~,) -
n- +oo 2
donc lim 2n sin ( 1Tn) = JT, et d'autre
n- +oo
part que:
R- _1_ <
2../n- 2../n
..!!:!!:_ ~l
2 n-+oo 2
@
....,
.c
Ol
lim
1
n-+oo cos ( ;, )
- 1 = O. Or lim
n-+oo R l
- - - = 1.
2../n
·;::
>- On en déduit que: lim (bn - an) = O.
n-+oo
On en déduit que lim u~
11-+002vn
= 1 d'après le théorème des
a.
0 • Par définition, les deux suites sont donc adjacentes. Par théo- gendarmes (version limites finies) puis que
u
rème, on sait donc qu'elles sont convergentes et ont la même
limite. Un - 2V,Ï
+oo

56
Suites réelles Chap,itre3

- • Réécrivons le terme un de manière plus standard. X -2 2 +oo


La suite (un) est en fait définie par : Signe de 1

/'(x) + 1
1
+

Variations de - - - - +oo
uo= v'2
{ f --------2
Vn € N, Un+ 1 =,/2 + Un 0

Vérifions rapidement par récurrence que, pour tout n 2:: 1, un On voit que pour O s x s 2, on a v'2 s f (x) s 2, donc que l'in-
existe et un 2:: O. C'est vrai pour n = O. De plus, si cette pro- tervalle 10,2) est stable par/. Comme uo € [0,2], une récur-
priété est vraie à un rang n 2:: 1 : alors Un+ 1 existe aussi car rence immédiate permet de montrer que:
un + 2 2:: 2 2:: 0 et iJ est clair que Un+ 1 = J2 + Un 2:: O. 'efnEN, UnE[0,2]
D'après le principe de récurrence, notre suite est donc bien
En particulier, on vient de montrer que la suite (un) est majo-
définie et positive.
rée par 2.
• Traçons la courbe représentative de f : x ...... ./2 + x ainsi que • De plus, pour tout n E N :
les premiers termes de la suite (un) :
Un+l - un = J2+ Un - un = h(un)

en posant h(x) = J2 + x - x.
La fonction h est dérivable sur ] - 2, +ool et :

'ifx>-2, h' (x) = 1 -1 = 1- 2v'x+2


2.Jx+2 .Jx+2

Or si x 2:: -2 : 1 - 2 .Jx + 2 s O <==> l s 2 .Jx + 2 <==> 1 s 4(x +


2) <==> -i S X.
Pour x 2:: 0, on a h' (x) s O. Donc h est décroissante sur IR!+, ce
qui donne:
Vx € [O, 21, h(x) 2:: h(2) 0 =
Or on a vu que un € [O, 21,pour tout n € N. Donc Un+ 1 - Un =
h(un) 2:: 0, pour tout n € N. Ainsi la suite (un) est croissante.
• On a donc (un) croissante et majorée. D'après le théorème
de la limite monotone, on sait donc qu'elle est convergente.
0 On note€ cette limite. Comme u 11 € [0,2]. pour tout n EN, le
théorème de passage à la limite dans une inégalité donne que
€€ [0,2].
Notre étude est maintenant guidée : nous allons montrer que
la suite est croissante et majorée par 2 puis montrer qu'elle Pour calculer€ on part de: V n € N, Un+ 1 = J2 + un. Or on sait
converge vers 2. que lim Un+l = l (suite extraite de (un)), et par continuité
n-+oo
"O • Commençons par déterminer les points fixes de f:
de f sur [-2,+oo( (donc en€) on a:
n-+oo
lim J2+ Un = ./z""+ë.
0
C:
::J
I:unicité de la limite donne alors€= J2 + €,
c'est-à-dire€ est
0 f(x) =X <==> V2+ X = X un point fixe de f. Alors, d'après notre étude préliminaire:
lfl € = 2.
,-i
0
<==> x 2:: 0 et 2 + x =x 2
• Conclusion: (un) est convergente vers 2.
N <==> X 2:: 0 et x2 - X - 2 =0
@
...., <==> X 2:: 0 et ( X = 2 OU X = - 1) - • Commençons par étudier la monotonie de ces
.c deux suites. Pour tout n E N :
Ol <==> x =2
·;::
>- 3un+Vn Vn - Un
a. Un+1-Un= -un =
0 • Dressons ensuite le tableau de variations de f sur son en- 4 4
u et:
semble de définition [- 2, +oo[, sans oublier d'y placer notre
Un + 3vn Un - Vn
point fixe: Vn+ l - Vn = - Vn = - - -
4 4

57
Chapitre 3 Suites rée lles

• Ces deux calculs nous amène à étudier la suite (un - Vn) .


Pour tout n e: N :
n- l 1 + uf + · · · + u~_ 1 u~1 { 1) u?1
3un + Vn Un + 3vn 1 Un+l = - - x +- = 1- - Un+ -
Un+1 -Vn+ 1= - = (un-Vn) n n-1 n n n
4 4 2
On en déduit que la suite (un - Vn) est géométrique de raison On a donc:
! · Donc :
2
Un Un Un
1 )n (uo - 1 'v'n ;;,: 2, Un+ 1-Un =- - +- =- (un-1) ~ 0
'v'ne: N, un - vn = ( - vo) = - - - <0 n n n
2 zn -1
car Un~ n.
• On a donc, pour tout ne: N: Un + 1 - Un ~ 0 et Vn+ 1 - Vn s O.
On a donc (un) croissante et (vn) décroissante. On en déduit que la suite (un) est croissante. Et comme
De plus: lim un - Vn = 0 (suite géométrique de raison l/2). 'v'n ~ l, Un ~ n, le théorème des gendarmes (version li-
n-+oo mites infinies) donne que lim Un existe et est égale à
Par définition, on a donc (un) et (v,i) adjacentes. Par théo- n-+oo
+oo.
rème, on sait donc qu'elle converge vers la même limite. No-
tons ecette limite commune. b) • On montre d'abord par récurrence forte que la suite est
• Pour calculer e, on remarque que : 'v'n e: N, Un+ l - un = bien définie et à termes strictement positifs. On va donc
-(Vn+ l - Vn) i.e. Un+ l + Vn+ l = Un + Vn. On peut donc dire démontrer que, pour tout ne: N, un existe et un > O.
que (un+ Vn) est stationnaire, ce qui donne:
C'est vrai pour n =O.
'v' ne: N, Un+ Vn = Uo + Vo = 2
Supposons que pour un ne: N, , la propriété est vraie pour
Comme les suites convergent on peut passer à 1a limite quand tous les rangs inférieurs ou égaux à n. On alors : 'v' k e: Œo, n],
n - +oo dans cette dernière relation: 2i! 2 d'où e 1. = = uk > 0 et donc, en additionnant ces inégalités: I:Z=o uk >
O. Ceci prouve que Un+ 1 existe et que Un+ 1 > O.

'"'
a) • Les premiers termes de la suite sont 1, 2, 3, 5, 10, 28, 154,
3520. Il semble donc que l'on ait: 'v' ne: N*, un ~ n. On va
le vérifier par récurrence forte.
• Commençons par regarder si cette propriété est hérédi-
D'après le principe de récurrence forte, la suite (Un) n e:N est
donc bien définie et strictement postive.

• On remarque que, pour tout n e: N : u:1+ 1 = L


n

k=O
uk, donc:

taire au sens fort. Supposons que pour un n e: N* , la pro- u:i+l = u~ + Un.


priété est vraie pour tous les rangs inférieurs ou égaux à n:
Or on a vu que: 'v'n e: N, Un > 0 donc u:i+ l > u~ et
1 + 12 + 2 2 + ... + n 2
donc, par croissance de la fonction racine cubique sur !Ai+ :
Un+ l ~ Un+l ~Un.On peut donc dire que (un) est croissante.
n
• Alors, d'après le théorème de la limite monotone :
~ -l{ l + -
n(n+1)(2n+l))
-----
n 6
soit elle est majorée et alors eUe converge vers UJn réel
l{n(n + l)(2n + ll) (2n+l) e~o;
~ - = (n+l)---
n 6 6
soit elle diverge vers +oo.
u
0 0 r. pour n ~ 3 , on a -2n+l 1 d 1
6- ~ , one Un+ 1 ~ n + .
C: Supposons qu'elle converge vers R. ~ O.
::J La propriété est donc héréditaire au sens fort pour n ~ 3.
0
lfl De plus, eUe est vraie pour n = 3 (car u3 = 3). Sachant que 'v'n e: N, u~+ l =
u~ + Un, en passant à la li-
,--i
D'après le principe de récurrence forte, on a donc: 'v' n ~ 3, mite on obtient R.3 = e3 + e c'est-à-dire e = O. Ceci est im-
0
N Un~ n. possible car, comme la suite est croissante, on a: 'v' n e: N
@ ,Un~ UQ > 0, donc i! ~ UQ > 0.
...., • De plus cette propriété est aussi vérifiée pour n = 1 et
.c n =2. On a donc : On en déduit que lim un = +oo.
Ol n -+oo

-
ï::::
>- 'v'ne:N *, Un~n
a.
0
u
• On remarque ensuite que pour tout entier n supérieur à
2: a) Étudions sur IAi+ la fonction fn : x ...... xn - x pour n ç 2

58
Suites réel les Chap,itre3

X 0 ,/3
T
CCt +oo c) On a, pour tout entier n?: 2 u~ = Un+ n et Un = v 11 + 1
Signe de
- @ + '
' +
+ ++
On obtient donc (vn 1f1 = Vn 1 n . Ces nombres étant
f,,(x) strictement positifs, on peut composer par le logarithme
' '
-------+oo pour obtenir :
Variations de 0~
J,, 2
-2,/3
9 ------- nln(vn + 1) =ln(vn + 1 + n)
Sur In = [0, "-z/Î],
ln est à valeurs négatives. réquation ce qui s'écrit encore nln(v 11 + 1) =lnn +ln{ 1 + 1+,:",) et fi-
(Enl n'a donc pas de solutions sur In. nalement:
Sur In = ] "-z/Î; +oo[,
ln est continue, strictement
ln(l + v ) ln (1 + _v,_~+_l)
croissante. D'après le théorème de la bijection, elle est _ _ _n_ =1+ - - - - -
Inn ln n
donc bijective de Jn sur lnOn) = ]ln( "-z/Î).+oo[. Or: Il

ln ( n-zlî) = "-z/î (i-1) < 0 pour n '?. 2. Donc, pour Or lim un = 1 donc lim v11 = O.
n - +oo n-+oo
n '?. 2, lie: ln0 11 ) et (En) a donc une unique solution dans Donc, par opérations simples sur les limites :
Jn.

Conclusion: pour n '?. 2, (En) admet donc une unique so- 1n(i + v,~+l) 1
lim =0
lution un dans IAi+. n-+oo lnn
OrVn;;,: 2, 111 (1) = O<ln(un) et 1 e:Jn. Comme ln est stric-
On a donc:
tement croissante sur Jn, on en déduit que: Vn '?. 2, Un> 1. . ln(l + Vn)
b) Comme n?: 2 on obtient successivement :
1un
n-+oo Inn
=1
n
ce qui donne: ln(l + Vn) - lnnn .
n-+oo
n?:2 ==> llln(n)?: 2ln(n) Mais on sait que ln(l + x 11 ) - x 11 , pour toute suite (xn)
lnn>O n - +oo
==> enln(n) ?: e21n(n) convergente vers O. On a donc ln(l + v,i) - Vn, d'où:
n-+oo
exp/
==> nn?: n2 lnn
Vn -
2 n-+oo n
==> llç lin
1
car la fonction x- xïi est strictement croissante sur IAi+.
On a déjà un minorant de (un). Comme on n'a pas étu-
iif:i
a) Vérifions par récurrence que, pour tout n e: N : un et Vn
dié jusqu'alors la convergence de la suite, il devrait être
existent, Un > 0 et Vn > O.
utile de trouver à présent un majorant. La fonction ln étant
croissante sur [l;
2
+oo[,
il devrait être instructif de calculer • C'est vrai pour n = O.
l'image de ce n ïi tout juste évoqué. Pour tout n '?. 2: • Supposons que c'est vrai à un rang n e: N. On a alors
ln ( n fi) = n2 - n fi ?: n 2 - n = n(n - 1) ?: n(2 - 1) = n u 11 + v11 > 0, donc u 11 + v11 t O. Ceci prouve que Un+l et
u 2
Vn+ 1 existent. On obtient alors facilement que Un+ 1 > 0 et
0 car Vli;;:: 2, n ii ,;,; n. Vn+l > 0.
C:

0
::J La fonction ln étant croissante sur (1,
2 2
+oo[,et comme un> • D'après le principe de récurrence les suites (un) et (vn)
lfl 1 et li ïi > l, on en déduit que Un ,;,; n n. sont bien définies et strictement positives.
,--i
0 Ainsi b) Étudions le sens de variation de (un). Pour tout ne: N:
N
u2 - (zi+u v) u v
@ 2 u n+l - u n = » u,,+v,,
n " " = - -u,,+v,,
< 0 car les deux
11
-
11
-

...., Vn?:2, l<un,;,;nïi suites sont à termes strictement positifs.


.c
Ol z !.!ili!.l ln(n) On en déduit que la suite (un) est strictement décroissante
ï::::
>- Or lim nn lim e 2
= n-+oo ,, = J car n-+oo
lim - - = O(crois- or elle est minorée par O : elle est donc convergente d'après
a. n - +oo n
0 sances comparées) donc d'après le théorème des gen- le théorème de la limite monotone.
u darmes (versions limites finies) on en déduit que lim Un
n-+oo Une démonstration analogue conduit à la même conclu-
existe et est égale à 1. sion pour (vn).

59
Chapitre 3 Suites rée lles

c) Notons fu et fv les limites respectives des suites (un) et


(Vn). Une première idée est de passer à la limite l'une des
égalités définissant les suites. On a, pour tout ne: N :

Un +1 =
2
Un 2
donc Un+ 1(Un+ Vn) = U11
-
a) Posons ln (x) = x 11 + xn- l + · · · + x 2 + x. La fonction In est
dérivable sur ~ + (fonction polynômiale) et pour tout réel
positif x et tout entier naturel non nul n :
un+ Vn
l~(x) = nxn-l + (n- l)x11- 2 + ··· +2x+ l :2:: l > 0
Par passage à la limite: €u(€u + €v) c'est-à-dire: = et De plus 111 (0) = 0 et lim ln(X) = +oo.
fu€ 11 = 0, ou encore: €u = 0 ou €v = O. (Attention à ne pas x-+oo
, · e e.,+ev -- e2u car on ne sait· pas s,· eu + ev ..,../. o...l
ecnre 11
X 0 a +oo
On peut vérifier que la deuxième relation de récurrence
Signe de 1
donne la même formule.
f~(x) + 1
1
+
D'autre part, dans ce genre d'exercice, il faut souvent cher-
cher une combinaison des deux suites formant une suite
constante. Ici c'est (un - Vn) qui va nous intéresser:

Vn € N, Un+ 1 - Vn+ 1 =
2 2
Un-Vn
Un+ Vn
= Un - Vn
Variations de
fn
0 -----1 -----+oo

Comme ln est strictement croissante et continue sur IR1+,


La suite (un - vn) est donc constante, d'où: elle est donc bijective de ~+ sur ln(~+) = ~+ (d'après le
théorème de la bijection).
'efn E 1\1, Un - Vn =UO - vo < 0 Comme 1 e: fn(~+), l'équation (En) admet donc une
unique solution Xn dans IR1+.
Par passage à la limite cela donne eu - ev
= uo - vo < 0 i.e.
0~€u<€v. On calcule facilement lnO) = n;;:: l.
Ainsi i'v > 0 et c'est donc i'u qui vaut O. Alors i'v vo - uo. = (!) = ! x :_\
D'autre part,/11
( l l )"
= ( 1 - ), ) ~ 1
Pour conclure, on a: 2

lim un = 0 lim Vn =vo - uo


On en déduit que Xn ! ;1] car In est croissante sur~+.
E [

n -+oo n - +oo b) On remarque que pour tout réel positif différent de 1

d) I.:introduction de a= ~~ nous incite à étudier la suite ( ~;;) 1-xn


ln(x)=x--
qui est bien définie car (v,i) est à termes strictement posi- 1-x
tifs. (somme des termes d'une suite géométrique de raison x).
'efn € I\J, U>1+l : .!i : (!!lz.)2.
Vn+l vf, Vn I.:idée est maintenant de calculer f n (xn+ 1) et de le com-
Comme on l'a vu dans l'exercice 3.6, on a donc: parer à 1 ce qui nous permettra de comparer x 11 et Xn+l
=
puisque ln(Xn) 1 et que .fn est strictement croissante sur
'efne: N, Un = a 2 " IR1+.
Vn
Pour n ;;:: 2, 1 n'est pas solution de (E,i). On peut donc
"O considérer que 'ef n;;:: l, Xn+l e: [ 1 [. !;
0 Un = a 2" Vn
C:
::J
On obtient ainsi que ,
Dune part fn+l (Xn+1l = Xn+i -1--x- =1
1-x;;:; (i).
0 {
Vn = Un+ vo - uo = Un + UO ( i - 1) , _
11+1
1- x~+ J _ l-x.;;+ , , •
lfl
En combinant les égalités on obtient tout d'abord que Dautre part, fn(Xn+1) - Xn+i -1 -x- - - n+i1 d apres (1).
,-i - ri+t 1-x,1+ 1
0
N Vn =a 2
11
Vn + uo ( 1a - 1) c, est-a-dlfe:
, ·
f
Or O< Xn+ 1 < 1 donc x~: < x~+ 1 . Nous en déduisons que
@
...., 1-a
.c Vne:1\1, V - uo - - - - 1-x~+l
Ol n- a(l-a2") fn(Xn+1) = n+l < 1 = fn(Xn)
·;::
1 - xn+l
>
a. puis
0 La suite (x 11 ) est donc strictement décroissante or elle est
u
(1 - a)a 2"-l minorée par !: elle est donc convergente d'après le théo-
Vne:1\1, Un = UQ rème de la suite monotone.
1-a211

60
Suites réelles Chap,itre3

c) On note e la limite de (xn), On a: ~ 2, e: [!,1[ ' · d e lareat10n.,n(Xnl=l=xn


Aparnr l · f' l - x;; b·
1 _x,, ono oent:
\;/n Xn

donc ee: [ 4, 1]. De plus, (x 11 ) est strictement décroissante Xn(2- X~)= 1.


Par passage à la limite, en appelante la limite de (x 11 ), on
et xi = 1 donc : (! ~ x i < 1. Ainsi : ee: [ 4, 1 [. obtient :
On a : fun x 1111 = O. En effet xn = enln(x,,) et lim lnx 11 = 1
n-+oo
ln e< ln 1 = 0, donc lim nln Xn = -=.
n n-+oo ec2 - oi = 1<==> e= -2
n-+oo
1
Dans toute la suite on considérera un entier n ~ 2. Ainsi fun Xn =-.
n-+oo 2

'O
0
C:
::J
0
lfl
,-i
0
N
@
....,
.c
Ol
·;::
>-
a.
0
u

61
CHAPITRE '

Systè1nes linéaires et calcul 1natri-


ciel

'Tlièmes a6ordés dans {es e~ercices


- Résolution d'un système linéaire
- Opérations sur les matrices
- Calcul des puissances d'une matrice
- Inversibilité d'une matrice
- Systèmes linéaires et calcul matriciel
- Suites récurrentes et calcul matriciel
- Calcul du rang d'un système linéaire ou d'une matrice
- Propriétés de la transposition

Points essentiefs du cours pour {a réso{ution


des e~ercices
- Matrices triangulaires, diagonales
u - Opérations de bases sur les matrices
0
C:
::J
- Matrices nulles, identités
0
lfl
- Produit matriciel
,-i
0 - Formule du binôme
N
- Lien entre systèmes linéaires et matrices
@
...., - Rang d'une matrice
.c
Ol
·;:: - Matrices inversibles
>-
a. - Transposée d'une matrice
0
u - Matrices symétriques et antisymétriques

62
Systèmes linéaires et calcul matriciel Chap,itre 4

Les méthodes à retenir

:Calgorithme de Gauss permet de mettre le système sous forme ré-


duite de Gauss. Son rang est alors égal au nombres d'équations
restantes. Si les conditions de compatibilité sont vérifiées, on ob-
tient alors soit une unique solution, soit une infinité de solutions.
~ Exercices 4.l et 4.4

Lorsque le nombre n d'équations est égal au nombre p d'inconn-


nues, le système est de Cramer si et seulement si sont rang est égal
à n(=p).
~ Exercices 4.l et 4.4
Pour étudier un système linéaire Dans le cas où l'on obtient une infinité de solutions, le nombre
de paramètres nécessaires est égal au nombre total d'inconnues,
auquel on retranche le rang du système.
~ Exercices 4.l et 4.4

Pour les systèmes linéaires dont les coefficients et/ ou le second


membre dépendent d'un ou plusieurs paramètre(s), la discus-
sion repose sur deux conditions : 1) un coefficient ne peut être
choisi comme pivot que s'il est non nul; 2) l'opération élémen-
taire L; - aL; + ~Lj ne peut être effectuée que si a t O.
~ Exercice 4.4

- Si A est carrée d'ordre n, alors elle est inversible si et seulement si


rg(A) =n. Le rang se calcule à l'aide de l'algorithme de Gauss.
~ Exercice 4.3 e t 4.10
'O
0
C: Lorsque les coefficients de la matrice sont connus, on a le choix
::J
0 entre la méthode du système linéaire ou la méthode dite« du mi-
lfl
,-i
Pour étudier l'inversibilité d'une roir>>. Elles permettent toutes les deux de conclure sur l'inversibi-
0
N
matrice lité de la matrice, et elle donne aussi la matrice inverse.
@ ~ Exercice 4.3
....,
.c
Ol
·;:: Dans tous les cas et quelque soit la méthode utilisée, si on trouve
>-
a. une matrice B telle que AB = BA = 111 alors par définition A est
0
u inversible et B = A- 1.
~ Exercices 4.3, 4.8, 4.10 et 4.15

63
Chapitre 4 Systèmes linéaires et calcul matriciel

On calcule les premières puissances, puis on conjecture une for-


mule générale. Cette formule doit ensuite être démontrée par ré-
currence.
c:........ Exercices 4.5, 4.6, 4.7, 4.8,4.11 et 4.12

On décompose la matrice comme une somme de deux matrices


plus simples. Si ces deux matrices commutent, on peut alors utili-
ser la formule du binôme matricielle.
Pour calculer les puissances d'une c:........ Exercices 4.5, 4.8, 4.11 et 4.12
matrice
- Si on a une relation polynômiale entre les premières puissances
de A, on généralise cette relation par récurrence.
c:........ Exercice 4.10

Si on a calculé les puissances entières positives, et si la matrice est


inversible, on peut alors regarder si la formule obtenue est valable
pour des entiers relatifs, grâce à la formu le : AP = (A- 1)-p.
c:........ Exercice 4.6

Le calcul matriciel permet d'avoir une seule relation de récur-


rence simple, entre des vecteurs colonnes, au lieu de plusieurs re-
lations de récurrence couplées sur des suites réelles.
c:........ Exercices 4.7 et 4.12
Pour étudier des suites récurrentes à
l'aide du calcul matriciel Lorsque l'on a une relation de récurrence matricielle, il faut en
déduire une expression générale du terme de rang n, en fonction
den.
c:........ Exercices 4.7 et 4.12

Il faut regarder les choses coefficient par coefficient, en choisis-


sant bien les notations.
c:........ Exercices 4.9 et 4.13
Pour étudier des propriétés générales
Pour démontrer l'existence d'une écriture ou de solutions à une
sur les matrices
équation, on raisonne par analyse-synthèse (ou encore par condi-
tions nécessaire et suffisante).
c:........ Exercices 4.14 et 4.15
'O
0
C:
::J
0
lfl
,--i
0
N
@
....,
.c
Ol
ï::::
>-
a.
0
u

64
Systèmes linéaires et calcul matriciel Chap,itre 4

Énoncés des exercices

---- - Exemples de résolutions de systèmes linéaires


Déterminer le rang puis résoudre les systèmes linéaires d'inconnues réelles suivants:

3x - y+z = 5
2x+ y-z = 1 2x+ y+z = 1
a){ 2x+y - z = l
x-y+z=2
b) j 3x+3y-z = 2 cl j x-y-z=2
2x+4y = 2 4x-y-z =3
4x+ y+z =3
x+y + z - t = l 3x - y+z = 5 XJ + 2X2 - X3 + 3X4=0
d) j x - y - z+t = 2 e) j x + y - z =-2 nj x2 + x3 - 2x4 + 2x5 =0
x- y- z- t = 3 - x + 2y + z = 3 2x1 + x2 - 5x3 - 4x5 =0

---- - Exemples de produits matriciels


Calculer, lorsque c'est possible, les produits de matrices suivants:

a) ( 2
-5 :H: ~Il bt' :){· :J
2
4

,) (; ~){:11
-5
-3 d) ('·"
_,;n} r,osy -s;ny)
smx cosx smy cosy
1

---- - Etude de l'inversibilité d'une matrice

0o 52 ]. Exprimer de façon simple la matrice u3 en fonction de u2,


a) Soit U = (:
1 0
U et 13. En déduire que U est inversible et donner u- 1 .
u
0

0
C:
::J

lfl
b) Soit U = ( : o :l 2
Oéw,m;ne, (a, b) <0 tel que (U - al3)(U - bl3) = 03. En

déduire que U est inversible et donner u- 1.


,-i
0
N
@
c) Inverser, sic'est possible, les matrices suivantes :
....,
.c
Ol
·;::
>-
a.
0
u

65
Chapitre 4 Systèmes linéaires et calcul mat ric iel

---- - Exemples de résolutions de systèmes linéaires à paramètres


Déterminer le rang des systèmes linéaires d'inconnues réelles suivants, en fonction
du paramètre m e: Dt On donnera l'ensemble des solutions dans le cas où le système
n'est pas de Cramer.

mx + y +z= m x-my+m 2 z=2m


(m+ l)x+ my =2m
a) x+my+z=m mx-m 2 y+mz =2m
{ mx + (m+ l)y = 1
x+y + mz=m mx + y - m 2 z = 1 - m

---- - Puissances d'une matrice à l'aide de la formule du binôme

Soient)=[:
}= (: 0
Il A= (: :)
et
0 0
a) Déterminer J" et Nn pour tout ne: I\J.
b) En déduire An en fonction de n e: N.

---- - Puissances d'une matrice

Soit A=(: :)
0
a) Etablir l'existence de deux suites réelles (an) nEN et (bn) nEN telles que :

an
Vne:N,
A"= (: 1
0
··i
an
1

"O b) En déduire la matrice An en fonction den EN.


0
C: c) La formule est-elle valable pour tout ne: Z?
::J
0
lfl
,-i
0
N
@
....,
.c

0
u
Ol
·;::
>-
a.
---- - Calcul matriciel et suites récurrentes

Soit lamattke A=( 1


11
- 1 ~')
a) Déterminer A pour tout ne: N.

66
Systèmes linéaires et calcul matriciel Chap,itre 4

b) On considère les deux suites (un)ne:N et (vn)ne:N définies par la donnée de uo et


vo et les relations de récurrence

J Un+l Un -vn

1 Vn+I - un + Vn

(i) On pose W n =(:: ). Etablit une telation entte W "", A et W,..


(ii) En déduire une expression de un et Vn en fonction de uo, vo et n pour tout

---- -
ne:N.

Puissances d'une matrice à l'aide de la formule du binôme


Soit (a, b) e: l!ll2 . On désigne par A et J les matrices de .,«4 (llll) définies par :

A=(~
b
a
b
b
b
b
a
b
b
b
b
a
et r{
a) Exprimer A en fonction de I4 et J.
b) Donner une relation entre A2 , A et 14 . Pour quelles valeurs de (a, b) A est-elle
inversible? Dans ce cas donner la matrice A- l.

---- -
c) Déterminer A11 pour tout ne: N.

Trace d'une matrice


Soit A = ((a;j)h ~ i.j ~ n e: .H11 (1K). On appelle trace de A, notée Tr(A), la somme de ses
coefficients diagonaux :
n
Tr(A) =L a;;.
i= l

a) Montrer que : \i(A,B) e: ..in(IK)2. \ii\ e: IK, Tr(AB) = Tr(BA) et Tr(i\A + B) =


i\ Tr(A) + Tr(B).

---- -
b) En déduire que \iA e: .Hn(IK), \iP e: <8 ln(IK), Tr(PAP- 1) = Tr(A).

u Puissances d'une matrice à paramètre à l'aide d'une relation polynômiale


0
C:
::J
0
lfl
,-i
SoitA= [: m }ùm<O
0
N
@ a) Donner le rang de A en fonction de m.
...., b) Montrer qu'il existe deux réels a et b tels que: A2 = aA + bl3. Pour quelles valeurs
.c
Ol
·;:: de m la matrice A est-elle inversible?
>-
a. c) Etablir qu'il existe deux suites réelles (an) ne:N et (bn) ne:N telles que: \in e: 1\1, An =
0
u anA + bnl3.
d) En déduire la matrice An en fonction de n e: N et m e: llll.

67
Chapitre 4 Systèmes linéaires et calcul mat ric iel

---- - Puissances d'une matrice à paramètre à l'aide de la formule du binôme

Soit A = [ ; :
m2]
: où m e: ~*.
ni'[ m
a) Calculer (A + l3 )(A - 213).
b) Soient B = ! (A+ l3) et C =! (213 - A). Déterminer B 11
et C11 pour tout ne: ~I.
c) En déduire la matrice An pour tout ne: 1\1.

---- - Calcul matriciel et suite récurrente d'ordre 3 (d'après Agro-Véto 2006)

Soit (a, b) c 0 2 . On pose )(a, b) = [ ~


1
a
0
a) Déterminer J(a, b) 11 pour tout ne: 1\1.
b) On considère la suite (vnlnEN définie par

(i) Pour n ;;:: 2, on pose : Vn =[ v:: 1 ] · Montrer qu'il existe une matrice Ne:
Vn- 2
.,«3 (~) telle que :
'tin ~ 2, Vn+l =NVn .

l
En déduire la valeur de V n en fonction de V2 et N.

(i;J On pose P = [ : : ~3 Montm que P est inve"ible et cakulet p- t .

Vérifier ensuite que que N = PJ(l, -3)P- 1 et en déduire N 11 en fonction de P,


p- l, J(l, - 3) et n e: 1\1.
(iii) Donner alors l'expression de v 11 en fonction den e: 1\1.

u
0

0
C:
::J

lfl
,-i
0
N
---- - Matrices positives et monotones
Une matrice A= ((aij )) 1( i (» e: 9J1 11p (~) est dite positive lorsque:
l(j(p

'v(i, j) e: Œl, n] x Œl, p], ai} ~ 0


@
...., On le notera alors : A ~ O.
.c Une matrice A = ((aij )) l(i(>r e: 9J111 (Ili) est dite monotone lorsque:
Ol
·;:: l(j(n
>-
a.
0
u (i) A est inversible
(ii) A- 1 ~ 0

68
Systèmes linéaires et calcul matriciel Chap,itre 4

a) Montrer que A e: 9"Rn (llll) est monotone si et seulement si :

2 -1 0 0

- 1
b) Montrer que A= 0 0 e: 9"Rn(IR1) est monotone.

-1
0 0 -1 2

---- - Parties symétrique et antisymétrique d'une matrice


On note .9'n (IK) l'ensemble des matrices symétriques d'ordre n à coefficients dans 1K
et dn(IK) l'ensemble des matrices antisymétriques d'ordre n à coefficients dans 1K.
Démontrer que pour tout Me: 9"Rn (IK), il existe un unique couple de matrices (A, S) e:

---- -
dn (IK) x .9'n (IK) tel que M = A + S.

Ensemble particulier de matrices (d'après Esc-Eap 2003)

a) Déterminer les fonctions dérivables f : llll - llll telles que:

\f(x, y) e: llll 2 , f (x +y)= f(x) + f (y)

b) Soit$ l'ensemble:

(i) Vérifier que II est stable par produit et passage à l'inverse, c'est-à-dire :
• pour toutes matrices Met M' de <fl', MM' e: <fl' ;
• toute matrice M de$ est inversible et son inverse appartient à$.
(ii) Deux matrices quelconques de g commutent-elles?
(iii) On dira que la matrice M(a(t), b(t), c(t)) est dérivable en t-o si les trois fonc-
tions a, b, c sont dérivables en to.
u Déterminer les applications <p définies sur llll à valeurs dans <fl' par
0
C:
::J <p : t ...... M[ a(t), b(t), c(t)]
0
lfl
,-i dérivables sur llll et telles que
0
N
1 1
@ ('v' t e: llll) ('v' t' e: IR1) <p(t + t ) = <p(t) X <p(t )
....,
.c
Ol
·;::
>-
a.
0
u

69
Chapitre 4 Systèmes linéaires et calcul matric iel

Du mal à démarrer ?

Ill Utiliser l'algorithme du pivot de Gauss. b) Utiliser le a).

. . Ne pas oublier de vér ifier l'existence de ces produits. 11D a) Ut iliser la version matricielle de l'algorithme de Gauss.
b) Procéder par identification.
Ill a) et b) Revenir à la définition de l'inversibilité. c) Raisonner par récurrence puis revenir à la définition de l'inver-
c) Utiliser la méthode du système l inéaire. sibilité.

Ill Ut iliser l 'algorithme de Gauss. La recherche de pivots va


d) Utiliser les relations obtenues en c) entre a 11 +t , bn+l , an et bn,

conduire à une discussion sur la valeur du paramètre m. 1111 a) et b) Conjecturer des formules puis les vérifier par récur-

Ill a) Conjecturer des formules puis les vérifier par récurrence.


rence.
c) Relier A à B et C puis utiliser la formule du binôme.
b) Utiliser la formu le du binôme.

1111 a) Util iser une démonstration par récurrence.


IIFI a) Conjecturer une formu le puis la vérifier par récurrence.
b) Utiliser une analogie avec les suites réel les géométriques.
b) Utiliser les relations obtenues en a) entre an+ 1 , bn+l, a 11 et bn,
c) Pour ne: l et A matrice inversible, (A-1 } 11 = A- 11 ... 1111 a) Montrer que si A n'est pas inversible alors 3Xo e: .,ttn1 {llll)
Ill a) Conjecturer une formule puis la démontrer par récurrence. tel que AXo = 0 et
b) Utiliser le a)
Xo ,J O.
b) Utiliser une analogie avec les suites réelles géomét riques.

1111 a) et b) reveni r à la définition de l' inversibilité. 1111 Procéder par analyse-synthèse.

c) Utiliser la formule du binôme.


1111 a) Dériver la relation vérifiée par f.
Ill a) Int rodu ire des notations pour les coefficients des matrices b) Donner une formule générale pour le produit de deux éléments
qui entrent en jeu. de 8. Pour déterminer la fonction (j), utiliser le a).

"O
0
C:
::J
0
lfl
,-i
0
N
@
....,
.c
Ol
·;::
>-
a.
0
u

70
Systèmes linéaires et calcul matriciel Chap,itre 4

Corrigés des exercices

-
a) Appliquons l'algorithme du pivot de Gauss :

(S) = 10;::2:: ~1
Géométriquement, les trois plans dans l'espace s'inter-
sectentselon une droite passant par le point M {
et dirigée par le vecteur d (2

On vérifie avec Python.


-1 3).
j â o)

3 y -3 z = -3
a =np.array([[2,1,-1], [3 ,3, -1 ],[2,4,0]])
5 y-3z =- 5 b=np.array([[l],[2],[2]])
print np.rank(a)
0 - y +z = 2 print np.linalg.solve(a,b)
II]-z=-1 (!L3)
(S)=
0=1 (L2 - jL3)
/)instruction sol ve est à utiliser avec du recul. Ici elle ne
donne qu'une solution particulière.
2 z =0 (L4 - ~L3)
c) On choisit la deuxième ligne pour le premier pivot.
0 - y +z =2
0 - y - Z =2
(S) =
II] - z =-1
0= O (L4)
(S) = {
3 y +3 z = -3
3 y +3 z = -5
0 =1 (L3)
Il y a trois pivots : le système est de rang 3 mais la dernière
équation rend le système incompatible. I.:ensemble des so- (S)= J
0
-0:::~,
lutions est vide: .9 =<t>.
Géométriquement, cela signifie que l'intersection des
1 0 = -2
Il y a deux pivots : le système est de rang 2 mais la dernière
quatre plans dans l'espace est vide.
équation rend le système incompatible. r.; ensemble des so-
On peut vérifier le calcul de rang avec Python. lutions est vide : .9 =</>.
import numpy as np Géométriquement, les trois plans dans l'espace sont d'in-
a=np.array( [ [3,-1,1), [2,1,-1),(1,-1,1), tersection vide.
[4,1,1)))
np.linalg.matrix_rank(a) d) On garde la première ligne pour le premier pivot.

l
Attention : ne pas confondre avec l'instruction 0+ r + z -t =l
np . rank (a) qui ne donne pas le bon résultat. (S) <==> 2y +2z-~=-l
b) La dernière ligne peut fournir le premier pivot : 12y I+2z = -2

~
0 +2y =l <!L3) 3
2
(S) = - 3y - z =- l (L1 - L3) -a- 1
u
0
C:
::J
{
- 3y - z =- l (L2-~L3) (S)=3ac•/ l a
0
lfl
,-i
(Sl = {0 +2 y =1
Il y a trois pivots : le système est de rang 3. I.;ensemble des
0 II] +} z = } (-} L2)
N solutions est:
@ X 1(1 +2a)
....,
.c
Ol
(S)=3aE!m/:
{
!(1-a) .9 ={(; ,- a - 1, a, - ~ ) E ~ 4 / a E ~}
ï::::
>-
a.
0 Le système est de rang 2. r.:ensemble des solutions est: Nous ne donnons pas d'interprétation géométrique, car
u
nous sommes ici dans un espace vectoriel de dimension
.9 = { ( ~ ( 1 + 2a), ~ (1 - a), a) E im 3 / a E lm} 4.

71
Chapitre 4 Systèmes linéaires et calcul matriciel

e) La deuxième ligne peut fournir le premier pivot : Il y a trois pivots : le système est de rang 3. r.:ensemble des
solutions est:
0+ y- z=-2 (L2)
(S) <==> - 4 y +4 z =11 (L1 - 3L2)
1 3 y =1 (L2 + L3)

(S) <==> 10fu - t + z : C!L3)


Nous ne donnons pas d'interprétation géométrique, car
nous sommes ici dans un espace vectoriel de dimension
5.
r:;i _ 37
~ - 12 C!L2 + }L3)
+ta
(Sl <==> 10ŒJ :: (L1 - L2 + L3) a) En utilisant la disposition pratique on obtient :

[fil
~ - 12
r:;, _ 37

Le système est de rang 3 et est composé de 3 équations à 3


(~ ) 2 -1
4 3
inconnues : on a donc un système de Cramer et donc une
unique solution. r.:ensemble des solutions est le singleton: ( 2 1
-5 0 )( )2 8
- 5 - 10 5
ŒJ !AxB !
Vérifions nos calculs avec Python :
import numpy as np
Géométriquement, les trois plans de l'espace s'inter- a =array([[2,1], [ - 5,0)))
sectent en unique point M ( i j r;). b=array( [ [1, 2, - 1], [O, 4, 3]])
np.dot(a,b)
Vérification pythonesque :
Attention : a* b ne donne pas le bon résultat.
a =np . array( [ [ 3, - 1, 1], [ 1, 1, - 1], [ - 1, 2, 1] ]) b) Le produit est impossible car les tailles ne correspondent
b=np.array([[S],[-2),[3]]) pas.
print np.linalg.matrix_rank(a)
print np.linalg.solve(a,b) c) En utilisant la disposition pratique on obtient :

[fil

l
f) On garde les deux premières lignes pour les deux premiers -1
pivots.

~ + 2X2
[ 2
5

l
- X3 +3 X4 =0

l l
0 -5 1 -5
(S) = 1 x2 !+ x3 -2 x4 +2
- 3x2 - 3 x3 - 6 x4 - 4 x5 = 0
x5 =0
(L3 - 2L1)
[ 2 -3 0
0 1 4 [ -8
22

~ + 2X2 ŒJ !AxB !

l
- X3 +3 X4 =0
'O
0 d) En utilisant la disposition pratique et les formules clas-
C:
::J
(S) <==> ~+ X3 -2 X4 +2 X5 =0
siques de trigonométrie, on obtient :
0
=0 (3L2 +L3)
l
- 12 X4 +2 X5
lfl
,-i [fil
~ +2 X2

l
0
N
- X3 +3 X4 =0
[ cos (y) - sin (y)
@
....,
(Sl <==> ~ + X3 - 2 X4 . +2 X.5 =0 sin(y) cos(y)
.c
Ol ~-àxs=0(- 1~L3)

l
·;:: cos(x+y) -(sin(x +y))
COS (X) - sin (X)
0
u
>-
a. xi 3cx + Jjl3 ( sin (x) cos (x) l[ sin(x+ y) cos(x+ y)
x2 -ex- i~ 0 !AxB !
(S) <==> 3(cx,f3) e: IR! 2 /

72
Systèmes linéaires et calcul matriciel Chap,itre 4

-
a) Calculons les puissances successives de U :

[ill
Ja+ b
l
=l
ab =-2
Ce sont donc les racines du trinômeX2 -X-2 c'est-à-dire
2 et-1.

l
0 0 5 Ainsi (U - 213)(U + 13) = 03. On obtient après développe-
( 0
0 2
0
ment

1 1

l( l
0 0 5 0 5 0 U x - (U - 13) = I3 et - (U - I3} x U = 13
2 2
( 1 0 2
0 1 0
0 2 5
1 0 2 La matrice U est donc inversible et
[ill lu2 I
lu I
2

l
0 5 0

( 0 2 5
0 2
c) • Considérnm lesystème AX = Yme X = ( ; ) et Y = ( : )

l( l
0 0 5 5 0 10
( 0 2
0 1 0
2 5 4
0 2 5 AX = Y = Jx +2y = a
[ill ~ l 2x + 5y = b
=
On remarque que U 3 2U + 513 donc U x k(U 2 - 213) =13 AX=Y= J 0+2y =a
!
et (U 2 - 213) x U = 13. l 0 =b - 2a
On en déduit que U est inversible et u- 1 = !(U 2 - 213).
AX=Y= J0 =5a-2b
l 0 =b - 2a
Le système est de rang 2 et est composé de 2 équations à 2
0 inconnues: on a donc un système de Cramer, et donc A est
0 inversible. De plus :

Avec Python.

i mport numpy as np
u=np . array( [ [0,0,5),[1,0,2], [0,1,0JJ)
np.dot(u,np.dot(u,u))-2*u-5*np.eye(3)
# matrice nulle
np. linal g. inv(u) # matrice inverse • Considérnns Jesystème BX = Y avec X = ( ~] et Y = ( : ]
'O Attention, les instructions u • * 3 et u • * ( - 1 ) ne donnent
0
C: pas les résultats escomptés.
::J 0+y+z =a
0
b) Calculons (U - al3)(U - bl3) : BX =Y= x + 2z = b
lfl
,--i {
0 2x +y+ 3z = c
N
@ ab+2 -a-b+l 0+Y +z=a
...., -a-b+l l
.c (U - al3)(U-bl3) = -a-b+ 1 ab+2 -a-b+l BX=Y= 0 - z = a- b (L1 - L2)
Ol
·;:: ( {
>-
a.
- a - b+ l - a - b+ l ab+2 y - z = 2a-c (2L1 - L3}
0
u 0+y + z = a
On a donc (U -al3)(U -bl3} =0 si, et seulement si, les réels
a et b vérifient le système somme-produit: BX =Y=
{
0-
z = -b + a
O= c - b - a
73
Chapitre 4 Systèmes linéaires et calcul matric iel

Le système est de rang 2 et est composé de 3 équations à 3 3 inconnues : on a donc un système de Cramer, et donc D
inconnues: on n'a donc pas un système de Cramer, et donc est inversible. De plus :
B n'est pas inversible.

• Cornidémn, lesystème CX = Y "'" X = ( ~) et Y = (:)


0- 1 =( :
-4
-5
-3)
-3
-1 6 4

l Mil
ix + 2y - 3z = a
ex = Y <=> i y + 2z = b
a) La troisième ligne fournit le premier pivot :
ÎZ = C
0

l
+y + mz = m (L3)
0- 2iy + 3iz = -ia (-iL1) (S)<=> x + my + z =m (L2)
ex= Y= III - 2iz = - ib (- iL2) mx + y +z =m (L1)
1 0 = -ic (-iL3) 0+ y + mz =m
0 = -ia+2b- (3-4i)c
(S)<=>
(l-m)y+(m-l)z =0 (L1-L2)
CX = Y = III = - ib+2c {
2
(m- l)y + (m - l)z = m(m-1)

1 0=-ic
Le système est de rang 3 et est composé de 3 équations à • Si m = 1 alors
(mL1 - L3)

3 inconnues: on a donc un système de Cramer, et donc C

l
0 + y +z= l
est inversible. De plus :
(S) <=> 0=0

l~
-i 2 0=0

c-1= ~ -i = l - a - 13
(
0 (SI = 3(a.~I <Ol2 / a
=
Il y a un pivot : le système est de rang 1. I.:ensemble
• Considé,on, lesy,tème DX = Y avec X = ( ~) et Y = ( :) des solutions est:

=a

l
2x + 2y + 3z • Si m i 1 alors
DX=Y<=> x -y =b
0+ y + mz =m
- X + 2y + Z = C
(S) <=> - III + z =0 ( m:1 L2)

u DX=Y= JŒJ-fu +z : :+b 1 y + (m + l)z = m (m:1 L3)


0 0+ y + mz =m

0
C:
::J 1 4y + 3z = a - 2b (L1 - 2L2) (S) <=> - III+ z =0
lfl
,-i
0
N
DX=Y= JŒJ -fu., : :+b • • Si m = - 2 alors
1 (m +2)z=m

@
....,
.c
1 0 = 4c +6b-a (- L3 +4L2)
(SI= f ŒJ+ fuu::
0
Ol
ï::::
>-
a. DX=Y = III
=a - 4b - 3c
= a - 5b - 3c
(Li +L2 - L3)
(L2 - L3) 1 0 =- 2
0
u
1 0= -a+6b+4c
Le système est de rang 3 et est composé de 3 équations à
Il y a deux pivots : le système est de rang 2 mais
la dernière équation rend le système incompatible.
I.:ensemble des solutions est donc vide: sP = </>.

74
Systèmes linéaires et calcul matriciel Chap,itre 4

• • Si mi { - 2, 1} alors • •Sim ~{- 1,- },o} alors

l l
0+y +mz=m fxl+lll+ly =..!..
(S)<==> ~ m m
(S) <==> Œ)+ z = o Œ) = 1- m ( 2,~+l L2)

[I) = m':2 ( m~2 L3) Le système est de rang 2 et est composé de 2 équa-
Le système est de rang 3 et est composé de 3 équa- tions à 2 inconnues : on a donc un système de Cra-
tions à 3 inconnues : on a donc un système de Cra- mer et donc une unique solution. I.;ensemble des so-
mer et donc une unique solution. I.;ensemble des so- lutions est le singleton :
lutions est le singleton :
Y = {(m, 1 - m)}
9 ={(- m(2m + 1), _ ___!!!:_, ___!!!:_)}
m+2 m + 2 m+2 On peut en fait regrouper les deux premiers cas avec le der-
b) Pour obtenir des pivots il faut commencer par distinguer nier et en conclure que :
trois cas :
- si m =-! alors le système est de rang 1 et les solutions
• Si m= 0 alors sont les couples (a - 2, a) avec a e: IJll ;
-!
l
(S)=
fxl
~
-o si m f. alors le système est de rang 2 (donc de Cra-
mer) et admet la solution unique (m, 1 - m).
Œ) =1
Le système est de rang 2 et est composé de 2 équa- c) La première ligne fournit le premier pivot :
tions à 2 inconnues: on a donc un système de Cra-
(S)
mer et donc une unique solution. I.;ensemble des so-
lutions est le singleton :

Y = {(O, 1)}

• Si m = - 1 alors 0 - my + m2 z = 2m
(m 2 + 1) y - m 2 (1 + m)z = -2m2 - m +1
(S)= J Œ:) = 2
10 =-1
m(l - m2 )z
(L3 - mL1)
=2m(l - m)
Le système est de rang 2 et est composé de 2 équa-
tions à 2 inconnues: on a donc un système de Cra- (L2 - mL1)
mer et donc une unique solution. r.:ensemble des so-
lutions est le singleton : Il y a donc trois valeurs de m qui posent problème :

Y = {(- 1,2)} • Si m = 0 alors

•Sim i{ -1,0} alors


0 =O
(S) ç:, Œ) =1
0 + m;;1 Y i ("#i L2)
1
l
=
O=O
(S) = (2m+ l)y = -2m2 + m+ 1
~
u
0 ((m+ l)L2 -mL1)
C: (S) <==> 3a e: llll/ { ;
0
::J
• • Si m =- } alors z a
lfl
,-i
0 (S) = J 0- y= -2 Il y a deux pivots : le système est de rang 2. I.;en-
N
@
1 O= O semble des solutions est :
....,
.c
Ol
·;::
(S) = 3ae:llll/ lJ YX <X-2
a
y, = { (0, 1, a) e: l!l 3/ a e: l!ll}

>-
a. Il y a un pivot : le système est de rang 1. I.;ensemble • Si m = I alors
0
u des solutions est:
0 -y

l
+z =2
(S) ç:, 12y 1- 2z = -2

0 =0
75
Chapitre 4 Systèmes linéaires et calcul matriciel

Il semble que 'rln e: N*, Jn = 3n- 1J. Cette propriété se


(S) = 3a e:!R!/ J; a- 1 montre rapidement par récurrence car :

z 1 a
Il y a deux pivots : le système est de rang 2. I.:en-
1n+l = 3n- 1J x] = 3n- l x3J = 3n].
D'autre part :
semble des solutions est :

.:/ = {(l,a-1,a) E 11i3 j aE Ili}


• Si m = -1 alors
0+ Y + z =-2
(S) ç> 12y 1 =O
1 0 = -4
Il y a deux pivots : le système est de rang 2. La der-
nière équation rend le système incompatible. r.:en-
semble des solutions est : .:/= <t>.
•Sim it{- 1,0, 1} alors
0- my + m2 z =2m
donc, par une récurrence immédiate: 'ri n:?: 3, Nn = 03.
(m 2 + 1) y - m 2 (1 + m)z = -2m2 - m +1
b) On remarque que A = 13 +N. Sachant que l3 commute avec
(L3 - mL1)
toute matrice de 9J13(1li) on peut utiliser la formule du bi-
m(l - m 2 )z = 2m(l - m) nôme de Newton :
(L2 - mL1)

Le système est de rang 3 et est composé de 3 équa- 'ri n E I\J,


tions à à inconnues: on a donc un système de Cra-
mer, et donc une unique solution. I.;ensemble des so-
lutions est le singleton : On en déduit que pour tout entier n ;:: 2 :
2
m(3 - 2m ) 1- m 2 )}
.:/ = {( (m2 + 1)(1 + m)' m 2 + 1' 1 + m

-
a) Calculons les premières puissances de J :
Donc :

Vn <NI {D, I}, A" =(:


n ----y-
n(n- l)

n
l
0 1
"O
0
C:
::J Or AO = 13 : la formule est encore valable pour n = O. Elle
0
l'est aussi pour n = 1. Elle est donc vraie pour tout n e: I\J.
lfl
,-i
0
N
@
....,
MIM
.c a) Démontrons par récurrence que, pour tout ne: I\J, il existe
Ol
·;:: deux nombres réels a 11 et bn tels que :
>-
a.
0 4
u 27
1 11] = (27
27 27 27]
27
(1
1 1 27 27 27 0
76
Systèmes linéaires et calcul matriciel Chap,itre 4

• En posant ao = 0 et ho = 0, on a bien : Donc: 'rln 2:: l, bn = <n 2l)n . On vérifie ensuite facilement
que cette formule est encore valable pour n O. =
On retrouve donc le résultat établi dans l'exercice précé-
1
bol
ao dent:

0 1
n
• Soit n EN fixé tel que que An puisses'écrire sous la forme
demandée. VneN, A" =(: 1
0 7]
l
Calculons A x An

(
0
0 0
IAn !
1 an bn
1 an

(:
n
1
•<;-•>
n
1
l
c) Pour n e: Z\N, on pose: p

.
= -n E N et Bn

l
0

[L:) (
1 an+ 1 an +bn
0 1 an + l Ona:
0 0 1
Œ] !An+l l n+ p •<1-•1 + np, ~ ]
AP xBn 1 n+p
En posant an+ 1 = an+ 1 et bn+ 1 = bn + an, on a bien :
(: 0 1

0
=
0 (: 0
:] = !3

• D'après le principe de récurrence, il existe donc deux


suites réelles (an)ne:N et (bn)ne:N telles que: De même, on vérifie que Bn x AP = l3. On en déduit que
1
AP = A-n est inversible et que (APr = Bn, ou encore que:
B11 = A-p = An.
VneN, A"= (: 1
bnl
an La formule du b) est donc valable pour tout n E Z.

-
0 1

On en déduit de plus que ces deux suites sont définies par :

a) Calculons les premières puissances de A :


ao = 0 et 'ri n EN, a 11+ 1 = an+ 1
{
b1 =Oet'rlne:N, bn+l =bn+an
u 2
(1 -11
b) La suite (an) est donc arithmétique et 'ri ne: N, an = n.
0
C:
::J On en déduit que 'ri n e: N, bn+ 1 bn + n. Donc : = - 1 1
=(2- 2 -2]
2
0
lfl
,-i
0
N
3
(-11 -111 =(4-4 -4]4
@
....,
.c
Ol bn-bn-1 n-1
·;::
>-
a.
0 En sommant ces égalités, on obtient:
u 4

(1 -11 =(8 -8]


(n-l)n
'rln 2:: l, bn -ho= o+ 1 +2+ ... + (n-1) = - -2-
-1 1 -8 8

77
Chapitre 4 Systèmes linéaires et calcul matric iel

Cela laisse augmer que rln E 1\.1*, An= zn- l A, ce qui se vé- quis' écrit encore :
rifie très simplement par récurrence.
On en déduit que : - a - 2b b b

A-1 = 3- b_2___ a_12___2_a_b b - a - 2b b


2n - 1 -zn- 1) b b -a-2b
rlnEl\.l*,An=
( -zn- 1 zn - 1
b b b

Attention cette formule est fausse pour n = O.


• Si 3b2 - a 2 - 2ab =O.
b) (i) On vérifie que: rln E 1\.1, Wn+ l = AWn.
Montrons que A n'est pas inversible. Par l'absurde suppo-
(i i) Une récurrence immédiate montre que sons que A est inversible.
ri n E 1\.1, W n = AnWo, c'est-à-dire : Alors: A(A - 2(a + b)l4) = (3b2 - a 2 - 2ab)l4 = 0 4 donne,
en multipliant à gauche par A- l : A = 2(a + b)l4 . En identi-
fiant les coefficients des deux membres de cette égalité, on
u =211- l uo - zn-1 vo trouve a = b = 0, i.e. que A = 04. Comme la matrice nulle
r/ne:I\J*, n
{
Vn = -zn - l UQ + zn- l VO n'est pas inversible, on a donc une contradiction!
On a donc: A non inversible.
Attention ces formules sont fausses pour n = O.

-
Par contraposée, on vient de montrer que: A inversible ==>
3b2 - a 2 - 2ab ;é O.
• Conclusion: A est inversible <==> 3if - a 2 - 2ab ;é O.
a) On vérifie que A= bJ + (a- b)l4. c) Comme A = bJ + (a - b)l4 et comme 14 et J commutent, on
peut calculer An à l'aide de la formule du binôme :
b) Les matrices J et T4 commutant, on a :
A2 = b2J2 +2b(a- b)J + (a- b)2 l4.
Or

on peut vérifier par une récurrence immédiate que, pour


tout k?:. l :Jk = 4k- 1J.
4 4 4 Ainsi, pour tout n ?:. 1 :

J
4 4 4
=
4
4
4
4
4
4
;]
On en déduit que A2 = 2b(a + b)J + (a - b) 2 l4.
Or bJ = A - (a - b)l4 donc

u
0
C:
::J A2 2(a + b)(A - (a - b)l4)+(a - b) 2 l4
0
lfl 2(a + b)A + (3b2 - a2 - 2ab)l 4
,--i
0
N
AlorsA(A - 2(a + b)I4) = (3if - a 2 - 2ab)l4 .
@
...., • Si 3b2 - a 2 - 2ab ;é O.
.c
Ol
·;:: Alors : A x 3 b2-aLzab(A - 2(a + b)l4) = l4 et:
>-
0
u
a.
3
b2_;2- 2
ab (A - 2(a + b)l4) x A = 14, donc A est inversible
et:
grâce à une simple application de la formule du binôme
dans llll.

78
Systèmes linéaires et calcul matriciel Chap,itre 4

On vérifie facilement que cette formule est vraie pour n = • Si m -:J 1 choisissons la deuxième ligne pour le premier
O. pivot :
On a donc, pour tout ne: 1\1 :

1 1

F
An = -(a+3b) 11 ] + -(a - 3b)nK m (L2)
4 4
rg(A)
rg ~ 1- m m~ll (L3 - L2)

K= r~l
-1
3 -li J-m2 1-m (L1 - mL2)

F J
où ~l . m
-1 -1

-
rg ~ ITJ 1!m(L2)
l+m 1!m(L3)

F l
m
a) Notons A = ((a;1)), B = ((b;1)). AB = ((c;1)) et BA=
((d;J )).
=
rg ~ ITJ -1
0 m+2 (L3 - (1 + m)L2)
n
• Nous savons que c;J =[ a;kbkj et donc que
k=l • •Sim= -2 il n'y a donc que deux pivots :

n. = [n(n.L a;kbki )

F ~},
Tr(AB) = [eu -2
i= l i= l k=l
rg(A) = rg ~ ŒJ
On obtient de même que 0

L
Tr(BA) = [n ( n b;kaki ) • •Sim 1/ {-2, o} il y a trois pivots :
i=l k= l

d
Il suffit de remarquer que m
rg(A) = rg [OJ
Tr(BA) = Ï: (Ï: b;kaki) =
i=l k= l
Ï: (f b;kaki)
k = l i=l
~ [TI
0 m~2 (L3)
en permutant les signes l:. Ainsi :

Tr(BA) = Ln. ( [n.akibik ) = Ln ckk = Tr(AB). Donc: rg(A) = 3.


k= l i=I k=l b) Calculons A2 :
n n n
• Tr(ÀA + B) = [ (.\a;; + bii) =À [a;;+ [ b;; donc ŒJ

il
i= l i =l i= l m

"O
Tr(ÀA + B) = .\Tr(A) + Tr(B)
( 1 m
1 1
b) Tr(P(AP- 1)) = Tr((AP- 1)P) = Tr(A(P- 1P)) = Tr(A) par as-

l
0
C:
m 2 +2 2m+l 2m +l

- l
::J sociativité du produit matriciel. m
0
lfl
,-i
0
N
( m
111
1 m [ 2m +l m 2 +2 2m +l
2m+l 2m+J m 2 +2
@
a) •Sim= 1 les trois lignes sont identiques : ŒJ ~
....,
.c
Ol
ï:::: ITJ On remarque que :
A2 = (2m+ l)A+(2- m - m 2 )I3 = (2m+l)A+(m+2)(1 - m)I3.
>- rg(A) = rg ~
0
u
a.
[ De plus: A x (A-(2m + l)I3) = (m + 2)(1 - m)I3.

Donc : rg(A) = 1. •Sim ft{-2,1).

79
Chapitre 4 Systèmes linéaires et calcul matric iel

Ona : Son équation caractéristique est :


1
x2 - (2m + l)X - (m + 2)(1 - m) = o.
Ax (A - (2m+l)I3) Son discriminant vaut 6. = 9 (ce qui est inespéré !).
(m +2)(1- m)
1 6. > 0 donc cette équation a deux racines réelles qui sont
- - - - (A- (2m + 1)13) x A
(m +2)(1- m)
~ = m - 1 et 2 mti+ 3 = m+2.
I3 On en déduit qu'il existe (À,µ) e: [ij 2 tel que:

Vn e:N, an = À(m- l)n + µ(m+2)n


Donc A est inversible et:
Or O = ao = À+µ et 1 = a1 = À(m - 1) + µ(m + 2). On en
A- 1 = l (A-(2m+l)I3)
(m+2)(1 - m) déduit que À=- ! =-µ.Donc :

• Sim = - 2 ou m = 1. 1
Vn E: N, an = ((m + 2) 11 - (m - l)n)
Montrons que A n'est pas inversible. Par l'absurde suppo-
3
sons que A est inversible. On en déduit, d'après (ii) que :
1
En multipliant à gauche par A- l'égalité: A x (A - (2m +
l}I3) = 03, on obtient A= (2m + l}l3, ce qui est absurde.
Vn e: N, bn+ l = ~ ((1- m)(m + 2)n+l + (m+2)(m- l)n+l)
Donc An' est pas inversible. donc (en posant n' = n + 1) :
• Conclusion: A est inversible= m fl {-2, 1}. 1
Vn ~ l, bn = -((1-m)(m+2) 11 + (m+2)(m-1) 11 )
c) On va démontrer par récurrence que, pour tout n e: 1\1, il 3
existe deux nombres réels an et bn tels que : An = anA + et on vérifie facilement que cette formule est aussi valable
bnl3. pour n = O.
• Cette propriété est vraie au rang 0, en posant ao = 0 et Finalement, on a :
bo = 1.
• Soit n e: 1\1 fixé tel que la propriété est vraie au rang n.
en utilisant la relation obtenue dans la question précé-
dente, on a:

An+l AxAn avec J =[:


1
1
l [
1etK = -1
1
2

-1
-1
2
-1
= anA2 +bnA 1

= (an(2m+l)+bn)A+an(m+2)(1 - m)I3

Si on pose: an+l = an(2m + l) + bn et bn+l = an(m+2)(1 -


m), on a bien : An+l = an+1A+ h 11+1I3.

• D'après le principe de récurrence, il existe donc deux


'"'
a) Effectuons le produit :
1 A-2T3 I

l
-2 m m2
suites réelles (an)nE:N et (bnlnE:N telles que: Vn e: 1\1, A11 = .l. -2 m
"O
0
C:
an A+ bn T3.
Elles sont définies par les relations :
( m

m2
1 .l. - 2
m
::J

l (~ ~ ~ l
0 m2
m
lfl ao = OetVne:N, an+l = a 11 (2m+l)+bn ( i)
,--i .l. m
0
{ m
N bo = 1 et Vn e: 1\1, bn+l = an(m+2)(1- m) (i i ) 1 .l. 0 0 0
@ m'l m
....,
d) Combinons les relations (i) et (i i)
.c
Ol
IA+I3 I
·;::
>- Donc : (A+ l3)(A- 213) = 03.
a.
0 Vn e: 1\1, a 11+2 = (2m + l)a11 + 1 + (m + 2)(1 - m)an b) On remarque que B2 = B et C2 = C donc une récurrence
u
immédiate montre que sn = B et en = C pour tout entier
Ainsi (an) est une suite récurrente linéaire d'ordre 2. naturel non nul.

80
Systèmes linéaires et calcul matriciel Chap,itre 4

c) On a: 28- C =A. On vérifie alors, grâce à la question a), b) i) On vérifie que 'rln ::: 2, Vn+l NVn avec N
que CB = 03 = BC, on peut donc appliquer la formule du
binôme de Newton :
[:' : ~']
Une récurrence immédiate montre que

Pour n ::: 2, on a donc :


Vn ~ 2, Vn =Nn- 2v2
ii) Déterminons si P est inversible.

Con,ldémn, le système PX = Y " " X = [ ~) et Y =

Or si k E [l,n- lil : skcn- k = BC x sk- lcn- k- l = 03 x


sk-lcn-k-l = 03. On a donc:

[:)
0+3y+9z= a
PX= Y<=> x +2 y - 3 z = b
{
X + y+ Z=C
Pour finir on vérifie que cette formule reste vraie pour n= 0
et n = 1. 0+3y +9 z = a
Conclusion : PX = Y<=> Œ)+l2 z = a - b
{
Vne:N, 2 y +8 z = a-c

0+3y +9 z = a

-
a) Regardons les premières puissance de J(a, b) pour nous
donner une idée :
PX = Y<=>

PX = Y<=>
{

{
0+3y +9 z
[)+12z= a-b

Œ)+l2z
16 z = a - 2b +c

=a
= a- b (L1-L2)
~ _ a- 2b+c ( 1L )
~ -16° 16 3
Le système est de Cramer : P est donc inversible.
0 = -5a-rr27c
PX= Y<=> [] = a+2:-3c
{ ~ _ a-2b+c

~: ~:,)
u ~ - 16
0
C:
::J -5
0
lfl 3·a 2 Ainsi p-l = 6 ! :
,--i [
0 a3
N
@ 0 On vérifie que Px J(l, - 3) x p-l = N.
...., Une récurrence immédiate montre que
.c
Ol
ï:::: On peut alors vérifier par une récurrence immédiate que:
>-
a.
0
u iii) On a donc:

Vne:N*, 'rln~2, Vn=PxJ(l,-3)n- 2 xp- l xV2

0 :.J 81
Cha pitre 4 Systèmes linéaires et calcul matriciel

l m. - 3)n- 2 I On a donc montré que si A est monotone alors, toute


matrice colonne X telle que AX soit positive, est elle-

l
n-2 0

[
même positive.
0 1 0
0 0 (- 3 )n- 2 G Si 'r:/X e: VJ11i1(~), ( AX '";if:. 0 => X '";if:. 0 ) • Supposons
que A ne soit pas inversible.

l
n+l (- 3)n
On a par théorème : rg(A) < n.

)
3 9
( 1 [
2 -3
1
n (- 3 )n- l

n - 1 (- 3)1!-2
En terme de systèmes linéaires, cela signifie que le
système linéaire AX = 0 d'inconnue X e: .,«,il(~) est
œ de rang< n. Il n'est donc pas de Cramer, et vu qu'il

l
1 p X J(l, - 3)11-2 1
est compatible (X = 0 est solution), on sait par théo-
puis on calcule Px J(l, - 3)n- 2 x p - 1 rème qu'il admet une infinité de solutions. On peut
4n+(- 3) 11 - l 4n-(-3)n+ l - 12n+(- 3) +l5
11 donc considérer Xo E: .,«111 (~) solution non nulle de
16 8 16 AX = O.
411+(- 3)n- t _ 5 4n- (- 3)"- 1 - 3 - 1211+(- 3) 11 - 1 +27
16
[ 411+(- 3)n- 2 - 9
8 16 Comme AXo = 0 :::: 0, notre hypothèse impose que
4n- (- 3) 11 - 2 - 7 - 12n+(- 3) 11 - 2 +39
l6 8 16 Xo '";if:. 0.

l
et enfin on obtient :
Mais on a aussi: A(-Xo) = -(AXo) = 0 :::: O. Or Xo
étant positive, -Xo ne l'est pas (si ce Xo et -Xo sont
v,. : P, )(J, - 3)•-2, p-1 , [ : =
simultanément positives alors Xo 0, ce qui est ex-
clu).

c'est-à-dire :

'r:/n '";if:.2, Vn =
(
411+(-3) -1
411+<-}In-1_5
11

411+ (-3)' 1- 2 -9
l Si on pose X1 = -Xo, on a donc trouvé une matrice
X1 E: 9J111 1(~) telle que A(X1) soit positive sans que X1
soit positive ce qui est absurde.
La matrice A est donc inversible.
=
• Notons: A- 1 ((b;j))l$i,}$II et, pour tout .i E:
16
[l. n] : BJ = ((b;J )) 1$i$1l" Remarquons que Bj repré-
On en déduit que: sente la _;ième colonne de A- l.
4(n - 2) + (- 3) 11- 2- 1 =
D'une part, AA- l 111 donc AA -l '";if:. O. D'autre part,
'r:/n::::2, V11-2 = 16 pour j e: [l; n], le vecteur colonne AB j est donné
par:
Finalement (en posant n' =n - 2) :
4n + (-3) 11 - 1
Vn = 16

u
0

0
C:
::J
-
a) 1 = 1Si A est monotone Soit X E: 9J1n1(~) tel que AX '";if:. O.
Notons B = AX : B est positive. Comme A est mono-
tone, elle est en particulier inversible.
Lk=l A[n, k) x Bj [k, 1]
Lk=l A[l,k] X (A- 1)[k.j]
lfl
,--i
On a donc X = A- 1 B. Or B est positive et A- l aussi
0 car A est monotone.
N
@ On a alors, pour tout i E: [l, nil : Lk=l A[i, k) x (A- 1)[k,j]
....,
.c n
Ol
·;:: xil = L (A- 1)1i.k) xB[k,1) ::::0
>-
a. k=l
0
u (sommes et produits de termes positifs). donc AB j est le _;ième vecteur colonne de AA- 1. Il
Donc X est aussi positif. est donc positif.

82
Systèmes linéaires et calcul matriciel Chap,itre 4

D'après notre hypothèse on a donc Bj ;;:: 0, ceci pour Pour cela on s'inspire des formules obtenues dans l'analyse.
tout entier j E. Œl; n]. Concrètement, on commence donc par poser:
On en déduit que A- l ;;:: O. 1 1
S = - (M + tM) et A = - (M - tM)
Finalement, A est monotone. 2 2

xi On va alors vérifier que A et S satisfont les conditions deman-


dées :
x2
b) Soit X = tel que AX soit positive. * On a, d'après les propriétés de la transposée:
Xn ts = t [ ~ (M + tM) = l~ ( tM + t (tM)) = ~ (fM+ M) = S
Alors AX ;;:: 0 signifie :
Ce calcul prouve que S est symétrique, donc: SE. S"n(IK).
2 x1 - ;;::o
x2
* De même, on a :
- x1 +2 x2 - x3 ;;::o
- X2 +2 X3 - X4 ;;:: o
Ce calcul prouve que A est antisymétrique, donc: AE. dn(IK).
- Xn-2 +2 Xn-1 - Xn;;:: 0
* Pour finir :
- Xn-1 +2 Xn ;;:: 0

2 x1 - x2
A+S = !2 (M +;M) + !2 (M - ;M) = M
3 X2 -2 X3 On a donc démontré 1'existence de la décomposition.
4 X3 - 3 X4 ~ 0 (L1 + 2L2 + 3L3) • Conclusion. On a donc démontré qu'il existe un unique
couple (A,S) tel que:

n Xn- 1 - (n - 1) Xn ~ o (L1 + ··· M =A+S où AE.d11 (1K) etS E. .9'11 (1K)


+(n - l)L 11 _i)
! (M + tM) et A = ! (M - rM).

-
On sait de plus que : S =
(n+l) Xn ~0 (L1 +···+nLn)
On en déduit que x 11 ~ 0 puis, de proche en proche, que
\:/ k E. Œl, n], Xk ~ 0, c'est-à-dire que X est positive. a) Comme dans l'exercice précédent on utilise un raisonne-
Ainsi, on a montré que : ment par analyse-synthèse.
\:/X1:.9'R 11 1(IRI), (AX~O = X ~ O) • Analyse (= conditions nécessaires). On suppose l'exis-
La matrice A est donc monotone. tence d'une solution f.

il@• On va raisonner par analyse-synthèse (ou encore On fixe y E.IRI. On a: \:/x E. IRI, f(x + y) = f(x) + .f(y). Comme
la fonction f est dérivable sur !RI, on en déduit que :
par conditions nécessaires et suffisantes). On se fixe M E.
.,ttn(IK).
\:/ x E. IRI, !'(X+ y) = !' (x) + 0 = f' (x)
• Analyse (=conditions nécessaires). On suppose l'existence
"O
0 de la décomposition de M : En particulier, pour x = 0 : .f' (y) = .f' (0). Ceci étant valable
C:
::J
pour tout y E. IRI, on en déduit que f' est constante sur !RI.
0 M = A + S où A 1:. d n (IK) et S 1:. .9,1 (IK) 2
lfl
=
Il existe donc (a, b) E. (IR1*) tel que:\:/ x E. IRI, f(x) ax + b.
,--i
0 On a alors: tM = tA+ ts = - A+S. • Synthèse (= conditions suffisantes). On vient de voir que
N
En additionnant et en soustrayant ces deux égalités, on en dé- les éventuelles solutions sont de la forme x - ax+ b avec
@ (a, b) 1:.1R12. On va maintenant vérifier si ces fonctions sont
....,
.c
!
duit que: S = (M + tM) et A = (M - tM) . ! bien toutes des solutions (peut-être y a-t-il des conditions
Ol Ceci prouve l'unicité de la décomposition (si elle existe),
·;:: supplémentaires sur les réels a et b ?... ).
>- puisque S et A sont définis de manière unique en fonction de
a. On se fixe donc (a, b) E. IR1 2 et on note fla fonction définie
0 M.
u sur !RI par: f (x) =ax + b. Vérifons que f est bien solu tion:
• Synthèse (=conditions suffisantes). On va prouver l'exis-
tence de la décomposition de M. *f est une fonction polynôme donc est dérivable sur !RI.

83
Chapitre 4 Systèmes linéaires et calcul matriciel

* Pour tout (x, y) e: [ij2 : En posant a' = -a, b' = ac - b et c' = -c, on a : M- 1 =
M(a', b1 , c1).
f(x+ y) = a(x+ y) + b = ax+ ay+ b Ces calculs prouvent que M- 1 existe et est encore un élé-
ment de g.
et: Ainsi~ est stable par passage à l'inverse.
f(x) + f(y) = ax+ay+2b (ii) D'après les calculs de la question précédente :

Donc: f vérifie la condition demandée ç:::::::> 2b = b ç:::::::> M(~.~,ci)xM~2.bz,~l


b = O. =M(a1 +a2,a1c2+b1 +b2,ci +c2)
• Conclusion. Les fonctions solutions sont toutes les fonc- et :
tions/ : x--. axoù ae:~.
b) (i) • Formons le produit de deux matrices M1 = M(a2,b2,c2) x M(a1,b1,cil
M(a1,b1,c1) etM2 = M(a2,b2,c2) de$: = M(a2 + a1,a2c1 + b2 +b1,c2 + c1)
!M2 I
Ainsi, deux matrices quelconques Tvl(a1, b1, ci) et

[~ 77]
M(a2, bz, c2) de 6f commutent si, et seulement si, a1 c2 =

l
a2c1. Ceci n'est pas toujours vérifié... (prendre par
exemple: a1 = b1 = c1 = a2 = b2 = 1 et c2 = 0).
(i i i) Comme au a), on utilise un raisonnement par analyse-
a1 + a2 a1c2 + b1 +bz
synthèse.
1 C1 + C2
0 1 • Analyse ( = conditions nécessaires). On suppose l'exis-
tence d'une solution <p.
!M3 =M1 xM2 !
La condition : rl(t, t') e: ~ 2• <p(t + t') = <p(t) x <p(t1 ) s'écrit :
En posant a3 = a1 + a2, b3 = a1 c2 + b1 + b2 et c3 = c1 + c2,
on a: M1 x M2 = M(a3, b3, c3). Donc M1 x M2 est encore un a(t+ t') a(t) + a(t')
élément de E. 2
rl(t, t') E [ij , c(t + t 1) c(t) + c(t')
{
Ceci prouve que 8 est stable par produit. b(t + t 1
) b(t) + b(t') + a(t)c(t1 )
• Soit M = M(a, b, c) un élément fixé quelconque de g.

~l [:l
D'après la question a), les deux premières conditions en-
traînent qu'il existe deux réels ex et y tels que :
Confildérons le système MV =W avec V =[ et W = rite:~. a(t) = cxt et c(t) = yt

On a donc, pour tout (t, t 1 ) e: ~ 2• b(t + t 1 ) = b(t) + b(t1 ) +


0+ay +bz =X cxytt'. Si on dérive cette relation par rapport à t (pour t'
MV=Wç:, Œ]+cz =Y fixé quelconque), on obtient:
{
rl(t,t') e: ~ 2• b1 (t+ t 1 ) = b1 (t) +O+cxyt'
ITJ = z
u Le système est de Cramer donc M est inversible. De plus : Donc pour t = 0:
0
C:
::J x +ay +bz =X
0 rit' e: [ij, b1 (t1) = p+ cxy t'
lfl MV = Wç:, y =Y-cZ (L2-cL3)
,-i
{ où on a posé 13 = b' (0).
0
N z=Z
Il existe donc une constante ô e: ~ telle que :
@
...., X =X - aY + (ac - b)Z
.c rlte:[ij, b(t) =Ô+j)t+ ~y t 2
Ol MV = W ç:, y = Y - cZ
ï:::: {
>-
a. z =Z Mais la relation de départ sur la fonction b avec t = t' = 0
0
u donne b(O) = o. i.e. ô = O. On a donc:
-a
On a donc M -
1
= [: rite: [ij, b(t) = pt+ cxy t 2
2
0
84
Systèmes linéaires et calcul matriciel Chap,itre 4

Ainsi, si une fonction <p vérifie les conditions imposées * D'après les calculs du al, pour tout (t, t') E (ij2 :
alors nécessairement elle est définie par :
<p(t) x <p(t) =M (cx(t + t'J.tHt + t') + ~y (t + t 1
)
2 , y(t + t'))
<p : t-M[cxt,cx/: +Pt,yt]
avec (cx,p, y) E ~ 3
Donc: <p vérifie la condüion demandée <==> cxy = 0 <==>
ex= 0 ou y= o.
• Synthèse (= conditions suffisantes). Vérifions que nos
conditions nécessaires donnent bien toujours une solu- • Conclusion. Les solutions sont toutes les fonctions :
tion.
On fixe (ex, p, y) E ~ 3 quelconque. On note <p la fonction dé- <p : t - M [cxt,pt,O]
finie par :
ou:
<p: t - M[cxt,cx/: +Pt,ytl
<p : t - M[o,pt,cxt]
* La dérivabilité sur~ ne pose pas de problème puisque
les fonctions qui entrent en jeu sont polynômiales.

u
0
C:
::J
0
lfl
,-i
0
N
@
....,
.c
Ol
ï::::
>-
a.
0
u

85
CHAPITRE

Espace vectoriel lK n et applications


linéaires de lK P dans lK n

'Ilièmes abordés dans {es e~ercices


- Familles de vecteurs libres/liées
- Espace engendré par une famille de vecteurs
- Dimension
- Bases et coordonnées
- Applications linéaires
- Représentation matricielle d'une application linéaire
- Rang d'une application linéaire

Points essentiels du cours pour {a réso{ution


des e~ercices
- Familles de vecteurs libres/liées
- Sous-espaces vectoriels de 1K 11
u - Familles génératrices d'un sous-espace vectoriel
0
C:
::J - Bases d'un sous-espace vectoriel
0
lfl
- Dimension d'un sous-espace vectoriel
,-i
0 - Coordonnées d'un vecteur dans une base
N
- Opérations sur les applications linéaires
@
...., - Rang d'une famille de vecteurs
.c
Ol
·;:: - Noyau et image d'une application linéaire
>-
a. - Représentation matricielle d'une application linéaire dans une base
0
u - Rang d'une application linéaire, d'une matrice, d'un système linéaire

86
Espaces vectoriels et applications linéaires Chap,itre 5

Les méthodes à retenir

- On peut essayer de trouver une relation "évidente" entre les vec-


teurs de la famille. Dans ce cas, elle est liée.
~ Exercices 5.1, 5.7 et 5.13

- Si la famille de vecteurs est incluse dans un sous-espace vectoriel


IE de 1K n, et si son cardinal est strictement plus grand que la dimen-
sion de IE, alors la famille est liée. En particulier, les familles libres
de 1K n ont au plus n vecteurs.
~ Exercices5.l et5.7

- Si § = (ui, ... , u;;) est une famille de p vecteurs de 1K n on tra-


duit l'équation vectorielle: a1 ui + ... + ap u;; = 0 d'inconnues
a 1, .. . , ap, par un système linéaire homogène (S) de n équations à
p inconnues. Pour cela on utilise qu'une égalité vectorielle et équi-
valente à l'égalité des composantes (ou des coordonnées dans une
Pour étudier la liberté d'une famille de base donnée).
vecteurs ~ Exercices 5.1, 5.6, 5.7 et 5.20

- Si le système (S) est de Cramer, il a pour unique solution: a1 =


... = ap = O. On obtient que§ est libre.
~ Exercices 5.1, 5.6, 5.7 et 5.20

- Si le système (S) n'est pas de Cramer, il a alors une infinité de so-


lutions (car il est homogène). On obtient que la famille§ est liée.
~ Exercices 5.1, 5.7 et 5.20

- J.:une quelconque de ces solutions autre que (0, 0, ... , 0) dlonne


alors une relation linéaire entre les vecteurs de §.
~ Exercices5.l et5.7

- rg(§) = Card(§) si et seulement si§ est libre.


~ Exercices 5.8, 5.9 et 5.14

- Si la famille de vecteurs est incluse dans un sous-espace vectoriel


IE de 1K n, et si son cardinal est strictement plus petit que la dimen-
"O
0
sion de IE, alors la famille n'est pas génératrice de IE. En particulier,
C:
::J
les familles génératrices de 1K n ont au moins n vecteurs.
0 ~ Exercice 5.2
lfl
,-i
0
N
Pour étudier le caractère générateur - Si§= (ui, ... , u;;) est une famille de p vecteurs de IE (IE sev de 1K n)
@ d'une famille de vecteurs on traduit l'équation vectorielle: a 1 uj + ... + apu;; = v d'incon-
...., nues a 1, ... , ap, par un système linéaire (S) de n équations à p in-
.c
Ol
·;:: connues. Pour cela on utilise qu'une égalité vectorielle est équiva-
>-
a. lente à l'égalité des composantes (ou des coordonnées dans une
0 base donnée).
u
~ Exercices 5.2, 5.7 et 5.20

87
Chapitre 5 Espaces vectoriels et appl ications linéai res

- Si le système (S) est compatible pour tout v E IE, alors la famille


est génératrice de IE. S'il est de plus de Cramer pour tout v
E IE,
alors on obtient même que§ est une base de IE.
c.......,. Exercices 5.2, 5.7 et 5.20

- v
S'il existe (au moins) un vecteur E IE pour lequel (S) est incom-
patible, alors la famille§ n' est pas génératrice de IE. Le vecteur v
en question, fournit un contre-exemple de vecteur de IE qui n'est
pas combinaison linéaire des vecteurs de§.
c.......,. Exercice 5.2

- Si§ famille de vecteurs de IE: rg(§) =dim(IE) si et seulement si§


est génératrice de IE.
c.......,. Exercices 5.8, 5.9 et 5.14

On commence par "intuiter" si la partie considérée est un sous-


espace vectoriel ou non : une façon d'y arriver est de voir si les
conditions définissant la partie sont "linéaires" et "homogènes".
c.......,. Exercices 5.3, 5.5 et 5.11

Pour montrer que Ac IK 11 n'est pas un sev de !Kn, on commence


par vérifier si Ô E A.
c.......,. Exercice 5.3

- Si c'est le cas, on essaye de trouver un contre-exemple montrant


que A n'est pas stable pour la loi+: on cherche (u, v) E A2 tel que
u + vri.A.
c.......,. Exercice 5.3
Pour étudier si une partie de 1K n a une
structure de sous-espace vectoriel Indifféremment on peut aussi chercher un contre-exemple mon-
trant que A n'est pas stable pour la loi·: on cherche(.\, u) E 1K x A
tel que .\.Û rt. A.
c.......,. Exercice 5.3

Inversement, pour montrer que IE c 1K 11 est un sev, il faut vérifier


que Ô E IE et que IE est stable par combinaison linéaire: si À e: 1K et
(û, v) E IE2 alors W = ÀU +VE IE.
u c.......,. Exercices 5.3, 5.5 et 5.11
0
C:
::J
0 - Une autre méthode pour montrer que IE c 1K 11 est un sev, consiste
lfl
,-i
à déterminer une famille de vecteurs § telle que IE = Vect(§).
0
N c.......,. Exercices 5.3 et 5.5
@
....,
.c
Ol
·;::
>-
a.
0
u

88
Espaces vectoriels et applications linéaires Chap,itre 5

- Si on sait que dim(IE) = Card(§) : il suffit de montrer que § est


libre ou qu'elle est génératrice pour obtenir une base.
~ Exercices 5.6, 5.9, 5.10, 5.11, 5.12 et 5.13

On peut toujours compléter une famille libre en une base grâce à


la propriété suivante : si § est une famille libre de vecteurs elle IE et
u u
si E IE est tel que ft. Vect(§), alors la famille§ u (ïl) est une
famille libre de vecteurs de IE. De plus on peut imposer que soit u
choisi dans une base de IE.
~ Exercice 5.6

On peut ainsi construire une base de IE en partant de v


E IE tel
v
que ~ Ô. On arrête de compléter§ lorsque Card(§) =dim(IE).
Cette méthode n'est donc utilisable que si l'on connaît dim(IE).
~ Exercice 5.6

Inversement on peut toujours extraire une base de IE à partir d'un


famille génératrice§ de IE grâce à la propriété suivante : si § u (u)
u
est génératrice de IE et si est combinaison linéaire de certains
Pour déterminer une base d'un vecteurs de § alors la famille § est encore génératrice de IE.
sous-espace vectoriel de 1K 11 ~ Exercices 5.9, 5.10, 5.11, 5.12 et 5.13

En pratique si § = (ui, ... , u;;) est génératrice de IE, on étudie si


u;;
elle est libre en résolvant l'équation vectorielle : cx1 ui + ... +exp =
Ô. Si la solution est unique,§ est libre, (donc c'est une base), si-
non§ est liée et toute solution non nulle de l'équation donne
alors une relation linéaire entre les vecteurs de §. En ôtant de §
pas à pas les vecteurs qui s'expriment en fonction des autres, on
obtient une nouvelle famille génératrice cg plus "petite". Si tout va
bien elle est libre (le vérifier) et on a donc une base de IE. Sinon on
doit encore enlever des vecteurs dans la famille cg.
~ Exercices 5.9, 5.10, 5.11, 5.12 et 5.13

Pour être sûr de tomber du premier coup sur une famille cg libre,
il faut trouver le même nombre de relations entre les vecteurs,
qu'on a introduit de paramètres pour décrire les solutions de
l'équation vectorielle. Ce résultat n'étant pas au programme de
"O
0 BCPST, il ne faut pas oublier de vérifier rapidement que '!J est
C:
::J libre.
0
lfl
~ Exercices 5.9, 5.10, 5.11, 5.12 et 5.13
,-i
0
N
@
....,
.c
Ol
·;::
>-
a.
0
u

89
Chapitre 5 Espaces vectoriels et applications linéaires

On dit que [ est défini par un système d'équations cartésiennes


lorsque [ est défini comme l'ensemble des solutions d'un système
d'équations linéaire et homogène. Il n'y pas unicité des ces équa-
tions, cela ve dépendre des différentes étapes choisies dans les cal-
culs.
<.......+ Exercices 5.3, 5.5, 5.11 et 5.12

- Si [ est défini par un système d'équations cartésiennes, la résolu -


tion complète de ce système donne un paramétrage de [, duquel
on déduit une famille génératrice de[.
<.......+ Exercices 5.5, 5.11 et 5.12
Autour des équations cartésiennes d'un
sous-espace vectoriel de 1K 11 Réciproquement si [ est défini par un paramétrage (ou par une
famille génératrice), on cherche à quelles conditions sur X1, ... , Xn
on a (x1, ... , Xn) E [ : ceci revient à calculer une CNS sur X1, ... , Xn
pour qu'un système linéaire soit compatible, et cette CNS d!onne
un système d'équations cartésiennes de [ . Les irlconnues du sys-
tème linéaire en question sont les paramètres qui irlterviennent
dans la définition de [ .
<.......+ Exercices 5.11 et 5.12

Connaître les équations cartésiennes de [ permet de dire très ra-


pidement si un vecteur ü E 1K n appartient à [ ou non.
c........ Exercices 5.11 et 5.12

- Si f est définie par une expression analytique, on doit vérifier


qu'elle est bien linéaire: si ex E 1K et (ü, ïi) E (1KP)2 on doit avoir
/(exü + ïi) = cxf(ü) + f(v).
c........ Exercices 5.4 et 5.9

- Toutes les applications linéaires doivent vérifier f (Ô) = Ô.


c........ Exercice 5.4

X E Ker f se traduit par : f (x) = ô.


u <.......+ Exercices 5.9, 5.10 5.13, 5.17 et 5.21
0 Comment étudier une application
C:
::J linéaire de 1K P dans 1K n r.;expression analytique de f donne un système d'équations car-
0
lfl
tésiennes de Ker f. Grâce à sa résolution on obtient une famille
,-i
0
génératrice, puis une base de Ker f.
N
<.......+ Exercices 5.9, 5.10 et 5.13
@
....,
.c f est injective si et seulement si Ker f ={Ô} .
Ol
·;:: <.......+ Exercice 5.10
>-
a.
0
u y E Imf se traduit par y= f(x) avec XE iKP.
<.......+ Exercices 5.17 et 5.21

90
Espaces vectoriels et a pp lications linéai res Chap,itre 5

- Si &ê est une base de [KP (par exemple la base canonique), on sait
que la famille f (&ê) est génératrice de lm f. On en déduit alors une
base de lm/ grâce aux techniques habituelles. Ensuite on peut
calculer un système d'équations cartésiennes de lm/.
c.......+ Exercices 5.9, 5.10 et 5.13

- f est surjective de [KP sur IK. 11 si et seulement si lm/= IK. 11 •


c.......+ Exercices 5.5, 5.9 et 5.10

- Tout endomorphisme de 1K n, qui est injectif ou surjectif, est un


automorphisme de 1K n.
<.......+ Exercice 5.13

- Si f est définie par sa matrice A relativement aux bases cano-


niques &ê et <efl, on détermine son expression analytique grâce à
la formule :

<.......+ Exercices 5.10, 5.13, 5.16 et 5.18

Comment manipuler la représentation - Si f est définie par son expression analytique alors on détermine
matricielle d'une application linéaire sa matrice relativement aux bases &ê = (ei, ... , ë;;) et <efl en calcu-
de [KP dans 1K n lant les coordonnées des vecteurs /(ei), ... ,f(ë;;) dans la base <efl.
<.......+ Exercices 5.9, 5.14, 5.15 et 5.19

Il est équivalent de faire des calculs avec f ou avec l'une de ses


matrices relativement à des bases fixées : pour calculer son rang,
pour trouver un polynôme annulateur, pour déterminer la bijec-
tion réciproque de f etc ...
<.......+ Exercices 5.10 et 5.16

u
0
C:
::J
0
lfl
,-i
0
N
@
....,
.c
Ol
·;::
>-
a.
0
u

91
Chapitre 5 Espaces vectoriels et applications linéaires

Énoncés des exercices

---- - Étude de la liberté d'une famille de vecteurs


Pour chacune des familles de vecteurs suivantes, déterminer si eUe est libre ou Uée.
Lorsqu'elle est liée, donner une relation linéaire entre ses vecteurs.
a) §1 = {Cl,2,3),(2,3,4),(3,4,5),(4,5,6))

b) §2 = ((1,i,l - i),(- i,l, - l - i))

c) §3 = ((1,i,0),(0, - i,l),(- i,0,i))

d) §4 = (<2,o,-1),(o,1,1),(-2,3,4))

e) §5 = ( (1, l, 0, 1), (0, 1, 1, 0), (0, 0, 3, - 1), (1, 0, 2, 0))

f) §6 = (<2,1,0,0),(0,0,l,O),(O,l,I.2),(2,0,0,-2))

---- - Étude du caractère générateur d'une famille de vecteurs


Pour les familles de vecteurs suivantes, déterminer si elles sont génératrices de l'es-
pace IKn considéré. Lorsqu'elles ne le sont pas, donner un exemple de vecteur de IK 11
qui n'est pas combinaison linéaire des vecteurs de la famille.
a) §1 =(<o,o,1,o),(0,1,1,2),(2,o,o.-2))

b) §2 = (0,i),(1,0),(i,i))

c) §3 = ((1,i,0),(0,i,l),(i,O,i))

d) §4 = (Cl,l,0,l),(0,l,l,0),(0,0,3, - 1),(1,0,2, 0))

e) § 5 = ( (1, 0, 2), (0, 2, 0), (0, 0, 2), (1, 1, 1))

u
0

0
C:
::J

lfl
,-i
---- - Exemples de sous-espaces vectoriels de 1K n
Déterminer si les parties suivantes sont des sous-espaces vectoriels de l'espace IK 11
adéquat.
a) IE1 = {(x,y,z)€~3/y = x + z}
0
N
b) IE2={(x,y)€~ 2/y=x+l}
@ c) IE3 = {(x,y,z) e: ~ 3/ y = x+z 2 }
....,
.c d) IE4 = {(a+ 2j) + y, j)) e: ~ 2/ (a,j), y) e: [ij3}
Ol
·;:: e) 1Es = {(a+2,j3)€~ 2 /(a,P)€~ 2 }
>-
a.
0 f) IE6 = 1\1
u
g) IE7 =Vect((9,4,l),(3,i,7),(0,0,l),(2+i,-5,-i))

92
Espaces vectoriels et a pp lications linéai res Chap,itre 5

---- - Étude de la linéarité d'une application


Étudier la linéarité des applications suivantes:

a) r: application fi de (ij2 dans (ij 2 définie par : fi (x, y) = (x + 2 y, - x - y)


b) I.:application h de (ij4 dans (ij 4 définie par :

[2(x,y,z, t) = (3x - 2y + 9z + t, - x - 2y + z + t, - x - z - t +2, t)

c) I:application 13 de ~ 2 dans ~ 3 définie par :

f3(x,y) = (3x-2y,-x2,lyl)
d) I.:application f 4 de (ij4 dans (ij3 définie par :

f 4(x,y,z,t) =(x+3z - t, - x - 2y + t, - x+ y+2z+ t)

---- - Manipulations des trois différentes façons de définir un sous-espace vec-


toriel de 1K n
Un sous-espace vectoriel de 1K n peut-être défini de trois façons différentes: par un
système d'équations cartésiennes, par un paramétrage ou par une famille généra-
trice. Pom chacun des exemples suivants, vérifier que l'on a bien un sous-espace
vectoriel de l'espace 1K n adéquat, et le mettre sous les trois formes énoncées précé-
demment.
a) IE1 ={(x,y,z,t)e:(ij4 jx+y=z+tety=-z+t}

b) IE2={(a-~+3y,y,a-~,O)e:(ij 4 /(a,~.y)e:[ij3}
c) IE3 = Vect ((l,l,0),(0,2,1),(1, - 1, - 1))

---- - Complétion d'une famille de vecteurs en une base


Pour les familles de vecteurs suivantes, vérifier qu'elles sont libres et les compléter
en une base de l'espace 1K n adéquat.
a) S-:1 = ((1,1,0),(0,1,l))

b) s>:2 = (o,o,o,o),(1,1,0,o).(2,o,1.l))

u c) 9""3 = ( (1, 0, l, 0), (0, l, 0, 0))


0

---- -
C:
::J
0
lfl
,-i
0
Extraction d'une base d'une famille génératrice
N Pour les familles de vecteurs suivantes, vérifier qu'elles sont génératrices del' espace
@ 1K n adéquat, et en extraire une base.
....,
.c
Ol a) S-:1 = ( (1, 0), (O, l), (l, l))
·;::
>-
a. b) S-:2 = ((1,0,0),(2,5,l),(1,l,l),(1,-1,1))
0
u
c) S-:3 = ((1,1),(2,1),(6,1),(-1,l))

93
Chapitre 5 Espaces vectoriels et applications linéaires

---- - Exemple de changement de base pour une famille de vecteurs


Dans 11i3, on considère les vecteurs ui = (1, 0, 1), Ïlz = (0, l, 1) et U3 = (1, l, 1).
a) Montrer que@= (UÎ, Uz, U3) est une base de 11i3.
b) On pose vi = (1,2,4) et vi = (3, - 1,0). DéterminerMat(vî, vï).
@

---- - Étude d'une application linéaire définie par son expression analytique
On considère l'application f définie par :

f: lli3
(x,y,z,t) (-2y+3z+ t,-x-2y+3z+ t,-x+2z+ t)

a) Montrer que f est une application linéaire de 11i4 dans 11i3.


b) Déterminer une base et la dimension de Ker f et lm f, puis donner rg(f).
c) Donner la matrice de f relativement aux bases canoniques de lli4 et 11i3.
d) On pose ui = (1,0,0,0), ïiz = (1, 1,0,0), U3 = (1, l, 1,0), Ï4 = (1, l, l, 1), et aussi vi =
(1, 1,0), vi = (l,0,0) et vj = (0,2, 1) . Vérifier que@ = (UÎ,Uz,U3,Î4) est une base
de 11i 4 et que <~ = (VÎ, VÎ, °ii3) est une base de 11i3.
e) Donner la matrice de f relativement aux base@ et <(f .

---- - Étude d'une application linéaire définie par sa matrice dans une base
On considère l'endomorphisme f de 11i4 dont 1a matrice dans la base canonique est:

A= )4 ~l i, ~l
a) Montrer que f est un automorphisme de lli 4 et donner la matrice de 1- 1 dans la
base canonique. En déduire l'expression analytique de f et de 1- 1.
b) Déterminer une relation simple entre / 2, f et Id11t4 . Retrouver alors le résultat du
a).
c) On pose g = f - IdlR4. Déterminer la matrice B de g dans la base canonique, puis
donner une base et la dimension de Ker g et Img. Que vaut rg(g)?

u
0

0
C:
::J

lfl
,-i
0
N
---- - Intersection de deux sous-espaces vectoriels
On définit deux parties IE et IF de 11i4 de la manière suivante :

IE=Vect((-l,2,l,0),(-l,2,0,l)) et IF= {(x,y,z,t) Elli 4 /y=-2z+t}


@
...., a) Vérifier que IE et IF sont des sous-espaces vectoriels de lli4 .
.c
Ol
·;::
b) Déterminer la dimension de IE ainsi qu'un système d'équations cartésiennes de
>- IE.
a.
0 c) Déterminer une base de IF et sa dimension.
u
d) On pose G = IE n IF. Déterminer une base de G et sa dimension.

94

www.bibliomath.com
Espaces vectoriels et a pp lications linéai res Chap,itre 5

---- - Inclusion de sous-espaces vectoriels


On considère le sous-espace vectoriel IE de llll 4 défini par

IE = Vect ((1, - 1,3, - 3), (2, - 2,4, - 4), (3, - 3, 7, - 7), (1, - 1, l, - 1)).

a) Donner une base et la dimension de IE.


b) Déterminer un système d'équations cartésiennes de IE.
c) Etablir que IE c IF où IF est défini par

IF= Vect ((1, 0.1. -1), (0.1. 2, -2), (1, o, o, 0), (0, 0, -1, l)).

---- - Calculs de noyaux et d'images


Pour chacun des exemples suivants, on définit une application linéaire
f E ~(IJllP, lllln) par la donnée de sa matrice relativement aux bases canoniques.
Déterminer une base, la dimension et un système d'équations cartésiennes du
noyau et de l'image de f.

Donner aussi rg(f}.

0 1
A= ( 2 -1 1 2 l
-1 -1 1 2

-2 1 -1 2 1 0 0
-1 1 1 5 4 2 -2
C= ( 2 D=
1 -1 1 1 0 0 3 0
-3 3 -1 3 0 0 0

u
0
C:
::J
---- - Exemple de changement de base pour des endomorphismes

a) On note f l'endomorphisme de llll3 défini par sa matrice dans la base canonique:

0
lfl
,-i
Ao (~ ~3 ~2 ]- On pose UÎ =(1, - 1, 1), uâ =(1,0,0) et "3 =(0, - 1,2). Montœ,
0
N
@ que~= (ui, Ït2, uj) est une base de lli3 et donner la matrice de f dans cette base.
...., Que remarquez-vous?
.c
Ol
·;:: b) On note g l'endomorphisme de 1Jll3 défini par sa matrice dans la base canonique:
>-
a. 3 2
0
u B= r ~2 - ~ ]. On pose iii = (1,0,1), V2 = (O,l,-1) et V3 = (1,-1,1).
3 -2 -2

95

www.bibliomath.com
Chapitre 5 Espaces vectoriels et applications linéaires

Montrer que~= (vi, Y2, ii;) est une base de (ij3 et donner la matrice de g dans cette
base. Que remarquez-vous?

---- - Réduction d'un endomorphisme nilpotent


Soit/ un endomorphisme de IK3 teUe que: / 3 =0.<e(w) et/2 f. O.<t(R3 ).

a) Montrer qu'il existe un vecteur û € (ij3 tel que la famille ~ =(û, f (û), f 2 (û))
soit une base de 1K3 .
b) Donner la matrice de f dans cette base.

---- - Calcul des puissances d'un endomorphisme à l'aide d'un polynôme annu-
lateur
Soit f l'endomorphisme de
3 -2 - 1 0
(ij4 dont la matrice dans la base canonique est donnée

0 1 0 0
par :A =
2 -2 0 0
0 0 2
a) Vérifier que: / 2 -3/ +2Idllf' = o.<e(ot1)·
b) En déduire que f est un automorphisme et donner une expression analytique de
ri.
c) Montrer qu'il existe deux suites réelles (an) n€N et (b 11 ) n€N telles que:
\:/ n EN, [ 11 =anf + bnl~1.
d) En déduire l'expression analytique de f n en fonction de n E 1\1.

---- - Quelques propriétés classiques de Ker(g of) et Im(g of)


Soient f et g deux endomorphismes de 1K n.
a) Montrer que Ker(f) c Ker(g o .f) et lm(g of) c lm(g).
b) Montrer que Ker(g) n Jm(f) = f (Ker(g of)).
c) On suppose que f et g commutent i.e. f o g = go f. Montrer que Ker(g) et Im(g)
sont stables par f .

u
0

0
C:
::J

lfl
,-i
---- - Étude des sous-espaces stables par un endomorphisme (d'après Agro-
Véto 2008)

On considère la matrice A E .,tt3 ((ij) définie par: A = ( ~ - l ~ J.


1
0
N
-1 0 0
@
...., On note f l'endomorphisme de ~ 3 qu'elle représente dans la base canonique.
.c
Ol
·;::
a) Déterminer une base et la dimension de lm(f) et de Ker([).
>-
a. b) Déterminer rg(f - aldw) en fonction de a E (ij_
0
u = = =
c) On pose ui. (0,1, - 1), ui (1,1, - 1). D1 Vect(ui.) et D2 Vect(ui). Montrer=
que D1 et D2 sont les seules droites vectorielles stables par f.

96

www.bibliomath.com
Es paces vectoriels et a pp lications linéai res Chap,itre 5

---- - Formules de changement de base


On considère l'espace vectoriel 1K n muni de deux bases $ 1 et $
vectoriel iKP muni de deux bases "€1 et "€2.
On pose P = Mat($i) et Q = Mat(<tf1)
8l12 'ft2
2, ainsi que l'espace

a) Déterminer l'application linéaire de 1K 11 , muni de $1, dans 1K 11 , muni de $2, re-


présentée par la matrice P. De même, déterminer l'application linéaire de 1K P,
muni de <tfi, dans IKP, muni de "€2, représentée par la matrice Q. En dédui re que
P et Q sont inversibles et donner une interprétation de leur inverse.
b) Soit x un vecteur de 1K n. Montrer que :
Mat(x) = p X Mat (x)
{i}J2 {i}JI

c) Soit .f une application linéaire de 1K n dans 1K P. Vérifier que :

d) Soit.fun endomorphisme de 1K n. Vérifier que:

Mat(!) = p - l x Mat(!) x P
8l11 ff82

---- - Somme de deux sous-espaces vectoriels


Soient IE et IF deux sous-espaces vectoriels de IK 11 • On définit un sous-ensemble de 1K 11
noté IE + IF par :
IE + IF = {e + f/
e E IE et E IF} f
a) Vérifier que IE + IF est un sous-espace vectoriel de 1K II contenant IE et IF.
b) Établir que : dim (IE + IF) !', dim IE + dim IF.
Lorsque tous les éléments x
de IE + IF s'écrivent de manière unique =ë + x f
f
avec ë E IE et E IF, on dira que IE et IF sont en somme directe et IE + IF sera notée
IEEBIF.
c) Établir que IE et IF sont en somme directe si et seulement si IE n IF= {Ô }.
d) Établir que IE et IF sont en somme directe si et seulement si la juxtaposition d'une
base de IE et d'une base de IF donne une base de IE +IF. En déduire que: dim (IEEBIF) =
dimlE+dimlF.
e) Établir que IE et IF sont en somme directe si et seulement: dim (IE + IF) =dim!E +

---- -
dimlF.
u
0
C:
::J Noyaux itérés d'un endomorphisme
0
lfl Dans tout l'exercice f est un endomorphisme de 1K n (n E N*).
,-i
0 a) Montrer que, pour tout p EN : Ker(/P) c Ker(/ p+l) et Im(!P+l) c Im(f P).
N
@ b) Établir qu'il existe un entier Po EN tel que Ker(/Po) = Ker(/Po+l ).
....,
.c c) Montrer que pour tout q ~ Po, on a Ker(.fq) =Ker(f Po) .
Ol
·;::
>- d) Vérifier que ker(f PO) n Tm(/ PO)= {Ô }.
a.
0
u

97

www.bibliomath.com
Chapitre 5 Espaces vectoriels et applications linéaires

Du mal à démarrer ?

. . Uti liser une résolu tion de système linéaire. 811 a) Faire un calcul de rang puis calculer les vecteurs f (ïi i) en
fonction des vecteurs Ü i.
IIJI Uti liser une résolut ion de système linéaire. b) Faire un calcul de rang puis calculer les vecteurs f(ïi jl en fonc-
tion des vecteurs v j-

Ill Essayer d 'intuiter le résultat pour savoir ce qui doit être


1111 a) Choisir Ü de tel sorte que J 2 (ü) ,t. O.
démontré.
b) Poser a= Ü , b = f(ü}, ë = J2 (ü} puis calcu ler leur image
Ill Essayer d 'intuiter le résultat pour savoir ce qui doit être par f en fonction d'eux-mêmes.
démontré.
8111 a) Faire un calcul matriciel.

Ill Uti liser une résolution de système linéaire.


b) Déterminer g tel que f o g = go f = Id.
c) Procéder par récurrence.
d) Utiliser les relations de récurrence sur (an) et (bn) obtenues au
811 On peut choisir le/les vecteur(s) manquant(s) dans la base c).
canonique.
1111 a) Utiliser les définitions.
Ill Il faut trouver les vecteurs qu i s'expriment comme une com- b) Procéder par double-inclusion.
binaison linéaire des autres. c) Util iser les définitions.

. . a) Faire un calcul de rang. 11111 a) Utiliser une résolution de système linéaire.


b) Calcu ler ïi I et ïi 2 en fonction de Ü I et Ü2. b) Uti liser l'algorithme de Gauss.
c) Remarquer que Ker/= 01 et que lm/= Vect(Ü1,ü2).

1111 a) Qµestion classique.


ml a) Revenir à la définition de la représentation matricielle.
b) Ut iliser une résolution de système linéaire.
c) Qµestion classique. b) Partir de l'évidence: X= ld(x).
d) Faire un calcul de rang. c) Traduire matricellement y= f(x) de deux façons différentes.
e) Déte rm ine r les vecteurs f(Ü ;) en fonction des vecteurs ïi j· d) Réécrire plus simplement le c).

111!1 a) Étudier l'inversibilité de A. 1111 a) Qµest ion classique.


b) Considérer la juxtaposition d'une base de IE et d'une base de IF.
b) Chercher une re lation entre A2 , A et 13 .
c) Utiliser une résolution de système linéaire. c) Pour ! = 1, remarquer que x = x + 0 = 0 + x.
Pour 1 = x
1, partir de ï
= ë 1 + 1 = ë 2 + 2.ï
1111 a) Question classique. d) Pour 1 = ~ montrer que la juxtaposition d'une base de IE et
d'une base de IF donne une fami lle libre.
b) etc) Uti liser une résolution de système linéaire.
d) Déterminer un système d'équations cartésiennes de G. Pour ! = 1, montrer que IEn lF est réduit à {0}.
e) Uti liser d).
u
0
C:
ll9 a) et b) Uti liser une résolution de système linéaire.
1111 a) Revenir aux définitions.
::J c) Déterminer un système d'équations cartésiennes de IF.
0 b) Raisonner par l' absurde et utilise r un argument de dime nsion.
c) Procéder par récurrence.
lfl
,-i
0
811 Commencer par Ker f. d) Ut iliser b).
N
@
....,
.c
Ol
·;::
>-
a.
0
u

98

www.bibliomath.com
Espaces vectoriels et applications linéaires Ch ap,itre 5

Corrigés des exercices

-
a) On a Card(~\) = 4 > 3 = dim(l!ll 3), la famille §
liée.
=-
On note § 1 ( a, b, c, -)--
d .
1 est donc

Comme on ne trouve pas de relation «évidente» entre les


vecteurs de g;l, on va en chercher une par le calcul. Pour
cela on résout l'équation vectorielle (E) : x â + y b + z ë +
t d = Ô d'inconnue (x, y, z, t) e: l!ll4 . On a :
0
~
0
+ 2y + 3z + 4t =0
(E) = (S)
{ 2x + 3y + 4z + 5t = 0 (SI= { 0
0
3x +4y+5z+6t = O
(S) est donc un système Cramer, ce qui prouve que §3 est

l
0 + 2y + 3. z + 4t =0 libre.
(Sl = - II]- 2z - 3t =O dl On note g:4 = (ü, w). v,
- 2y - 4z - 6t = 0
On remarque que: Ü + w= 3v. La famille g:4 est donc liée.
e) On note$5 = (â, b, ë, d).
0 + 2y + 3z + 4t =0 Pour savoir si § 5 est libre, on résout l'équation vectorielle
(S) <==> - II] - 2z - 3t =o (E): x â + y b +zë + td = Ô d'inconnue (x,y, z, t) e: !R4 . On

1 0 =0
(S) a une infinité de solutions, ce qui prouve que g;I est liée
a:
0 + t =0

(on le savait déjà). Pour obtenir la relation cherchée entre (E) <==> (S) x +y =0
les vecteurs, il faut poursuivre la résolution de ce système. { y+ 3z + 2t =0
a+2(3 X - z =0
-2a-3(3
0 +t =0
a
(S)<==> II] - t =0

y + 3z + 2t = 0
On a donc, pour tout (a, (3) e: llll 2 : - z - t =0
(a+2(3)â -(2a+3(3)b +aë +(3d =Ô 0 +t =O

u
0
C:
::J et poura = 0 et J3 = 1 : 2a -
3b + d = 0.
- --
Par exemple pour a = 1 et 13 = 0 : a - 2 b + c = 0 - (S)<==>
II]
0
-t=O

+ t=O
0 -z -t=O
lfl b) On remarque que: (-i,l,-1- i) = -i.(l,i,1- i). c'est-à-
,-i
dire v =(- i).û, en notamg:2 =(ü, v).
0
N 0 +t=O
La famille $2 est donc liée.
@
...., (S)<==>
II] - t =O
c) Onnoteg:3 = (Ü,v,w) .
.c 0 + t=O
Ol
ï:::: Pour savoir si g:3 est libre, on résout l'équation vectorielle
>- 0=0 (4 +L3)
a. (E) : x -u + y -v + z ~
w = -d'
0 inconnue (x, y, z) e: C3 . On a:
0 (S) a une infinité de solutions, ce qui prouve que §5 est liée.
u
~ . - iz = 0 Pour obtenir une relation entre les vecteurs, il faut pour-
(E) <=> (S) lX - zy =0 suivre la résolution de ce système.
{
y + iz = 0
99

www.bibliomath.com
Chapitre 5 Espaces vectoriels et applications linéaires

-ex =
Par exemple pour û (0, 1, 0, 0) : 0 - 2 x 1 + 0 - 2 ;: 0 =
donc û n'est pas combinaison linéaire des vecteurs de §1.
ex
- ex Cet exemple montre aussi que §1 n'est pas génératrice de
Ol1 4 , mais on le savait déjà.
ex
On a donc, pom tout ex e: IR : b) Onnote$2 =(a,b,c).
- - -
- ex a + ex b - ex c + ex.....,.
d =-0 Pour savoir si § 2 est génératrice de C 2 , on résout l'équa-

---- tion vectorielle (E) : X + a y-,; + z ë = û d'inconnue


Par exemple pom ex = 1 : - a + b - c + d = .....,.
0. (x,y,z) e: ( 3 , avec Û = (u1, u2) e: ( 2 . On a:

~ + y + ~z =u1
f) Onnote$s=(a,b,c,d).
(E) <==> (S) {
On remarque que: a+ b - ë =d. La famille $s est donc IX + IZ = U2
liée.

+ta
a) On a Card($1) = 3 < 4 = dim(IR 4), la famille $1 n'est donc
pas génératrice de Ol1 4 •
(S) est compatible pour tout Û =(u1, u2) e: iC 2. La famille
Onnote$1 = (a,b,c) . §2 est donc génératrice de C2 .
Comme on ne trouve pas de façon « évidente» un vecteur
de lll1 4 qui n'est pas combinaison linéaire des vecteurs de c) On note §3 = (â. b. ë).
$1, on va en chercher un par le calcul. Pour cela on résout
Pour savoir si §3 est génératrice de C 3 , on résout l'équa-
l'équation vectorielle (E) : X a+ y-,;+ z ë = û d'inconnue
(x, y, z) e: IR 3 , avec Û =(
u1, u2, u3, u4) e: IR 4 . On a :
tion vectorielle (E) : X + a y-,;+ z ë = û d'inconnue
(x,y,z) e: ( 3 , avec Û = (u1, u2, u3) e: C 3 . On a :
2z = u1
0 + iz = UJ
(E) = (S) y = u2 (E) <==> (S) ix + i y =u2
.
{ X+ y = U3 {
y + lZ =U3
2y - 2z = U4
0+ Y = U3
(S) <==>
ŒJ =u2
2y - 2z =U4
2z = u1 (Li)

ŒJ+y

u (S)<==>
ŒJ = u2
0
- ~ = U4-2U2
C:
::J
(S) est compatible pour tout û = (u1,u2,u3 ) e: C3 . La fa-
0 2z = u1 mille §3 est donc génératrice de c 3 .
lfl
,-i
0 ŒJ+ y d) Onnote§4=(a,b,c,d).
N
@
...., (Sl <=> ŒJ Pour savoir si § 4 est génératrice de !R4, on résout l'équa-
.c
Ol
- ~ = U4-2U2 tion vectorielle (E) : X a + y b + Z c + t d = Û d'inconnue
(x,y,z, t) e: iR4. avec Û = (u1, u2, u3, u4) e: IR • On a :
ï:::: 4
>- 0 = u1 -2u2 + u4
a.

~
0 (S) est compatible si et seuJement si u1 - 2u2 + u 4 = O.
u
Donc : Û = (u1,u2,u3,u4) est combinaison linéaire des
vecteurs de §1 si et seulement si u1 - 2u2 + u 4 O. = (E) <==> (S) { +y + t : ::
y+ 3z + 2t =U3
100 X - z = U4

www.bibliomath.com
Espaces vectoriels et applications linéaires Chap,itre 5

0 +t = u1 Xw À.Xu +Xv
Rema,quons quec {
(S) <=> 0 - t = u2-u1 (L2 -L1) Yw À.Yu+ Yv
y + 3z + 2t = U3 Zw À.zu + zv
On a:
- z -t = U4 - U1 (L4 - L1)

+ t = u1
Xw+Zw = (>..xu + Xv} + (>..zu + Zv}
- t = u2 - u1
= >..b) +b )
(S)<==> =yu =yv

~ + 3t = U1 -
carüe:E1 carVe:Œ1
U2 + U3
- z - t =u4 - u1
= À.Yu + Yv
Yw
0 +t = u1 ce qui prouve que w= À.Ü + v E IE 1.
[IJ -t = u2 -u1
• Ces deux vérifications montrent que IE 1 est un sous-
(S)<==> !3z 1+ 3t = u1 - u2 + u3 espace vectoriel de IR!3 .
0 =-2u1 - u2+u3+3u4 b) Comme l'équation y = x + 1 est linéaire mais non homo-
gène, on conjecture que IE2 n'est pas un sous-espace vecto-
(3L4 +L3)
riel de IR! 2.
(S) est compatible si et seulement si -2u1 - u2 + u3 +3u4 =
En effet Ô = (0,0) fi'. IE2 car: 0 ;é O+ 1 = 1.
O. Donc: Ü = (u1, u2, u3, u4 ) est combinaison linéaire des
Ceci prouve que IE2 n'est pas un sous-espace vectoriel de
vecteurs de §4 si et seulement si -2u1 - u2 + u3 + 3u4 = O.
IR!2.
Par exemple pour Ü = (l,0,0,0): -2x l-0+0+3x0 = -2 ;é 0
donc Ü n'est pas combinaison linéaire des vecteurs de §4. c) Comme l'équation y = x + z 2 n'est pas linéaire, on conjec-
ture que IE3 n'est pas un sous-espace vectoriel de IR! 3 .
Cet exemple montre que § n'est pas génératrice de IR! 4.
4
En effet:
a, --+b , --+
e) On note §5 = ( --+ c , -d ) . Û = (l,2,l)EIE3car: 2 = 1 +1 2 .
Pour savoir si § 5 est génératrice de IR!3, on résout l'équa- (- 1).Ü = (- 1, - 2, - 1) 11'.IE3 car: - 2;é - 1+(- 1) 2 =O.
tion vectorielle (E) : X a+ y b + Z C + t d = Û d'inconnue
(x, y,z, t) E IR!4, avec Ü = (ui, u2, u3) E IR! 3. On a:
Ceci prouve que IE3 n'est pas un sous-espace vectoriel de
IR!3.
0 + t = u1
d) Comme le paramétrage (a+ 213 + y,13) est linéaire et homo-
(E) <=> (Sl 2y + t = u2
{ gène, on conjecture que IE4 est un sous-espace vectoriel de
2x + 2z + t =u3 IR!2.
• Ô = (0, 0) E IE4 car Ô = (a + 213 + y, Pl avec a= 13 = y = O.
+t = u1
(S)<==>
• Soient À. E iR! et (ü, v) E IE4 X IE4.
Comme (Ü, v) E ~ X IE4, on a: û = (au+ 213u + Yu,13u)

"O
avec (au,13u, Yu) E IR! 3 et v
= (a v + 2Pv + Yv,13v) avec
0 (S) est compatible pour tout Ü = (u 1, u2, u3) e: IR!3 . La fa- (av,Pv,YvlEIR! 3 .
C:
::J
mille $5 est donc génératrice de IR!3 . On pose w= À.û + v. On a :

-
0
lfl
,-i
0
N (À.au+ av+ 2À.13u + 213v +À.Yu+ Yv, À.l3u + 13v)
@ a) Comme l'équation y = x + z est linéaire et homogène, on
...., conjecture que IE 1 est un sous-espace vectoriel de IR! 3 .
.c aw À.au +av
Ol
·;:: • Ô = (0,0,0) e: IE1 car O = 0 + O.
>- On pose alors: 13w >..Pu +Pv et on a :
0
a. v) E IE1 x IE1.
• Soient À. E IR! et (Ü, {
À.Yu +yv
u Yw
On note Ü = (xu, Yu, Zu) , v = (xv, Yv, Zv) et w= À.Ü + v =
(Xw,Yw,Zw) .

101

www.bibliomath.com
Chapitre 5 Espaces vectoriels et applications linéaires

Ceci prouve que üi = À Ü + ïi e: IE4. Ona:


• Ces deux vérifications montrent que IE4 est un sous-
espace vectoriel de 11i2 •
/1 (üi) fi(xw,Yw)
e) Comme le paramétrage (a+ 2, 13) est linéaire mais non ho-
mogène, on pourrait conjecturer que IE5 n'est pas un sous- (xw +2yw,-Xw -yw)
espace vectoriel de 11i2 . Mais nous allons voir que contrai- (J...xu +xv +2À.yu +2yv, - À.Xu - xv - À.yu - Yv)
rement à ce que nous dit notre intuition, IE5 est bien un J....(xu +2yu, - Xu - Yu)+ (xv +2yv, - Xv - Yv)
sous-espace vectoriel de ~ 2.
J....f1(xu,Yu) + fi(xv,Yv)
• Ô = (0,0) e: IE4 car Ô = (a+2,13) avec a = - 2 et 13 = O. À.fi (û) + fi (ïi)
• Soient À e: ~ et (ü, v) e: IE5 x IE5 .

Comme (Û, v) e: IEs x IE5, on a: Ü = (au+ 2,13u) avec


(au,13u) e: ~ 2 et ïi = (av +2,13v) avec (av,13v) e: ~ 2 . Cette vérification prouve que fi est linéaire, et donc que fi
est un endomorphisme de ~ 2.
On pose üi =ÀÜ + ïi. On a :
-
w À..(au + 2,13u) +(av+ 2,13v)
b) On remarque que:
(À.au +av +2J... + 2,J...13u +13v)

On pose alors :
aw = et on a:
h(Ô) [2(0,0,0,0)
{ (0,0,2,0)
13w
ô
üi = (aw +2,13w)

Ceci prouve que üi = À Ü + ïi e: IE5.


Ceci prouve que h n'est pas linéaire.
• Ces deux vérifications montrent que IE5 est un sous-
espace vectoriel de ~ 2 .
f) Comme l'ensemble N est "discret", c'est-à-dire que ses c) Intuitivement, on peut conjecturer que [3 n'est pas li-
points sont "isolés", on peut conjecturer que c'est n'est pas néaire.
un sous-espace vectoriel de Ili.
En effet: En effet, pour û = (l, l, l) et À= -1, on a:
Ü = 1 e:Nmais !.ü = ! 'l'.N. f3(J....ü) = /3(-1,-1,-1) = (-3+2,-1,1) = (-1.-1.1)
J....f3(û) = (-1)./3(1,l,l) = (-1).(1,-1.1) = (-1,1,-1)
Ceci prouve que N n'est pas un sous-espace vectoriel de~. et donc : [3(J....ü) t J....[3(Ü).

g) On a IE7 =Vect(9'), où 9' est une famille de quatre vecteurs Ceci prouve que 13 n'est pas linéaire.
deC3 .
u Par théorème, on sait donc que IE7 est un sous-espace vec-
0
C: toriel de 4::3 (c'est même le plus petit contenant la famille d) Montrons que ft est linéaire de ~ 4 dans ~ 3 .
::J

-
0 ~).
lfl
,-i Soient À.€~ et (û. v) e:~4 x ~4.
0
N
@ a) Montrons que fi est un endomorphisme de ~ 2 .
...., On note : Û = (xu,Yu,Zu, tu) et ïi = (xv,Yv,Zv, tv) et üi =
.c Soient À e:~ et (û,ïi) e: ~2 x ~2. À.Û +V = (Xw,Yw,Zw,tw) .
Ol
ï::::
>- On note: Ü = (Xu,Yul. v = (Xv,Yvl et üi = J...Ü + ïi =
a. (Xw,Yw).
0 Xw À.Xu +Xv
u
Xw À.Xu +Xv Yw À.Yu+ Yv
Remarquons que: Rema<quoru;quec {
{ Yw = À.Yu + Yv Zw = À.Zu +zv
102 tw = À.tu+ tv

www.bibliomath.com
Espaces vectoriels et ap p lications linéaires Chap,itre 5

Ona: b) IE2 est défini par un paramétrage.


On va directement donner une famille génératrice de IE2,
t(w) ce qui prouvera aussi que IE2 est un sous-espace vectoriel
f(xw,Yw,Zw, tw) de lll1 4 . On déterminera ensuite un système d'équations car-
= (xw+3zw - tw, - Xw - 2Yw + tw, tésiennes de IE2.
- Xw + Yw + 2zw + tw) • Ona:
= (.\xu + Xv +3.\zu +3zv - Àtu - tv, IE2
-.\xu - Xv -2.\yu -2yv +.\tu+ tv, {(a -13 + 3y, y,a -13,0) e: ~ 4 / (a.~. y) e: ~ 3 }
-.\xu -xv +.\yu+ Yv +2.\zu +2zv +.\tu+ tv) {a.(1,0, l, 0) + l3.(- l, 0, - 1,0)
.\.(xu +3Zu - tu,-Xu -2yu + tu, +y.(3, 1,0, 0)/ (a,l3, y) E im 3 }
-Xu + Yu +2Zu + tu)
= Vect(0,0:1,0):~·,(3,l~O,O))
+(xv + 3Zv - tv, - Xv - 2yv + tv ,
=Ü = ïi =w
- Xv + Yv + 2Zv + tv)
Ceci prouve que IE2 est un sous-espace vectoriel de Ol14, et
= .\.f(xu,Yu,Zu, tu) + f(xv,Yv,Zv, tv)
que la famille (ü, ïi, w) est génératrice de IE2 .
.\.f(ü) + f (ïi)
• Soit (x,y,z, t) e: ~4 . On a:
Cette vérification prouve que f est linéaire de lli 4 dans lli 3 . @] - =X

-
l3 + 3y
y =y
(x, y, z, t) e: IE2 <==> 3(a, 13, y) e: lll1 3 / (S)
{ a -13 =z
a) IE1 est défini par un système d'équations cartésiennes.
0 = t
On va directement donner une famille génératrice de IE1, Autrement dit : (x, y, z, t) e: IE2 <==> (S) est compatible
ce qui prouvera aussi que IE1 est un sous-espace vectoriel
de~4 . Or:
@]- ~ + 3y = X
• Soit (x, y, z, t) e: IAl 4 . On a :
(S) <==>
[TI= y
l
ŒJ+y -z-t=O
(x, y, z, t) e: IE 1 <==> (S) - 3y =z-x
Œ)+ z -t=O
0 =t

2a @] - ~ + 3y = X
- a+l3
(S) <==>
[TI =y
a 0 = -x+3y+z
0 =t
On obtient ainsi IE1 sous forme d'un paramétrage:
u
0
Donc: (S) est compatible = lJ -x+3y+zt 0
0
C:
::J Mais on a : (x, y, z, t) e: IE2 <==> (S) est compatible.
0
• De plus: On obtient donc un système d'équations cartésiennes de
lfl
,-i
0 IE2 :
N IE1 {(2a,-a+l3,a,l3) e:lli 4 / (a,13) e:~2 }
@ {a.(2, -1, l, O) + l3.(0, l, 0, 1)/ (a,13) e: ~2 }
IE2 ={(x,y,z, t) e: lll14/ -x+3y+z =Oet t =o}
....,
.c Vect ((2, - ~.' l, 0): ,(0, 1:0, ll) c) IE3 est défini par une famille génératrice :
Ol
·;::
>-
a. =Ü =v IE3 = Vect( (1, 1,0), (0,2, 1),.(1, - ~· -1))
.__.,..._., .__.,..._.,
0
u =Ü =v =w
Ceci prouve que IE1 est un sous-espace vectoriel de ~ 4, et
que la famille (ü, ïi) est génératrice de IE1. Ceci prouve que IE3 est un sous-espace vectoriel de !Al3.

103

www.bibliomath.com
Chapitre 5 Espaces vectoriels et applications linéaires

~
On va donner un paramétrage de IE3 et en déduire un sys- 0
tème d'équations cartésiennes. (E) <==> (S) J: + y + z : <==> J; 0
• Ona: 1 y=O
ce qui prouve que @1 est libre.
z 0 1
IE3 = Vect(û, v, w) Mais on a: Card(@ 1) = dimlli 3 = 3, donc, d'après le théo-
{o:ü +13v + yw / co:,13, y)€ 11i3} rème de la base incomplète, @1 est une base de lli 3.
= {(o:+y,o: + 2j3 - y,j3 - y)Elli 3 /(o:,j3,y) Elli 3}
b) On a: §2 = (.._,._,
(1, 0,0,0), (l, l, 0, 0), ,(2,0: l, 1)).
.._,._,
On vient donc trouver un paramétrage de IE3. =Ü = ÏÎ = ÏÛ
• Vérifions que §2 est libre. Pour cela, on résout l'équation
• Soit (x,y,z) € lli 3. On a: v w
vectorielle (E) : x û + y + z = 0 d'inconnue (x, y, z) E IR! 3.
Ona:

l
@J +y=x
(x, y, z) E [3 = 3(o:, 13, y) E lli3 / (S) a + 213 - y = y
X+ y+ 2z = 0
y =0
0
13 - y = z (El= (Sl 0
{ z =0
Autrement dit: (x,y,z) E IE3 = (S) est compatible 0
z =0
Or: ce qui prouve que §2 est libre.
@] +y =X • dim(~ 4 ) = 4 donc il faut compléter §2 avec 4 - 3 = 1 vec-
(Sl = ~-2y=y-x teur, pour obtenir une base de IR! 4 . De plus, on sait qu'on

1 13 - y =z
peut choisir ce vecteur dans la base canonique de IR! 4 .
Onposedonc@2= ( ---+ - ---+---+ ---+
u, v,w,a)avec a = (0,0,1,0).
@] +y =X Vérifions que @2 est libre. Pour cela, on résout l'équa-
(Sl = ~-2y=y-x v
tion vectorielle (E) : x Ü + y + z üi + tâ = 0 d'inconnue

1 0 =x - y+2z
(x,y,z,t) Elli4 .0na:

X + y + 2z =0 { X = 0
Donc : (S) est compatible = x - y+ 2z = 0
Mais on a: (x, y, z) € IE3 = (S) est compatible. (El = (Sl y =0 = y 0
{
z+t=O z 0
On obtient donc un système d'équations cartésiennes de
IE3 : z =0 0
IE3 = {(x,y,z) Elli 3/ x-y+2z =0} ce qui prouve que ~2 est libre.

-
Mais on a: Card(@2) = dimlli 4 = 4, donc, d'après le théo-
rème de la base incomplète, @ 2 est une base de IR! 4 .

c) On a: §3 = ((1, 0: l,Ol_.,(O, 1:0,0l).


a) O n a : § 1 = ~ ~ -
=Ü =ÏÎ
"O
=u = ÏÎ
v
• Comme et û et sont non colinéaires (car leurs compo-
0
C:
::J
v
• Comme et Û et sont non colinéaires (car leurs compo- santes ne sont pas proportionnelles), on sait qu'ils forment
0 santes ne sont pas proportionnelles), on sait qu'ils forment une famille libre.
lfl une famille libre.
,-i • dim(~ 4 ) = 4 donc il faut compléter §3 avec 4 - 2 = 2 vec-
0
N • dim(~3) = 3 donc il faut compléter § 1 avec 3 - 2 = l vec- teurs, pour obtenir une base de 11i4 • De plus, on sait qu'on
@ teur, pour obtenir une base de IR! 3. De plus, on sait qu'on peut choisir ces vecteurs dans la base canonique de m! 4 •
...., peut choisir ce vecteur dans la base canonique de ~ 3.
.c On pose donc@3 =
__.
(ü,v,a,b) avec â = (0,0,l,O) et
Ol
·;::
>-
On pose donc @ 1 =(û, v, i) avec 7 = (1,0,0). b = (0,0,0, 1).
a.
0 Vérifions que @ 1 est libre. Pour cela, on résout l'équation Vérifions que @3 est libre. Pour cela, on résout l'équa-
u
v
--+ --+ 3
vectorielle (E) : x û + y + z i = 0 d'inconnue (x, y, z) € ~ . v
tion vectorielle (E) : x û + y + z â + t b = 0 d'inconnue
Ona: (x,y,z, t) E IR!4 . On a:

104

www.bibliomath.com
Espaces vectoriels et applications linéaires Chap,itre 5

(E) = (S) { >~


t=O
+z : ={ ~
0
0
0
0
• dirn(~3) = 3 donc il faut extraire 4 - 3 = 1 vecteur de $2,
pour obtenir une base de ~3. De plus, on sait qu'il faut ex-
traire un vecteur qw s'exprime comme une combinaison
linéaire des autres.
* Comme on ne trouve pas de relation« évidente» entre les
ce qui prouve que ~3 est libre. vecteurs de g:2, on va en chercher une par le calcul. Pour
Mais on a: Card(.@3) = djmill1 4 = 4, donc, d'après le théo- cela on résout l'équation vectorielle (E) : x â + y b + z 7 +
rème de la base incomplète, @3 est une base de lll14 .

-
t -d = -0 d'inconnue (x,y,z, t) E ~ 4 . On a:
0 + 2y + z + t = 0
(E) <==> (S) ~+z - t =0
a) Ona:$1 = ~ ~ ~ -
=u = 11 =w 1 y + z+t = O

• Pour savoir si $1 est génératrice de Ol1 2, on résout 0 + 2y + z +t =0


v
l'équation vectorielle (E): xÜ + y + zw = â d'inconnue (S) <==> 1 sy 1 +z - t =0
(x,y,z) E lll1 3, avec â = (a1,a2) E lll1 2. On a:

= 0 + z = ai
(S)
1 ~+6t= O
a une infinité de solutions, ce qui prouve que $ 2 est
(SL3 - L2)
(E) (S)
{ [IJ + z = a2 liée (on le savait déjà carCard($2) = 4 > 3 = dim(Ol1 3)). Pour
obtemr la relation cherchée entre les vecteurs, il faut pour-
(S)est compatible pour tout â = (ai, a 2 ) E C 2. La fa-
swvre la résolution de ce système.
mille g;l est donc génératrice de ~ 2• c'est-à-dire : Ol1 2 =
Vect(ü, w). v, a
• dim (lll1 2) = 2 donc il faut extraire 3 - 2 = 1 vecteur de g:1, -a
pour obtenir une base de lll1 2. De plus, on sait qu'il faut ex- 3a
traire un vecteur qui s'exprime comme une combinaison
linéaire des autres. -2a
On a donc, pour tout ex E ~ :
On remarque que : = Ü + w v.
Par théorème, on saü
donc que@1 = (ü, v) est encore génératrice de ~ 2 : ~ 2 = aci - ab + 3a7 - 2ad = 0
Vect(Ü, v).
Par exemple pour a= 1: â = b -37 +2d.
Mais on a: Card(@1) = dim ~ 2 = 2, donc, par théorème,@1
est une base de ~ 2. * Par théorème, on sait donc que @2 = (b, 7, d) est en-
core génératrice de Ol13 : Ol13 = Vect (b, 7, d).
b) Ona:g:2 = ~·~ · ~ ·!!.:_-.~.
Mais on a : Card(@2) = dim~ 3 = 3, donc, par théorème, @2
=a =b =d est une base de ~3.
• Pour savoir si $2 est génératrice de ~3. on résout l'équa-
c) O n a : $ 3 = ~ · ~ · ~ ~ -
tion vectorielle (E) : x â + y b + z 7 + t d = Ü d'inconnue
(x,y,z, t) E ~4, avec Ü = (ui, u2, u3) E ~ 3. On a : =a =b =ë =ri
• Pour savoir si $ 3 est génératrice de lll1 2, on résout l'équa-
"O
0 0 + 2y + Z + t = Ut
tion vectorielle (E) : x â + y b + z 7 + t d = Ü d'inconnue
0
C:
::J (E) = (S) l sy 1 + z - t = u2 (x, y, z, t) E lll14, avec Ü = ( u1, u2) E ~ 2. On a :
lfl
,-i
0
1 y +z+t =u3 (E) = (S) { 0 + 2y + 6z - t = u1
N
@
0 + 2y + z +t = u1 X + y + Z + t = U2
...., (Sl <==> ~+z - t = u2 0 + 2y + 6z - t = Ut
.c
Ol
ï::::
>-
a.
1 ! 4z 1 + 6t = Su3 - u2
(S)=
{ -[IJ- Sz + 2t = u2 - ui

0 est compatible pour tout Ü = (u1, u2, u3) E ~ 3. La fa-


(S) (S) est compatible pour tout Ü = (u1,u2) E ~ 2. La fa-
u mille g:2 est donc génératrice de ~ 3• c'est-à-dire : Ol1 3 = mille g:3 est donc génératrice de lll1 2, c'est-à-dire : ~ 2 =
Vect(â, b, ë, d). Vect(â, b,ë, d).

105

www.bibliomath.com
Chapitre 5 Espaces vectoriels et applications linéaires

• dim(llll 2) = 2 donc il faut extraire 4-2 = 2 vecteurs de §3,


pour obtenir une base de l!ll2 . De plus, on sait qu'il faut ex-
traire un vecteur qui s'exprime comme une combinaison
linéaire des autres.
-
a) Déterminons Je rang de la famille @ :

rg(@)
* Comme on ne trouve pas de relation« évidente» entre les
vecteurs de §3, on va en chercher une par le calcul. Pour
cela on résout l'équation vectorielle (E): X a+ y b + Z C +
0
-
td 0 d'inconnue(x,y,z,t)e:llll4 . Ona :
= ---+
= [TI
(E) <==> (S) { 0 + 2y + 6z - t =0 : ) (Ls - L1I
x + y +z + t =O

~]
0
[TI
0
(S) <=>
l + 2y + 6z - t
-II]-5z+2t = 0
=0 0

(S) a une infinité de solutions, ce qui prouve que §3 est On a rg(@) = Card(@) = dim(llll3), donc par théorème on
liée (on le savait déjà car Card(§3) = 4 > 2 = dim(llll2 )). Pour sait que@ est une base de l!ll3.
obtenir la relation cherchée entre les vecteurs, il faut pour-
suivre la résolution de ce système. b) On note (7, j, k) la base canonique de l!ll3.

4cx -313
Ona:
ïi 1 =(l,2,4)= i +2j +4k -
- 5a + 213 et:
--,1,, ; ~
a v2 = (3,-1,0) =3 1 - J

Déterminons les formules de changement de base entre la


base canonique et@ :
On a donc, pour tout (a, 13) e: llll 2 :
- +-
i k -Û2+ Û3

- - - --
j + k - Û l + Û3
(4a - 3j3) a +(- 5a+213) ~ ~
b + ac +Pd =0
= -+- +-
i j k = Üi + Û2 - Û3

On a donc:
Par exemple pour ex= 1 etl3 = 0: - c = -4a
---+ +5 -b

et pour a = 0 et 13 = 1 : d = 3a - 2b.

u
0
C:
::J
* Comme = c -4a+ 5b + Od, on sait donc par théo-
Ainsi:
0 rème que (a, b, d) est encore génératrice de l!ll 2 : l!ll3 =
lfl
,-i
Vect(a, b, d).
0
N
@
...., Ensuite, comme d = 3a - 2b, on sait donc par théo-
.c rème que @3 = (a, b) est encore génératrice de l!ll 2 : l!ll 2 =
Ol
ï::::
>-
a.
Vect(a, b). Mil
a) On peut procéder comme à l'exercice 5.4.
0
u
Mais on a : Card(@3) = dim llll 2 = 2, donc, par théorème, @3 Pour aller plus vite (et pour changer un peu...), on peut
est une base de l!ll 2. aussi remarquer que, si@cano = (ë 1, ë 2, ë 3, ë 4) désigne

106

www.bibliomath.com
Espaces vectoriels et applications linéaires Chap,itre 5

la base canonique de [ij 4 et '€cana = (71, 7 2, 7 3) celle de On a donc:


~ 3 , on a pour tout û =
(x, y, z, t) e: ~ 4 :
Ker f {(0,a,2a,-4a)/ a e: [ij}
= {a.(0,1,2,-4)/ a e:~}

l~
]
-2 3 = Vect((0,1,:, - 4))
Mat (f(û))
't'cano
[ ~]
-1
-2
0
3
2
=u
Et comme û '7" 0 , on a donc obtenu une base de Ker[.
Ainsi : dim ( Ker f) = 1.
= Ax Mat(û)
~cano
• Base de Tm f.
Par théorème on sait que l'image de f est engendrée par la
-2 3 famille [(@cana) = (t(ë i),t(ë 2).f(ë 3),f(ë 4) ). avec:
avec: A= (~l -2 3
-1 0 2 (0, - 1,-1)
{ /(7,)
f(ë2) (-2,-2,0)
Ceci prouve que f est linéaire de [ij 4 dans [ij 3 _
f(ë3) (3,3,2)
Ceci prouve aussi que: Mat (!,@cana, '€cana) = A
f(ë4) (l, l, l)
(mais ce n'était pas demandé ...).
Il reste à extraire une base de cette famille génératrice.
b) • Base de Ker[.
* Comme on ne trouve pas de relation« évidente » entre les
Soit Û = (x, y, z. t) e: [ij 4 . On a : vecteurs de /(~cana), on pourrait procéder comme dans
l'exercice 5.7, mais cela risque d'être un petit peu long et
on va certainement refaire plusieurs fois le même calcul. ..
Reprenons donc la base de Ker[ déterminée précédem-
ûe:Kerf = f(û)= 0 ment. Chaque vecteur de cette base nous donne en fait une
relation entre les vecteurs de !(~cana).
- 2y + 3z + t =0
Plus précisément: (0,l,2,-4) = ë2+2ë3 - 4ê4 e: Ker[
(S) - x - 2y + 3z + t : 0
{ donc f(ë 2 + 2ë 3 - 4ë 4) = 0 et donc, comme f est li-
- X + 2z + t - 0 néaire: f(ë2) +2f(ë3)-4f(ë 4) =O.
Comme f(ë 2) = Of(ë i) - 2f(ë 3) + 4f(ë 4), on sait par
et:
théorème que la famille (t(ë i).f(ë 3).f(ë 4 )) reste géné-
0 ratrice de lm/.

l
+ 2z + t =0 (L3)
(S) <==> =x - 2y + 3z + t =0 * Si cette famille est libre alors on aura déterminé une base
de Tm f. Pour le savoir, on résout l'équation vectorielle (E):
-2y+3z+t=O (L1)
xf(ëi) + zf(ê3) + tf(ê4) = 0 d'inconnue (x,z,t) e: [ij 4 .
"O
0 -0 + 2z + t =0 Ona:
C:
::J
(S) <==> - l 2y 1 + z =0 (L2 - L1)
3z + t =0
0
lfl
,-i 1 - 2y + 3z + t =0
(E) = (S)
{
- x + 3z + t =0
0 - X+ 2z + t = 0

t: ~
N
@
= 1-0 :z
l
...., (S) - 12y 1: + - 0+ 2z + t = 0
.c
Ol
·;:: ~+t=O (L3-L2)
(S) = -x + 3z + t = 0
>-
a. 3z + t = 0
0 0
u - 0 + 2z + t =0
a
(S)= 0 =0

=
2a
- 4a 1 3z +~ = O
107

www.bibliomath.com
Chapitre 5 Espaces vectoriels et applications linéaires

0 Déterminons les formules de changement de base entre


~cano et~:
(S) = { : 0
0
-V1 = 71 + 72
La famille
est donc libre. On en conclut que
4))
(t(ë i),f(ë 3),f(ë est une base de Tmf.
Donc rg(f) = dim ( lm f) = 3.
{ v2
ïi 3
=
=
71
272+ 73

c) D'après la question a), on a: 71 -


v2

-2 3
=
{- 72 - -
V 1 - V2
E3 -2ïi 1 + 2ïi 2 + ïi 3
Mat(f,@cano,~cano) =A= (~l -2 3
- 1 0 2 :] On a donc:

Une autre méthode donnant ce résultat consiste à calculer,


à partir de l'expression analytique de f, les coordonnées
f(ûi) ÏÎ1-ïi2-ÏÎ3
des vecteurs f(ë i), i e: [ 1, 4], dans la base canonique de f(û2) -ïi1-ïi2-ÏÎ3
11113. f(Ü3) = - 2ïi 1 + 3v 2 + ïi 3
d) • Déterminons Je rang de la famille @ : f(Ü4) = - 3ÏÎ 1 +SÏÎ2 + 2ÏÎ3

[TI 1 Finalement:

rg(@) = rg
0 [TI =4
0 0 [TI -1 -2

0 0 0 Mat(f,IJs, <C) = [~·.l -1 3


-1 -1
On a rg(@) = Card(@) = dim(~ 4 ), donc par théorème on

-
sait que fJs est une base de ~ 4 .
• Déterminons le rang de la famille~:

a) • D'après Je cours, il suffit de montrer que A est inversible

F ;]
1
et de déterminer A- l. Pour cela, considérons Je système
rg('C)
rg ~ 0
0

"O
3
[[Q Œ
rg :
1

0
~l (L2 - L1 J
=a
0
C:
::J On a rg('G') = Card('G') = dim (~3), donc par théorème on AX=Y=
3y -0 =b
0
lfl
sait que <C est une base de ~ 3 . ~ =C
,-i
0 e) On doit calculer les vecteurs f(û i). i e: [1,4], en fonction - x - 4y +4z + ~ = d
N des vecteurs f(v j), je: [1,3].
@
...., On a, d'après l'expression analytique de f: 0 =a
.c
Ol
·;:: f(ûi) (0, -1, -1) =- 7 2 - 7 3 AX=Y=
ŒJ =!c
>-
a. f(û2) (-2,-3,-1) = -271-372- 7 3 0 =-b+~c
0
u t(û3) = (1,0,1) = 71 + 73 [D = !a +2b - 2c+ !d
t(û4) (2, l, 2) = 27 1 + 7 2 + 27 3 On a un système de Cramer, donc A est inversible. De plus :

108

www.bibliomath.com
Espaces vectoriels et applications linéaires Chap,itre 5

Donc:A2 =3A-24 .
0 0 0 Par unicité de la représentation matricielle dans une base,
0 0 on a donc: f 2 = 3f - 2Id1R4.

-2 3 0 • On a donc:

4 -4 -1 -1
f O 2(f - 31dw) = 2 (f - 31dlR4) of = ldlR4
Par théorème on sait donc que f est un automorphisme
de lli 4 et que, si ~cano = (ë 1, ë 2, ë 3, ë 4) désigne la base Par théorème f est donc un automorphisme de IR4 et:
canonique de lli 4 , on a : r 1
= -,l (f-3IdlR4).
c) • On a g = f - ldlR4 et donc B = A - 4.
2 0 0

• Ona:
Mat(f- 1)=A- 1 =!
alcano
0
2 0 -2
0

4
3
-4 l
f B=
0

0
0
- 1
0
2
2
-4
0
-1
- 1
4

A, } 3y - z
2y
X

- x - 4y+4z+2t
l • Base de Ker g.
Soit û = (x,y,z, t)

û E Ker g
4
E 11i .

ç::::::;,
On a:

f (û) = Û
et donc, pour tout (x, y, z, t) E 11i4 : =x
(S) { x 3y-z =y
f(x,y,z,t) = (x,-3y-z,2y,-x-4y +4z +2 t)
2y = z
De même : - X - 4y + 4z + 2t =t

}!
et:
2x

A- 1 x z J O=0
z 2 -2y+3z (S) ç::::::;,
t x+4y - 4z + t

et donc, pour tout (x, y, z, t) E 11i4 :


LX_;;::z +' : :

-@- 4y
l
+4z+ t = O
(S) ç::::::;,
u
0 r 1(x,y,z, t) = i(2x,z, - 2y +3z,x +4y - 4z + t) Ej-z =0
C:
::J X 4a+j3

l
0
lfl b) • Pour touver une relation entre les fonctions f et IdlR4, 2, f
,--i (S) ç::::::;, 3(a,13) E 11i2 / y: a
0 on va chercher une relation entre les matrices A2 , A et 14 . 2a
N
@
....,
Ona: = 13
.c On a donc :
Ol
ï::::
>-
a. Kerg {(4a + 13, a, 2a, 13) / (a, 13) E lli2 }
0

~]'
u 0 0 0 0 0
{a.(4, l ,2,0) +j3.(1,0,0, l)/ (a ,13) E 11i2 }
0 3 -1 0 7 -3 0
Vect((4, 1~2,0)_, p. o~o, 1))
0 2 0 0 6 -2 0
=Ü =v
- 1 -4 4 -3 - 12 12 4 109

www.bibliomath.com
Chapitre 5 Espaces vectoriels et applications linéaires

Et comme Ü et v
ne sont pas colinéaires, on a donc ob- (X
tenu une base de Ker g.
- 213 + y
Ainsi : dim ( Ker g) = 2.
13
• Base de Img.
y
Par théorème on sait que l'image de g est engendrée par la
On obtient donc:
famille g(~cano) = (g(ê i).g(ë 2),g(ê3),g(ê4) ). avec:
IF
g(ë 1) (0,0,0, - 1) = {(ex, - 213 + Y, 13, y) E: ~ 4 / (ex, 13, y) E: ~ 3 }

g(ê2) (0,2,2, - 4) {cx.(l, 0, 0, 0) + j3.(0, -2, l, O) + y .(O, l, 0, 1) / (ex, 13, y) e: ~ 3}

{ g(ê3) (0, - 1, - 1,4) Vect ((1, 0~0, 0)1~ ,(O, l~O, l))
g(ë 4) (0, 0, O, l) =a =b

Tl reste à extraire une base de cette famille génératrice. Ceci prouve que IF est un sous-espace vectoriel de ~4, et
que la famille (â, b, ë) est génératrice de IF.
* Reprenons la base de Ker g déterminée précédemment :
(4,l,2,0) =4e 1 + ê2 +2e3 E: Kerg donc b) La famille (ü, v) est génératrice de IE, et elle est libre car
g( 4e l + e 2 + 2ë 3) = 0 et donc, comme g est linéaire : v
Ü et ne sont pas colinéaires.
4g(ëi)+g(ë2)+2g(ê3)= ô. Ainsi (û, v) est une base de IE et donc : dim(IE) = 2.
(1,0,0, 1) = ë 1 + ë 4 e: Kerg doncg(ë 1 + ë 4) = 0 et donc,
Déterminons ensuite un système d'équations cartésiennes
comme g est linéaire : g(ë i) + g(ê 4) = Ô . de IE. On a :
Comme g(ë 4) =- g(ë i) + Og(ë 2) + Og(ë3). on sait par
théorème que la famille (g(ë i),g(ë 2),g(ë 3)) reste géné-
IE = Vect(û, v)
{aÛ + l3v j (a, j3) E~
2}
ratrice de Img.
De plus comme g(ë 2) = -4g(ë i) - 2g(ë3), on a donc
= {(- a - l3,2o:+2l3,cx,l3) 4
E:~ / (cx,13) E:~2 }
aussi que la famille (g(ë 1),g(ë3)) est génératrice de Img. Pour (x,y,z, t) E: ~ 4. On a:
* Si cette famille est libre alors on aura déterminé une base
de 1mg. C'est bien le cas car les vecteurs g(ë i) et g(ë 3)
ne sont pas colinéaires.
(x,y,z,t) E:!E = 3(a,j3) E: ~ 2 / (S) { - : : ~::
La famille est donc libre. On en conclut que
(g(ë i),g(ë3)) est une base de Img. 13 = t
Donc rg(g) =dim ( Jm g) = 2. Autrement dit: (x, y. z, t) E: IE <=> (S) est compatible
Or :

"O
0
li•+
a) • On pose Ü = (- 1,2,1,0) et v
= (- 1,2,0,1). On a IE =
{-@H ==x 0 y+2x (L2 + 2L1)
C: (S) <=>
::J Vect(ü. v). donc par théorème on sait que IE est un sous- - 13 = z+x (L3 + L1)
0
espace vectoriel de ~ 4 .
lfl
,-i
13 =t
0 • Pour montrer que IF est un sous-espace vectoriel de ~ 4 on
N
peut procéder comme dans l'exercice 5.3. - @] - 13 =x
@
....,
.c Mais on peut aussi chercher une famille génératrice de IF, (Sl <=>
@] =t (L4)
Ol
·;:: ce qui montrera que IF est un sous-espace vectoriel de ~ 4 0 = y+2x (L2)
>- et répondra en plus en partie à la question c) !
0
a.
4. On a :
0 =x+z+ t (L3 + L4)
u Soit (x. y, z, t) E: ~
Donc: (S) est compatible= y+ 2x =0 et x+ z+ t = 0
(x,y,z,t) e:IF = (S) [1+2z- t = 0 Mais on a : (x, y, z, t) e: IE <=> (S) est compatible.

110

www.bibliomath.com
Espaces vectoriels et ap p lications linéaires Chap,itre 5

On obtient donc un système d'équations cartésiennes de


(E:
0 +z +t=O
(S) <==> Œ) - 2z. - 2t = 0
IE = {(x, y, z, t) e: ~4/ 2x + y = Oet x + z + t = O} 1 ~ + t =0

30'.
c) A la question a), nous avons vu que la famille (a, b, 7) est
génératrice de IF. -60'.
O'.
Pour savoir si elle est libre, on résout l'équation vectorielle
(E): xa + y b + zê = 0 d'inconnue (x,y,z) e: ~3. On a: = - 40'.

=0 On obtient donc :

-2y +ŒJ=O G { (3a, -6a, ex, -4cx) e: ~ 4 / ex e: ~}


(E) <==> (S)
0 =0 = {a.(3, - 6,1, - 4)/ cx e:~}
z =O = Vect ((3, - 6~ 1, - 4))
=n
0

(S)= { : = 0 Et comme n -:/, Ô, on a donc trouvé une base de G. Ainsi:


dim(G) = 1.
= 0

La famille est donc libre. On en conclut que (a, b, 7) est


une base de IF. Donc djm(IF) = 3.
iiti
a) On a IE = Vect(a, b, c, d) donc la famille (a, b, 7, d) est
d) Pour déterminer une base d'une intersection d'espaces génératrice de IE. On doit en extraire une base.
vectoriels, une méthode efficace consiste à passer par un
système d'équations cartésiennes.
----
On remarque que : a = b - d = b + 0 ~
c- ~
d, et on a donc
(b, 7 , d) génératrice de IE.
• On a déjà une équation cartésienne de IF:
De même: 2b = ë + d, donc (7, d) est aussi génératrice
IF= {(x,y.z, t) e: ~ 4
/ y+2z- t = o} de IE. Mais comme cette famille est composée de deux vec-
teurs non colinéaires, on a donc une base de IE.
et un système d'équations cartésiennes de IE: On a donc : dim(IE) = 2.

IE = {(x, y, z, t) e: ~ 4/ 2x + y = 0 et x + z + t = 0} b) Pour (x,y,z, t) e: ~ 4 . Comme IE = Vect(ê, d), on a:

~+13 = x
On en déduit très simplement un système d'équations car-
tésiennes de G = IE n IF : 3
(x, y, z, t) e: IE <==> 3(cx, 13) e: ~2 / (S) - cx - l3 = y
{ 70'. + 13 = z
G = {(x,y,z, t) e: ~ 4/ y+2z- t = 0 et 2x+ y = 0 et x+z+t = o}
- 70'. - 13 = t
"O
0
C: Autrement dit : (x, y, z, t) e: IE <==> (S) est compatible
::J
• On peut en déduire une famille génératrice, puis une
0 base, de G. Or:
lfl
,-i
0
Soit (x, y, z, t) e: ~ 4. On a : ~+13 =x
N
@
....,
.c
Ol
ï::::
(x, y. z, t) e: G <==> (S)
{02x + y
H
+t=O
=O
(S) <==>
0

413
= x+ y
-1413 I= 3z - 7 X
= 3t + 7x
(L2 + L1)
(3L3 - 7L1)
(3L4 + 7L1l
>- y+ 2z - t = 0
0
u
a.

(S) <==>
10 + z + t =O
ŒJ-2z-2t = 0 (L2-2L1) (S) <==>
~+13
0
- ~
=X
= x+y
= 3z - 7x
y + 2z - t = 0
0 = z+ t !(L3 +L4) 111

www.bibliomath.com
Chapitre 5 Espaces vectoriels et applications linéaire s

Donc : (S) est compatible<==> x+ y= 0 et z+ t = 0 et:


Mais on a : (x, y, z, t) € IE =
(S) est compatible.
On obtient donc un système d'équations cartésiennes de
IE:
A x ( :] =(2x ~:: 2zl
z - x - y- z
IE = { (x,y,z, t) E ~ 4 / x +y = Oet z + t = o}
c) IF est un sous-espace vectoriel de ~ 4 donc pour montrer ce qui donne, pour tout (x, y, z) € ~3 :
que: IE = Vect (ë, d) c IF, il suffit de vérifier que ë E IF et f(x,y,z) = (x+z,2x-y-2z,-x-y-z)
que d € IF.
Le meilleur moyen de savoir si un vecteur appartient ou Commençons par déterminer une base de Ker f.
non à IF, est d'en déterminer un système d'équations carté- Soit Ü = (x,y,z) € ~ 3 . On a:
siennes.

--- ---
On note : IF = Vect ( u, v , w, -)n . Comme
0 v + w - n on a aussi. : IF = Vect (-
'
v , w, -)
n .
u = w- n =
-
- -- Ü E Ker f <==> f (ü) = Ô
Pour (x, y, z, t) € ~ 4, on a donc : 0 +z =0
=x <==> (S) 2x - y - 2z = 0

(x, y, Z, t) E IF = 3(a, 13, y) E~ 3 / (S) =y 1 -x -y-z =O

2a -y=z et:
-2a +y=t
Autrement dit: (x,y, z , t) €IF<==> (S) est compatible (Sl <==> J0 - y : [fil: :
Or:
=x
1 -Œ] =O

=y
(S) <==>
-III= z-2y (L3 -2L2) (S) = { ; : :

+y = t+2y (L4 + 2L2)


On a donc: Ker/ = {Ô}. Par théorème on sait donc que f est
=x injective sur ~ 3 . De plus f est un endomorphisme, et donc,
par un autre théorème, on sait que f est un automorpihisme
(Sl <==>
=y de~3 .
- III = z - 2y On en déduit que lm f = ~ 3 . Une base d e lm f est donc
0 = z+ t
7,
(7, k) et rg(f) = dim(Imf) = 3.
Donc : (S) est compatible <==> z + t = 0 • On note (7. 7. k) la base canonique de ~3.
Mais on a: (x, y, z, t) €IF<==> (S) est compatible. Ona:
On obtient donc un système d'équations cartésiennes de f(i) f(j) f (k)
u IF:
0

0
C:
::J

lfl
IF = { (x, y, z, t) € ~ 4 / z + t = 0}
!.!_ est alors immédiat que ë E IF (car 7 + (- 7) = 0) et que
B= (=: :J
-1 2
,-i
0 d €IF(carl+(-l) =0).
N
et:
On a donc mo ntré que : IE c IF.
@
....,
.c
Ol
·;::
-
On a:
• On note (7, 7, k) la base canonique de ~3 . 8 x ( :] =
z
(=::~:::]
-x+ y+2z
>-
a.
0
f(i) [(} ) f(k )
u ce qui donne, pour tout (x, y, z) € ~3 :

A= ( : ~l ~2] f(x,y,z) = (-x+ y+2z,-x+ y+2z,-x+ y+2z)


- 1 - 1 - 1
112

www.bibliomath.com
Espaces vectoriels et applications linéaires Chap,itre 5

* Commençons par déterminer une base de Ker f. * Commençons par déterminer une base de Ker f .
Soit Û = (x,y,z) e: ~3 . On a: Soit Û = (x, y, z, t) e: ~4 . On a :

û e: Ker/ = f(û) = ô
Û e: Ker f f (û) =Ô
= -x +y+2z =O <===>

~ - 2y + z - t = 0
~
ex+ 213
= 3(a.~)<R 1 {
2
ex (S) - x
X
+ y + z + 5t = 0
-y +z + t =0
J3 {
On a donc: - 3x + 3y - z + 3t =0

Ker f {(ex + 2J3,ex,J3)/ (ex,J3) e: ~2 } et:


2
{ex.(1, 1,0) +J3.(2,0, 1)/ (ex,J3) e: ~ } ~ - 2y + z - t =0
Vect(~~
(S) <===>
z + 3t =0 ! (2L2 + L1)
=U =V z + 3t = 0 (2L3 - L1)
{
v
Et comme Û et ne sont pas colinéaires, on a donc obtenu z + 3t = 0 (2~ +3L1)
une base de Ker f.
Ainsi: dim (Ker f) 2. = = ex + 213

l'
* Déterminons ensuite une base de Imf.
Par théorème on sait que J'image de f est engendrée par la (Sl = 3(ex,13l e: ~21
y
z
a

(f
famille (t).f(i).f(k) ). = - 313
= 13
0~ pose üi =f(i) = (1, l, 1). Comme f(k) =2w et f(i) =
- w, on a : On a donc :

lmf = Vect(f(i),f(i),f(k) )) = Vect(üi)


Kerf = {(a + 2l3,a, - 3l3,l3)/ (a,13) e: ~2 }
Et w
i: Ô, on a donc une base de lm f. Ajnsi rg(f) {ex.(l, l, 0, 0) + J3.(2, 0, -3, 1) / (ex, 13) e: ~ 2 }
dim(Imf) = 1. Vect ~1. 1~0,0): (2,0, -3, 1))
• On note (ê 1, ê 2, ê 3, ê 4 ) la base canonique de ~ 4 .
=U =V
Ona:
v
Et comme û et ne sont pas colinéaires, on a donc obtenu
une base de Ker f.
Ainsi: djm ( Ker f) 2. =
* Déterminons ensuite une base de lm f.
'O
0

0
C:
::J

lfl
,-i
0
N
et:
2x - 2y+ z - t
- x + y+z+5t
x-y+z+t
l Par théorème on sait que l'image de f est engendrée par la
famille {t(ê i),f(ê 2).f(ê 3),f(ê 4) ).
Reprenons la base de Ker f déterminée précédemment:
(J,l,0,0) = ê1 + ê2 e: Ker/ donc f(ê 1 + ê2) = Ô et donc,
comme f est linéaire : f(ê i) + f (ê 2) = Ô.
@ (2, 0, -3, 1) =2e 1 - 3ë 3 + ë 4 e: Ker f donc
...., -3x+3y-z+3t
.c f(2ë 1-3ë 3 + ë 4) =Ô et donc, comme f est linéaire:
Ol
·;::
ce qui donne, pour tout (x, y, z, t) e: ~4 :
2f(êi)-3f(ê3)+f(ê4)= ô.
>-
0
a. f(x,y, z, t) Comme f(ê 2) =- f(ê i) +Of(ê 3) +Of(ê4), on sait par théo-
u rax-2y+z-~-x+y+z+5~x-y+z+t rème que la famiUe (f(ê 1).f(ê3),f(ê4)) reste génératrice
,-3x+3y-z+3t) delmf.

113

www.bibliomath.com
Chapitre 5 Espaces vectoriels et applications linéaires

De plus comme f(ë 4) = -2f(ë 1) +3f(ë 3), on a donc aussi X 0:


que la famille (f (ë i)J(ë 3)) est génératrice de lmf. y - 20: - 313
De plus les vecteurs f(ë 1) et f(ë 3) ne sont pas colinéaires. (S) <==> 3(o:,13) e: lli2 / z 13
On en conclut que {t(ë i)J(ë3)) est une base de Tm f. -313
Donc rg(f) = dim ( lm f) = 2. u 13
• On note (ë 1, ë 2, ë 3, ë 4, ë s) la base canonique de lli5 et Onadonc:
(71,Ë2,Ë3,Ë4) celledelli4.
Ona:
Ker f {(o:, -2o: -313, 13, -313, 13) / (o:, 13) e: lli2}
frê1l J(ê 2) f(ê3) J(ê 4) f(ê5)
-- {o:.(l, -2, 0, 0, O) + l3.(0, -3, l , -3, 1) / (o:, l3) e: 11i2 }

D=(
2
4
0
0
2
0
0
l 0
-2
3
0
1
0
0
3

-Il3
-1
l-
E1
E2
Ê3
E4
Vect ( (1, -2,0,0,0), (0, -3, l, -3, 1))
--..,.-
=a
--.....--
=b

l
Et comme â et b ne sont pas colinéaires, on a donc obtenu
et: une base de Ker f.
Ainsi : dim ( Ker f) = 2.
x [ 2x+y+3u
Dx ~ = 4x+2y-2z+t+llu * Déterminons ensuite une base de lmf.
3z-3u
t Par théorème on sait que l'image de f est engendrée par la
z -u famille {t(ë i)J(ë 2).f(ë 3),f(ë 4),f(ë s) ).
u

ce qui donne, pour tout (x, y, z, t, u) e: llis : Reprenons la base de Ker f déterminée précédemment :
(1,-2,0,0,0) = ë1 -2ë2 e: Ker/ donc f(ë1 -2ë2) = 0 et
f(x,y,z, t, u) donc, comme f est linéaire : f(ë i)-2f(ë 2) = 0.
(2x+ y +3u,4x+ 2y-2z + t + llu,3z-3u, z - u) (0,-3,1,-3,1) = -3ë2 + ë3-3ë 4 + ës e: Kerf donc
f( - 3ë 2 + ë 3- 3ë 4 + ë s) = 0 et donc, comme f est linéaire:
* Commençons par déterminer une base de Ker f . -3f(ë2)+f(ë3)-3f(ë4)+f(ës)= O.
Soit Ü = (x,y,z, t, u) e: llis. On a:
Comme f(ë i) = 2f(ë 2)+of(ë 3) +Of(ë 4) +of(ës), on sait
par théorème que la famille (t(ë2)J(ë3)J(ë4)J(ës))
Ü e:Kerf = f(ü) = o reste génératrice de lmf.

{ ~+y + 3u = O De plus comme f(ë s) = 3f(ë 2)- f(ë 3) +3f(ë 4), on a donc
(S) 4x + 2y - 2z + t + Jlu = O aussi que la famille ~ = (f (ë 2).f(ë3),f(ë 4)) est généra-
= 3z - 3u = 0 trice de lmf.
'O
0 z - u =0
C: Vérifions que ~ est libre : pour cela, on résout l'équation
::J
0 et : vectorielle (E) : xf(ë 2) + y f(ë 3) + zf(ë 4) = 0 d'inconnue
lfl
,-i
(x,y,z) e: lli3 . On a:
0 lli]+ y +3u = 0
N X =0
@
...., (S) = - 2z + [!] +Su = 0 (L2-2L1)
2x - 2y + z = 0
X = 0
.c
Ol
~ -3u = 0 (E) <==> (S)
3y =0
<==> { y = O
{
·;:: z -u =0 z 0
>-
a. =0 y
0 lli]+ y +3u = 0 ce qui prouve que~ est libre.
u
- 2z +[!] + Su = 0 (L2 -2L1)
(Sl = Donc~ est une base de lm f et rg(f) = dim ( lm f) = 3.
~ - 3u = O
11 4 0 =0 (3L4 - L3)

www.bibliomath.com
Espaces vectoriels et applications linéaires Chap,itre 5

-
a) • Déterminons le rang de la famille ~ :
b) • Déterminons le rang de la famille 8l! :

([Q
~·l
0

rg(~)
,g~ 0
0
~.]
2
rg(~)
rg :
-1

rg :
F
1
[TI (L2 + L1)
rg
([Q
:
0
[TI
~Il (L1 - L3)

l
-1
~l l (L3 - L1)

,g(~ 0

= 3
([Q
rg : [TI
0 ~l (L3 +L2 )

=
3

=
[TI
0 [TI

On a rg(8ll) Card(@) dim (~ 3), donc par théorème on


- 11
(L3 - L2)

On a rg(@) = Card(8ll) = dim (~ 3 ), donc par théorème on


sait que 8l! est une base de ~ 3 .
sait que 8l! est une base de ~3.
• On note (7, j, k) la base canonique de ~ 3 . • On note (7, j, k) la base canonique de ~ 3 .

Ona: Ona:
f(i ) f(j) f(k) g( i) g(J) g(Î)

et:
A= (~ ~3 ~2 l et:
B= (~2 ~3 ~21
3 -2 -2

Ax ( : ] =(x-+3;_+2: ]
Bx ( : ] = (~:: : : : : :]
z 4y+3z z 3x-2y-2z
ce qui donne, pour tout (x, y, z) E ~3 :
ce qui donne, pom tout (x, y, z) e: ~3 :
f(x,y,z) = (x + 4y + 2z, - 3y - 2z,4y +3z)
g(x,y,z) =(3x - 3y - 2z, - 2x + y+2z,3x - 2y - 2z)
On doit calculer f(û1).f(û2).f(Ü3) en fonction de
Û 1, Û2 et Û3.
On doit calculer g(v\),g(ïi2),g(v3) en fonction de
Ona: ïi 1, ïi2 et v3.
'O
0 f(ûi) /(1, - 1,1) = (- 1,1, - 1) =-Û1 Ona:
C:
::J f(û2) f(l,0,0) = (l,0,0) = Û2
0
lfl t(û3) = Jco,- 1,2) = co, - 1,2) = U 3
g(ïi i) = g(l,O,l) = (l,O,l) = zÎ1
,-i
0
g(ïi2) = g(O,l, - l) = (- l ,- l,O) =-ïi1 - ïi.2
N On obtient donc: g(ïi3) g(l,-1,1) = (4,-1,3) =?
@
...., f(Ûi) f(Û2) f(Û3)
.c --
l
0 0 u1 Déterminons les formules de changement de base entre la
Ol ( -1
·;:: l'vlat,91 (/) = ~ 1 0 u2 base canonique et 8l! :
>-
a. 0 1 U3
0
-- -i -
=l-
u

l
V1 +k ïi2+ïi3
-- -V -V3
On remarque que cette matrice est diagonale. v2
-V3
j - k
-i - -j + k - j
k
1-
-v1-v2-v3
- - 115
Chapitre 5 Espaces vectoriels et applications linéaires

On obtient donc: b) Posons:

a u
b f(ü)
g(v 1> g(v 2> C = f 2 (û)
-1
Mat,.(g) = ( : - 1 On doit calculer f(a).f(b)J(ê) en fonction de â, b et
0 ê.

f(â) = f(û) = b
On remarque que cette matrice est triangulaire supérieure.

-
f(b) J2(û) =ê
f(ê) J3 (û) =0
a) On a supposé que _r 2 fc 0, on sait donc qu'il existe au moins
On obtient donc :
un vecteur û e: 11<3 tel que f 2 (û) fc Ô .
Montrons que ce vecteur û est solution du problème, c'est- J<a> f(b) t<ê>
a

il
à-dire que la famille (ü .f(û)J 2 (û)) est libre. 0
Mat,o({I =( : 0 b
Pour cela, on résout l'équation vectorielle (E) xû + 1 C
yf(û) +zf2 (û) = Ô d'inconnue (x,y,z) e: 11<3 .

-
Comme on ne connaît pas les composantes du vecteur û,
on ne peut pas utiliser une résolution de système linéaire.
Il faut donc trouver une autre méthode.
a) On a:
2
Composons l'équation (E) par / = f of:

f2(xü + yJ(û) +zf2(û)) =f(Ô) 2


-2 - 1 0 7 -6 -3
et corrune f est linéaire :

Or on a supposé que / 3 = 0 et donc / 4 = O. Ceci nous


-2
0
0
0
0
0
2
0
6
0
-6
3
0
-2
0
~]
donne .f3 (û) = / 4 (û) = Ô. En réinjectant dans l'équation Donc : A2 = 3A - 2l4.
précédente, on a: Par unicité de la représentation matricielle, on a bien:
x/2(û) = Ô /
2
-3/ + 2Id11t1 =02 (11t1)·
et donc I x=O ~ puisqu'on a choisit û de telle sorte que
b) On a donc: f O :f (t-3IdR4) = :f (t-3Idlli4) 0 f = IdR4 .
u J2 (ü)tô.
0 Par théorème on sait donc que J est un automorphisme de
C:
::J (E) devient donc y f (û) + z/2 (ü) = Ô. En la composant r
llll 4 et que: 1 = -,i1 (f-3Idw ).
0 par .f, on obtient:
lfl Ona:
,-i
0
N yf2(û) +zf 3 (û)= Ô 3x - :y - z l
--...,._, --...,._,
@
...., io =o Ax
2x - 2y
.c
Ol
ï::::
>- et donc I y=O1· y +2t
a.
0
u (E) nous donne alors: zf 2(û) = Ô, et donc ! z=O ~ ce qui donne, pour tout (x, y, z, t) e: l!ll4 :

En conclusion, on a donc (û J(û), f 2 (û)) libre. f(x,y,z, t) = (3x-2y- z,y,2x-2y, y+ 2t)

116
Espaces vectoriels et applications linéaires Chap,itre 5

On en déduit que, pour tout (x, y, z, t) e: l!ll 4 : On en déduit, d'après (i i) que :

_r 1 (x,y,z, t) - 21 (f(x, y, z, t) - (3x,3y,3z,3t)) rlne:N, bn+I =2-zn+l

- 1 (-2y-z.-2y,2x-2y-3z,y- t) donc (en posant n' = n + 1) :


2
c) On va démontrer par récurrence que, pour tout n e: l"\J, il rln~l. bn=2-2n
existe deux nombres réels an et bn tels que: _rn = a,i.f +
et on vérifie facilement que cette formule est aussi valable
bnl~1.
pour n = O.
• C'est vrai pour n = 0 en posant ao = 0 et ho = 1.
Finalement, on a:
• Soit ne: l"\J tel que la propriété soit vraie au rang n.
Ona:

= .f o .f n
Ajnsi, pour tout ne: l"\J et tout (x, y, z, t) e: llll 4 :

= .f o (a,i.f + bnI~4)
an/2 + bn.f
f 11 (x, y, z, t)
flinéaire (c2 11 + 1 - l)x - c2n+l - 2)y - (2 11 - l)z,y.
an (3 .f - 2IdlR4) + bn.f
=
(2n+l - 2)x- (21!+ 1 - 2)y + (2- 211 )z, (2n - l)y + zn t)
(3an + bn) .f - 2an IdlR4

Si on pose: a 11 +1 = 3an + bn et b,a 1 = - 2an, on a bien

fn+I = an+lf + bn+1l~4


• D'après le principe de récurrence, il existe donc deux
suites réelles (an)ne:N et (bn)ne:N telles que:
-
a) • Soit x e: Ker.f fixé quelconque. On veut montrer que
Xe: Ker(go /).

On a x e: Ker f. donc f (x) = Ô. On compose par g :

g(f(x)) = g(Ô) = ô car g linéaire


Elles sont définies par les relations :
Ainsi: go .f(x) = Ô et donc x e: Ker(go .f).
J ao =0 et ri ne: l"\J, an+ 1 =3an + bn (i) Ceci prouve que: Ker/ c Ker(go f).
l bo = letrln e:N,bn+1 =-2an (i)
• Pour la seconde inclusion à démontrer, procédons autre-
ment, bien que la même méthode marche encore ici.
d) Combinons les relations (i) et (ii):
Par définition: 1mg = g(IK n) et Im(g o .f) = (go .f)(O< n) =
ri ne: N, an+2 =3a 11+1 + hn+ 1 =3an+ 1 - 2an g(.f (IK n)).
Ainsi (an) ne:N est une suite récurrente linéaire d'ordre 2. Mais f est une application de 1K II dans 1K 11 donc: f (IK 11 ) c
IKn.
u Son équation caractéristique est :
0
C: =
X2 - 3X + 2 O. En composant par g: g(.f (IK n)) c g(IK n).
::J
0 Son discriminant vaut 6. = 1. Ceci prouve que: lm(g o .f) c 1mg.
lfl 6. > 0 donc cette équation a deux racines réelles qui sont 1
,-i
0
b) Procédons par double-inclusion.
et 2.
@] Soit x e: Ker g n Tm f
N
On en déduit qu'il existe (À,µ) e: l!ll 2 tel que : fixé quelconque.
@
....,
.c
Ol
On veut montrer que x e: !( Ker (go f) ), c'est-à-dire qu'il
·;:: existez e: Ker(go f) tel que x = f(z).
>-
a. Or, O= ao = À+µ et 1 = 3ao + bo = a 1 = À+ 2µ. On en déduit
0
u que À= - 1 etµ = 1. Donc: On axe: Kergnlmf, donc x
e: Kerg et e: Jmf. Ceci x
nous donne que g(x) = Ô et qu'il existe y e: 1K n tel que
rlne:N, an=-l+2n x =f(y) .
117
Chapitre 5 Espaces vectoriels et applications linéaires

Posons alors z = y et vérifions que ce vecteur est celui


que nous cherchions.
* On a : go f(z) = g(t(z)) = g(f(y)) = g(x) = O. Donc
z e: Ker(go f).
-
a) On note~= (7. j, k) la base canonique de lli 3 .
Ona:

* t(z) = tG) = x.
Comme expliqué plus haut, ceci prouve que: x e:
!( Ker(go f) ).
et donc, pour tout (x, y, z) e: 11i3 :
0 Soit x e: t( Ker(go f)) fixé quelconque. f(x,y,z) = (x- y-z,x+ y+ z, -x)
On veut montrer que x e: Ker g n lm/, c'est-à-dire que
g(x) = ô et qu'il existez e: 1K n tel que x = f(z). • Base de Ker f.

On axe: f( Ker(go f) ). donc il existe y e: Ker(go f) tel que Soit Ü = (x,y,z) e: 11i3 . On a:
x =f(y).
* On a : g(x) = g(t(y)) = Ô car y e: Ker(go f). Ü e: Ker/ <=> f(û) = Ô
* Posons _.z = _.y. On a bien
. _.
z e: 1K n· et : f (_.)
z = f (_.)
y = x. -+
x - y - (IJ = O
Comme expliqué plus haut, cela prouve que : x e: Ker g n = (S) { - : +y +z : :
lm/.
Par double-inclusion, on a donc:
et :
Ker gn Imf =!( Ker(go f)) X -y-[TI=O

c) • Montrons que Ker g est stable par f, c'est-à -dire que:


(S) <=> 2x =o
{
rlx e: Ker g, f (x) e: Ker g. -0 =0
Soit x e: Ker g fixé quelconque. On a :

g(f(x)) = gof(x) = f og(x) = t(g(x))


(SJ= 10-y-[!j::
0
car f et g commutent.
ex
Mais x e: Ker g, donc g(x) =Ô. On obtient donc : - ex

g(f(x)) =f(Ô) = o car clinéaire On a donc:

Ker/ {(0, ex, - ex)/ ex e: ~}


Ainsi : f (x) e: Ker g.
{ex.(0,1,-1)/ exe:lli}
Ceci prouve que Ker g est stable par f.
"O Vect@:.l:~
0 • Montrons que 1mg est stable par f, c'est-à-dire que :
C:

0
::J
'rlx e: 1mg, f(x) e: 1mg. =J -k
lfl
,-i
Soit x e: Jmg fixé quelconque. Il existe donc y e: IK 11
tel
Et comme j - k -:/, Ô, on a donc obtenu une base de Ker f.
0 que: x g(y).
=
N Ainsi: dim (Ker f) = l, Ker f est une droite vectorielle.
@ Alors:
...., • Base de lm/.
.c f(x) = t(g(y)) = f og(y) = gof(y) = g(f(y))
Ol
·;::
On sait que Imf est engendrée par la famille f(~) =
>- (t(t).f(i).f (k) ).
0
a. Posons z =
f (y). On a alors: f (x) = g(y). et donc
u f(x) e: Jmg. Mais on remarque que: f(i) = f(k). Donc Imf est en-
Ceci prouve que 1mg est stable par f. gendrée par (f(i).f(i) ).

118
Espaces vectorie ls e t ap p lications li néaires Chap,itre 5

Comme f (i) et J(J) ne sont pas colinéaires, on a donc


trouvé une base de lm/.
Donc rg(f} = dim ( lm f) = 2, lm f est un plan vectoriel.

b) Soit o: e: ~- On a rg(/ -o:IdR3) = rg(A- o:I3) et:


ŒJ 0
rg(A - o:l3) = rg ~
[
[TI
0 1
- 1o:

o:(o:- l)
l
1
=3

rg(A - o:13) En conclusion :

-1 si o: = 0 ou o: = 1
r· - a ,g(/ - •ldo,J ={ :
rg
-1
1 1 - o:
0
~·1
- (X
si o: ;: 0 et o: ;: 1

c) Remarquons que, d'après la question a) : Ker f = D1 =

rg (Œ1-o:
1
0
1 - o:
-1
- • ] (L3)
1
-1 (L1)
Vect(û 1) et lm/= Vect(û2, Ü3) en posant Û3 = (-1, 1,0).
• Analyse ( = conditions n écessaires). Soit D une droite
vectorielle stable par f (on suppose qu'il en existe au
moins une).

rg (Π0
0
0
1 - o:
-1
1- a
-o:

o:(o:-1)-1
] (L2+L1)
(L3 + (1 - cx)L1)
Comme D est une droite vectorielle, on a dim(D)
D = Vect(Û) avec û ;: 0.
D stable par f signifie que: Vx e: D, f (x) e: D. En particu-
= 1 et

lier on doit avoir f(û) e: O, ce qui donne qu'il existe o: e: ~


tel que J(û) = o:û .
• Si o: =1.
Mais f (û) = o:Û donne (f- cxldR3 )(û) = 0 . Co mme Û
;:
O. on en déduit que f n'est pas un automorphisme de ~ 3,
0
0 -li
0 =2
et donc que rg(f-o:ldR3) < 3.
La question b), nous donne alors que o: = 0 ou o: = l , c'est-
à-dire que f (û) = 0 ouf (û) = Û
DJ -1
* Si J(û) = 0, alors û e: Ker/ = Vect(û1) = D1. Donc
• Si o: ;: 1. Oc O 1 et, par égalité des dimensions: O = O l ·
* Si f(û) = Û alors Û e: lmf = Vect(û2, Ü3).
Il existe donc (À,µ) e: ~2 tel que û = Àû 2 + µ û 3. Alors, par

l
rg(A- o:I3) linéarité de f:

F
0
-a f(û) = .\f(Ü 2) + µ /(Ü3) = ÀÛ 2 + µ(-2,0, 1)
rg ~ [TI ( 1~cx L2) '-,.-'

-1 o:(o:-\) -1 = u2

"O
0
rg
F ~ [TI
0

0
-a
1
o:(o: - 1)
l (L3 + L2)
Mais alors f (û) = û donne :
~ + µ( - 2,0,1) =~+µÛ3

C: donc:
::J
0
µ(-2,0,1) = µ(-1,1,0) d'où µ=0
lfl
,-i Ainsi û = ÀÛ2 e:Vect(û2) = D2.
0
N
• • Si o: =O. On a donc Oc 02, et par égalité des dimensions: O = 02.
@ On a donc prouvé que si D est une droite vectorielle stable
....,

:l
par f alors D = D1 ou D = D2 .
.c
Ol
ï::::
(qJ 0
[TI
>- rg(A) = rg l ~ • Synthèse (=conditio ns s uffis antes). Vérifions que les
a. =2 droites vectorielles D1 et D2 sont stables par f.
0 0
u
* Pour tout À e: ~: f(A. Û i) = A.f(Ü i) = 0 donc f(A. Û i) e:
• • Si o: ! {O, 1}. Di.

11 9
Chapitre 5 Espaces vectoriels et applications linéaires

Autrement dit: pomtout x e: D1, on a f(x) e: D 1. Donc D 1 b) Soit x e: 1K n fixé quelconque.


est stable par f · On a : x = IdlK" (x) par définition de l'application identité.
* Pour tout À E Ili: /('A. û2) = 'A.J(û2) = ÀÛ2 donc Matriciellement, cette relation devient:
f('A. Ü2) E D2 .
Autrement dit: pour tout XE D2, on af(x) E D2. Donc D2 r~f(x) = Mat(Id0< ,@1,@2) X 1~t(x) 11

est stable par/.


ou encore :
• Conclusion. D 1 et D2 sont les seules droites vectorielles Mat(x) = p X Mat(x)
stables par f. .':ll2 .':ll1

-
a) • Soit f l'unique application linéaire de 1K n dans 1K n défi-
nie parMat(/, @1,@2) = P.
Onnote@1 = (û1, .. . , Ün) et@2 = (v 1, ... , v n), On a :
c) Soit x e: 1K n fixé quelconque. On pose y = f (x) e: 1K P.
Si on munit O<n de@ 1 et n<P de '€1, la relation
s'écrit matriciellement :

Mat(y) = Mat(f,@i, 'Ci) X Mat(x)


'6'1 '--,,--' {i}J\
y = f(x)

u1
Pll
- -
Un
-VJ et donc : Y= AX.
--...-,
=Y
=A --...-,
=X

p Mat(@1) = : De même, si on munit K11 de @2 et 0<P de '€2, la relation


{i}J2 ( •
Pnl -Vn
y =f(x) s'écrit matriciellement:
Mat(y) = Mat (f,@2, '€2) x Mat(x)
f(Û1) '6'2 '--,,--' {i}Jz

Pu -
v1
--...-,
=Y'
=B --...-,
=X'
Mat(/,@1, @2) = :
(
Pnl -
V n.
et donc : Y'= BX'.
Mais on a vu à la question précédente que : Y' = QY et
X' = PX.
On a donc: On a donc: QY = BPX et donc Y= Q- 1BPX.
Autrement dit pour tout x e: 1K n :
Mat(/(x)) = Q- l BP x Mat(x)
Et donc, comme une application linéaire est entièrement 'é'1 {i}JI

déterminée par l'image d'une base: f = ldK" ·


et donc, par unicité de la représentation matricielle :
Ainsi: P = Mat(@1) = Mat(ldl)(11,@1,@2).
{i}Jz

• De même on montre que:


Q = Mat('C1) = Mat (Idl)(P , '€1, '€2). On a donc démontré la formule :
'G'2
'O • Comme ldl)( 11 et l<ll)(p sont bijectives, on sait donc que P
0
C:
::J
et Q sont inversibles.
0 d) On a: Mat(/) = Mat(/,@1,@1) et de même Mat(/) =
lfl
De plus, ldl)(" est égale à sa bijection réciproque, donc: {i}J\ {i}J2
,-i Mat(/,@2, @2).
0
N p- l (Mat(ldw1,@1,@2)t 1 D'après la question précédente :
@
....,
.c
Tv!at( (Tdl)(n
1
r
,@2, @1)
Ol = Mat(Idwr ,@2,@i)
·;:: avec P = Q = Mat(@1).
>-
a. Mat(@2) {i}Jz
0 {i}J\
On obtient donc :
u
De même: o-1 = Mat(Idll(P,'€2,'Ci) = Mat('C2).
'«1
Mat(/) = p - l x Mat(/) x P
{i}J1 {i}Jz

120
Es pa ces ve ctoriels et ap p licatio ns linéaire s Chap,itre 5

-
a) • Soit x e: IE.
Comme IF est u n sous-espace vectoriel de IKn, on a Ô e: IF.
On a donc:
Si on montre que !Y- est génératrice de IE + o=, on saura que:

dim(IE + IF) s Card(~) = dim(IE) + dim(IF)

ce qui est Je résultat demandé.


Montrons donc que !Y- est génératrice de IE + o=.
• Co mme IE + IF contient IE et o=, on a !Y- c IE + o= et donc
Vect(/Y-) c IE + IF.
On a donc x e: IE + IF.
• Soit x e:IE+o=.
Ceci prouve que IE c IE +IF.Avec le même type d'argument,
on peut montrer que IF c IE + o=. On sait qu'il exist e (ê, f) e: IE x IF tel que x =ë + f.
• Vérifions ensuite que IE + o= est u n so us-espace vectoriel Alors ë e: IE et BBE = (ë 1, . . . , ë p) est une base de IE, il existe
delKn. donc (cx1, ... ,cxp) e: !KP tel que:

* Comme IE et IF sont deux so us-espaces vect oriels de 1K n, p


on Ô e: IE et Ô e: IF. Donc : e = 'L ex; -+
-+ ei
i= l
0 =
~
0 + 0
~
De même, il existe donc Cl31, ... ,l3q) e: !Kq tel que :
=ëEIE =fEF

ce qui prouve que Ô e: IE + IF.

* Soient À e: 1K et (x, y) e: (IE + IF) 2 . On p osez= À x +y .


On a donc:
Comme x e: IE + o= on sait qu'il existe (ê x, f x) e: IE x IF t el
que: x = ë x + 7x· De même, y e: IE + IF donc il existe
(ê y, 7y) e: IE x IF tel que: y = ë y+ 7y·
On a donc :
ce qui prouve que x e: Vect(~).
z = Ax + r Ainsi: IE + IF c Vect(~).
= A(ë x+ f x)+(ë y+ f y) • Par double-inclusion, on a donc IE + IF = Vect(~).
(A
..._,__, ..._,__,
7
ë x + ë y)+ (A/ x + y) La famille~ est donc génératrice de IE+IF, et donc, comme
dit p lus haut :
=ë z =fz
êz + fz dim(IE + IF) s Card(/Y-) = dim (IE) + dim(IF)

=
Mais ë x e: IE et ë y e: IE, donc ë z À ë x + ë y e: IE, puisque c) Procédons par double-implication.
IE est un sous-espace vector iel de 1K n. Pour les mêmes rai-
==> 1 On suppose que IE et IF sont en somme direct e. On
u
7z = Af x + Î y e: IF.
1
0 sons
C: doit montrer que IEnlF = {O}. Pour cela on procède par
0
::J
Ceci prouve que z = Àx + y e: IE + IF. double-inclusion.
lfl
,-i On a donc démontré que IE est un sous-espace vectoriel de @ Comme IE et o= so nt des sous-espaces vectoriels de 1K n,
0 1K n contenant IE et IF. on sait que IE n IF est aussi un sous-espace vectoriel de 1K n,
N
et donc :
@
...., b) Soient BBE = (ë 1, . . . , ë p) une base de IE et &Bp = {Ô ) c!Eno=
.c (f 1, . . . , f q) une base de IF.
Ol
ï::::
>- On considère la famille ~ BBE u &Bp
BSoit x e: IEnlF.
a.
0
u ( e 1, ••• , e p,
-+ -+ -+f , ... , -+f q.
1
) Remarquons que: On a donc x e: IE et x e: o=. Remarquons que:
Card(~) =Card(BBEl + Card(@p) =p + q =dim(IE) + dim(IF) x = x+ô=ô+x
~~~~
EIE EF EIE Ef
121
Chapitre 5 Espaces vectoriels et applications linéaires

Mais cette décomposition doit se faire de manière unique .....,. p ~ -


e = [ ex; e i = 0 nous donne cx1 = · · · = exp = 0 car la fa-
(car on a supposé IE et IF en somme directe), on a donc: i= I
x= o. mille @E = (ê 1, ... , ë p) est libre (c'est une base).
Ceciprouveque!EnlFc {OJ. - q - ~
De même f = [ PJ .f J = 0 nous donne 131 = · · · = tiq = O.
Par double-inclusion, on a donc: IE nif = {OJ. j =l

1 <= 1On suppose que IE n IF = {0 J. On doit montrer que IE On a donc (E) <==> cx1 = · · · = exp = 131 = · · · = 13q = O.
et IF sont en somme directe, c'est-à-dire que tout E IE + IF x Ceci prouve que la famille§ est libre.
s'écrit de manière unique x = ë + 7 avec (ê, 7> E IE x IF. § est donc une base de IE EBIF puisqu'elle est libre et généra-
x
Soit E IE + IF fixé quelconque. Supposons que = ë 1+ x trice de IE EB IF.
71 = e2 + 72 avec (e1, 71) E IE xlf et (e2, 72) E !Ex IF. On 1<= 1 On suppose que la juxtaposition d'une base de IE et
doit vérifier que ë 1 = ë 2 et 71 = 72. d'une base de IF donne une base de IE + IF. On doit montrer
que IE et IF sont en somme directe. D'après c) il suffit de vé-
On a : ë 1+ 7 1 = ë 2 + 7 2, et donc: ê 1- ë 7 l· 2 =7 2-
rifier que lE n IF = {O}.
Comme IE et IF sont des sous-espaces vectoriels, on a : ë 1 -
@ Comme IE et IF so nt des sous-espaces vectoriels de 1K n,
ë 2 E IE et 7 2 - 71 E IF. Donc si on pose z = ë 1- ë 2 = ona{O}clE n lF.
7 2 - 71 alors z E IEnlF = {0}, c'est-à-dire: z = O.
@] Soit x E IEn lF. On se donne @E = (ê 1, . . . , ê p} une base
On a donc ë 1 = ë 2 et 71 = 72· La décomposition de x de IE et @p = (7 1, ... , f q} une base de IF. Par hypothèse on
est donc unique.
sait que § = @Eu &lJF = (ê 1, ... , ê p, f 1, ... , f q} est une
Ceci prouve que IE et IF sont en somme directe. base de IE + IF.
Par double-implication, on a donc : Comme x E IE on sait qu'il existe (a1, ... ,ap) e: !KP tel que:
IE et IF en somme directe = IE n IF = {0 l -
x = [p __.
a; e;
i =l
d) • Procédons par double-implication.
1 ==> 1 Onsuppose que IE et IF sont en somme directe. On car @E est une base de IE.
doit montrer que si @E = (ê 1 , ... , ë p) est une base de IE De même x E IF nous donne qu'il existe (131, .. . ,l3q) E !Kq
et §6F = (Î 1, ... , 7q) une base de IF, alors§ = §6E u§6F = tel que:

(ê 1, ... , ë p,71, ... , 7 q) est une base de IEEBIF.


On a déjà prouvé au b) que§ est génératrice de IE EB IF. On
va donc vérifier qu'elle est libre. Pour cela on résout l'équa- Ainsi on a:
tion vectorielle (E) :

Mais la famille§ est libre (c'est une base) donc


'O = .. · = CXp = 131 = .. · = l3q = 0.
CX1
0
C:
::J
d'inconnue (cx1, .. ,,cxp,l31, ... ,l3q) e:IKp+q. On a donc x = O. Ceci prouve que IE nif c {O }.
0
lfl
,-i
0
On a : (E) = [ a; ê; =- L 13 f
p

i=l
q

j= l
j
-
j
Par double-inclusion on a donc: !En IF = {O }.
Par double-implication, on a donc:
N
@ Comme IE est un sous-espace vectoriel de IKn, on a ê =
...., p - q - -
IE et IF en somme directe
.c
Ol
·;::
[ a; ê;
i=I
E IE. De même f = [
j=l
13 j .f j E IF. Alors ê =- .f = la juxtaposition d'une base de IE et d'une base de IF
>-
0
a. nous donne que ê E IEnlF et 7 E [nif. donne une base de IE + IF
u Mais comme on a supposé que IE et IF sont en so mme di- • Soient @E une base de IE et @p une base de IF. On a donc
recte, onsaitd'aprèslec) quelEnlF = {OJ.Ainsi ê = 7=O. Card(@E) = dim(IE) et Card(@p) = dim(IF).

122
Espaces vectoriels et applications linéaires Ch ap,itre 5

D'après ce qui précède on sait que si IE et IF sont en somme b) Raisonnons par l'aburde: supposons que Vp e: I\J,
directe alors § = @E u@p est une base de IE E& IF, et donc: Ker(.f P) Ç Ker(.f P+l).
Card(g;) = dim(IE œIF).
Ona:
Mais: Card(§) = Card(@E) + Card(@p). On a donc:

dim(IE œIF) = dim(IE) + dim(IF) {Ô} = Ker(/0 ) Ç Ker f Ç Ker(/2) Ç · · · Ç Ker(/P) Ç · · ·


e) Procédons par double-implication. et donc, en passant aux dimensions:
1 ==> 1On suppose que IE et IF sont en somme directe. On a
vu au d) que: 0 < dim(Ker f) < dim(Ker(/2)) < · ·· < dim(Ker(f P)) < ·· ·
dim(IE + IF) = dim(IE) + dim(IF)
Ainsi dim(Ker f) ::: l, dim(Ker / 2 )::: 2 etc ...Par une récur-
1 = 1On suppose que: dim(IE + IF) = dim(IE) + dim(IF). On rence immédiate, on peut montrer que:
doit montrer que IE et IF sont en somme directe. Pour cela
iJ suffit de montrer, d'après d), que la juxtaposition d'une
Vp e: I\J, dim(Ker f P)::: p
base de IE et d'une base de IF donne une base de IE + IF.
Soient @E une base de IE et @p une base de IF. On pose
§ =@EU@p. Mais alors pour p = n + 1 : dim(Ker fn+ 1) ::: n + 1. Ceci est
impossible car fn+l est un endomorphisme de IKn et doit
On a vu au b) que§ est génératrice de IE + IF. De plus: donc vérifier dim(Ker f) :,; n. On aboutit donc à une contra-
Card(§) = Card(@E) + Card(@p) diction.
dim(IE) + dim(IF) On a démontré qu'il existe un entier Po pour lequel:
= dim(IE + IF) Ker(/Po+l) = Ker(.f Po).

on sait donc, par théorème que la famille § est une base c) Procédons par récurrence.
de IE + IF.
• Pour q = Po, on a évidemment: Ker(fq) = Ker(.f PO).
Donc, d'après d), IE et IF sont en somme directe.
Par double-implication, on a donc: • Supposons que Ker(r) = Ker(/ Po) pour un q::: Po fixé.

IE et IF en somme directe <=> dim(IE + IF) = dim(IE) + dim(IF) * D'après a), on a: Ker(fq) c Ker(/q+l).

-
a) Soit p e: I\J fixé quelconque.
• Soit x e: ker(fP).
Par hypothèse on a donc f P(x) = 0, d'où:
* Soit x e: Ker(fq+ 1). On a alors: r+ 1(x) = r
Ô, ce qui donne que f (x) e: Ker(/q)_
Mais on a supposé que Ker(fq) = Ker(f/lO), donc on peut
dire que f(x) e: Ker(JJlO), cest-à-dire que JJl0+ 1 (x) =
f Po (f (x)) = O.
of (x) =

t p+i (x) = t o t P(x) = t(ô) = ô On a donc x


e: Ker(/ Po+ 1). Or par définition d!e Po:
car f est linéaire. Ker(/Po) = Ker(/Po+ l ), donc on a: x
e: Ker(/Po). Et par
"O
0
C: On a donc: x e: ker(f p+l). hypothèse de récurrence: Ker(/ PO) = Ker(fq), donc: e: x
::J Ker(ftf).
0 •f est un endomorphisme de 1K n, donc: f (IK n) c 1K 11 •
lfl
,-i
Ceci prouve que Ker(ftf+ 1) c Ker(fq).
0
On a donc:
N Par double-inclusion, on a donc: Ker(ftf+ 1) = Ker(ftT).
@ tp+ l (IK n) = f P (f (IK n)) c fP (IK n)
....__., Mais on a supposé que Ker(/q) = Ker(/ P0 ). Donc:
...., ...._,.-.,
.c =lm(/ p+ IJ =lm(/ P) Ker(/Cf+l) = Ker(/ PO). La propriété est donc vraie au rang
Ol
·;:: p+l.
>-
a. c'est-à-dire: lm(/ p+l) c lm(/ P) .
0 • D'après le principe de récurrence, on a donc:
u On a donc prouvé que, pour tout p e: I\J :

Ker(/ P) c Ker(f p+ 1) et lm(f p+ 1) c lm(/ Pj

123
Chapitre 5 Espaces vectoriels et applications linéaires

d) Procédons par double-inclusion. En combinant ces deux égalités : f Po(! Po (z)) =Ô c'est-à-
@] On sait que f Po est un endomorphisme de 1K 11
• et donc dire: f 2Po(z) = O.
que Ker(/ P0 ) et Im(f P0 ) sont des sous-espaces vectoriels
de !Kn. Ceci nous donne que: z e: Ker(f2PO) = Ker(fPo) d'après c)
On a donc : {O} c Ker(/ Po) nlm(f Po). avec q= 2po.
~ Soit
o.
x e: Ker(/Po) n Tm(/Po). On doit montrer que x = Donc f Po (-) - --
z = O, c'est-à-dire x = O.

Par hypothèse x e: Ker(/ Po) et x e: Im(f Po). Donc: Par double-inclusion, on a donc:
f Po (x) = ô et il existez E 1K n tel que X =f Po (z). Ker(/PO) nlm(f PO) = {Ô}.

"O
0
C:
::J
0
lfl
,-i
0
N
@
....,
.c
Ol
ï::::
>-
a.
0
u

124
CHAPITRE 6
Fonctions usuelles, polynônies en
une indéterniinée et continuité des
fonctions nuniériques

'Tlièmes a6ordés dans {es e~ercices


- Fonctions usuelles
- Limite d'une fonction en un point dei
- Comparaison de fonctions
- Continuité d'une fonction numérique
- Polynômes en une indéterminée

Points essentiefs du cours pour {a réso{ution


des e~ercices
- Propriétés des fonctions usuelles
- Calculs de limites
- Calculs d'équivalents
u
0 - Continuité en un point, prolongement par continuité
C:
::J
0 - Théorème des valeurs intermédiaires
lfl
,-i - Théorème de continuité sur un segment
0
N - Théorème de la bijection
@
...., - Fonctions circulaires réciproques
.c
Ol
·;:: - Degré, coefficients et racines d'un polynôme
>-
a.
0
u

125
Chapitre6 Fonctions, polynômes, continuité

Les méthodes à retenir

En encadrant la fonction, le théorème des gendarmes prouve


l'existence et permet de calculer la limite. Cette méthode est très
efficace avec les fonctions cosinus, sinus et partie entière.
<---+ Exercices 6.1, 6.6, 6.14, 6.19 et 6.21

- A l'aide des limites connues sur les fonctions usuelles et des ré-
sultats de croissances comparées, on peut prouver l'existence et
calculer une limite, sauf en cas de forme indéterminée.
<---+ Exercices 6.1, 6.2, 6.3, 6.5, 6.8, 6.9, 6.12, 6.14, 6.20 et 6.21

En cas de forme indéterminée, on peut multiplier les différences


de radicaux par leur quantité conjuguée, afin de simplifier l'ex-
Pour étudier une limite de fonction pression.
<---+ Exercice 6.3

- Toujours en cas de forme indéterminée, on peut aussi essayer de


trouver un équivalent simple de la fonction (car deux fonctions
équivalentes ont la même limite).
<---+ Exercices 6.5, 6.8, 6.9 et 6.21

- Pour montrer que lim f(x) n'existe pas, on peut trouver deux
x-xo
suites (un) et (vn) convergentes vers xo telles que: lim
n-+oo
f (un) -:J.
lim f(vn).
n-+oo
<---+ Exercices 6.14 et 6.20

On fait apparaître les équivalents usuels.


<---+ Exercices 6.5 et 6.8

- Si on cherche un équivalent quand x - x0 , on se ramène en O en


posant x = Xo + h.
<---+ Exercice 6.8
u
0
C: On utilise les opérations sur les équivalents. Attention : en général
::J
0 Pour déterminer l'équivalent d'une on ne peut ni les composer, ni les additionner.
lfl
,-i fonction <---+ Exercices 6.5 et 6.8
0
N
@ - Si on se retrouve devant une situation où il faudrait composer un
...., équivalent, on revient à la définition pour voir si le résultat est vrai
.c
Ol
·;:: (le quotient doit avoir pour limite 1). Dans le cas d'une compo-
>-
a. sition par ln, on calcule le quotient moins l, qui doit avoir pour
0 limite O.
u
<---+ Exercices 6.8 et 6.9

126
Fonctions, polynômes, continuité Chap,itre 6

- Si on se retrouve dans une situation où l'on doit additionner des


équivalents, on commence par conjecturer l'équivalent: si les
deux termes ne sont pas du même ordre alors on garde le "domi-
nant", sinon s'ils sont du même ordre on les additionne. Poll!r voir
si le résultat est vrai, on calcule la limite du quotient (qui doit être
égale à 1).
~ Exercice 6.8

- A partir d'un encadrement, on conjecture l'équivalent, puis on le


démontre à l'aide du théorème des gendarmes (en vérifiant que le
quotient tend vers 1).
~ Exercice 6.6

- Si f admet une limite infinie (à gauche ou à droite) en un point xo,


alors la droite d'équation x = x 0 est asymptote à la courbe repré-
sentative de f.
~ Exercice 6.5

- Si f admet une limite finie l en +oo ou en - oo, alors la droite


d'équation y= lest asymptote à la courbe représentative de f.
~ Exercice 6.5

- Si f admet une limite infinie en +oo ou - oo, alors on peut recher-


Pour étudier les branches infinies d'une cher une éventuelle asymptote oblique.
fonction ~ Exercice 6.5

Pour savoir si la courbe représentative de f admet une asymptote


oblique en +oo, on commence par déterminer la limite en +oo de
f~x) : elle doit être égale à un réel a non nul. On détermine alors
la limite en +oo de f(x) - ax: elle doit être égale à un réel b. Si ces
deux conditions sont vérifiées alors la droite d'équation y= ax+ b
est asymptote en +oo. On peut appliquer le même raisonnement
en -oo.
~ Exercice 6.5

u - On peut chercher des racines évidentes en essayant: - 2, -1, 0, l,


0
C:
::J
2, - i et i ... Il ne faut surtout pas oublier de déterminer ensuite
0 leur ordre de multiplicité, à l'aide des dérivées successives du po-
lfl
,-i lynôme.
0
N ~ Exercice 6.4
Pour déterminer les racines/factoriser
@
...., un polynôme On peut aussi tenter de les trouver par une résolution d'équation
.c
Ol dans C : il faut alors soit se ramener à une équation du second
ï::::
>-
a.
degré à coefficients réels (az2 + bz + c =0 avec (a, b, c) E ~ 3 ), soit à
0
u une recherche de racine nème (zn =a avec n EN * et a E C*}.
~ Exercice 6.4

127
Chapitre6 Fonctions, polynômes, continuité

- Lorsqu'on sait que a est racine d'ordre p, on sait que le polynôme


se factorise par (X - a)P. Le quotient se calcul par identification
des coefficients.
~ Exercice 6.4

Dans C[XJ tout polynôme se factorise en un produit de binômes


du premier degré.
~ Exercice 6.4

Dans IRl[X) tout polynôme se factorise en un produit de binômes


du premier degré et de trinômes du second degré à discriminant
strictement négatif. On passe de C[X] à ~[X] en regroupant les ra-
cines complexes conjuguées.
~ Exercices 6.4, 6.17 et 6.21

Dans C[X] tout polynôme a un nombre de racines (comptées avec


leur ordre de multiplicité) égal à son degré. Dans IRl[X) tout poly-
nôme a un nombre de racines (comptées avec leur ordre de mul-
tiplicité) inférieur ou égal à son degré.
~ Exercices 6.17 et 6.21

- Si la relation de récurrence est du premier ordre (entre le rang net


le rang n + 1), on utilise des démonstrations par récurrence "clas-
sique". Si la relation de récurrence est du second ordre (entre le
rang n, le rang n + 1 et le rang n + 2), on utilise des démonstrations
par récurrence double(= à deux pas).
~ Exercice 6.17

Pour vérifier que tous les termes de la suite sont dans !RI [X) ou dans
C[XJ, on procède par récurrence.
~ Exercice 6.17

Pour déterminer le degré de Pn en fonction de n, on fait une


Pour étudier une suite de polynômes conjecture que l'on vérifie par récurrence.
définies par une relation de récurrence ~ Exercice 6.17
u
0
C:
::J Pour déterminer le coefficient dominant de P n en fonction de n,
0 on fait une conjecture que l'on vérifie par récurrence. On peut
lfl
,-i aussi noter an ce coefficent dominant et tirer une relation de ré-
0
N currence sur la suite (an), à partir de celle qu'on connaît sur (P11 ).
@ ~ Exercice 6.17
....,
.c
Ol
ï:::: Pour déterminer les racines de Pn il faut se rappeler qu'il ne peut
>-
a. en avoir plus que son degré. Il suffit donc souvent d'en calculer
0 certaines pour en fait les avoir toutes.
u
~ Exercice 6.17

128
Fonctions, polynômes, continuité Chap,itre 6

Les fonctions usuelles sont continues sur leur ensemble de défi-


nition. Les opérations sur les fonctions donnent encore une fonc -
tion continue.
<--+ Exercice 6.5

- Pour étudier la continuité en un point xo on doit vérifier


lim f (x) = /(Xo) ou encore que lim f (x) = /(Xo) et lim_f(x) =
~~ ~~ ~~
f (Xo) (i.e. f est continue à droite et à gauche en Xo)
Pour étudier la continuité d'une <--+ Exercice 6.15
fonction
- Si a est une borne de l'ensemble de définition où f n'est pas dé-
finie, et si lim f (x) existe et est finie égale à l, alors en posant
x~a
f (a)= l on prolonge f en une fonction continue en a.
<--+ Exercice 6.5

- Si on "recolle" plusieurs fonctions, il faut toujours étudier la conti-


nuité au(x) point(s) de recollement.
<--+ Exercice 6.15

- On raisonne par analyse-synthèse (ou par condition(s) néces-


saire(s) et suffisante(s)). On commence par supposer qu'on a au
moins une solution et on la calcule (analyse =condition(s) néces-
saire(s)), on ne trouve alors que des candidats solutions. Il faut
alors vérifier si tous ces candidats sont bien solutions du pro-
blème (synthèse= conidtion(s) suffisante(s))
<--+ Exercices 6.10 et 6.11
Pour résoudre une équation
- Si on a une équation fonctionnelle valable en tout point x E !RI, il
fonctionnelle
faut prendre des choix particuliers de x : 0, l, -x, ±...
<--+ Exercice 6.20

- Si f est cherchée continue on pourra dire que lim f (x)


x-xo
=f (xo) ou
encore que lim f(un) = f(Xo) pout toute suite (u 11 ) convergente
n -+oo
vers xo.
<--+ Exercice 6.20
"O
0
C:
::J
0 D'après le théorème des valeurs intermédiaires, f continue sur
lfl
,-i [a, b] prend toute valeur entre f(a) et f(b) . Si f(a) et f(b} sont
0
N de signes opposés, on peut en déduire que f s'annule sur [a , b].
@ <--+ Exercices 6.7 et 6.10 et 6.11 et 6.18
...., Pour utiliser les théorèmes généraux
.c
Ol
·;::
sur les fonctions continues D'après le théorème de continuité sur un segment, f continue sur
>-
a. [a, b] est bornée, mais surtout "ses bornes sont atteintes", c'est-à -
0 dire qu'elle admet un maximum et un minimum global sur [a, b].
u
<--+ Exercices 6.18 et 6.19

129
Chapitre6 Fonctions, polynômes, continuité

- D'après le théorème de la bijection, I continue sur I et strictement


monotone est bijective de I sur J = 10). De plus 1- 1 est alors conti-
nue sur J. Pour déterminer 1- 1, on calcule l'unique solution x E I
de l'équation y= l(x), avec y E J fixé quelconque.
c........ Exercices 6.12, 6.15 et 6.16

- Les fonctions x .......... arccos(cosx) et x - arcsin(sinx) sont défi-


nies sur~ mais ne sont pas toujours égales à x.
c........ Exercices 6.12, 6.13 et 6.16

- La fonction x .......... arctan(tanx) est définie sur ~ \{~+ k11/k E Z}


mais n'est pas toujours égale à x.
c........ Exercices 6.12, 6.13 et 6.16

- Si a = b alors sin a = sin b. Mais sin a = sin b donne a = b [211) ou


Pour faire des calculs faisant intervenir a= 11 - b [211). Pour avoir a= b il faut, par exemple, avoir vérifié
les fonctions trigonométriques que(a,b)E [-~.~]2.
réciproques c........ Exercices 6.13 et 6.16

- Si a= b alors cos a= cos b. Mais cos a= cos b donne a= b [211) ou


a = - b [211). Pour avoir a = b il faut, par exemple, avoir vérifié que
(a, b) E [O, 11)2 •
c........ Exercices 6.13 et 6.16

- Si a= b alors tan a= tan b. Mais tan a= tan b donne a= b [n]. Pour


avoir a =b il faut, par exemple, avoir vérifié que (a, b) E ] - ~, ~ [ •
2

c........ Exercices 6.13 et 6.16

u
0
C:
::J
0
lfl
,--i
0
N
@
....,
.c
Ol
ï::::
>-
a.
0
u

130
Fonctions, polynômes, continuité Chap,itre 6

~ ~
Enonces des exercices
.

---- - Calculs de limites par encadrement


Déterminer les limites suivantes.
a) lim xcos(é)
x-+oo ~
b) lim ex-sinx
x- +oo
C) lim E(i)
x- o+
d) lim xE(i)
x- 0
e) limx2 E(i)
x- o

---- - Calculs de limites par opérations usuelles


Déterminer les limites suivantes (envisager éventuellement la limite à droite ou à
gauche).
a) lim x-./x
x- +oolnx+x
b) lim lnxln(lnx)
x- 1+
c) lim ~ ln(l +x2 )-lnx
x- +oo
d) lim (ln(sin x) - ln x)
x-o+
e) lim x2e-./x
x- +oo
f) lim xx
x- o+
g) r i
x-lm0 x(x+l)
h) ru n
Jxi
-
x-0 X
i) lim ln(l +ex)
u x- +oo X
0
C:

---- -
::J
0
lfl
,-i
0 Calculs de limites par quantité conjuguée
N
@ Déterminer les limites s uivantes.
...., ) lim x2
.c a x- ovl+x2 - l
Ol
·;::
>- b) lim vx+4-v'3x+4
a. x- o v'x+l-1
0
u ) lim x- Jxi+ï
C -v
x- +oo x 2 x2 +1

131
Chapitre6 Fonctions, polynômes, continuité

---- - Factorisations de polynômes


Factoriser les polynômes suivants :
a) x 4 - x 2 + 1 dans C[Xl puis llll[Xl .
b) X4 - zX3 +2X2 - 2X+ 1 dans llll(X].
c) x6 +x3 + l dansC[XJ.
d) x6 - 3X5 + 2X4 - x 2 + 3X - 2 dans llll [X] sachant que i est racine dans C

---- - Branches infinies d'une fonction


On considère la fonction f définie par f (x) = x 2 ln ( 1: x ).
a) Donner l'ensemble de définition de f. Montrer que f est prolongeable par conti-
nuité en O.
b) Étudier les branches infinies de la courbe représentative de f. Pour cela, on ad-
2 3 2 3
mettra que, pour tout x > 0 : x - ~ - \ ~ ln(l + x) ~ x - ~ + \ .

---- - Le théorème des gendarmes permet de trouver des équivalents


Soit f une fonction telle qu'au voisinage de +oo, on ait:

2
x +-
X
1
~ f(x) ~
?
r +x.

Déterminer un équivalent de f en +oo.

---- - Une utilisation classique du théorème des valeurs intermédiares


Soit f : (0, l] -
point fixe.
[O, l] une fonction continue. Montrer qu'elle admet au moins un

---- - Calculs de limites à l'aide des équivalents usuels


Déterminer les limites suivantes.
a) Logarithme et exponentielle.
1
• lim x(n(l+x) - 1nx) XX - l
• lim - - • Ji m (xe !x - x )
x-+oo x-O+ X X-+oo
u
0 • lim ln(l +-;+x2) • lim ( x~2 )2x • lim (1 + I)3x- 2
C:
::J x-0 x x- +oo x x- +oo x
0 1
lfl
,-i
lim (x2-2x+l)x
• x-+oo x 2 -4x+2
• lim (111(1 +
x-0
x)) x • lim ln(x)
x-1 v'x- 1
0
N b) Puissances réelles.
@ • lim Vx+î-Vx" • lim _ x _ - _ x_
...., x- +oo Vx+l - Vx" x- +oo v'x+Î vx+2
.c
Ol c) Fonctions trigonométriques.
·;:: lim sin(2x) lim tan(nx) • lim xsin 11 )1. sin(3x)
>-
a. • x- Osin(nx) • x - O tan(3x) x-+oo x •
x-3
"1. l - 2cos(x)
0
u . cosrnxJ
• 1un _ JIz _ JI
x-I e 2x -e 2

132
Fonctions, polynômes, continu ité Chap,itre 6

---- - Composer un équivalent par le logarithme (î)

a) Soient f et g deux fonctions définies au voisinage de xo E Di et à valeurs stric-


tement positives. On suppose que f(x) - g(x) et que lim g(x) = E IPJ+\{1}. e
x-xo x-xo
Établir que ln (f (x)) - ln (g(x)).
x-x-0
b) En déduire un équivalent en +oo de f(x) =ln (x2 + zx).

---- - Équation fonctionnelle trigonométrique


Déterminer les fonctions f continues sur~ à valeurs réelles telles que:

'v' x E ~. cos(f(x)) = sin (f (x))

---- - Équation fonctionnelle polynômiale


Déterminer les fonctions f continues sur~ à valeurs réelles telles que:

'v' XE IR, f (x) = (f (x) ) 2

---- - Utilisation du théorème de la bijection


On considère la fonction f définie par: f(x) = Jtan(x).
a) Montrer qu'elle est bijective de
réciproque est co ntinue.
lü,; [ sur un intervalle à déterminer, et que sa
b) Déterminer l'expression de 1- 1(y).

---- - Formules pour les fonctions trigonométriques réciproques

a) Vérifier que: 'v' x E 1-1, l i, arccos(x) + arcsin(x) =~.


Si X> 0
b) étabfu- que, Vxc ••, a,ctan(x) + a,ctan ( 1) = { _;~
si x<O

c) Montrer que: arctan (!)+arctan ( ! ) = î.


u
0

0
C:
::J

---- - Comment montrer qu'une limite n'existe pas î


On pose: f(x) =eX sin(x). Étudier la limite de f en -oo et +oo.

---- -
lfl
,-i
0
N
@ Une fonction bijective n'est pas nécessairement continue ou m onotone...
....,
.c . , . , . Si - 1 :( X :( Û
Ol
·;:: On cons1dere le fonct10n f defime par : f (x) ={ X

>- 1- X Si Û < X :( 1
a.
0
u a) Vérifier que f est bijective de [-1, l] sur [- l, l].
b) Montrer que f n'est ni continue ni monotone sur [- 1, 1].

133
Chapitre6 Fonctions, polynômes, continuité

---- - Équations avec des fonctions trigonométriques réciproques

a) Soit t E lll1* . Donner le nombre de solutions de l'équation : arcsin(x) +


l.
arctan( v'î+x) = en fonction de t.
b) Résoudre dans lll1 l'équation: arcsin(2x) = arcsin(x) + arcsin(xJ2).

---- - Polynômes de Tchebychev


On définit une suite de polynômes (Pnln€N de lll1[X] par Po = l, P1 = X et:

'rlnEN, Pn+2 = 2XP11+1-Pn

a) Soit n EN. Déterminer le degré et le coefficient dominant de Pn.


b) Vérifier que, pour tout n E N: 'ri x E lll1, Pn(cosx) =cos(nx).
c) En déduire les racines de Pn et la factorisation de P11 dans lll1[XJ.

---- - Soit f une fonction continue...


Soit f: (0, +oo[- lll1 une fonction continue.
a) On suppose que la limite de f en +oo existe et est finie. Montrer que f est bornée
sur (0, + oo[.
b) On suppose que lim f(x)
x- +oo
=+oo. Montrer que f est minorée sur [O, +oo[ et que
sa borne inférieure est atteinte.
c) On suppose que f (0) < 0 et que lim f(x) existe et est strictement positive. Mon-
x-+oo
trer que f s'annule au moins une fois sur JO, +oo[.

u
0

0
C:
::J

lfl
,-i
0
N
---- - Continuité du maximum sur un segment (d'après Escp-Eap 2008)
Soient f et g deux fonctions numériques définies et continues sur [- 1, l ).
a) Soit u e: lll1. Montrer que la fonction hu définie sur (-1, 1) par: hu(x) = f(x)+ug(x),
est bornée et atteint ses bornes sur (-1, 1).
@ On note M la fonction: 1Jl1-.. lll1, u - M(u) = max hu(x).
...., - l ~ x ~l
.c
Ol b) Soient (u, v) e: lll1 2 et (x, y) e: (-1, 1) 2 tels que: Tvl(u) = hu(x) et M(v) = hv(y). Véri-
·;::
>- fier que:
a. (v- u)g(x) :::; M(v) - M(u):::; (v - u)g(y)
0
u
c) Montrer que la fonction M est continue sur lll1.

134
Fonctions, polynômes, continu ité Chap,itre 6

---- - Étude d'un ensemble de fonctions continues


On note E l'ensemble constitué des fonctions f continues de Ili dans Ili vérifiant les
deux conditions suivantes :
(i) pour tout (x,y) E lli 2 , f(x+ y)+ f(x - y) = 2/(x)f(y),
(ii) 3xE Ili/ f(x) < 1.
a) Déterminer un élément de E non identiquement nul.
b) Soient a E Ili* et f E E. On définit une fonction g par g(x) = f(ax). Montrer que g
est encore un élément de E.
c) On suppose dans cette question que .f est un élément non nul de E.
• Calculer f (0).
• Pour tout x E Ili, comparer f (x) et f (- x).
• Trouver une relation entre .f(x) et .f(x/2).
• Vérifier que : ri x E Ili, f (x) + 1 :::: O.
d) On suppose dans cette question que f est un élément de E qui ne s'annule pas
sur~.
• Étudier le signe de f.
• Soit a E Ili tel que f (a)< 1. On pose : rln E 1\1, Un = f (2n a). Montrer que la suite
( Un) ns:N est convergente.

---- -
• En déduire que ri x E Ili, f (x) ;;;: 1.

Factorisation d'un polynôme


On considère le polynôme P de lli(X] défini par:

P(X) = Ln (2n+ 1) (- l)pxn-p


p=O 2p+l
..,, on pose aussi· xk -_ cotan2 ( n+l
k1r ) , _ cosa
Pour k E "-• 2 , ou cotan a - sina .

a) Montrer que x1, x2, ... , Xn sont des réels deux à deux distincts.
b) Déterminer le degré de P et son coefficient dominant.
c) Montrer que:
P(X2) = ;i ((X+ i)2n+ l _ (X- i)2n+ J)
où i est le nombre complexe tel que: i 2 = -1.
d) En déduire que les Xk, k E Œl, n], sont les racines de P. Factoriser P dans lli[X].
e) Al' aide du coefficient de xn- l de P, vérifier que :

u ~ cotan2 ( -krc- ) = -
n(2n -1)
0 L, --
C:
::J k=l 2n+l 3
0
lfl
,-i f) Pour 0 E ] 0, ~ 1, on admet que : cotan2 (0) < ~ < 1+ cotan 2 (0). Al' aide des racines
0
N de P, en déduire que:
@
....,
.c
Ol
·;::
>-
a.
0
u

135
Chapitre6 Fonctions, polynômes, continuité

Du mal à démarrer ?

al Utiliser les encadremen ts classiques de cos(x), sin(x), et E(x) lll9 a) Utiliser le théorème de la bijection.
avec le théorème des gendarmes. =
b) Résoudre l'équation /(x) y d'inconnue x.

ID a) Factoriser numérateur et dénominateur par le terme pré- 1111 a) Commencer par montrer que cos(arccos(x))
pondérant. cos ( ~ - arcsin(x) ).
b) Faire apparaître une croissance comparée. b) Commencer par x > O.
c) et d) Utiliser les propriétés algébriques de la fonction ln. .. ( b) ran(a)+tan(bl
c) U t1 11ser tan a+ = l - tan(aJ tan(b) .
e) et f) Faire apparaître une croissance comparée.
g) et h) Disti nguer lim ite à droite et à gauche.
i) Utiliser une mise en facteur dans le ln.
111D En +oo introduire des suites.

. . Tout est dans le titre.


CID a) et b) Revenir aux définitions.

111111 a) Utiliser le théorème de la bijection.


Ill a) Résoudre dans C : z 4 - z 2 + 1 = O. b) Commencer par montrer qu'on peut prendre le sinus de l'équa-
b) Chercher des racines évidentes. tion sans changer les solutions.
c) Résoudre dans C : z6 + z3 + 1 = O.
d) Chercher des racines évidentes.
ll9 a) Faire une conjectu re puis u ne récurrence double.
b) Récurrence double.
ID a) Utiliser ln(l + u)
0 u. c) Commencer par résoudre cos(nx) = 0 d'inconnue x.
b) Utiliser le th éorème des gendarmes.
1111 a) et b) Uti liser la définition de la limite et le théorème de
11111 Conjecturer l'équivalent de f (x) en +oo puis montrer que le continuité sur un segment.
c) Uti li ser le théorème des valeurs intermédiaires.
quotient tend vers 1.

Ill Introduire g: x ~ f(x) - x.


11111 a) Utiliser le théorème de continuité sur un segment.
b) Comparer hu(x) avec hu(Y) ...
c) Conclure à l' aide du théorème des gendarmes.
. . a) Utiliser ln(l + u) -0 u et e" -1 -0 u.
b) Utiliser (1 + u)a - 1- au.
0
IE!I a) Penser à la trigonométrie...
u2 b) Vérifier les conditions de l'énoncé.
c) Uti liser sin(u) - u, tan(u) - u et 1- cos(u) - - . c) Utiliser (i) pour des choix judicieux de x et y.
0 0 0 2
d) Trouver une relation simple entre Un+l et u,,.

11111 =
a) Distinguer trois cas e o+ , I? e:]O, l[u]l, +oo[ et e=+oo.
ml a) Utiliser la stricte monotonie de cotan 2 .
b) Faire apparaître une croissance comparée.
b) Simple vérification.
c) Développer la formu le proposée à l'aide de la formule du bi-
u
0
CIII Uti liser sin(a) = cos ( ~ - a). nôme.
C: d) Utiliser le degré de P.
n
0
::J

lfl
11111 Ne pas conclu re trop vite. e) Le coefficient de xn- l est -(2n+ l)
k=l
L
x k·

,-i
0
N
@
....,
.c
Ol
·;::
>-
a.
0
u

136
Fonctions, polynômes, continuité Chap,itre 6

Corrigés des exercices

-
a) Sachant que rite: IR:, 1cos(t) 1 ~ l, on obtient, pour x > 0 :

O ~ 20
1 x: ::x 1 ~ 1 x 2: 1 I = x/+ 1 ~ :i = t
limxE(~) ex~te 1
x- o
x>O
X

Une démonstration similaire donne le même résultat pour


x < O. Finalement :
car x 2 + 1 ~ x 2 > O.
1
Or lim - = 0 donc, d'après le théorème des gendarmes,
lim xE
x -0
(.!.)
X
ex~te 1
x-+oo X
on en déduit que e) On a x 2 E ( ±) = x x xE ( ±), donc par produit de limites :
. xcosex existe
=

,,-
1un - - - 0 lim x2E(~) extJte Ox O= 0
x-+oo x2 + 1 x -0 X
b) Sachant que rite: IR:, - 1 ~ sin(t) ~ l, on obtient pour tout
xe:IR: :
. x-Ji _ l-
a) pour x > O, on a . x+ln(x) - --iiilxJ.
lx
x-1 ~ x-sin(x) ~ x+ 1 l +-x-
. 1 li ln(x) ( .
et donc: 0 r 11m r.:. = 0 et m - - = 0 cr01ssances compa-
x-+oo yX x-+oo x
rées) donc, par quotient de limites, on obtient :

Or lim é- 1 = +oo donc, d'après Je théorème des gen- . x - ,/x existe


1,m = 1
x-+oo x-+oo x+ ln(x)
<larmes :
b) D'après les croissances comparées :
lim é-sin(x) ex~te +oo
x-+oo lim ln(x) =O+ l parcomposition ·
x- i+ ==> lim ln(x)In(In(x))ex!lte o
c) Par définition de la partie entière d'un réel : lim Tln(T) =O x- 1+
T -o+
rlte:IR:, E(t) ~ t<E(t) +l c) On remarque d'abord que pour x > 0:

rite: llll, t - 1 < E(t) ~ t 21 In(l + x2 ) - lnx = ln ( ~


x ) = ln ( yi
~ +;; )

On en déduit que : puis

1 (1) -1
"O
0
C:
::J
1
ri x > 0, - - 1 < E -
X X

Or lim - = +oo donc, d'après le théorème des gendarmes :


~
X
·R
hm
x- +oo
l+ -

lim ln(T) = O
x2
=l
}
.
==> lim ( -1 Jn(l + X 2 )
x- +oo 2
- ln X ) existe
= Û

0 x- X T- 1
X>O

1x-.1m0 E( -X1) existe par composition de limites.


lfl
,--i
0 = +oo d) On remarque d'abord qu'au voisinage de o+ :
N x>O
@ In(sin(x)) - ln(x) = ln (sin?)) puis :
...., d) Multiplions par x > 0 chaque membre de la double inéga-
.c lité obtenue à la question précédente :
Ol
·;:: fun sin(x) = 1 l .
>-
a. x- o x ==> lim ln(sin(x) ) - ln(x) ex!f'te 0
0
u ri x > 0, 1 - x < xE ( ~) ~1 lim ln(T) =0 x-o+
T- 1
On en déduit, d'après Je théorème des gendarmes : par composition de limites.

137
Chapitre6 Fonctions, polynômes, continuité

e) On remarque d'abord que x2 e- ./x = ( Jx) 4 e- ./x puis :

lim ../x = +oo


x-+oo
fun T4 e-T = O
} .
r.;)4 e -./x exl_§te
==> lim (vx
x- +oo
- o
-
a) Utilisons l'indication donnée par le titre :

T- +oo

par composition de limites. On a donc: donc par opération sur les funites :

lim x2e- ./x ex~te 0 x2


x-+oo
fun exif te 2
x-o v' 1 + x2 - 1
f) On remarque d'abord que xx =
exlnx puis, par croissances
b) Même tactique :
comparées et composition de limites:

../x+4- ffx+4 - 2x( ../x+T + 1)


lim xln(x)
x- o+
=0 } . .
x existe
==> 1lm X = 1 ../x+T- 1 x( Jx +4+ J3x +4)
lim eT = 1 x- o+
T-0 Les indéterminations sont maintenant levées après simpli-
fication par x :
g) Il faut discuter selon le signe de x :
1 . Jx+4 - v3x+4 existe 2
lim x(x+ 1) = o- donc lim - - - = -oo; hm = - 2x - =- 1
x-o
x< O
x- o x(x + 1)
x<O
x-0 ../x+T - 1 4
1 c) Le dénominateur n'est pas une forme indéterminée. JI suf-
lim x(x+ 1) = o+ donc lim - - - = +oo.
x-o x-o x(x + 1) fit donc de multiplier et diviser par la quantité conjuguée
x>O x>O
du numérateur :
La fonction n'admet donc pas de lilnite en O mais une li-
mite à droite et une limite à gauche djstinctes.
x2 - (x2 + 1)
h) On remarque que J? = l~I = J Si X> 0
. Donc:
X-~

l l J;
1 - 1 Si X <0 x2 - v'x2 + 1 ( x2 - 2 + ; 4 ) (x + v' x 2 + 1)
-1

x-o
n
lim - -
X
=- 1 ·
'
x2 ( 1 - J; +
2 x~ ) (x + v' x 2 + 1l
x<U

/?
lim _v_
x~ =1. x- ~ existe
x-o X On en déduit que lim
x-+oo x2 - V x2 + 1
~ = 0
x>O

'O
0

0
C:
::J

lfl
,--i
La fonction n'admet donc pas de funite en O mais une li-
mite à droite et une limite à gauche distinctes.
i) On remarque d'abord que :

ln(l+ex)
---- =
ln(ex(l+e-x))
=l+ - - - -
ln(l + e- x)
-
a) DansC[X]
Résolvons dans C l'équation (E) : x 4 - x2 + 1 O.
Posons t = x2 alors (E) = t2 - t + 1 = 0, t = x 2 .
=
0 x X X Le discriminant est t:, = -3 = (iv"3)2 donc les solutions de
N
@ Or fun ln(l+e-x) = 0, donc par quotient de lilnites (E)sont les racines carrées des deux nombres :
...., x-+oo
.c ln (1 + e-x)
Ol lim = O. 1 +iv'3 jl!
·;:: x-+oo x t1 = --- = e 3
>-
a.
2
Finalement par somme de limites :
0
u · ff ·ff ·ff
c'est-à-dire e1 6, -e1 6 et e- 1 6, -e- 1 6
·ff

. ln(l +ex) existe


fun = 1 Ainsi, Pa(X) = (x-eii) (x + eilf) (x-e-ii) (x + e-ilf ).
x - +oo x

138
Fonctions, polynômes, continuité Ch ap,itre 6

Dans!Rl[X] :
On regroupe deux par deux les facteurs correspondant à
Pd(X)
des racines conjuguées :
(X2 + l)(X - 1) 2 (aX2 + bX + c)
Pa(X) = aX6 + (b- 2a)X5 + (c - 2b + 2a)X 4 + (2b- 2a - 2c)X3
(x - ei ~)(x-e-i~) x (x+ei ~)(x+e-i~) + (a+2c - 2b)X2 + (b - 2c)X + c

(X2 - ( ei ~ + e -i ~) X + 1) x (X2 + ( ei ~ + e-i ~ ) X + 1) On en déduit que a = 1, b = - 1 etc = - 2.

2 2
Pd(X) = (X2 + l)(X - 1) 2 (X2 - X- 2). De plus 2 et - 1 sont les
(x - 2cosrn)x+ 1) (x +2cosrn)x+ 1) racines du dernier facteur.
(x2 -hx+ 1) (x2 + hx+ 1) Finalement :

P d(X) = (X2 + l)(X - 1)2 (X+ l)(X - 2)

-
Ces deux derniers facteurs sont bien sûr irréductibles dans
IR1 car leurs racines sont complexes.

b) Pb(X) admet 1 comme racine évidente. On peut donc le


factoriser par X - 1. Mais on remarque que P~(l) = 0 et a) La fonction f est définie là où 1tx est strictement positif :
P~(l) f. 0, donc 1 est racine d'ordre 2 de Pb(X) et on peut
donc le factoriser par (X - 1) 2 : X -oo - 1 0 +oo
Pb(X) = (X - 1)2 (aX2 + bX + c) = aX4 + (b - 2a)X3 + (c - 2b + Signe de -
l+x
() + +
a)X2 + (b-2c)X + c. 1

Signe de - -
On en déduit que a = l, b = 0 etc = 1. X 1
Q +
Ainsi Pb (X) = (X - 1)2 (X.2 + 1). Le deuxième facteur admet i Signe de 1

et - i comme racines et est donc irréductible dans IR1 [X). l+x


-- + Q - +
1
X 1

c) On remarque que x 6 + x 3 + 1 = (x3)2 + X3 + 1.


I.;équation x 2 + x + 1 = O admet deux racines conjuguées @r =J- oo ;- l ( u]O; +oo[.
-l-iv'3 _ _ 2in _ --: -l+iv'3 _ 2in _ . Calculons la limite de J en O.
-- 2- - e 3 - J et -- 2- - e 3 - J.
J (x) = x2 ln(l + x) - x2 ln(x).
On en déduit que les racines de l'équation x 6 + x3 + l = O
Or lim ln(l + x) = 0, donc par produit de limites
sont les racines cubiques de ces deux nombres. x-0
Les racines cubiques de 1 sont l, j et] et des racines eu- lim x2 ln(l + x) = O.
- 2in - 2în 2în 2în x-o
biques évidentes de j = e- T et j = e T sont e- T et e T. De plus par croissances comparées lim x2 ln(x) = O.
En les multipliant par les racines cubiques de 1 on obtient x- o
que les racines de l'équation x 6 + x 3 + 1 = 0 sont: Finalement, lim f (x) ex!1 te O. On peut donc prolonger la
2in 2irr 2in - 2in 2irr 2in -
x-0
e-T xl,e-T xj,e-T xj,e-T xl,eT xj,eT xj. fonction .f par continuité en O en introduisant une fonc-
On en déduit que la forme factorisée de Pc (X) est : tion J définie sur J - oo; - 1[ u [O; +oo[ par :

u
0 ilrr)(X-e _T2in ) (X- e T2in) (X- eT41n ) (X-e Sin)
_ 8in) (X- e _ T .f-(x) ={ f (x) si x f. 0
C: (X- e T T
0 six= 0
::J
0
lfl
,-i d) Ce polynôme est à coefficients réels et vérifie Pd (i) = 0 et b) !~) =xln(1+±)= ln(~+i ) _
0 P~(i) f. O. Tl admet donc une racine imaginaire i d'ordre l, X
N
et admet donc aussi comme racine d'ordre 1 le conjugué . ln(l + u) . . . .
@ Or lim = 1 donc, par compos1t10n de limites,
...., dei à savoir -i . u- 0 u
.c
Ol lim f (x) ext]te 1
·;:: De plus 1 est aussi racine évidente. Pd(l) = P~(ll = O et
>- x-+oo x
a. P~(l) f. 0 donc 1 est racine d'ordre 2.
0 Étudions à présent la limite de f (x) - x au voisinage de +oo.
u
Pd(X) est donc factorisable par (X - i)(X + i)(X - 1)2 = (X2 + À partir de l'inégalité proposée on obtient successive-
1) (X - 1) 2 . On en déduit que : ment:

139
Chapitre6 Fonctions, polynômes, continuité

~ __l___l_ ~ln(l+ ~) ~ ~ __l_ + - 1_.


x zx2

1
X- - - -
2
3x3

3x
1 ~X
2
x x

Jn ( 1 + -1) ~ X - -1 + -1
x 2 3x
zx2 3x3 -
I ntroduisons la fonction g : x - f (x) - x. Elle est continue
sur (0; l] comme différence de fonctions continues sur œt in-
tervalle.
Or g(O) =f(O) ~ 0 et g(l) =/(1) - 1 ~ 0 car /(1) e: [O; l J.
La fonction g étant continue sur (O; l i et changeant de signe,
1 1 1 1 elle s'annule au moins une fois d'après Je théorème des va-
- 2- 3x ~f(x) - x~ - 2+ 3x leurs intermédiaires.
Il ne reste plus qu'à composer le 17 : d'après le théorème On en déduit qu'il existe au moins un réel xo de l'intervalle

-
des gendarmes, on en déduit que (O; li tel que f (x) = x.

.( X ) -X ex{§te 1
lliTI - --
x- +oo / 2
a) - On se débrouille pour retrouver une forme ln(l + u) au
Finalement, la courbe représentative de f admet la droite voisinage de O :
!
d'équation y= x - comme asymptote au voisinage de ln(l +x)-ln(x) = 111(1 + 1).
+=.
Or ln(l + u) - u et lim ~ = 0 donc par substitution
O x -+oo X
ln(l + x) - Jn(x) -
+oo
1donc, par produit d'équivaftents,
x(ln(l + x) - ln(x)) - 1
+oo

2- et donc:

.
hm x (ln(l+x)-ln(x) ) existe
= 1
x-+oo
On introduit la fonction exponentielJe :
1- xx = exln(x) or Jim xln(x) = 0 et eu - 1 - u donc par
('l&' f) x- o+ 0
substitution xx - 1 - x ln(x) . On en déduit que
o+
XX -1
- - - ln(x)
X o+
xx -1 existe
donc lim = -=
x-o+ x
1 1 1
On factorise par x :
1 2 3 xe ~ - x =x (e ~ - 1).
l
Or llin - = 0 et eu - 1 - u donc par substitution
- Comme les membres de gauche et de droite de l'in- x - +oo x 0
'O l
0 égalité sont proches en += de x 2 , on conjecture que f (x) ex-1- -l .
C: +oo X
::J va être équivalent en +ex, à x2 . Il reste à le démontrer! On en déduit que :
0 Pour x non nul, multiplions chaque membre de l'inégalité par
lfl l .
,-i L
x2 . xex-x-l
0
N +oo
@
1 f (x) 1
l+ - 3 ~ - 2- ~l+ - . ! existe
...., x X X donc Jim xe x - x
x-+oo
= 1.
.c
Ol On en déduit, d'après le théorème des gendarmes que :
·;:: Comme lim x 2 + x = 0 et Jn(l + u) -0 u on a, par substi-
>-
a.
x-o
0 lim f (x) ex[Jte
1
tution et quotient d'équivalents:
u x- +oo x2
ln(l+x+x2) x+x2 1
On en déduit que f (x) - x2 .
· x- +oo x2 o"7' o:;
140
Fonctions, polynômes, continu ité Chap,itre 6

donc On introduit la fonction exponentielle :


. ln(l +x+x2) existe
]1m (ln(l + x)) i = e i ln(l+x)
= +oo
x-o+ x2
et Or ln(l + u) - u donc± ln(l + x) - !. = 1
. ln(l +x+x2) existe 0 0 X
]1m = -oo
x-o- x2 On en déduit que fun .: ln(l + x) exgte 1 et
x-Ox
On introduit la fonction exponentielle : donc par continuité de l'exponentielle au point 1 :
( x;2 )2x = e2xln(l+ ~). . l existe
fun(ln(l + x))x = e
x- o
On obtient, par substitution et produit d'équivalents,
que: ln(x) ln(l + u)
Posons u=x-1 alors~= ~
vx - 1 vl+u - 1
2xln - 2x x ~ =4
{1 + ~) +oo Or ln(l + u) - u et Jf+ü - 1 - - u donc
1
X X 0 0 2

On en déduit que lim 2xln (1 + ~) exgte 4 ln(l + u)


------=2
u
x-+oo X
et donc par contuinité del' exponentielle au point 4 : Vl+u - 1 o !u
2
1.,m (x 2) x existe e.
-+- 4
= . . ln(x) existe
x-+oo x On en déduit que hm ~ = 2.
x-lvx-1
On introduit la fonction exponentielle :
b) On factorise par t'x numérateur et dénominateur :
(l + i )3x-
2
= e<3x- 2)1n(l+~).

On obtient, par substitution et produit d'équivalents, Vx+l-Vx x!


- ---- =- {/1:1-1
que: X--'------

Vx+ 1- Vx A {/1:1-1
2) 2(3x - 2) 6x
(3x - 2) ln ( 1 + - - - =6
X +oo X +oo X
Or t,'1 + u - l - .: x u donc
0 k
On en déduit que lim (3x - 2)ln (1 + ~) exgte 6 1
x-+oo x Vx+l - ijx 1 4x 3
et donc par continuité del' exponentieUe au point 6: - -x-=--
3r.::-:,- _ 3fx +oo ..L ..l.. 4 12'x
3x- 2 v ,t -r l V ..c X 12 3x v "'
. existe 6
11m 1 + -2)
x-+oo ( x
= e.
Vx+l-Vxexiste "- 3
On introduit la fonction exponentielle : d one lim = wu - - = 0 .
x- +oo Vx+ 1- Vx x-+oo 4 1V,i
2 2
x -2x+ l)x = (x -4x+2+ (2x-1) )x =
( x 2 -4x+2 x 2 -4x+2 On factorise par x, on réduit au même dénominateur
et on utilise la quantité conjuguée du numérateur pour
2x.-l )x
x1n(1+ x22x-l ) lever l'indétermination :
( 1 + x 2 -4x+2 =e - 4x+2

Or ln(l + u) - u donc par substitution et produit d'équi-


u
0
o _ x_ _ _x_ = X (Jx+2 - VX+l)
C: valents Jx + l Jx+2 J(x+2)(x+l)
::J
0
lfl xln{l + 2x-l ) _ x ~ 2_x_ - _I_
,--i
x 2 - 4x + 2 +oo x 2 - 4x+2 J(x:~x+l) ( R - Ji + ~)
0
N
@ 1
...., x1n(1 + /x-1
x -4x+2 +oo x
) - 2x22 = 2 = J(x:~x+l) [( R - l) - ( R- )]
.c
Ol
ï:::: , . . ( 2x- l ) existe,
>- On en dedmt que fun xln 1 + 2 = 2 Or Jf+ü - 1 - ~ donc par substitution :
a. x- +oo x - 4x+2 0
0
u et donc, par continuité de l'exponentielle au point 2:
.
1,m
x-+oo x2-4x+2
2
(x - 2. x+ 1 )x existe
= e.2 . (R'
fun x
x- +oo
l+ - -1 )existe
x
= .
fun xx -1 x -2 =l
x- +oo 2 x

141
Chapitre6 Fonctions, polynômes, continuité

et et donc par opération sur les limites :

. (R )
lim x
x-+oo
l+ - -1 existe
X
= .
hm xx -1 x -1 = -1
x-+oo 2 X 2
.
hm 1 x ( 1- cos(u) + vr:::
;;;
u- o v3u
.
3sm(u) ) existe
= 0 x 1+ 1 = 1

' ,fi ,fi 1 Ceci prouve que: 1 - cos(u) + v'3sin(u) - V3u.


Dautre part : v'(x+ 2)(x+ll +~ R = ,fi donc: 0
Alors par quotient d'équivalents et substitution:
lim v'x ex~te lim - 1-· = 0 -sin(3u) _ - 3u = - V3
x-+oo J(x+2)(x+ 1) x-+oo v'x
1 - cos(u) + v'3sin(u) u- 0 v'3u
Ainsi, par opérations sur les limites : et donc:
.
hm -X- - -X- existe (
= Ox 1- - =0
1) sin(3x)
- - V3
x- +oo Jx+ 1
Jx+2 2 l-2cos(x) x-i
c) - On utilise sin(u) - u et le théorème de substitution: On obtient que :
0
sin(2x) 2x fun sin(3x) ex~te -V3
sin(nx) o nx x-j l - 2cos(x)
. sin(2x) 2
donc hm - . - - = - . - On se ramène au voisinage de Oen posant u = x-1. On
x-0 SII1(1TX) 1T
obtient :
- On utilise tan(u) - u et le théorème de substitution :
0
tan(nx) nx
----- cos(;x) cosrn u+ ;)
tan(3x) o 3x
e- fx2 - e - f e- f <u+ l l2 - e - f
tan(nx) n
On en déduit que lim - - - = - . -sin(; u)
x- 0 tan(3x) 3 =
- On utilise sin(u) - u et le théorème de substitution:
0

Or sin(v) - v et ev -1 - v donc :
Onendéduitque lim xsin(~) = n. 0 0
x-+oo x
u2
- On cherche à utiliser sin(u) - u et 1-cos(u) - - .
0 0 2
COS rn X)
- - 2- - - - -e 2 x
!! ; U !!
- -e 2 x - -
U 1 !!
=-e 2
Posons u = x - ~. On obtient e-~x - e-~ o ;(- 2u - u2) o -2u 2

sin(3x) sin(3u+n) cosrnx) existe 1 n


l-2cos(x) l - 2cos(u+ ~) On en déduit que lim " " = - e2.
ïx2 - e- 2

-
x- 1 e- 2
u
0
C:
::J sin(3u+n) -sin(3u)
0 Or - - - - - -
l-2cos(u+~) l - cos(u) + v'3sin(u)
lfl
,-i
Attention, on ne peut pas additionner les équivalents a) •Sie= o+.
0
N u2 Dans ce cas, on a par composition de limites
@
mais comme sin(u) - u et 1- cos(u) - - . on conJ·ec-
o O 2 lim ln (g(x)) = - oo.
...., x - xo
.c ture que 1-cos(u) + v'3sin(u) - V3u. Démontrons le:
Ol 0 De plus, il existe un voisinage de xo tel que g(x) f. 1.
·;::
>-
0
u
a.
!
v3u
x ( 1- cos(u) + V3sin(u))
Sur ce voisinage on peut écrire :

u 1 - cos(u) sin(u) ln (f (x)) ln (g(x)) + ln (f (x)) - ln (g(x)) ln ( ~ )


-- x +-- ---,,-~ = = 1 + -,--~
'3
2 V.:> Tu2 u ln (g(x)) ln (g(x)) ln (g(x))

142
Fonctions, polynômes, continuité Chap,itre 6

Or f (x) - g(x) donc lim f (x) 1 et donc b) D'après les croissances comparées :
xo x-xo g(x)
x2 .
lim ln (f(x)) = O. lim _ = lim .x2e-xln2 exg te 0
x-xo g(x) x-+oo 2x x-+oo
De plus lim ln (g(x)} = -inf ty. car ln2 > O.
x- xo
On a donc x2 + 2x - 2x puisque :
ln(f(x)) +oo
On en déduit que lim g(xJ ext]te _Q__ = O. x2+zx
x- xo ln(g(x)) -oo _,- - = 1 + .x2e- xln2 x-+oo
- 1 +0 = 1
zx
ln (f(x)) existe
Finalement lim
x- xo ln g(x)
( ) = 1 + O = 1. Or lim 2x = lim exlnC2J = +oo donc, d'après la ques-
x-+oo x-+oo
tian a),
On en déduit que ln(f(x)) - ln(g(x)).
Xo

•Si e e:]O, l [u] 1, +oo[.


Puisque f(x) - g(x), on a:
Xo
,,,.,
Notons (E) l'équation proposée.
lim f(x) exgte lim g(x) =e •Analyse(= conditions nécessaires):
x- xo x- xo
Supposons qu'il existe une fonction continue sur ~ solution
et donc par continuité de ln au pointe : de (E).
(E) <==> 'v'xe:~, cos(f(x)) = cos(f(x)-;).
lim ln (f (x)) = lim ln (g(x)) =ln(€) ;z! 0 On en déduit que :
x- xo x- xo

carl!;z!l. f( x) - f(x) - 11
2 + 2k rr
(E)<==>'v'xe:~,3kxe:l./ - x
Alors, par quotient de limites:
{
f(x) =- f (x) + ; + 2kxn
(kx est un entier dépendant de x).
. ln (f (x)) existe ln(€)
hm = -- = 1 On en déduit que :
x-xo ln(g(x)) ln({!)

On en déduit que ln(f(x)) - ln(g(x)). (E) = 'v'xe: ~.3kx E: l./ { ; = 2kxn


Xo
f (x) = î + kxn
•Si e= +oo.
Le premier cas est impossible car le nombre kx doit être en-
Puisque f(x) - g(x), on a: tier.
xo
î
Le second implique que f (Ili) c { + krr, k e: Z}. Or f est une
fonction continue sur Ili. L'image de~ par f doit donc être un
x- xo
= x-liIngx=+oo
lim /( x ) existe
xo
( )
intervalle. La seule possibilité est que f (~) soit un intervalle
î
réduit à un point, de la forme { + krr}. l'entier k étant cette
u Il existe donc un voisinage de xo sur lequel les fonctions -} fois fixé (c'est-à-dire qu'en fait kx ne dépend pas de x).
0
C: Ainsi, les solutions de (E) sont nécessairement des fonctions
::J et 'k sont bien définies.
0 constantes définies sur Ili par f (x) = f + krr, k étant un entier
lfl 1 1 1 relatif fixé.
,--i
0
Elles vérifient - - -- et lim - - = o+.
xo g(x)
N 1 (x) x-xo g(x) •Synthèse(= conditions suffisantes).
@ On peut donc appliquer le premier point qui nous donne: Réciproquement, si f est définie sur Ili par f (x) = + krr, k î
...., 1 1 étant un entier relatif fixé, alors f est continue sur~ et :
.c ln( -f (x) ) xo
- ln(-
Ol g(x) ) ·
ï:::: - 'v'x E: ~. cos(f(x)} = cos(f+krr) cosrn)cos(krr) -

0
>-
a. Mais alors d'après les propriétés du logarithme:
-ln(f(x)) - - ln(g(x)).ce qui nous donne après multi-
sin(i)sin(krr) = (-l)k f;
u xo 'v'x E: ~. sin(f(x)) = sin(i + krr) = sinnJ)cos(krr) +
plication par (- 1) : ln (f (x)) - ln (g(x)) .
Xo
cos(J)sin(krr) = (-l)k f
143
Chapitre6 Fonctions, polynômes, continuité

car cos(kn) = (- l)k et sin(kn) = O.

1 -
a) La fonction tan est continue sur I = [O,; [, à valeurs dans
J = ~+. De plus, la fonction x ...... ../x est continue sur J = ~ +.
Donc f est continue sur I, comme composée de fonctions
continues.

1
De même f est strictement croissante sur I, comme com-
posée de fonctions strictement croissantes.
De plus lirjl _ tan(x) = +oo donc liCJl _f (x) = +oo.
Srr x-ï x-ï
T Finalement, on obtient Je tableau suivant :

lT
On en déduit que toutes les fonctions constantes définies sur X 0 2
~ par f (x) = !} + kn, k étant un entier fixé vérifient (E).
+oo
• Conclusion. Variations de
Les solutions de (E) sont donc toutes les fonctions : f 0~
avec k e: Z
X
La fonction f étant strictement croissante et continue sur

-
I, elJe est bijective de I surf (I), d'après le théorème de la
bijection. De plus on sait que sa fonction réciproque est
continue surf (I).
N otons (E) l'équation proposée. D'autrepart:/(1) = [/CO), lil'jl_/(x) [ = ~+.
•Analyse (= conditions nécessaires). x~ ï

Supposons qu'il existe une fonction f continue sur~ solution b) Soit y e: ~ +. On considère l'équation y= f (x) d'inconnue
de (E). X E J.
(E)<==>'v'xe:~, f(x)(I - f(x)) = O.
On a, puisque y~ 0 :
(E) <==> 'v' XE~. (t(x) = 0 OU f(x) = 1).
Attention! Cela ne signifie pas que f est constante égale à 0 y = Jtan(x) = y2 = tan(x)
ou l : à priori f (x) peut être égal à O pour certaines valeurs de
x et à 1 pour d'autres valeurs de x. Mais on sent bien que la Mais arctan est injective sur~ (car bijective) donc:
continuité de f va intervenir.
En effet : on en déduit que/(~) c {O, 1}. Or f est une fonc- y= Jtan(x) <==>arctan(y2) =arctan(tan(x))
tion continue sur~- I.;irnage de~ par f doit donc être un inter-
valle. Les seules possibilités sont/(~) = {Ol et/(~) = Ill c'est- Orx e: I, donc x e: ]- ~, ~ [ et donc: arctan(tan(x)) = x. On
à-dire que f est la fonction constante égale à Oou la fonction obtient donc:
constante égale à 1 :
'O y= Jtan(x) =arctan(/)= x
0
C: ('v' X E: ~. f (X) = 0) OU ('v' X E ~. f (X) = 1)
::J
0 Ceci prouve que l'équation y = f(x) a une unique solution
(comparer la place des quantificateurs avec notre première
lfl x dans~- Mais comme arctan(/) e: I, l'équation y= f (x) a
,-i
traduction de (E)).
0 en fait une unique solution x e: I: x = arctan(/).
N
•Synthèse(= conditions suffisantes).
@ Ceci prouve que f est bijective de I sur~+ (on Je savait
...., Réciproquement, si f est définie sur~ par 'v' x e: ~. f (x) = 0 ou déjà!) et don ne la fonction r
l :
.c rixe:~. f (x) = l, alors, de façon évidente, f est continue sur~
Ol
·;::
>- et f vérifie (E).
a.
• Conclusion. [o,~[
0
u
(E) a deux solutions: la fonction constante égale à 1 et la fonc- r l (y) = arctan(/)
tion constante nulle.

144
Fonctions, polynômes, continui té Chap,itre 6

- = = -
a) Posons f (x) arccos(x) et g(x) ~ arcsin(x). Les fonc-
tions f et g sont définies sur [-1, l] et à valeurs dans [0, TC]
(car arcsin à valeurs dans [ - ; , ; ]).
On a donc, pour tout x > 0, f (x)

'ri x > O,

• Six< 0, on pose x'


=g(x) et donc:
arctan(x) + arctan ( ~)

=- x > O. D'après ce qui précède:


= i
• On a, pour tout x e: [- 1, 1) :

cos([ (x)) = cos(arccos(x)) = x par définition de arccos arctan(x') +arctan(-!,)


X
= ~2
et:
et donc, comme arctan est impaire sur~ :
cos(g(x)) = cos ( i- arcsin(x)) = sin(arcsin(x)) = x

d'après les formules de trigonométrie usuelles et la défini-


- arctan(x) - arctan ( 1) = i
tion de arcsin. c'est-à-dire:
On a donc : 'rixe: [-1, l). cos(f(x)) = cos(g(x)).
• On sait que cos est injective sur [0, TC] (car strictement dé-
arctan(x) + arctan ( ~) =-i
croissante), donc, pour tout x e: [- 1, l i :
En conclusion, on a donc :
[(X) E: [0, TC] f cos injective sur (0,JT] six> 0
g(x) E: [0, TC) => f (X) =g(x) 'rixe:~·.
six<O
cos([ (x)) =cos(g(x))
On a donc, pour tout x e: [- 1, l). f (x) = g(x) et donc: c) • Utilisons la formule tan(a + b) = 1-tan(a)
tan(a)+tan(b) valable
tan(bJ •
dès que a, b et a+ b sont dans l'ensemble de définition de
. TC tan.
'rixe: [-1,1) arccos(x) + arcsm(x) =
2 Ici a = arctan} et b = arctan! donc (a, b) e: ]- ; , ~ [ . De
2

plus:
b) Posons f(x) = arctan(x) et g(x) = ; - arctan(±). Les fonc- 1 1 1
tions f et g sont définies sur~~ (car définies sur~*) et 0<-<-<-
3 2 v'3
alors à valeurs dans ]0,; [.
donc, corrune arctan est strictement croissante :
• On a, pour tout x > 0 :
O<a<b<~
tan([ (x)) = tan(arctan(x)) = x 6

et: et donc:
O<a+b<~
3
tan(g(x)) = tanrn - arctan(1)) • Ainsi on peut appliquer la formule :
1
'O
0 tan (arctan U))
C:
::J
tan(a + b) = tan ( arctan ( ~) + arctan ( ~))
0
lfl
,-i
TX ! ))+ tan (arctan {! ))
tan (arctan {
0
N
= X
1-tan ( arctan (})) tan( arctan(!))
@ On a donc : 'ri x > 0, tan(f(x)) = tan(g(x)).
...., l3 + l2
.c
Ol
·;::
• On sait que tan est injective sur ]o.~ [ (car strictement 1-! X!
>- croissante), donc, pour tout x > 0:
a. 5
0
= I
u
f(x) e:
g(x) e:
Jo, ~ [
]o, ~ [
f tan injective sur Jo,~ [
==> f(x) =g(x)
5
6
1

tan([ (x)) =tan(g(x))


145
Chapitre6 Fonctions, polynômes, continuité

Donc tan(a+ b) = 1 = tan(~l Soient (u 11 )n€N et (v11 )n€N deux suites définies respective-
ment par
• On sait que tan est injective sur]-~,~ [ (car strictement 7T 7T
croissante), donc: Un= - - +2nn Vn = - +2nn
2 2
Ces deux suites tendent vers +oo et f(u 11 ) =-eu", f(vn) = ev"

l
a+be:]-~.~[
tan injective sur ]- ~, ~ I 7T donc, par composition de limites :
îe:J-;,~[ ==> a+b=-
4
tan(a+b) = tan(~) lim /(un) = - oo lim f(v 11 ) = +oo
n-+oo n-+oo
On a alors e = -oo = +oo ce qui est absurde (il y a to11.1jours
On a donc:
unicité d'une limite).
Donc .f ne peut pas admettre de limite en +oo.
arctanU) +arctan(i) = arctan(!) = ~
•i~j
-
Graphiquement, on comprend bien Je problème :
import numpy as np
import matplotlib. pyplot as plt
E n-oo:
'v'x e: ~. -1 ~ sin(x) ~ l donc -ex~ f(x) ~ ex. def morceaux(x) :
if X<=û :
Or llin é = 0 donc, d'après Je théorème des gendarmes : return x
x- - oo
else:
lim f (x) ex~te 0
return 1-x
x--oo xl=linspace(-1,0,10)
yl=np .vectorize(morceaux)(xl )
En +oo: # 011 veut appliquer la fonction
Le sinus «oscille » entre - 1 et 1 mais cette fois ex tend vers # morceaux à un tableau
x2=linspace(0 . 01,1,10)
+oo. Observons ce que cela donne graphiquement: y2=np.vectorize(morceaux)(x2)
plt. plot(xl. yl)
plt . plot(x2, y2)
plt . show()

On obtient la figure:

,..

..,
0.0

u
0
C: - (i.<4
::J
0
lfl
,--i
0 Le dessin nous inspire : suivons les« Lignes de crête » et raison-
...
N
nons par l'absurde en utilisant des suites. -o.s - !l . f, -0. 4 - 11.2 ,., ,.,
@
...., En effet, supposons que f admette en +oo une limite e. finie
.c ou infinie.
Ol
ï:::: a) D'une part, 'v'y E (- 1,0]. 3x E (- 1,0J/ f(x) = y, en effet on
>- Alors toute suite (u 11 ) divergeant vers +oo est telle que la suite
a. (f(u11 )) tend verse. prend x = y et alors .f (x) = x = y.
0
u
Les« lignes de crêtes» correspondent aux valeurs telles que le D'autre part, 'v'y e:JO,l]. 3x e:JO,l]//(x) = y, en effet on
sinus est égal à 1 ou - 1. prend x = 1 - y et alors f (x) = 1 - x = 1 - (1 - y) = y.

146
Fonctions, polynômes, continuité Chap,itre 6

On en déduit que 'vyE [-1,1). 3xE (-1,1)/f(x) = y, donc On a - !:,; x:,; ! donc -1:,; 2x:,; 1 et donc, comme arcsin
que f est surjective de (- 1, l ) sur (-1, 1). est strictement croissante: - ~ :,; f (x) :,; ~.
Soient (x1,x2) E (- 1;1] 2. Supposons que f(x 1) = f(x2) •
De meme - 11 :,; x:,; 1ldonne - 0n :,; arcsm
· ()
x :,; n0 et - 4
n ::;
alors deux cas se présentent : . ( r,;2) t d . n Sn ( ) Sn rr
arcsm xv ~ :,; 4 , e one. - 2 ::; - 12 ::; g x ::; 12 :,; 2 .
1l

soit [(xi)= f (x2) E [-1;0) donc xi = x2 ;


• Ainsi on a pour tout x E [- !, ! ], (f(x),g(x)) e: [- ~. ~ ]2.
soit f (x1l = f(x2) E]O; 1) donc l - x1 = l - x2 i.e. xi = x2.
Alors, comme sin est injective sur 1-~. ~]
(car strictement
= =
Dans tous les cas, f(x1) f(x2) => XJ x2. On en déduit croissante) :
que f est injective. Étant aussi surjective, elle est donc bi-
jective de [-1, l) sur [-1, l).
b) D'une part f (0) = 0 et d'autre part lim f(x) = 1 : on en dé- (E) <=> 2x = sin ( arcsin(x) + arcsin (xv'Z))
x-o
x>O <=> 2x = xcos(arcsin(xv'2)) + xv'Zcos(arcsin(x))
duit que f n'est pas continue en O.
D'autre part, f est croissante sur [- 1; 0) et décroissante sur Or rite: (-1, l i, cos(arcsin(t)) = ~ -
JO; 1) donc f n'est pas monotone sur (-1; 1).
On en déduit que :
Remarque : ce contre-exemple montre donc que la conti-
nuité et la stricte monotonie de f sont des conditions suf-
fisantes mais non nécessaires de bijectivité. (E) <=> 2x = xVl-2x2 +xV2(1-x2)

''''
a) Soit fla fonction x ...... arcsin(x) + arctan ( v'I+x).
La fonction arcsin est définie sur [-1, 1) tandis que la fonc-
tion x ..... arctan( v'I+x) est définie sur [- 1,+oo[ donc f est
(El admet Ocomme solution évidente.

(E) <=> 2 = Vl - 2x2 + ,J2(1 - x2) ou x = o

Les deux membres del' équation étant positifs, on obtient :


définie sur [-1,l] .
Elle est composée et somme de fonctions strictement
croissantes donc elle est strictement croissante sur [-1, 1).
(E) 4 =1 - 2x2 + 2 - 2x2 + 2 J 1 - 2x2 J 2 - 2x2 ou x =o
X - 1 1
2x2 +! = J1-2x2J2-2x2 oux=O
!! + v12 2
Variations de
f 1T~2
-2 Les deux membres de l'équation étant positifs, on obtient :

La fonction f est donc strictement croissante et continue


sur [-1, 1) à valeurs dans [-~.~+arctan ( v'2) ].
Elle réalise donc une bijection de [-1,1) sur
[- ~.~ + arctan( VZ)].
(E)
2
( 2x + ~ r
= (1 - 2x2)(2 - 2x ) OU X
2
= 0

Ainsi: 4x4 + 2x2 + ! = 2 - 2x2 - 4x2 + 4x4 ou x = O


4
u
0
C: si te: [-~,~+arctan ( v'2) ], c'est-à-dire si x2 = !_OU X = 0
::J 32
0 t e: ]- oo, - i ] u [ 2 (./2) , +oo [, alors l'équation
lfl " n + 2arctan 2
,-i
admet une unique solution ;
0
N
sinon, l'équation n'admet pas de solution. i
• Or O< { 2 < donc O< Jfi < !- On en déduit que l'en-
@ semble des solutions est :
....,
.c b) • T.:équation est définie sur[ - !,! ] n [- 1, 1) n [- }i, 12 ]

0
Ol
·;::
>-
a.
c'est-à-dire [-!,!].
• Notons (E) cette équation.
Y--
{o - ~~}
'
- - 32' 32

u
Posons aussi f (x) = arcsin(2x) et g(x) arcsin(x) +
arcsin(xv'2).
+MEi
147
Chapitre6 Fonctions, polynômes, continuité

a) On obtient que P2 = 2X2 - l, P3 = 22x 3 - 3X. Montrons par Posons ak = cos(xk). La fonction cos étant strictement dé-
récurrence double que le degré de Pn est n et que son coef- croissante sur JO; n [ :
ficient dominant est 211- 1 pour tout entier naturel non nul
n.
l>ao>a1 >· ·· >an-1 > - 1
• Cette propriété est vraie pour n = 1 et n = 2.
• Si elle est vraie pour un certain entier naturel non nul n
et son successeur n + 1 alors Pn = 2n-lxn + Qn et Pn+l = Donc Pn (ak) = cos(nxk) = 0 pour n valeurs distinctes de
2nxn+l +On+l avec On et On+l des polynômes de degrés ak·
respectifs inférieurs à n - 1 et n. Ces n valeurs sont donc n racines distinctes de Pn. Or P11
On obtient que : est de degré n donc admet au plus n racines réelles (comp-
tées avec leur ordre de multiplicité).

Pn+2 2X(2nxn+1 + On+1) - 2n- 1xn - Qn Donc P11 admet exactement n racines simples qui sont
2n+2- lxn+2 +Qn.+2 donc

(2k + l)n)
avec On+2 un polynôme de degré au plus n + 1. ak=cos ( n ,avecke:{O,l,···,n-1}
2
La propriété est donc vraie au rang n + 2.
• D'après Je principe de récurrence double, la propriété est
donc vraie pour tout entier naturel non nul n.
b) On va procéder par récurrence double comme à la ques-
tion précédente.
• Pour tout x E llll, on vérifie que Po(cos(x)) = 1 = cos(O x x)
et P1 (cos(x)) = cos(l x x).
-
a) Comme lim
x-+oo
f (x) = eavec eE llll, alors

'v'E > 0, 3xo ?: 0, \i X> Xo,


:

f (X) e:J€ - E, € + E[
• Si la propriété est vraie pour un certain entier naturel n
et son successeur n + 1 alors, pour tout x € llll: y

€+E i -- - - - - - - - - - - - - - - -
Pn+2(cos(x)) 2cos(x) cos((n+ l)x) - cos(nx) € --------------------------
cos((n + 2)x) + cos(nx) - cos(nx) €- E
COS((n + 2)x)

!
sachant que cos(a) cos(b) = (cos(a + b) + cos(a - b)).
La propriété est donc vraie au rang n + 2.
• D'après Je principe de récurrence double, la propriété est xo X
donc vraie pour tout entier naturel n. 0

c) Résolvons dans un premier temps l'équation cos(nx) = 0 PosonsE = 1.


u dansllll.
0
C: Comme f est continue sur l'intervalle fermé borné [O;xol.
0
::J f est bornée sur cet intervalle et atteint ses bornes, d' après
lfl cos(nx) =0 3k e: Z/ nx = ~ + krr le théorème de continuité sur un segment.
,--i 2
0
N
(2k + J)n Or nous venons de voir que si x > xo, f(x) e: [€ - l, e+ li.
3 k EZ/x= - - -
@ 2n donc f est bornée sur Jxo; +=[.
....,
.c Nous en déduisons que f est bornée sur llt
Ol Or pour k e: {0,1, ·· · ,n - 1}, Xk = <2\+nl)1t prend n valeurs
ï::::
>- distinctes dans JO, n[ :
a. b) Cette fois lim
x-+oo
f (x) = +oo donc :
0
u
n 3n (2n - l)n
0 < - = xo < - = xi < x2 < · · · < Xn- 2 < Xn- 1 < 'v'A> 0, 3xo ?: 0, \ix> xo, f(x) >A
2n 2n 2n

148
Fonctions, polynômes, continu ité Chap,itre 6

y
b) M(v) = hv(J) et M(u) = hu(x) car hu et hv atteignenilleurs
bornes, d'après a).
donc: Tvl(v)- M(u) = hv(Y) - hu(x).
A Or hv(J) = max hv ~ hv(x) donc :
1-1 .11

M(v) - M(u) ~ hv(X)- hu(x)


~ f(x) + vg(x)- f (x)- ug(x)
~ (v - u)g(x)
xo X

De même, hu (x) = max hu ~ hu (y) donc - hu (x) ~ - hu (y)


1- 1,11
Posons A= If (O)I + 1 de tel sorte que A> 0 et A> f(O). etdonc :
La fonction f est donc minorée par A sur Jxo, +oo(.
D'autre part : f est continue sur l'intervalle fermé borné
M(v)-M(u) ~ hv(y)- hu(y)
[0,xo) : elle est donc bornée sur cet intervalle et atteint
ses bornes, d'après le théorème de continuité sur un seg- ~ f(y) + vg(y) - f (y) - ug(y)
ment. En particulier elle admet un minimum m atteint en ~ (v- u)g(y)
un point ex :
Finalement, en regroupant les deux inégalités :
\f x e: [O, xoL f (x)?. /(ex) = m ( *)
(v- u)g(x) ~ M(v) - M(u) ~ (v- u)g(y)
Remarquons que pour x = 0 : f (0) ?. f (ex) = m. Alors avec
notre choix de A, on obtient que six> xo, c) Comme x et y sont fixés, on a
f(x)?. A> f(O)?. /(ex) = m (** ).
lim (v - u)g(x) = lim (v - u)g(y) = 0
En regroupant (*) et (* *), on a : v-u v-u

'ri x?. 0, f (x) ?. f (ex) = m donc, d'après le théorème des gendarmes :

Ceci prouve que f admet un minimum global sur IR+, ou lim M(v) - M(u) ex!f te 0
autrement dit, que f est minorée sur IR+ et que sa borne v-u
inférieure est atteinte. Et donc par somme de limites
c) Comme lim
x-+oo
f (x) = eavec e> 0, alors :
existe
lim M(v) lim (M(v) - M(u)) + lim M(u)
'rle>O, 3xo~O, 'rlx>xo, f(x)e: Je-e,e+e[ v-u. v-u v-u
O+M(u)
En particulier, prenons E = !>0 : M(u)

3xo ~ O.'rlx>xo,f(x)e: [e;3e] ce qui est la définition de la continuité de Mau pointu.


22
Comme cette propriété est vraie pour tout u E IR, on a donc
'O
0 Si on fixe un réel xi > xo on a donc f(xi) ~ > O. Or ! démontré que M est continue sur IR.

-
C:
::J /(0) < 0 : f étant continue sur l'intervalle [0,xi] et chan-
0 geant de signe, le théorème des valeurs intermédiaires per-
lfl
,-i met d'affirmer que f s'annule au moins une fois sur (0, x1 )
0

-
N et à fortiori sur IR+. a) On reconnaît une célèbre formule trigonométrique en pre-
@ nant la fonction cosinus:
....,
.c 'rl(x,y) E !R2 , cos(x +y)+ cos(x - y) = 2cos(x) cos(y)
Ol a) La fonction hu est continue sur l'intervalle fermé borné
·;::
>-
a. (-1, l ) comme combinaison linéaire de fonctions conti-
0
u nues sur cet intervalle : eUe est donc bornée sur [-1, l) et
atteint ses bornes, d'après le théorème de continuité sur De plus cos est continue sur IR et cos rn) = 0 < 1.
un segment. Donc cos est un élément de E non identiquement nul.

149
Chapitre6 Fonctions, polynômes, continuité

b) Il est clait que g est continue sur !RI, comme composée de =


d) • On sait que f (0) 1 > 0 et que f est continue sur IR1 et ne
fonctions continues sur !RI. s'annule pas sur !RI.
D'une part, pour tout (x, y) e: IR1 2 : D'après le théorème des valeurs intermédiaires, une fonc-
tion continue qui ne s'annule pas est de signe constant
strict.
2g(x)g(y) 2f (ax)f (ay) On en déduit que Vx e: !RI, f (x) > O.
f(ax+ay)+ f(ax-ay) • Tout d'abord, pour tout entier naturel :
f (a(x +y)) + f (a(x - y))
g(x+ y)+ g(x - y)
Un+l = f (2 x 2na) = 2{f(2na)) 2 -1 = 2u;1 -1
* Nous avons montré que f était à termes strictement po-
donc g vérifie (i). sitifs donc :

D'autre part, soit xo e: !RI tel que f (xo) < 1 alors, comme a
est non nul, g ( 7J-) = f (a x 7J-) = f (xo) < 1 donc g vérifie
(ii) avec x~ = !. Montrons par récurrence que Vn e: N, un < 1.
Tout d'abord, uo = f (a) < 1.
Ceci prouve que g est un élément de E.
Ensuite, supposons que un < 1 pour un rang n e: N fixé.
c) • En appliquant (i) avec y= O: Nous savons que O < un < 1 donc u;1 < 1 et par suite
Un+l = 2u;;_ -1 < 2 x 1-1 = 1.
Vx e: !RI, f (X + 0) + f (x - 0) =2/ (X) = 2/(x)/(O) La propriété est donc héréditaire et vraie au rang O donc
vraie pour tout entier naturel n.
Or f n'est pas identiquement nulle donc il existe au
On a donc:
moins un réel xo tel que f(xo)-:/, O.
Vne:I\J, O<Un<l
Alors, f (xo) = f (xo)f(O) donne en divisant par f (xo) :
/(0) = 1. * Toujours pour tout entier naturel n :
En appliquant (i) avec x' = 0 et y'= -x, on a:
Un+l - Un 2u~ - Un -1

Vxe:IRI, f(O + x)+ f(O - x) = 2/(0)f(x) (Un - 1) (2un + 1)

Or O < Un < 1 donc Un+ 1 - Un < O.


Comme /(0) = l, on en déduit que Vx e: !RI, f(x) +
f( - x) = 2/(x). Finalement : La suite (un) est donc décroissante. Or elle est minor,é e par
0 donc elle est convergente, d'après le théorème de la li-
mite monotone.
Vxe:IRl,f( - x) = f(x)
• * La suite (un) est définie par une relation Un+ 1 = h(un)
Donc f est paire sur !RI. où h est une fonction continue sur !RI définie par h(x) =
2x2 -1.
"O
• En appliquant (i) avec x' =y'= f, on a: Or (un) est convergente et à valeurs dans )O, l [ : sa limite
0 est donc une solution sur (0, 1) de h(x) = x.
0
C:
::J
h(x) = x = 2x2 - x - l = 0 = (x - 1)(2x + 1) = 0
lfl Les points fixes de h sont donc l et-!. Comme - ! 1/. [0, 11,
,-i
0 On en déduit que on a donc : lim un = 1.
N
@
* Or Vne: N, 0 < un s u0 < l (suite décroissante) donc la li-
...., mite ede (u 11 ) vérifie O ~ e= 1 ~ uo < 1. Nous aboutissons
.c Vxe:IRI, f(x) =2(f(~})2-1
Ol donc à une contradiction.
·;::
>-
a. • D'après ce qui précède :
* Il n'existe donc pas de solution de (El ne s'annulant pas
0 sur !RI et telle qu'il existe a e: !RI tel que f(a) < l.
u
Ainsi, une solution de (E) ne s'annulant pas sur !RI vérifie
Vxe:IRI, f(x)+l =2(f(~))\~o forcément f (x) ;;:: 1 pour tout réel x .

150
Fonctions, polynômes, continu ité Chap,itre 6

- lT klT nlT
a) 0 na 0 < 2n+l < ··· < 2n+l < ··· < 2n+l < 2 ·
lT
La fonction f : x ...... cotan 2(x) est strictement décroissante
sur ]0;;] (comme inverse de la fonction tan2 qui est stric-
Ainsi:
P(X2) = ;i ((X+ i)2n+ l _ (X- i)2n+ l)

klT , on o b tient
d) • En posant ak = 2n+
{1,2,· · ·, n} :
1
. pour tout k e:
tement croissante) :
lT
X 0 2
-
1 (( cos(ak) ·)2n+
- - - +J -
1 (----1·)2n+ 1)
cos(ak)
+oo 2i sin(ak) sin(ak)
Variations de
f - - -1-2- -1 (( cos(ak) + 1sm(ak)
. . )2n+l
~o 2i(sin(ak)) n+
- ( cos(ak) - isin(ak) )2n+ 1)
On en déduit que
1 (ei(2 n+ l)ak _ e - i(2n+ l)ak)
2i(sin(ak)) 2n+l
t(-2nn+ l )>···>t(~)>···>!(~)>o
2n + l 2n + l 1 ( eiklT -e-iklT)
2i(sin(ak)) 2n+l
C'est-à-dire :
l (( 1ik ( 1)-k)
2i(sin(ak)) 2n+ l
x1 >x2 >··· >Xn 0
en particulier ces n réels sont deux à deux distincts.
b) P(X) = (2nt1)(- l)Oxn + Q(X) avec deg(Q) :( n - 1. On en déduit que P admet au moins n racines distinctes.
Or P est de degré n donc admet au plus n racines réelles
On en déduit que P est de degré n et de coefficient domi-
(comptées avec leur ordre de multiplicité).
nant 2n+ 1.
c) En utilisant la formule du binôme de Newton on obtient : Donc P admet exactement n racines simples qui sont les
Xk, k E [1, nn.
• Orle coefficient dominant de Pest 2n + l, donc :
(X+ i)2n+ l _(X_ i)2n+ l

2[ l (2n + l)ikx2n+l-k - 2[1 (2n + l)(- iikx2n+l-k


k=O k k=O k
P(X) = (2n+l) fr [x - cotan2 (2n+l
k=l
~)]

I:
2n+l(2n+l) (1 - (- 1ik)lx2n+1-k e) D'après la définition de P, le coefficient de Xn- l est :
k =O k

Or l - (-1) 2P = Oet l - (-1) 2P+ 1 = 2 pour tout entier natu- 1 (2n + 1) =- (2n + 1)(2n)(2n - l) = (2n+ 1)(2n - l)n
(- 1)
rel p. Donc dans la somme précédente on ne garde que les 3 3! 3
termes où l'indice k est impair: k = 2p + 1 avec p e: [o, nil
(les autres termes sont nuls). On en déduit que : D'autre part :
"O
0
C: 1
::J
(X+ 0 2n+ l _ (X _ 0 2n+ l --P(X) = (X-x1) x ··· x (X - xk) x ··· x (X - xn)
0 2n+l ·
lfl
,--i [ (2n+ 1)zi2p+lx2n+l- (2p+l) En développant, on voit que les termes de degré (n- li) sont
0
N p=O 2p + l
de la forme :
@
...., = [. (2n+l)2i(i2yx2(n-p)
.c
Ol p=O 2p+ 1 ..____..._.,
XX···XX x(-xk)x
. ..____..._.,
Xx ···XX =-xkxn
. -l
·;:: k-1 termes n -k termes
>-
0
a. 2i Ln (2n+l) (- 1)P (x 2) 2p + 1
u p =O 2p+ 1 En les regroupant on obtient que le coefficient de xn-l
n
2 dans zii+l P(X) est- L
= 2iP (x ) xk.
k=I

151
Chapitre6 Fonctions, polynômes, continuité

On en déduit que
n(2n - 1) (2n + 1) 2 ~ 1 n(2n - 1)
(2n+ 1)(2n - l)n f
- - - - - - = - (2n + l) L, xk 3
<
n
2 L. 2<n+
k=l k 3
3 bl .

d'où n 2 n(2n - l)
-
f
< L,
1
- 2 < n2
n
+
n 2 n(2n - l)
- ---
3 (2n + 1) 2 k=I k (2n+l) 2 3 (2n+1) 2
f n(2n - 1)
L, Xk =
k= l 3 n(2n - 1) 1 n
Or lim = - et lim =0
f) Écrivons la double inégalité proposée avec 9 = xk n - +oo (2n + 1)2 2 n -+oo (2n + 1)2
donc, d'après Je théorème des gendarmes :
1 2
cotan (~)<(Zn+ ) < 1 +cotan 2 ( ~ )
2
n I . ~2
2n+ 1 kn 2n+ 1 lim L -- ex!Jre .'. .'.__
pourtoutke: Œl,n] . n-+ook=lk2 6

Additionnons maintenant ces inégalités pour k variant de Euler füt le premier à établir cette formule en 1 735! On ne
1 à n: compte plus aujourd'hui le nombre de démonstrations dis-
tinctes découvertes par les mathématiciens...
n (Zn+ 1)2 n 1 n Pour aller plus loin :
L xk < 2 L 2 <n+ L xk http ://fr.wikipedia.org/wiki/Problème_ de_ Bâle
k=l 7T k=l k k=l

u
0
C:
::J
0
lfl
,-i
0
N
@
....,
.c
Ol
·;::
>-
a.
0
u

15 2
CHAPITRE

Dérivabilité des fonctions nu1né-


riques et développe1nents li1nités

'Thèmes abordés dans {es e~ercices


- Dérivabilité d' une fonction numérique
- Propriétés de la fonction dérivée
- Dérivées successives
- Développements asymptotiques/limités

Points essentiefs du cours pour {a réso{ution


des e~ercices
- Dérivabilité des fonctions usuelles
- Calculs de dérivées
- Théorèmes de Rolle et des accroissements finis
- Monotonie et signe de la dérivée
u
0
C:
::J
0
lfl
,-i
0
N
@
....,
.c
Ol
·;::
>-
a.
0
u

153
Chapitre 7 Dérivabilité, développements limités

Les méthodes à retenir

On se base sur la dérivabilité des fonction usuelles (attention à


la valeur absolue, aux fonctions puissances, qui sont en certains
points continues mais pas dérivables). Les opérations sur les fonc-
tions donnent encore une fonction dérivable.
<---+ Exercices 7.1, 7.2, 7.3, 7.8, 7.9, 7.11, 7.13, 7.15, 7.16, 7.17, 7.18, 7.19 et 7.20

Pour la dérivablité en un point, une première méthode consiste à


revenir à la définition (la limite du taux de variation doit exister et
Pour étudier la dérivabilité d'une être finie). Éventuellement on distingue dérivabilité à gauche ou
fonction à droite, si les nombres dérivées sont les mêmes alors la fonction
est dérivable en ce point (sinon on a un point anguleux).
<---+ Exercices 7.6 et 7.16

Une seconde méthode consiste à déterminer un développement


limité à un ordre supérieur ou égal à 1 de f au voisinage du point,
ce qui prouve la dérivabilité, donne le nombre dérivé et peut
même positionner la courbe par rapport à la tangente.
<---+ Exercice 7.7

- Le signe de la dérivée (au sens large) donne la monotonie de la


fonction. Le signe strict de la dérivée, donne la monotonie stricte
de la dérivée. De plus si la dérivée a un signe strict sauf en un
nombre fini de points d'un intervalle (où par exemple elle n'existe
pas), la fonction est strictement monotone sur tout l'intervalle (à
condition qu'elle soit continue sur tout cet intervalle).
<---+ Exercices 7.1, 7.2, 7.15 et 7.19

Pour étudier les variations d'une - Si la dérivée est nulle alors la fonction est constante, mais atten-
fonction tion ceci ne s'applique que sur un intervalle. Lorsque la dérivée est
nulle sur différents intervalles disjoints, la valeur de la constante
peut changer d'un intervalle à l'autre. De plus si la dérivée est
"O nulle sur un intervalle sauf en un nombre fini de points (où par
0 exemple elle n'existe pas) alors la fonction est constante sur tout
C:
::J
0 l'intervalle (à condition qu'elle soit continue sur tout cet inter-
lfl valle).
,-i
0
N
<---+ Exercice 7 .8
@
....,
.c
Ol
·;::
>-
a.
0
u

154
Dérivabilité, développements limités Chap,itre 7

- On commence par étudier les variations de f. On subdivise l'en-


semble de définition en sous-intervalles disjoints I1, •.• , In sur les-
quels f est continue et strictement monotone (donc bijective de
Ik sur Jk = f(Ik)) .
c.......+ Exercices 7.2, 7.18 et 7.19

On peut ainsi déterminer le nombre de solutions de f (x) = t: si


Pour étudier les solutions de l'équation t Eh alors on a une unique solution xk(t) dans Ik, sinon pas de
f(x) = t, en fonction de t E ~ solution dans Ik. Il faut ensuite faire la synthèse de ces résultats.
c.......+ Exercices 7.2, 7.18 et 7.19

Pour étudier la solution Xk ( t) en fonction de t sans la calculer, il


1
faut Utiliser le fait que Xk(t) = (/11kr (t) OÙ /iik désigne la restric-
tion de f à Ik.
<.......+ Exercice 7.18

- Si f est en au voisinage de x0 alors la formule de Taylor-Young


assure l'existence du DLn(Xo). A part pour de petites valeurs den,
cette formule n'est pas utilisable pour le calcul des coefficients.
<.......+ Exercice 7.13

La méthode classique de calcul consiste à faire des opérations sur


les développements limités usuels. On se ramène en O grâce aux
t.
changements de variables x = Xo + h ou x = Attention pour le
DLn(O) de go fil faut déterminer le DLn de g en f(O) (donc pas
toujours en 0).
Pour calculer un développement
<.......+ Exercices 7.4, 7.5, 7.7 et 7.19
limité/ asymptotique
- Si on sait que le DLnCxo) de f existe et si on dispose d'une équa-
tion fonctionelle vérifiée par f, alors on peut en déduire un sys-
tème d'équations vérifié par les coefficcients du DLn(.xo). La réso-
lution de ce système donne alors les coefficients.
<.......+ Exercice 7.13

Le théorème de substitution dans un développement limité


"O
0 donne souvent un développement asymptotique.
C:
::J <.......+ Exercice 7.19
0
lfl
,-i
0
N
@
....,
.c
Ol
ï::::
>-
a.
0
u

155
Chapitre 7 Dérivabilité, développements limités

- Le théorème de Rolle permet de montrer qu'une dérivée s'annule.


Appliqué "en cascade", on peut obtenir que la dérivée nème s'an-
nule.
<----,. Exercices 7.12 et 7.20

Le théorème des accroissements finis permet de relier les proprié-


tés de f' et de f. En particulier, un encadrement de J'(x) permet
Pour utiliser les théorèmes généraux d'avoir un encadrement de f(x) - f(y) .
sur les fonctions dérivables <----,. Exercice 7.10

Pour la dérivabilité d'une bijection réciproque il faut s'assurer que


f est dérivable mais aussi que f' ne s'annule pas. Si /'(x0 ) = O
alors 1- 1 n'est pas dérivable en Yo = f(xo), et sa courbe représen-
tative admet une tangente verticale en ce point. Sinon on a la for-
mule: u-l)'(yo) = rlxo) = f'of~ l(xo)"
<----,. Exercices 7.9, 7.13 et 7.18

"O
0
C:
::J
0
lfl
,--i
0
N
@
....,
.c
Ol
·;::
>-
a.
0
u

156
Dérivabilité, développe ments limités Chap,itre 7

Énoncés des exercices

---- - Étude de fonction


On considère la fonction f définie sur llll: par :

f (t) = t - ln(t) - -1
t

a) Étudier les branches infinies de f.


b) Faire une étude des variations de f et donner l'allure de la représentation gra-
phique de/.
c) Résoudre l'équation f (t) = O.

---- - Questions classiques sur les fonctions

a) Déterminer le nombre de solutions de l'équation lnx = i.


b) Établir que : 'v' x > 0, ln(x) ~ x - 1 et que ln(x) = x- 1 si et seulement six= 1.
=
c) On note h l'application de llll+ dans llll définie par h(O) 0 et pour tout x > 0,
h(x) = xln(x). Montrer que h est continue sur llll+ et tracer l'allure de son graphe
en précisant la tangente au point d'abscisse 1. Donner le nombre solutions de
l'équation h(x) = le·

---- - Calculs de développements limités


Déterminer les développements limités suivants.
a) DL2(2) de±·
l
b) DL4(l) de X Lnx- 1.
3 r-:,- 3 r-;,-
C) DL4(+00) de v x~ +x- v x3-x.
d) DL2 (1) de ln(l + Jx).
e) DL2(0)deé05<v'xl.
f) DL2 (+oo) de ;i/ xZ+x+
2
1.
V x +1
g) DL3(0) de ev'l+sinx_
u
0

0
C:
::J

lfl
,-i
0
N
---- - Calculs de limites à l'aide de développements limités
Déterminer les limites suivantes (en utilisant des développements limités).
a) lim ex2 - cos(x)
x-0 X
@
...., vi+x- v'Ï-x l
.c b) lim - - -
x-o X
--
Ol
·;::
>- c) lli sin(x)-x
a. x-11tan(x)(e2x+1- 2ex) ·
0
u d) lim ex+2sin(x)-l .
x-0 ln(l +X)+ v'l-tx- 1

157
Chapitre 7 Dérivabilité, développem ents limités

---- - Peut-on dériver une formule'? ...

a) Soit f une fonction dérivable en O telle que : f (0) = 1. Peut-on en déduire que
f'(O) =0?
b) Soient f et g deux fonctions dérivables en xo e: llll et égales au voisinage de xo.
=

---- -
Établir que:!' (xo) g' (xo).

Un développement limité permet d'étudier facilement un prolongement


=
On considère la fonction f définie sur lm* par: f (x) si~ x. Montrer que f est p,rolon-
geable par continuité en 0, que ce prolongement est dérivable et positionner courbe

---- -
et tangente au voisinage de O.

Nouvelle démonstration des formules pour les fonctions trigonomé-


triques réciproques

a) Vérifier que : 'v' x e: [-1, l i, arccos(x) + arcsin(x) = ~.


six> 0
b) etabli, quec Vxe o•, aœtan(x) +aœtan ( ~) = { _;;

---- -
si x<O

Étude de la stricte monotonie d'une fonction


On considère la fonction f définie par : f (x) = 1+Jxl .
a) Montrer que f est bijective de lm sur un intervalle à déterminer.
b) Déterminer l'expression de 1- 1(y).
c) Étudier la dérivabilité de f en O et positionner sa courbe représentative par rap-

---- -
port à sa tangente en O.

Constante d'Euler
On considère la suite (un) neN* définie par: Un = 1 + ! + -k + · · · + ti - ln(n) .
a) Vérifier que, pour tout n;, 1 : n~ 1 ~ ln(n + 1) - ln(n) ~ fi.
b) Montrer que la suite (un) 11; , 1 est décroissante et minorée par O.
c) En déduire qu'il existe un réel positif y (appelé constante d'Euler) tel que:

u
t'1 k n-=:..oo
n 1
ln(n) +y + o(l)
0
C:

0
::J
d) Donner un équivalent de I:
n
t quand n - +=.

---- -
lfl k=l
,-i
0
N
Application des développements limités à l'étude d'une fonction
@
...., On considère la fonction f définie par: f (x) =2x - 1- J x2 - 4x.
.c
Ol
·;:: a) Déterminer l'ensemble de définition de f et ses éventuelJes limites aux bornes de
>-
a. cet ensemble.
0
u b) Étudier les branches infinies de f, et préciser la position locale de la courbe par
rapport à ses éventuelles asymptotes.

158
Dérivabilité, développe ments limités Chap,itre 7

---- - Une application classique du théorème des accroissements finis


On considère une fonction f définie et continue sur Ili+, dérivable sur Ili: et telle que
f (0) =0 et f' est croissante sur Ili:.
a) Vérifier que, pour tout x > 0 :
f(x) ~ !'(x)
X

b) En déduire que g: x - f~x) est croissante sur Ili:.

---- - Théorème de Rolle en cascade


On considère une fonction f de classe C2 sur [a, b] et telle que /(a) = f(b) = O. En
considérant la fonction <p : t - f (t)-1 (t-a)(t-b), où A est une constante à choisir
judicieusement, établir que:

rlxE [a,b], 3cE]a,b[/ f(x) = (x-a~x-b) f"(c)

---- - Dévelopement limité d'une fonction réciproque (sans utiliser la dérivée)


On considère la fonction f définie par : f (x) = arctan (In(l + x) ).
a) Montrer que f est définie et de classe C00 sur J - l, +oo[.
b) Donner un DL3 (0) de f.
c) Montrer que f est bijective de J - l, +oo[ sur un intervalle à préciser.
d) Déterminer un DL3 (0) de 1-1 .

---- - Équation fonctionnelle


On souhaite déterminer toutes les fonctions f: Ili ---+ Ili, de classe C 1, vérifiant la
relation :
X
(E) : ri XE Ili, (/ o f)(x) = - + 3
2
a) Donner les fonctions affines solutions du problème
(i.e. les fonctions f: x - ax + b avec (a, b) E ~ 2 ).
b) Soit <p : x - i +3. On pose <po = 1~ et pour n ~ 1 : q, 11 = q,o<pn-l = <pn-l o<p. Pour
x E ~. vérifier que la suite (4>n(x)) n€N est arithmético-géométrique et déterminer
son terme général en fonction de n et x.

u c) Soit f solution du problème. Vérifier que: ri x E Ili, f (~ + 3) = f~x) + 3. En déduire


0
C:
que f' est constante sur~-
::J
0 d) Résoudre le problème posé.
lfl
,-i
0
N
@
....,
.c
Ol
·;::
>-
---- - Suites définies implicitement
On pose: rit> 0, g(t)
1
= 1e- ï .
a) Montrer que g peut-être prolongée en O en une fonction dérivable à droite en O.
b) Soit ne: I\J*. Montrer que pour n assez grand, l'équation (En) : g(t) =i admet
a.
0 deux solutions Xn et Yn sur Ili+ vérifiant: 0 < Xn < 1 < Yn·
u
c) Donner la monotonie et la limite des suites (x1i) et (Yn).

159
Chapitre 7 Dérivabilité, développements limités

---- - Une dérivée n'est pas toujours continue

On considère la fonction f définie pac [(x) 1x2


= sin( } )
0
si X f. 0
si x = O
a) Montrer que f est définie et continue sur~-
b) Montrer que f est dérivable sur~ et donner l'expression de .f' (x).
c) Montrer que lim f' (x) n'existe pas. Conclure.
x-o+

---- - Suite récurrente et accroissements finis


On considère la fonction f définie sur~: par:

=
x+ 1 x ln(x)
Si X 1" 1
.f(x)
1 x-I
1
2
Si X = 1

a) Montrer que f est C1 sur son ensemble de définition.


b) Montrer que: ri x > 0, ln(x) ~ x - 1. Construire le tableau de variations de f et
montrer que: ri x > l, f(x) < x.
c) Soit a> l. Montrer que la suite (xn) neN définie par xo = a et ri n € N, Xn+ 1 = f (xn)
est bien définie et à valeurs dans ] l, +oo[. Établir qu'elle converge et déterminer
sa limite e.

d) Montrer qu'il existe un entier no tel que:

En déduire que:
1
rln ~no, lxn-€1~
311
_n
0
1xn0 -€1

e) Endéduireque:xn =
n-+oo
€+o( 2~,)-

u
0
C:
::J
---- - Fonction définie implicitement
Soit a un réel positif ou nul. Pour x € ~. on pose Pa (x)
a) Montrer que ce polynôme admet une unique racine réelle u(a).
On note u l'application définie sur~+ qui a tout a associe le réel u(a).
=x3 + ax - 1.

0
lfl b) Montrer que : u(~+l ç ~:.
,-i
0
N c) Montrer que u est strictement décroissante sur~+.
@ d) Calculer u(O), puis lim u(a).
...., a-+oo
.c e) Déterminer l'application réciproque de u.
Ol
·;::
>- f) Tvlontrer que u est continue sur~+-
a.
0 g) Montrer que u est dérivable sur~+ et calculer u' (a), pour tout a E ~+-
u
h) Esquisser l'allure de la courbe représentative de u.

160
Dérivabilité, développe ments limités Chap,itre 7

---- - Développement asymptotique d'une suite définie implicitement

a) Soit n e: N* . Montrer que l'équation tan(x) = x admet dans l'intervalle


]-~ + mr,; + mr[ une unique solution, notée Xn.
lT
b) Montrer que Xn - mr. Remarquer que, pour tout n ;:: 1: Xn 2 -
n-+oo
arctan{] ) + mr. En déduire: Xn
n
- ~ - mr n-+oo
- - n\1.
·
c) Effectuer un développement limité à l'ordre 2, selon les puissances de de t,,,
x 11 - mr. En déduire un développement asymptotique à 4 termes de x 11 lorsque

---- -
n-+oo.

Théorème de Rolle généralisé


On considère la fonction f définie par :

1
/(x)=-12
+x

a) Montrer que f est de classe C00 sur Ili.


b) Soit ne: N. On pose, pour tout x e: Ili : P11 (x) = (1 + x2)n+l fCn) (x) . Établir que :
'tin E: N, Vx E: Ili, (1 + ,l)P~1 (x) =2(n + l)xPn (x) + Pn+l (x)
En déduire que, pour tout ne: N, P11 est un polynôme dont le terme de plus haut
degré est (-l)n(n + l)!Xn.

c) Soient a e: Ili et g une fonction continue sur [a, +oo[, dérivable sur Ja, +oo[, et qui
=
vérifie g(a) =O et lim g(x) O. On considère la fonction G définie sur [O, 1J par:
x-+oo

G:x-
g(lx + a - 1) six e:)0, 1)
{ 0 six= 0

Vérifier que Gest continue sur [0, l]. dérivable sur JO, l [. Montrer que G' s'annule
en un point de JO, 1[; en déduire que g' s'annule en un point de )a, +oo[.
d) Soit hune fonction continue sur J-oo, al, dérivable sur ) -oo, al, telle que h(a) =0
=
et lim h(x) O. Montrer que h' s'annule en un point de] - oo,a[.
x--oo
e) Montrer par récurrence sur n que le polynôme P11 admet exactement n racines
réelles distinctes.
"O
0
C:
::J
0
lfl
,-i
0
N
@
....,
.c
Ol
ï::::
>-
a.
0
u

161
Chapitre 7 Dérivabilité, développements limités

Du mal à démarrer ?

. . a) Rechercher une asymptote obli que en +oo. 11D a) Chercher les valeurs possibles de a et b.
b) Déterminer le signe de f'. b) Trouver une relation simple entre (j) 11 +l (x) et (j) 11 (x).
c) Uti liser le théorème de la bijection. c) Calculer de deux façons différentes f of of.
d) Combiner le a) et le c).
.. Uti liser le théorème de la bijection.
8l!a a) Déterminer lim g(t) puis revenir à la définition de la dé-
Ill Écrire f comme une composée de fonctions usuelles. Wlllil'
rivabilité.
r- o+

b) Utiliser le théorème de la bijection.


Ill Une fonction est équ ivalente au premier terme non nu l de c) Introduire Yt (respectivement yz) bijection réciproque de f sur
son DL. (0, l i (respectivement )1, +oo[).

. . a) Chercher un contre-exemple.
11111 a) Faire appel au théorème des gendarmes.
b) Revenir à la définition de la dérivabili té en O.
b) Reven ir à la défin ition de la dérivabil ité en xo.
c) Raisonner par l'absurde.

1111 Déterminer un DL2(0) de f.


fm a) Établir le développemen t limité à l'ordre 1 de f et de J'. b)
Ill a) et b) Dériver le m embre de gauche de l'égalité. Utiliser la concavi té de ln: '</x > 0, ln(x) ~ x - 1.
c) Montrer que l'i ntervalle I =)1, +oo[ est stable par f, puis étud ier

1111 a) Utiliser le théorème de la bijection.


la m onotonie de (x11 ).
d) Appliquer le théorème des accroissemen ts finis.
b) Résoudre l'équation y= f (x) d' inconnue x. e) Revenir à la définition du « peti t o » .
c) Reven ir à la définition de la dérivabilité en O.
ml a) Utiliser le t héorème de la bijection.
Ill a) Utiliser le théorème des accroissements finis. b) Calculer Pa (0).
b) Utiliser le a). c) Comparer Pa(u(a)) avec Pb(u(b)).
c) Montrer que (u11 ) converge vers un réel y. d) Résoud re Po(x) = 0 puis e ncadrer u(a).
n 1 e) Déterminer a en fonction de u(a).
L -k
k-1
f) et g) Remarquer que u = (u- 1 . r 1

d) lntuiter une suite (v 11 ) telle que lim - -- - = 1. h) Commencer par l'allure de la courbe représentative de z[ 1.
n-+oo v,,

GD a) Mettre x en facteur dans l'expression de f (x). 11D a) Utiliser le théorème de la bijection.

b) Déterminerer un DL(+oo) et un DL(- oo) de f. b) Se rappeler que arctan(x) + arctan ( ±) = ; pour x > O.

811 a) Appliquer le théorème des accroissements finis. c) Chercher un développement asymptotique de }n.
b) Utiliser le a).
11!1 a) Facile.
1111 a) Éc ri re f comme une composée de fonctions usuelles. b) Raisonne r par récu rrence.
b) Déterminer d'abord un DL5(0) de x - ln(l + x) et de arctan. c) Appliquer le théorème de Rolle à G.
u c) Uti liser le théorème de la bijection. d) Poser g(x) = h(- x).
0
C: d) Calculer de deux manières d ifférentes u n DL5(0) de 1- 1 o f(x). e) Uti liser le b), le c) et le d).
::J
0
lfl
,--i
0
N
@
....,
.c
Ol
·;::
>-
a.
0
u

162
Dérivabilité, développements limités Chap,itre 7

Corrigés des exercices

-
a) On remarque que f(t) =

d'après les croissances comparées que


Donc, par produit et sommes de limites:
t( 1- ";tl - tz ).De plus on sait
1

ln t
lim -
t-+oo t
= O.
t
Signe de
!' (t)
0

+
+oo

+oo

~
Variations de
f
lim f (t) ex{pe +oo - oo
t-+oo

Utilisons Python pour le tracé :


Étudions alors le comportement asymptotique de !.ip.. import numpy as np
Or !.ip. =1 - lnvl - tz.On obtient donc : import matplotlib.pyplot as plt

x =np.linspace(0.1, 10, 1000);


liin f (t) exgte
1 y=x -np.log(x)-1/ x
t-+oo t
plt. plot(x, y)
plt . show()
Étudions cette fois f (t) - t = - ln(t) - 7.
lim f (t) - t exgte -oo
t-+oo

Nous en déduisons que 'et admet une branche para-


bolique dans la direction y = x, mais pas d'asymptote
oblique.

Étudions à présent le comportement de f au voisinage de


0:

f(t) =- ~(1 + tln(t) - 2


t )

Or d'après les croissances comparées lim t ln (t) =0, donc


t-o+
par produit et sommes de limites:

lim f(t) exgte - oo


t-o+

La courbe CCf admet donc une asymptote verticale d' équa-


c) Le graphique nous laisse penser que f s'annule en 1 ce que
u
0
tion x =0 au voisinage de O.
C:
confirme le calcul :
::J b) La fonction f est deux fois dérivable sur!R: (car somme de
0 trois fonctions dérivables sur IR:). 1
lfl f(l) = 1- ln(l) - - = 0
,-i 1
0
N / 1 1 t2-t+l Or sur Ili:, f est continue, strictement croissante et change
@
'vt>O, f (t) = 1- - + - = - - -
t t2 t2 de signe, donc, d'après le théorème de la bijection, l'équa-
....,
.c =
tion f(t) 0 admet une unique solution sur IR:. Or nous
Ol Comme t 2 > 0, f 1 ( t) est du signe de t 2 - t + 1. connaissons une solution, l, c'est donc l'unique solution
ï::::
>-
a. Or t 2 - t + 1 est un trinôme du second degré dont le dis- de l'équation.
0 criminant est strictement négatif. Il est donc strictement
u
positif sur Ili. Nous en déduisons que la fonction f est stric- Sl'={l}
tement croissante sur Ili:.

163
Chapitre 7 Dérivabilité, développem ents limités

-
a) Introduisons la fonction f :x- ln(x) -
Elle est définie et dérivable sur JO;+=[ (somme de fonc-
tions dérivables).
±.

Au voisinage de O : lim f (x) = -= (somme de limites)


x-o+ -,
Au voisinage de += : lim
x- +oo
f (x) = += (somme de limites) _,

VX > 0, f
f
(x) = -XJ + 2X1 >0
-,
La fonction f est donc continue, strictement croissante et
change de signe sur JO;+=[.
-•
D'après Je thorème de la bijection, l'équation f (x) = 0 ad-
_,....____________________,
met donc une unique solution.
Or sur JO;+=(, f (x) = 0 <==> ln (x) = } . I.:équation ln(x) = }
admet donc une unique solution sur JO;+=[.
Au voisinage de O : lim g(x) =- = (somme de limites)
x- o+
X 0 a +oo
Signe de '
=
Au voisinage de += : g(x) x ( lnix} - 1 + donc ±)
/'(x) + '' + lim g(x) = += x (0-1 +O) = -= (croissances comparées
x- +oo
-----+oo et opérations sur les limites)
Variations de
f -----0
- oo
/ 1 1-X
Vx > 0,
g (x) = - - 1 = - -
x X
On peut avoir une approximation de la solution avec Py-
On en déduit Je tableau suivant :
thon:
from math import log
X 0 1 +oo
def f(x) : Signe de 1
return log(x)-1/x
g'(x) + (i)
1
-

import scipy. optimize as op

op.bisect(f,1,2)
Variations de
g ~o~
- oo - oo
On obtient une valeur approchée par la méthode de dicho-
tomie. On a utilisé le tableau de variation pour savoir que On voit sur ce tableau de variations que la fonction g est
la solution est dans le segment Il, 2). donc négative sur JO;+=[ et s'annule en un unique p oint:
u
0 b) Étudions cette fois la fonction g : x - ln(x) - (x - 1). Elle 1.
C:
::J est définie et dérivable sur JO;+=[ (somme de fonctions En effet:
0
lfl
dérivables). - sur JO, l J : g est continue et strictement croissante, donc
,-i
0
Commençons par l'observer avec Python : d'après Je théorème de la bijection, elle est bijective de
N
import numpy as np
JO, 1) sur g(]O, 1)) = Ili-. Or O€ Ili- donc l'équation g(x) 0 =
@ admet une unique solution sur JO, l J. De plus g(x) :S g(l) =
...., import matplotlib. pyplot as plt
0 pour x €]0, 1) .
.c
Ol - sur )1, +=I : g est continue et strictement décroissante,
ï:::: x=np .l inspace(0.1,10, 1000)
>-
a. y =np .log(x) - x+ l donc d'après Je théorème de la bijection, elle est bijective
0
u plt.plot(x,y) de ]1, +=[ sur g() 1, +=Il =~:'...
Or O, ~:'.. donc l'équation
pH.show() g(x) = 0 n'a pas de solution sur Jl, +=[. De plus g(x) <
=
g(l) 0 pour x €Jl, +=[.

164
Dérivabilité, développements limités Ch ap,itre 7

Doncsur!RI: =]O, l]u] l, +oo[.


g(x) s Oet l'équation g(x) = 0 y2=0.S*exp(-1)•np.ones((lOOO,l))
admet une unique solution. Comme g(l) 0, cette solu- = plt.plot(x,y)
tion est 1. plt .plot(x, y2)
plt.show()
Ainsi on a bien :
'v' x e: !RI:, ln(x) s x - 1 et ln(x) = x - 1 = x = 1.

c) • Les fonctions x ...... x et x - ln(x) sont continues sur !RI:,


donc h est continue sur !RI:, comme produit de fonctions
continues. ,.,.~ - - - - - - - - - - - - - - - - - - ~

On sait d'après les croissances comparées que


lim xln(x) =0 donc:
x-o+ ,. '
lim h(x) = h(O) ,.
x-o+
1.,
Par définition , la fonction h est donc continue à droite en
O. 1.

On a donc h continue sur !RI+. o. '

De plus : lim h(x) =


x-+oo
+oo.
• Les fonctions x ...... x et x ...... Jn(x) sont dérivables sur !RI:.
donc h est dérivable sur !RI:, comme produit de fonctions
dérivables.

'v' x > 0, h 1 (x) =1 + ln(x) • On voit sur ce tableau de variations que l'équation h(x) =
fe a une unique solution o: dans !RI+ et qu'elle vérifie o: > i-
. h(x) - h(O) existe .
De plus Ltm
x-o+ x- 0
= hm ln(x) =
x-o+
-oo. En effet:
Donc, par définition, h n'est pas dérivable à droite en O et
- sur [0, i]:
h est continue et strictement décroissante,
donc d'après le théorème de la bijection, elle est bijective
<(/f admet en O une demi-tangente verticale.
[o. i] sur h([o. i]) = [-i,o]. Or ie fl [-i,o] donc
Pour x > 0 : h1 (x) ~ 0 = ln (x) ~ - 1 <=> x ~ e- 1
de
l'équation h(x) = ie n'a pas de solution sur [0, i].
On en déduit le tableau suivant :
- sur ] i, +oo[: h est continue et strictement croissante,
X 0 ! a +oo
e donc d'après le théorème de la bijection, elle est bijec-
Signe de
li'(x)
- .
Q + ..
:
+ tive de ]i,+oo[ sur h(H,+oo[) = ]-i,+oo[. Or }e E:
Variations de 0~ 1 ------- +oo
]-i, +oo[ doncl'équation h(x) = ie admet une unique so-
lution o: sur]}, +oo[.
h
_ l ------- ïë
'
u Comme h'(l) = 1 et h(l) = 0, la tangente au point d'abs- Donc sur !RI+ = [0, i] u H, +oo [, h(x) 2~ admet une
0
C:
::J
cisse 1 a pour équation unique solution o:. De plus o: > i.
0
lfl y=x-1 Python nous donne une approximation de cette solution :
,-i
0
N from math import log,exp
Python nous donne l'allure de la courbe et de la droite
@
...., d'équation y =ie : def f(x) :
.c return x*log(x)-1/(2•exp(1))
Ol from math import exp
·;::
>- import numpy as np
a. import matplotlib. pyplot as plt import scipy.optimize as op
0
u op.bisect(f,1,2)
x=np.linspace(0.01,3,1000)
y=x•np.log(x)

165
Chapitre 7 Dérivabilité, développem ents limités

-
a) On commence par se ramener à un problème au voisinage
de Oen posant
1
1- ln(l+h)
h2 h3 h4

h4
2

h2 h3
4
1 + h - - + - - - + o(h )
3
u
4

u=x- 2
+ h 2 + - -2h- +2h- + o(h 4 )
4 2 3
u2
Alors :
h2
+ h 3 -3h2 - + o(h 4 )
2
U3

+ h4 +o(h4 )
'--..,.-'
u4

h2 h3 h4 4
On pose alors h = ~ = x 22 - 1+ h + - + - + - + o(h )
h-0 2 3 6
On obtient donc :

X Hh~l) 1-
1
1 - ln(l + h) h-o
=h2 h3 h 4
- h - - - - - - +o(h4 )
2 3 6
~ (1- h + h 2 + o(h 2 ))
Posons à présent v = 1 - h
1_ 10 +h).
2 On a lim v = O.
1( X- 2 (X - 2)
- - 1- -- +- - - + o(x- 2) ) h-0
2
x- 2 2 2 4
Ore v = l + v + 2v 3 v4 + o(v4 ).
+ 6v + 24
v-0
1 x- 2 (x - 2) 2
- - - - - + - - - + o(x - 2) On en déduit que :
x- 2 2 4 8

h2 h3 h4 )
b) Il va falloir procéder avec ordre.... A l+ -h- - - - - - +o(h4 )
h-0 ( 2 3 6
Tout d'abord, ramenons le problème au voisinage de O en
V
posant x = 1 + h. 2 3
h2 h4 2h h 2h h
+ _ + _ + __2_ + _ ·_3_ + o(h4)
2 8 2 2
_ 1_ v2 /2
A = xln(xl - 1
.J!!l&_ fz3 3h2 (-h2)
e ln(xl- 1 -- + 2 +o(h4)
6 6
= e~
(X) 1
v3 16
"O = el+ ln(.:l- 1 h4
0
C: + - + o(h 4 )
e 1+ ln(l}hl - 1 24
0
::J
--.....-
lfl el- 1-1n/i+hJ v4/24
,-i
0 fz4
N - 1 - h + - + o(h 4 )
h-0 12
@
...., 2 3 4 (x-1)4 ( 4)
.c Oru = ln(l + h) = h - !:!:.... 4 - 1 - (x - 1) + - - - + o (x - 1)
Ol h-02 + !:!:....
3 4 + o(h ) et lim u = O.
- !:!:....
h-0 x-1 12
·;::
>-
a. De plus I~u = 1 + u + u 2 + u3 + u 4 + o(u4 ) au voisinage c) Ramenons-nous au voisinage de O. Pour x > 0:
0
u
u-o
de O.
On en déduit que :

166
Dérivabilité, développements limités Chap,itre 7

Posons u =~ - On a bien lim u = O. Donc


x x- +oo
Comme x est en facteur, il faut un développement à l'ordre
5 en } des racines cubiques. Mais u = ~ donc il suffit
ln(l+Jx} 1n(1 + Vl+h)
d'avoir des développements limités à l'orc&e 2 en u.
ln(2)+ln(l+ ~ - l )

1 1 !(!-1)
3 3 Orona:ln(l+u) = u - ~ +o(u2 ),
2
(1 + u) :ï - l+ - u+ u 2 +o(u 2 ) u-o
u- 0 3 2
1z2 v"J+h - l
-
u- 0
1 1 2 2
l+ - u- - u +o(u)
3 9
!EJi::l h-0
= -4h - TI, + o(h
2
2) et: Jim
h-0
=O.
Donc en composant ces deux développements limités:
On en déduit que :
h h2 h2 2
ln(2) + - - - - - +o(h )
1 1 1 1 2 2
4
"-..--'
16 ._.,_.
32
(1 + u) :ï - (1 - u) :ï -
u-o
l +- u - - u +o(u)
3 9
u u2/2
2
h 3h
- ( l - l u- l u 2 + o(u2 ) ) - ln(2) + - - - + o(h 2)
h- 0 4 32
3 9
2
- - u+o(u2 ) Finalement :
u-0 3
ln(l+vx) = ln(2)+ x-l - ~(x - 1)2 +o((x -1) 2 )
x-1 4 32
Comme u = 4-:
X
e) Commençons par le cosinus. Pour avoir un développe-
ment à l'ordre 2 en x, il faut un développement à l'ordre
3~_3C1
v1- -zi
V1T-zi -
x-+oo
~+o(~)
3x2 x4
4 de cos(y) et substituer Jx à y.
2 4
On a cos(y) = 1 - ~ + {4 + o(y 4 ) et lim Jx = 0, donc par
y-0 x-o
- ~+o(~) substitution :
x-+oo 3x2 x5
(..fi) 2 ( ..fi)4
car, la fonction x - (/1
+ ~2 - (/1- ~ cos( vx) - 1- - - + - - +o(CvxJ2)
2 est paire et de x-o 2 24
classe c5 au voisinage de +oo, donc la formule de taylor- X x2 2
Young nous permet de passer à un DL5( +oo) et nous donne 1- - + - +o(x )
x-o 2 24
que Je coefficient d'ordre 5 est nul.
Ensuite, on remarque que :
Par suite :
éos(v'x) = ex éos( v'x)- 1

- Comme lim cos( vx)-1 = Oet:


u x-+oo x-0
0
C: u2
::J -
x-+oo eu= l+u+ - +o(u 2 )
0 u-0 2
lfl
,-i
0 d) Ramenons-nous au voisinage de O en posant x = I + h. On en déduit que :
N
@ Alors ln (1 + Jx) = ln ( 1 + v"î+"h).
....,
.c
Ol
ï::::

0
>-
a.
Or:

x- 0
- e[1-~ +
'-..-"
2
x2 +
24
x
8
2

._.,_.
+o(x2)]
u h-0 u u2/2
ex ex2 2

h- 0
-
x-o
e-- + - +o(.r)
2 6

167
Chapitre 7 Dérivabilité, développements limités

f) Transformons un peu l'écritme pour revenir au voisinage Alors :


deO:

eJl+sin(x) ex e J 1+sin(x) -1
=
-
x-o
e 1 + -X - -x2 - -x + o(x )
2 8 48
3
3]
[
Remarquons ensuite que - 1-
l+~ x-+oo
= 1- ...!.. +
x2-
o(J,_)
x 2 car
X
. 1
hm - =O.
x- +oo x
On en déduit que :

x2 + x + 1
-
x2 + 1 x-+oo

-
x-+oo

-
x- +oo x-o
J
Or (1 + u) li = 1 + 1 u - 1 u 2 + o(u 2 ) et -
u-o g g x-0

-
2 1
lirn x + x + - 1 =O. donc :
x-+oo x2 + 1

2 a) À l'aide des développements limités usuels on obtient :


3 x 2 +x+l 3 l+ (x +x+l) -1
x2 + 1 x2 + 1

- l+_!_ __l_ +o( _!_) ex2 -cos(x) 1 + x2 + o(x2l - (1 - f +o(x2l)


x- +oo 3x 9x2 x2
x2 -
x-o x2

g) Ona: ~ +o(x 2
)
-
x-0 x2
sin(x) = x- x3 + o(x3 ) 3
x-0 6 - - + o(l)
x-0 2
Sachant que v'I+u = 1 + u - ! fi 2
u + / 6 u3 + o(u 3 ), et que 2
=
lim sin(x) 0, on obtient :
x-o
d li
one x!..11
ex - cos(x) exi ste 3
x2 = 2

l
b) À l'aide des développements limités usuels on obtient :

~
v11 +x = 1 + lx - lx2 + ...!..x3 +o(x3 )
"O x-O 2 8 16
0
C:
J1 +sin(x) 1+
3
( X - ~ : o(x ) v11 - x = I-!x - }x2 - 1~x3 +o(x3 )
::J x-o
0 On en déduit que :

~ (x +
lfl
,-i
0 2 3 3 3
N - o(x )] + _!_ ( x + o(x )]
8 '-,--' 16 ' - , - - ' x+kx3+o(x3) - 1
@ u2 u3 v'î+x-X /Gr - 1
...., X
.c x x2 x3 3 X x-0 X
Ol
·;:: = 1+ - - -
2 8
- - + o(x )
48 1
-x+ o(x)
>-
a. x-o 8
0 2 3
u De plus ev = 1 + v + L + L + o(v3 ) et
v- 0 2 6 /I+x-JI=x 1
---- x - existe li 1
lim JI +sin(x)- 1 =0 d one liIn = m - x= 0
x-o x-0 X x-08

168
Dérivabilité, développements limités Chap,itre 7

c) À l'aide des développements limités usuels on obtient :

sin(x)-x =
x- 0
x-~x3-x+o(x3)=-!x3+o(x3)

Donc: sin(x) - x -
6

_!x3.
6
-
a) Si f est dérivable en O avec f (O) = l, alors on peut juste dire
que:
lim f (X) - 1 f' (0)
x- 0 X
=
o 6
De plus: Considérons alors f : x - x + 1. On a bien f (O) = 1 mais
f' (0) = 1 ,t o.
On ne peut donc pas dériver une formule valable seule-
4 ment en un point.
e2 x+l-2ex = l+2x+ x2 +1-2(1+x+ x2)+o(x2 )
x- 0 2 2 b) Puisque f et g sont égales au voisinage de xo, il existe a> 0
tel que :
=x
x-o
2 2
+o(x )
'v' x € ]xo -a,xo +a[, f(x) = g(x)

ce qui nous donne e 2x + 1 - 2ex - x2. On en déduit que :


0
D'autre part: tan(x) - x.
0 f(x) - f(xo) g(x) - g(xo)
'v'x€ Jxo-a,xo [u]xo,xo +a[,
Donc par opérations sur les équivalents : x - xo x - xo

1 Or f et g sont dérivables en xo donc les limites en xo de


- - x3
sin(x) - x 6 f(x) - f<xo ) et de g(xl-g(xol existent.
x- xo x - xo
6
D'après ce qui précède, elles sont égales.

On en déduit que : Ainsi, f' (xo) = g' (xo) , On peut donc dériver une fOJmule
valable sur un voisinage du point.
lim sin(x) - x extJte 1 Dans le contre-exemple du a), si on appelle g la fonction
x-o tan(x) ( e2x + 1 - zex) 6 identiquement égale à l, f et g étaient égale en O mais pas
sur un voisinage de O.
d) À l'aide des développements limités usuels on obtient :
- f est dérivable sur or comme quotient de fonctions
dérivables.
ex +2sin(x)-1 - 1 + x + 2x - 1 + o(x) En utilisant le développement limité de la fonction sinus au
x-0
3x+ o(x)
voisinage de Oà l'ordre 3 on obtient :
x-o
f(x) = 1- !x2 + o(x2 )
x-o 6
X On en déduit que lim f (x) = 1. On peut donc prolonger la
ln(l +X) + Vl+x - 1 -
x- 0
x+ 1 + - -1 + o(x)
2
x-0
fonction f en O en posant f (0) = 1 (le coefficient d'ordre O).
'O
0 3
- x + o(x) Comme le développement limité est à un ordre supérieur ou
C:
::J x- 0 2 égal à l, on en déduit que f est dérivable en O et que f' (0) 0 =
0
lfl (le coefficient d'ordre 1). La tangente à 'Ilf en O a donc pour
,-i On en déduit que :
0 équation y = 1.
N
eX +2sin(x) - l 3x De plus, f(x) - 1 - - àx2 < 0 donc la courbe représentative
@ x-o
...., - -=2 de f est en dessous de sa tangente au voisinage de O.
.c ln(l + X)+Vl+X - 1 x-0 ~X
Ol
·;::
>-
a. Donc Mi
0
u lim eX + 2 sin(x) - 1 extJte a) Introduisons la fonction f définie sur (- 1; li par
2
x-o ln(l + x) + v'l + x - 1
f (x) =arccos(x) + arcsin(x)

169
Chapitre 7 Dérivabilité, développem ents limités

Cette fonction est dérivable sur ] - 1; 1[ (comme somme On en déduit que f est strictement croissante sur~ (c'est
de fonctions dérivables) et continue sur (- 1,1] (comme la continuité en Oqui nous permet de passer de~· à~).
somme de fonctions continues).
Or f est continue sur~-D'après le théorème de la bijection,
1 l 1 f est donc bijective de~ sur] lim f(x) , lim f(x) [.
Vxe:] - 1,1(, f(x)= - =0 x--oo x-+oo
Vl-x2 Vl-x2
Or lim /(x) = - 1 et lim f(x) = 1.
x--oo x-+oo
On en déduit que f est constante sur [- 1, l] (c'est la conti-
nuité qui nous permet de repasser à l'intervalle fermé). Finalement, f est bijective de~ sur l - l; 1 [.
Or f (0) = ~. Finalement, on a donc : b) Soit y E [O, l [, alors

• 7T
Vx e: [- 1, l], arccos(x) + arcsm(x) =
2 y;;,,Odonc x;;,,o 1
b) Introduisons la fonction g définie sur~* par
Y= f(x) = y=l+x
y+xy-x =0
g(x) = arctan(x) + arctan ( i) = X = _2'.__
1-y
x = -y-
i
La fonction x ...... est dérivable sur~· et la fonction arctan 1-IYI
est dérivable sur~. On en déduit que f est dérivable sur~·
(comme somme et composée de fonctions dérivables).
On obtient de même que si y e:)- 1, 0(. alors x = i!iyl.

1 1 1 1 1 On en déduit
VxE~*, g1 ( x ) = - - - + - x - - = - - - - - =
l+x2 x2 1+ \ l +x2 2
x +1
X
vy e:1-1. l[. ricy) = ---1'._I
1- y
1
On en déduit que g est constante sur chaque intervalle
] - oo; 0( et JO; +oo( (attention~· n'est pas un intervalle, et
c) Étudions la dérivabilité de f en O :
le théorème utilisé ne s'applique que sur un intervalle).
Or g(l) = 2arctan(l) = 2 xi=~ et de même g(-1) = -~ . f(x) - f(O) f(x) 1
On en déduit que : Vx ;é 0,
x- 0 =7 = l+lxl

1
Vx E ~ • , arctan(x) + arctan (1)
- = { l!six>O
2 Or lim - - = l, donc:
x-0 1 +lxl
X 1T •
-2 SI X<
0

. f(x) - f (0) existe


1Jill = 1
81 1 1 x-o x-0

a) La fonction .f peut être définie par morceaux :


On en déduit que f est dérivable en Oet que f' (0) = 1.
u
0
C:
::J
Une équation de la tangente en O à la courbe représenta-
0 f(x)= { 1:x six ;, o tive de .f est donc y = x.
lfl
,--i l ~x si x<O
0 Or pour tout x E ~· :
N
Donc f(x) > 0 sur JO, +oo(. f (x) < 0 sur ] - oo,O[ et f (0) = O.
@
...., f est continue sur ~ (quotient de fonctions continues) et six>O
.c
Ol dérivable sur Ili* (on ne sait rien en O car x ...... lxl n'est pas
ï:::: Si X <0
>- dérivable en O) :
a.
0
u
_ l >0 si x>O On en déduit que la courbe est au-dessus de la tangente
f'(x) = (l~x)z sur 1- oo; 0( et en dessous sur JO; +oo[.
{
(1-x)z > 0 si x<O
170
Dérivabilité, développements limités Chap,itre 7

-
a) Soit fla fonction définie sur [0; +oo[ par f (t) = Ln(l + t).
Soit n:::: 1 fixé quelconque.
On remarque que ln(n + 1) - ln(n) = 1n ( n7i 1 ) = f (fi).
d) Revenons à la définition du « petit o » : il existe une suite de
réels (Enln€N de limite nulle et vérifiant

'v'n e: 1\1,
n 1
L k =ln(n)+y+En
k=I

Comme f est dérivable sur [O; +oo[, on peut appliquer


n l
l'égalité des accroissements finis à f sur [0; 11 existe ti]. On conjecture que [ -k - ln(n). En effet, par simples
k= I +oo
donc ce: [O; fi] vérifiant : opérations sur les limites:

f (;;)- f(O) = ;/' (c) ln(n) +Y+ En existe


liIn = 1
n- +oo ln(n)
C'est-à-dire
ce qui est bien la définition de :

t(;;) - n(/+c) [
n 1
- - ln (n)
OrO~c~idonc n~I ~ cli ~l.Onendéduitque: k=I k +oo

1
--~! ( -1) ~-
1

Donc:
n+l n n Ili••
1 1
a) Étudions le signe de x - J x2 - 4x :
'v'n:::: l, - - sln(n+ 1)- ln (n) s -
n+l n 0 4

b) Comme Un+i - un = nl i - ln(n+l)+ln(n) pour tout n e: 1\1*, + +


on obtient, en utilisant le résultat de la question a)
La fonction f est donc définie sur] -oo;O] u [4; +oo[. Pour
tout x;;,: 4 :
On en déduit que (un) est décroissante.
Or ln(k + 1) - ln (k) ~ "t pour tout k e: 1\1* donc f(x) = 2x -l-lx1J1 - ~ =x(2- ~ - J1 - ~)
n n 1
[ ln(k+ 1) - ln(k) = ln(n + 1) - ln(l) ~ L k = Un+ ln(n) On en déduit que lim
x- +oo
f (x) = +oo.
k=I k= I
De plus, par somme de limites : llin
x - -oo
f (x) = - oo.
car la somme de gauche est télescopique. On en déduit
que: b) • Étudions le comportement de f au voisinage de +oo à
l'aide d'un développement limité.
0 ~ ln (n+ 1)-ln(n) ~ Un
"O
0 donc 'v'n e: 1\1*, Un;;,: O.
C:
::J
0 c) La suite (u 11 ) étant décroissante et minorée, le théorème
lfl de la limite monotone nous donne qu'elle converge vers
,-i
0 un réel positif y, et on peut donc dire que
N
@ Un = y+o(l)
...., n-+oo
.c
Ol On en déduit que
ï::::
>-
0
u
a.
f~
k=I k
Un + ln(n)

n- +oo
ln(n) +y+ o(l) Ainsi f(x) - (x+ 1) = ~ + o ( t ).
171
Chapitre 7 Dérivabilité, développements limités

On en déduit que la courbe représentative de f admet la


droite d'équation y = x+ 1 comme asymptote au voisinage
de +oo et se trouve au-dessus de cette asymptote au voisi-
10
nage de +oo.

• Étudions Je comportement de f au voisinage de - oo à


l'aide d'un développement asymptotique.
·5

f(x) 2x - 1 - lxl J
1- ~
-10

-15

-
a) Soit x > O. La fonction f étant continue sur (0, xi (car conti-
nue sur~+) et dérivable sur JO, xi (car dérivable sur~~). on
peut appliquer le théorème des accroissements finis :

Ainsi f(x) - (3x-3) = -i + o ( ±). 3c E:)0,x[/ f (x) - f (O) =f' (c)


x-0 ·
On en déduit que la courbe représentative de f admet la Or f (O) = 0 et comme f' est croissante sur~~ :
droite d'équation y = 3x - 3 comme asymptote au voisi-
nage de -oo et se trouve au-dessus de cette asymptote au Ü<C<X = f 1( C) ~ f' (X)
voisinage de - oo.
, . f(x)
On en dedmt que pour tout x > 0, -· - ~ f 1 (x).
• Observons à l'aide de Python: X
b) Comme f est dérivable sur~~. la fonction g l'est aussi.

import numpy as np
"' 0
I f' (x) - f (X)
~ ' 0
import matplotlib. pyplot as plt vX> , g'(X ) -- xf (x) 2- f(x) -- .:;:;
X X
xm=linspace(-5,0,500) On en déduit que g est croissante sur Ili~.
xp=linspace(4 , 10,600)
u
0
C:
::J
def f(x) :
return 2*x- l -np.sqrt (x**2 - 4*x)
UtJ • Remarquons tout d'abord que, pour tout c e:
]a, b[:
0
lfl
,-i y1 =np.vectorize(f)(xm) (a-ak(a-b) f" (c)
0 y2=np.vectorize(f)(xp) /(a) 0
N
f(b) 0 (b-ak(b-b) !" (c)
@ plt .plot (xm,f(xm))
....,
.c plt.plot(xp, f (xp)) Donc six = a ou x = b alors tout choix de c E: [a, bl convient .
Ol plt.plot(xm,3*xm*np.ones((SOO,l)) - 3)
ï:::: • Soit maintenant x e:Ja; b[ fixé et quelconque.
>- plt.plot(xp,xp•np.ones((600 , 1)) +1)
a.
0 plt .show() Choisissons A afin que 4>(x) soit nul :
u
2/(x)
(j)(X) = 0 ~ A=-----
(x-a)(x- b)

172
Dérivabilité, développements limités Ch ap,itre 7

Or on a aussi <.p(a) = I (a) -0 = 0 et <.p(b) = I (b) - 0 = O. c) La fonction x - ln(l + x) est continue et strictement crois-
Comme <.p est dérivable sur [a;b) puisque I l'est, il existe sante de J - 1; +oo[ dans~-
die: ]a;x[ tel que <.p1 (di) = 0 d'après le théorème de Rolle et La fonction arctan est continue et strictement croissante
il existe d2 e: Jx; b[ tel que <.p1 (d2) = 0 pour la même raison. Re- de~ dans ]- ; ; ; [.
marquons que di< d2.
La fonction I est donc continue et strictement croissante
Or <.p1 est dérivable sur [a; b] puisque I est de classe C 2 sur cet de) - 1; +oo[ dans ]- ; ; ; [.
intervalle. 1T 1T
Il existe donc c e:Jd1;d2(c)a;b[ tel que <.p11 (c) = 0, toujours lim
x --1
I (x) = - - et lim
2 x-+oo
I (x) = -2 . On en déduit le tableau
d'après le théorème de Rolle. suivant :
Or: X - 1 +oo
A A

.~~
I
rite: [a,b]. =I I (t) -
2 (t - a) - 2 (t -
<.p (t) b)
Variations de
et f
rit e: l a, b],
11
<.p (t) =I" (t) - A -2

.
0 no btient done.f" (c) = A. 0 r A = (x- 2/ (X) b) .
a)(x- D'après le théorème de la bijection, la fonction f est donc
2fi\~-bl bijective de J - l; +oo( sur J-;;; [.
On en déduit que I" (c) = (x- d'où le résultat.
d) La fonction I étant de classe C00 sur J -1; +ool, de dérivée
Attention ! Vous noterez que c dépend de x (« pour tout x il
existe un c... ») donc I n'est pas une fonction polynomiale de strictement positive (f'(x) = l~x x l +ln 2\ 1 +x) > 0), sa réci-
proque est elle-même de classe C00 sur ]- ; ; ; [.

-
degré 2 car f" (c) n'est pas constant.
La fonction 1- 1 admet donc un développement limité à
tous les ordres au voisinage de 0, d'après la formule de
a) La fonction x -. ln(l + x) est définie et de classe C00 sur Taylor-Young.
J - 1; +ool et à valeurs dans~- On peut donc poser 1- 1 (x) = a + bx + cx2 + dx3 + o(x3)
x- 0
La fonction arctan est définie et de classe C00 sur~- avec (a, b, c, d) e: ~ 4 .
On en déduit que I est elle-même de classe C00 sur Or r1 (f(x)) = x et d'autre part comme 1 co) = 0 (car r
J - l;+ool. l(O) = 0), on peut composer le développement limité de
b) Commençons par déterminer un développement limité de 1-1 avec celui de I :
arctan au voisinage de O.
1
r 1 2
(f(x)) = a + bl(x) + cl (x)+dl3 (x) + o(x3)
Comme arctan'(u) = - 2 = J-u 2 +o(u2 ), on en déduit x- 0
1 +u u-0
que On en déduit que:
u3
arctan(u) = arctan(O) + u- - + o(u 3 )
u-o 3 X - a + bl (x) + cl 2 (x) + dl3 (x) + o(x3 )
x- 0
Donc finalement
x2 ( 2)2 + d (x - x2 )3 + o(x3 )
2 )+ c x -
u u3 a+ b(x - :
0
C: arctan(u) = u - - + o(u3 ) x-o 2
::J u-o 3
x2 x3
0
lfl - a+ bx-b - + cx2 -2c - + dx3 + o(x3 )
,-i Deplusln(J+x) = x - i + f +o(x3 ). x-o 2 2
0 x-0
N
On peut donc composer ces deux développements limités -
x-o
a + bx + ( c - ~)x2 + (- c + d)x3 + o(x3 )
@
...., au voisinage de O:
.c Donc par unicité de la partie régulière d'un développe-
Ol
·;:: ment limité :
>-
a. x2 x3 x3
0 l(x) X - - +- - - + o(x3) a =O
u x-o 2 3 3
x2 3 b=l
- x- - + o(x)
x-o 2
c- q= O
-c+d = 0
173
Chapitre 7 Dérivabilité, développements limités

Finalement La suite (vn)ne:N est donc géométrique de raison ! et de


= =
premier terme vo <.p0 (x) - 6 Id~(x) - 6 x - 6. =
r 1(x) = x+ -x2 + -x3 + o(x3 )
x-o 2 2 Ainsi: 'v' n E 1\1, Vn = x2-;,6 et on en déduit que

-
a) Soit f une fonction affine définie sur~ par f : x - ax + b
avec (a, b) E ~ 2 . Alors f est de classe C 1 sm ~- De plus :
'v'nEl\l,'v'xE~. <.pn(x) = zn:- + 6
x- 6

c) Comme f est une solution de (E). On a donc f of = <.p puis

f of of= f o<.p =(f of) of = (j> of


f solution de (E)
1 ce qui s'écrit encore :
'v'xE~, fof(x) = x + 3
2
1
'IXE~, a(ax+b) + b = - x+3
2
1
'v'xE~, a 2 x+b(a+l) = -x+3
2 Dérivons membre à membre cette égalité (/ est dérivable
sm ~). On obtient:
<::>
unicité des cocffs
dans RIXI
{a - !2
2-

b(a+ l) =3 'v'x e: ~. ~!' (~ + 3) = !11(x)


2 2 2

(a,b) = { 1;3(2 - Jz)) c'est-à-dire :


ou
{ f' = f'o<.p
(a,b) =(- 1;3(2+ Jz))
Une récurrence immédiate permet alors d'obtenir qu e:
(E)admet donc deux solutions affines:
X-. 1
X+ 3 ( 2 - v'2) et X ...... -1
X+ 3 ( 2 + v'2).

Comme on a raisonné par équivalence, on est dispensé de


vérifier que ce sont bien des solutions. c'est-à-dire :
b) Soit x E lll1 fixé quelconque. Alors, pour tout entier naturel
n : 'v' n E 1\1, 'v' XE ~, f t (X) = f ' (6 + zn
X-6)
1
<.pn+l (x) = (j> ( <.pn (x)) = -<.pn (x) + 3 x-6
2 Fixons ensuite x E Ili quelconque. On a lirn 6 + - P
2•
6.
n-+oo
=
La suite (<.pn (x)) nEN est donc bien arithmético- Or f' est continue au point 6 car continue sur Ili (f est de
géométrique. classe C1 sur ~l donc :
u I.;équation e= ! + 3 a pour solution e= 6.
0
C: Introduisons donc la suite (Vn) ne:N définie par : lim !' (6 + x - 6) ext]te !' (6)
::J n-+oo zn
0
lfl
,-i
'v' n E 1\1, Vn =<.pn(x) - 6 et donc par unicité de la limite f' (x) =f' (6).
0
N Alors, pour tout n E 1\1 : Finalement :
@
...., l;f XE 1Jl1, t' (X) = !' (6)
.c 1
Ol
ï:::: Vn+l - <.pn(x)+3 - 6 ce qui nous donne que f' est constante sur Ili, et donc que
>- 2
a. f est une fonction affine.
0 !(<.pn(x)-6)
u 2 d) D'après a) etc), les solutions de de (E) sont les fonctions :
1
2vn x-1 x+3(2-J2) et x- -1 x+3(2+ v'2).
174
Dérivabilité, développements limités Chap,itre 7

-
a) Tl est clair que g est définie et dérivable sur~: (produit de
fonctions dérivables).
On remarque que:
l'équation g(t) = ti
admet donc une unique solution
dans ]O, 1[ pour n ~ 3.

De même, la fonction g étant bijective de J l, + oo[ sur


l'équation g(t) = iadmet une unique solution Yn dans
Xn

]o. 1[.
] l, +oo[ pour n ~ 3.
v't>O, g(t) =- ( - i)e-f Donc sur IR?+ = [0, J]u] 1, +oo[, ]"équation g(t) = ,!; admet
exactement deux solutions Xn et Yn, pour n ~ 3.
1
Or lim - - = -oo et lim xé = 0 (croissances comparées)
1- 0 t x--oo De plus g(O) = 0 ;é ,!; et g(l) = 1;é ,!;, donc:
1>0
donc
lim g(t) ex[pe 0 O<xn < 1 < Yn
1-0
1>0
c) • On note Yl la réciproque de g sur [0, 1].
La fonction g peut donc être prolongée en O en une fonc-
tion continue à droite, en posant g(O) = O. On a pour tout n ~ 3, g(xn) = fi donc Xn = Yl (-!; ).
De plus, on a alors:
Or Yl est strictement croissante comme g donc
v't > 0, g(t) - g(O)
t- 0
= g(t)
t
= (-~)\-f
t

1
Or lim - - = -oo et lim x 2 ex = 0 (croissance comparées)
1-0 t x--oo
1> 0 La suite (x11 ) n:::3 est donc décroissante. Or elle est minorée
donc par Odonc, d'après le théorème de la limite monotone, eUe
lim g(t) - g(O) ext]te converge.
0
1- 0 t-0
1>0
De plus, la fonction y 1 est continue sur [0, ~] (car réci-
Par définition, g est donc dérivable à droite en O et~ (O) = proque d'une fonction strictement monotone et continue),
o. donc
b) Étudions la fonction g sur [O, + oo[.
1
On a lim - - = 0 et lim xex = oe0 (par continuité en 0)
t-+oo t x-0
lim Yl
n-+oo
(.!.)
n
= Yl { lim .!.) = Yl (0) = 0
n- +oo n
donc
lim g(t) exgte 0 car g(O) = O.
t- +oo

De plus: On en déduit donc que


1
e- ï
v' t > 0, g'(t) = t3(1- t) lim Xn =O
n -+oo
'O Donc g' (t) est sur~: du signe de (l - t). On en déduit le
0
C: tableau de variation suivant : • En introduisant Y2 la réciproque de g sur ]l, +oo[, qui
0
::J
est continue et strictement décroissante sur ] 0, i [,
une dé-
lfl monstration analogue permet alors de montrer que (yn)
,--i 0 Yn +c'
0 Signe de est croissante.
N
g' (t) + +
@ 1 De plus lim g(t) = o+ donne lim y2 (t) = +oo et donc,
...., Variations de 1 - - - - - - ë -------... 1 t- +oo r-o+
.c
Ol
·;::
g
0
n •-------...
0 puisque Yn = Y2 ( i) :
>-
a.
0 La fonction g est continue et strictement croissante sur
u [0, l]. D'après le théorème de la bijection, elle est donc bi-
lim Yn
n-+oo
= +oo
jective de [O, 1] sur [0, 1]· Comme i E [ 0, } ] pour n ~ 3,

175
Chapitre 7 Dérivabilité, développements limités

-
a) Tl est clair que f est dérivable donc continue sur ~*,
comme produit et composée de fonction usuelles déri-
vables.
En conclusion, on peut donc dire que lim f'(x) n'existe
pas.
x-o
Remarquez que l'expression de f' (x) n'est pas définie en 0,
et n'admet même pas de limite en O. Pourtant la fonction
est bien dérivable en O. Cet exemple illustre le fait que cal-
Étudions la limite de f en O.
culer l'expression de J'(x), puis donner son ensemble de
définition pour savoir où f est dérivable, est un raisonne-
Vz e:~. - 1,,,;sin(z),;,;1 ment complètement faux! (et qui vous attirera les foudres
du correcteur). On ne peut pas non plus passer à la limite
donc
dans l'expression de .f' (x) pour étudier la dérivabilité aux
bornes de l'ensemble de définition de f.
Vxe:~*. -x2 ,;,;f(x) ,;,;x2
2
Or Lim x = Odonc, d'après le théorème des gendarmes,
x-0 iifi
a) • La fonction/ est continue sur JO; l (u] l; +oo[ comme quo-
lim f(x) exgte O = f (O) tient et produit de fonctions continues.
x-o
Au voisinage de 1, utilisons un développement limité à
Par définition, f est donc continue en O. Ainsi .f est conti- l'ordre 1 de f (x) en posant x = 1 + h avec h tendant vers
nue sur~. 0 (un équivalent suffirait pour la continuité, mais ce déve-
loppement limité nous sera utile pour la dérivée) :
b) La fonction f est dérivable sur~*. Étudions la dérivabilité
en O.
(h + 2) ( h - ~ + o(h2))
Vx f. 0, f (x) - /(O) = xsin (.!.) (h + 2) ln (l + h)
f (x) = ___2_h_ __
X-0 X h-0 2h
Une démonstration analogue à celle du a) permet de prou- 1 + o(h)
h- 0
ver que
lim f (x) - f (0) exgre donc lim f (x) exgte 1 = f(l) .
0
x-0 x-0 x-1
Par définition, .f est donc dérivable en O et .f' (O) =O. Par définition, f est donc continue en 1.

La fonction f est donc dérivable sur~- On en déduit que f est continue sur ]O, +oo[.

De plus: • La fonction f est dérivable sur ]0, 1[ et sur )1, +oo[,


corrune quotient et produit de fonctions dérivables.
Vx f. 0, !' (x) = 2xsin (~)-cos {~) Pour x e:]O, l [U] l, +oo[ :

. , existe - '( x)= x-1-ln(x) 1


c) Par l'absurde supposons que hm f (x)
x-o
= e
e: ~- ! (x - 1)2
--
2x

"O
On a vu que lim xsin
x-0
(.!.)
X
=0, donc par sorrune de limites: La fonction/' est continue sur JO; l[u]l; +oo[, comme quo-
tients et sorrunes de fonctions continues. Au voisinage de
0
C: l, utilisons un développement limité de /' (x) en posant
- O- "n -_ - e
1) ex~te
::J
0 limcos ( - x = 1 + h avec h tendant vers O :
lfl x -0 X
,-i
0

l
N Mais alors par composition de limites :
@ f 1(1 + h)
...., h - ln(l + h)
.c liln - = O+ x=l .
Ol z-. +oo z1
1)
z "· ( ) exi ste
==> wn cos z = -
e h2
- ---
2(1 + h)
·;::
>- lim cos ( - =- e z- +oo 1 2 1
h-(h- ~ + f)+o(h 3 ) 1
a. x-o+ x
0 h2 - (1-h+o(h))
u
Ceci est absurde puisqu'on sait que cos n'admet pas de li-
h-0 2
1
mite en +oo. - h+o(h)
h-0 6

176
Dérivabilité, déve loppe ment s lim ités Chapitre 7

On en déduit que lim .f' (x) exgre lim !!_ = 0 = f' (l), Ja der- On a montré que Vx > 0, ln(x) ~ x - 1 donc
x-i h- 0 6
Ilière égalité provenant du développement limité à l'ordre
\J /'( ) x+ 1 ln(x) x+ 1 x+x
1 de/. vX> 1 X =--x--~--<--=x
' 2 x- 1~ 2 2
La fonction f' est donc continue en 1, et donc sur JO, +=I.
La fonction f est par conséquent de classe ci sur 10, +=I c) D'après Je tableau de variations précédent, l'intervalle
et f'(l) = O. ) 1, += I est stable par f.

b) • Linégalité V x > O,Jn(l + x) ~ x se montre classiquement Comme xo e:]1, +=I, une récurrence immédiate nous
en étudiant la fonction x ...... x - ln(l + x) sur~;. donne que:
• Étudions la limite de f en +oo : Vne: N, Xn e: )1, +=I
x +1 ln(x)
lim - -
x-+oox - 1
= I et x-+oo
lim - - = +oo donc, par produit de
2 Or on a montré que 'rfx > l, f(x) < x. Pour x = Xn > 1 on a
limites, donc:
lim f (X) exgte += 'rlne:N, Xn+i =f(xn)<Xn
x-+oo·
Étudions la limite de f en O : La suite (xn) est donc décroissante, or elle est minorée par
. x+ 1 ln(x) I : elle est donc convergente, d'après le théorème de la li-
hm - - = -1 et Jim - - =
x- o x - 1 x- o 2
-= donc, par produit de li- mite monotone.
mites,
lim f (x) exgte +oo La suite (x11) est définie par la relation Xn+i = /(x71 ) avec
x- 0 Xn e: ] l, +oo[ pour tout entier naturel n. Or f est conti-
nue sur [l, +=[ et (Xn) converge. On sait alors que (Xn)
• On a vu que :
converge vers un point fixe de f appartenant à (1. +oo[ : le
2 seul point fixe est 1 (car f (x) < x six> 1) donc, finalement,
vXE: l 0,1 [U l l,+(X),
\J [ f''(x) = x - l - 2xln(x)
2x(x - 1) 2 on obtient que
lim Xn = 1
11- +oo
On en déduit que f' (x) est du signe de g(x) =x 2 - 1-
2xln(x). d) Soit n e: N fixé quelconque. La fonction f est continue sur
Or g est dérivable sur~: et ll;x11 l et dérivable sur ll;x,if (car ci sur~:). D'après le
théorème des accroissements finis, il existe donc un réel
'rfx>O, g'(x) =-2(ln(x) - (x - l))?O en e:] 1; Xn [ tel que :

La fonction g est donc croissante sur ]O; +=[ :


Xn+ i -1 = f(Xn) - f (1) = !' (C11) (Xn -1)
X 0 1 +=
Or 1 < en < x 11 , pour tout n E N, donc, d'après le théorème
Signe de
+ + des gendarmes,
g'(x)
. existe
11m
n-+oo
Cn = 1
Variations de
g 0-------- et donc puisque f' est continue en 1 (/ est C1 sur~:):
'O
0

0
C:
::J
--------
On remarque que g(l) = O. On en déduit le tableau de va-
lim J'(cn)exg te f'O)=O
n-+oo
lfl riations de f :
,-i
0
On en déduit que :
N X 0 1 +oo
@
...., Signe de
- Q + 3no e: N, 'rln? no, If'(cnll ~ ~3
.c
Ol
f' (x) 1
·;::
>- +oo +oo Alors, en combinant avec le théorème des acroissements
a. finis, on a donc trouvé un entier no tel que :
0
u
Variations de
f
~1/ 'rfn? no, lxn+i -
1
l i ~ - lxn - l i
3

177
Chapitre 7 Dé rivabilité, développem ents limités

e) Une récurrence immédiate permet alors d'en déduire c) Soit (a, b) e: llll; tel que a< b alors :
que:

1 Pa(u(b)) (u(b)) 3 +a · u(b) - 1


Vn ~ no, lxn - li ~ - - lxn0 - li
311- 110
= (u(b)) 3 + a · u(b) + b · u(b) - b · u(b} - 1

On en déduit que = Pb(u(b)) + (a - b)u(b)


(a- b)u(b)

Vn::: no, lxnl-11 ~(~)11 x 311olxno - l l


2" 3 '--v---'
ne dépend pas de n Or a - b < 0 et u(b) > 0 donc Pa ( u(a)) >Pa ( u(b)).
La fonction Pa étant strictement croissante sur llll, o n a
Comme lim
2)11 = 0 car Î < l, on en déduit, d'après
n-+oo (-
3
I 1 donc u(a) > u(b).
le théorème des gendarmes, que : Ceci prouve que la fonction u est strictement décroissante
surllll+ .
. lxn - 11 existe
hm - - - = 0 d) Le nombre u(O) est la racine positive de Po(x) = x3 - 1 : on
n-+oo ...!..
2" a donc
u(O) = 1
ce qui signifie que
De plus Pa(u(a)) = 0 =
a· u(a) = 1 - (u(a))3 < 1 car
u(a) > O.
Xn =
+oo l+o{~) 211 On en déduit que, pour tout a> 0, 0 < u(a) < i.

-
Donc d'après le théorème des gendarmes :

a) La fonction Pa est dérivable sur IJll (fonction polynôme)


avec e) Comme Va ~ 0, u(a) > 0 alors :

Vxe:llll, P~(x)=3x2 +a 1- (u(a)) 3


Pa(u(a)) = O= a =----
u(a)
Donc pour x t O: P~(x) >a ~ O.

Pa est donc strictement croissante sur IJll (c'est la continuité Deplusu(llll+)=] lim u(t),u(o) ] =JO,lJ .
t-+oo
qui nous permet de passer de llll* à llll).
Ainsi, l'applicatio n réciproque de u est définie par :
Or lim Pa (x) = +oo et lim Pa (x) = - oo. On en déduit
x-+oo x- -oo
Je tableau suivant : JO; lJ llll+
l - t3
X - oo u(a) +oo -t-
Signe de 1

"O P~(x) + 1
1
+ f) La fonction u- 1 est co ntinue et strictement décroissante
0
C: sur JO, l l (car u strictement décroissante), donc on sait que
::J ----+oo sa réciproque ( u- 1 r 1
= U est co ntinue sur llll+ .
0
lfl
,--i
0
N
Variations de
Pa
- oo ----0 g) La fonction u- 1 est dérivable sur JO, l J et

@
....,
.c
D'après Je théorème de la bijection, Pa étant continue,
strictement croissante et changeant de signe sur llll, l' équa-
VO< t ~ 1, (u-1)' ( t ) =t2-1 - - 2 t =1-+-t22t3
-<0
Ol tion Pa (x) = 0 admet une unique racine u(a) sur llll.
ï::::
>- La dérivée de u- 1 ne s'annulant jamais, on en déduit que
a. b) Quelque soit a ~ 0, Pa(O) = -1 < 0 et Pa(u(a)) = O. Or Pa
0 u dérivable sur u- 1 (10, l J) = llll+.
u est strictement croissante sur llll donc u(a) > O.
D'après la formule de dérivation de la réciproque d'une
Ainsi u(llll+) ç llll:. fonctio n dérivable on a :

178
Dérivabilité, développements limités Chap,itre 7

b) • Comme Xn e: In :
1 (u(a)) 2
ria;,,: 0, u'(a) =- - - - - 7T 7T
(u- 1)' (u(a)) 1 + 2 (u(a)) 3 rlne:N*, - -2 + nn ~
~
Xn ~ -
~ 2 + nn
Or (u(a}) 3 = 1- a· u(a) donc donc
7T Xn 7T
, (u(a>2) rlne:N*, --- + 1 ~ - ~ --+ 1
ria;,,: 0, u (a) = - - - - - 2nn nn 2nn
3 - 2a · u(a)
D'après le théorème des gendarmes on en déduit que
h) On trace la courbe représentative de u- 1 et on déduit celle . Xn existe
de u par symétrie par rapport à la première bissectrice : hm - = 1 donc que
n-+oc nn
1 Xn - nn
1 +oc
1
1
1 • De plus Xn tan(x11 ) donne arctan(xn)
1
1 <t'u- I arctan (tan(x,i)).
1
1
1 Mais par n-périodicité de la fonction tan:
1
1
1
1 arctan(tan(x11 )) = arctan (tan (Xn - nn))
1
1
1 Comme Xn - nn e: ]- ~; ~ [, arctan (tan (x11 - nn)) = Xn - nn.
1
1 Donc:
1
1 rln ~ 1, arctan(x11 ) = x 11 - nn
1
1
1 1
\
D'autre part arctan(x11 ) + arctan ( 1»)= ; (car x 11 > 0).
On en déduit donc que:

0 X rln ~ l, x11 =~-arctan{_!_) + nn

-
1 2 Xn

1
• On a montré que lim
n-+oc
Xn = += donc lim -
n-+oo Xn
=O.
a) Introduisons la fonction gn : x ...... tan(x) - x définie sur
In = ]- ;+ nn;; + nn[. Or arctan(u) - u et donc en substituant -x1 à u, on obtient:
0 n

Cette fonction est dérivable sur In et


arctan ( - 1 ) - - 1 - - 1
rixe: In, g~ (x) = 1 +tan2 (x) - 1 = tan2 (x) Xn +oo Xn +oc rm

On en déduit donc que:


Doncg~1 (x) >0 si xe: 111 \ {nn}.
La fonction g11 est donc strictement croissante et continue 7T 1
u
0
sur ln. Xn -
2 - rm +-;, - nn
C:
::J Or lim g 11 (x) = - = et lim g 11 (x) = += donc g 11 c) • Comme Xn = ; - arctan (},,) + nn, on en déduit que
0 x--f+mt x-f+nn
lfl change de signe sur In.
~ = - arctan{_!_)
,--i
0
x,,
Xn - nn -
N X -J+nrr nrr J+nrr 2 Xn
@ Signe de
~ :

---0
+ + +
...., g;,(x) Or arctan(u) = u + o(u2 ) donc
.c _ _ _ . +oo 0
Ol Variaùons de
·;:: g,, _ _ . . - 1111
>-
a. -oo Xn-nn-~ = _ _!_+o(_!_)
0
u
2 +oc Xn x;,.
On en déduit, d'après le théorème de la bijection, que g 11
s'annule en une unique valeur de I 11 • Sachant que Xn - rm
+oo
on en déduit que o {:k-)
x,.
= o (~) .
n

179
Chapitre 7 Dérivabilité, développements limités

Ainsi : • On remarque que


Xn - nn - ~ = _ _!__ + o(_!__)
2 +oo Xn n2
Po(X) = (1 +X2) · ~ =1 = (- 1)0 (0+ l)!X0
l+X
1T 1
• De plus Xn - -2 - rm - - - donc
+oo nn donc la propriété est vraie pour n =0 ;
1T Supposons qu'il existe un entier naturel n tel que la pro-
Xn =
+oo
nn+ - + o(l)
2 priété soit vraie au rang n.
On en déduit que : Alors on peut poser :

PnCX) = (- l)n(n + l)!Xn + Rn- 1 (X)


1
Xn
=
+oo nn +; + o(l) avec Rn- 1 polynôme de degré n - 1.

:, L k1l
Pn+l e: ~[X) car:
+oo + ,~ ~. ( Vxe:~. Pn+1(x) = (1 +x2)P~(x)-2(n+ l)xPn(x)
= _!_(1 - _!_+o(!)) et Pn e: ~[X).
+oo nn 2n n
On a ensuite :
1 l 0 ( l )
=
+oo nn - 2nn 2 + n2
Pn+ l (X)
• Finalement on obtient :
2
(1 + X )P~(X) -2(n + l)XPn (X)

Xn - nn - ~2 +oo
= -
1
_!_ + - - + o { _!_)
nn 2n n2 n2
(1 + x 2 ) ( (-l)n(n + l)!nxn-l + R~_ 1(X))
- 2(n + l)X ((- l)n(n + l)!Xn + Rn-1 (X))
c'est-à-dire
( (-1) 11 n(n + 1)! - 2(- l)n (n + l)(n + 1)!) xn+ l

Xn
+oo
n ---+--+o -
= nn+
2 nn 2n n2 n2
l 1 (1) +QnCX), avecdegQn ~ n
(-l) 11 (n + l)!Xn+ 1 (n - 2n - 2) + QnCX)

-
a) La fonction x ...... 1 + x2 est de classe C00 sur~ et strictement
positive. Son inverse f est donc aussi de classe C00 sur~-
b) La fonction Pn est dérivable sur Ili car f est de classe C00

(- l)n+l(n+2)!Xn+l +Qn(X)

avec deg On ~ n.
La propriété est donc héréditaire or elle est vraie pour
n = 0 donc elle est vraie pour tout entier naturel n.
sur Ili. c) Six e: ]O, l] alors ±+a -
l e: [a, +oo[. Or g est continue sur
[a, +oo[ donc Gest continue sur JO, li (par composition de
fonctions continues).
V X e: ~, P~ (x) l
Or lim - + a - 1 = +oo et fun g(u) = 0 donc, par com-
"O (n + 1) x 2x(l + x 2 )n J'n) (x) + (1 + x2 )n+l J<n+l) (x) x-o+ x u-+oo
0 position de limites, on a
C:
::J
0 donc
lfl =
fun G(x) exgte O G(O)
,-i x-o+
0
N
VXe:~, (1 + x2)P~1 (X) On en déduit que G est continue en O donc continu e sur
@ [O, l].
...., (n + 1) x 2x(l + x2 )n+l/nl(x) + (1 + x2)n+2 /n+l )(x)
.c
Ol La fonction G est de même dérivable sur J0, 1[ car g est dé-
·;:: 2(n + l)xP n (x) + Pn+ 1 (x)
>- rivable sur) a, +oo[.
a.
0 Montrons par récurrence que Pn e: ~[X) et que son terme Or G(O) = 0 et G(l) =g(a) =O. G vérifie donc les cond itions
u
de plus haut degré est (- 1) 11 (n+ l)!Xn, pour tout entier na- d'utilisation du théorème de Rolle : il existe donc un réel
turel n. c e: JO; 1( tel que G' (c) = O.

180
Dérivabilité, développe ments limités Chap,itre 7

Mais G' (c) =-ti g' (i + a - l) =0 donc g' (i + a- 1) =O. En effet, f'n ) est continue sur [an; +oo[, dérivable sur
]a11 ; + oo[ et s'annule en an . Il reste à vérifier que
Etc E JO; 1[ donne t + a - l E Ja, +oo[. lim f'n )(x) =O.
x-+oo
On a donc montré que g' s'annule en un point de l'inter-
valle Ja, +oo[. Il suffit d'appliquer le b) :

d) Il suffit d'appliquer le résultat précédent en posant Pn(X)


Vx ~ - a, g(x) =h(- x) (1 +x2)n+I
(- l)n (n + l)!xn
donc Vx > - a, g' (x) = - h' (- x). n-±oo x2n+2
On obtient que g' s'annule en un réel de l'intervalle (- l)n(n+ 1)!
J - a, +oo[ donc que h' s'annule en un réel de l'intervalle xn+2
J -oo,a[.
e) Montrons par récurrence sur l'entier naturel n que P11 ad- On en déduit que,
met exactement n racines réelles distinctes.
• Comme Po =1 on en déduit que Po admet exactement lim J'n) (x) exgte lim
x-+oo x-+oo
(- W' (n + 1)! = 0
xn+2
0 racine.
Supposons que Pn admette exactement n racines
La question c) appliquée à la fonction f'n) nous donne
réelles distinctes qu'on notera
donc que u<n))' = t<n+l) s'annule en un point bn sur
l'intervalle
l an, + oo[.
avec n ~ l fixé.
Le même raisonnement avec la question d), nous
* La fonction f'n ) s'annule donc en ces réels. donne que f<n +l) s'annule en un point ho sur finter-
Sur chaque intervalle )a;, ai+ il, avec i E {l; 2; ... ; n - l}, valle J- oo, a1 [.
la fonction f'n) vérifie les conditions d'application du
théorème de Rolle. * t<n+ ll s'annule donc en (au moins) n + 1 réels dis-
tincts
Il existe donc n - 1 réels h; avec i E {1; . . . ; n - 1) véri-
fiant : ho< b1 < ... < bn

donc P n+ 1 admet au moins n + 1 racines réelles dis-


a;< hi < a;+1 et (J'n) )' (h;) = J'n+l\h;) =0 tinctes.
Ces n - 1 réels h; sont donc tous distincts et sont des Or Pn+l est de degré n + 1 donc admet au maximum
racines de Pn+l· Pn+l s'annule donc au moins (n-1) n + 1 racines. Donc Pn+l admet exactement n + 1 ra-
fois. cines réelles distinctes.
* On voudrait ensuite montrer que t<n+l) s'annule sur
J - oo; a1 [ et sur Jan; +oo[ en utilisant les résultats des • La propriété est donc héréditaire. Elle est donc vraie
questions c) et d). pour tout entier naturel n.
u
0
C:
::J
0
lfl
,-i
0
N
@
....,
.c
Ol
·;::
>-
a.
0
u

181
CHAPITRE j

Intégration sur un seg1nent et équa-


tions différentielles linéaires

'Tlièmes a6ordés dans {es e~ercices


- Primitive d'une fonction continue sur un intervalle
- Intégrale d'une fonction continue sur un segment
- Calcul intégral
- Fonction définie par une intégrale
- Equations différentielles

Points essentiefs du cours pour {a réso{ution


des e~ercices
- Primitive d'une fonction continue sur un intervalle
- Calculs de primitives
- Intégrale d'une fonction continue sur un segment
u
0 - Intégration par parties
C:
::J
0 - Formule de changement de variable
lfl
,-i - Décomposition en éléments simples des fractions rationnelles x - x 2:~+x~q
0
N
- Équations différentielles linéaires du premier ordre résolues
@
...., - Équations différentielles linéaires du second ordre résolues à coefficients constants
.c
Ol
·;::
>-
a.
0
u

182
Intégration, équations d ifférentielles Chap,itre 8

Les méthodes à retenir

On connaît une primitive de l'intégrande (= la fonction à intégrer).


<.......+ Exercices 8.l e t 8.2

On utilise une intégration par parties.


<.......+ Exer cices 8.1, 8.2 et 8.14

On utilise un changement de variable.


<.......+ Exercices 8.l e t 8.2

Pour les fonctions ln, arcsin, arccos et arctan, il est utile d'utiliser
une intégration par parties pour les dériver.
<.......+ Exercices 8.1 et 8.2

Pour calculer une intégrale ou une Pour une fonction du type P(x)f(x) où P est une fonction
primitive polynôme, des intégratio ns par parties successives permettent
d'abaisser le degré d u polynôme.
<.......+ Exercices 8.1 et 8.2

Pour les fonctions cos et sin, on peut passer à l'exponentielle com-


plexe et faire les calculs dans C. On peut aussi "linéariser" l' expres-
sion à l'aide des formules de trigonométrie.
<.......+ Exercices 8.1 et 8.2

Pour les fractions rationnelles, il faut essayer de se ramener à


:~:!q
l'exemple du cours: x .......... x2 et utiliser une décomposition
en éléments s imples.
<.......+ Exercices 8.1, 8.2 et 8.6

Pour pouvoir appliquer les résultats du cours, elle doit être sous
forme résolue.
<.......+ Exercices 8.3, 8.4, 8.6, 8.7, 8.8, 8.14 et 8.17

u Pour les équations différentielles linéaires résolues, les solu tions


0
C: so nt obtenues en additio nnant une solution part iculière avec la
::J
0 solution générale de l'éq uation homogène.
lfl Pour résoudre une équation
,-i <.......+ Exercices 8.3, 8.4, 8.6, 8.7, 8.8, 8.14 e t 8.17
0
N différentielle
@ Pour trouver une solution particulière d'une équation différen-
...., tielle linéaire du premier ordre, on décompose le problème à
.c
Ol
·;:: l'aide du principe de superposition. Ensuite on peut utiliser lamé-
>-
a. thode de variation de la constante ou chercher une solution qui
0 "ressemble" au second membre (polynômes etc ...).
u
<.......+ Exercices 8.3, 8.4, 8.6 et 8.8

183
Chapitre8 Intégration, équations différentielles

- Pour une équation différentielle linéaire du second ordre résolue,


les coefficients doivent être constants pour appliquer les formules
du cours.
<.......+ Exercices 8.3, 8.6, 8.14 et 8.17

Pour trouver une solution particulière d'une équation différen-


tielle linéaire du second ordre à coefficients constants, on décom-
pose le problème à l'aide du principe de superposition. Ensll!ite, si
le second membre est de la forme P(x)eiax avec P e: C[XJ alors on
cherche une solution sous la forme Q(x)x~eiax où Q est un poly-
nôme de même degré que Pet 13 est égal à l'ordre de multiplicité
de a dans l'équation caractéristique.
<.......+ Exercices 8.3 et 8.17

- Lorsque le second membre est de la forme P(x) cos(ax) ou


P(x) sin(j)x), on se ramène à P (x)eiax et on fait les calculs dans
C
<.......+ Exercice 8.3

Lorsque l'équation différentielle n'est pas d'une forme connue, on


peut s'y ramener par changement de variable ou de fonction.
c........ Exercices 8.3 et 8.8

La fonction x ........... J: f (t) dt est l'unique primitive de f nulle en a.


c........ Exercices 8.9, 8.17, 8.18 et 8.19
u
0
C:
La fonction <.p: x - ggj
f(t)dt se met sous la forme F(u(x)) -
::J F(v(x)) où Fest une primitive de f .
0
lfl <.......+ Exercices 8.9, 8.17 et 8.18
,-i
0
Pour étudier une fonction définie par
N une intégrale La formule précédente permet d'étudier limites, continuité, déri-
@ vabilité ... de <.p •
....,
.c c........ Exercices 8.9, 8.17, 8.18 et 8.19
Ol
ï::::
>-
a. La croissance de l'intégrale permet d'obtenir un encadrement de
0 <p(x)à partir d'un encadrement de f(t).
u
<.......+ Exercice 8.9

184
Intégration, équations d ifférentielles Chap,itre 8

Pour montrer que la suite d'intégrale converge, il suffit de montrer


qu'elle est croissante majorée ou décroissante minorée.
c........ Exercices 8.10, 8.13 et 8.15

Une judicieuse intégration par parties mène souvent à une rela-


Pour étudier une suite d'intégrale tion de récurrence entre les termes de la suite.
c........ Exercices8.l3 et8.l5

La croissance de l'intégrale permet aussi d'obtenir un encadre-


ment de la suite d'intégrales à partir d'un encadrement de f (t).
c........ Exercices 8.10 et 8.15

Le but est d'utiliser le théorème de la valeur moyenne :


11
·
hm ,il L ! (k)
n-+oo k =O
ri
ri =JofCt)dt.
c........ Exercices8.5 et8.l6

Dans une somme où l'indice est k, il faut le mettre sous la forme


~ (en mettant n en facteur).
Pour calculer une somme infinie c........ Exercices 8.5 et8.l6

Il faut ensuite mettre i en facteur devant la somme.


c........ Exercices 8.5 et8.l6

On peut enlever un nombre fini de termes dans la somme sans


changer la limite.
c........ Exercices8.5 et8.l6

'O
0
C:
::J
0
lfl
,-i
0
N
@
....,
.c
Ol
ï::::
>-
a.
0
u

185
Chapitre8 Intégration, équations différentielles

~ ~
Enonces des exercices
.

---- - Calculs d'intégrales


Calculer les intégrales suivantes.
a) I1 =fon vl + cos(t)dt.
b) 12 = J( cos(ln x) dx (poser t = ln x) .
C) J3 = jJ (x2 + x + l)eX dx.
d) T4 = JJ Vl - x2 dx (poser x = cos t).
e) 15 =JJ J--x~ 2 dx.
f) l 6 =JO
rl x+ 1 d
x 2+4x+4 X.

l7 =fJ xz:\ + 1 dx.


1
g)
!!.
h) la =fo4 ln (1 + tan(x)) dx (poser u =i - x).

---- - Calculs de primitives


Déterminer les primitives suivantes.
a) F1 (x) = Jln(x) dx sur~:.
b) F2(t) = Jarctan(t)dt sur~.
c) F3(t) = J v'-e-'; - 1 dt sur~: (poser x = e-t) .
d) F4 (x) = J cos(x)ex dx sur~-

---- - Résolution d'équations différentielles linéaires


Résoudre les équations différentie11es suivantes sur 1'interva11e donné.
a) (E 1): (x2 +l)y'+xy=Vx2 +lsur~.
b) (E2) : y'+ y = ex + x2 - cos(x) sur~-
c) (E3): sin3(x)y' = 2(cosx)ysur]O,n[.
d) (E 4) : y" + y' + y = cos(2x) sur~-
u e) (E 5 ) : y" - y= xsin (x) sur~-
0
C:
::J f) (E5) : y11 - 2y' +y=exsur~.
0

---- -
lfl
,-i
0
N
Calculs de sommes infinies
@
...., Déterminer la limite de la suite (un) définie par :
.c
Ol
·;::
>-
a) 'v n ?:- l, Un = fi k[= I COS rn) sin ( ~).
a.
0 n- 1
u b) 'vn?:-1,un = [ v'4fT-kZ·
k =O 4n -k

186
Intégration, équations différentielles Chap,itre 8

---- - I.:énoncé donne une solution particulière de l'équation différentielle


On considère l'application f de~ dans~ et l'application g de~+ dans~ définies par :

f (x) =e- x ln(l + ex) et g(x) =~


l+x
- ln(l + x)

a) Déterminer le signe de g(x) en fonction de x.


b) Etudier les variations de f ainsi que ses éventuelles limites aux bornes de son
ensemble de définition.
c) Résoudre sur~ l'équation différentielle (E) : y'+ y= }ex.

----
1

- Une solution particulière qui demande des calculs...

1
a ) V'en"fierque.·V t > O, (l+t)(t2+l) -- l(_l_
2 l+t + t2 + 1
1-t) ·
b) Résoudre sur JO, +oo[ (E) : x(x + l)y' +y= arctan(x).

---- - Recollement en Odes solutions d'une équation différentielle

a) Résoudre sur~~ et~~ (E) : xy' + 2y =1 :x


2.
b) Montrer que cette équation différentielle admet une unique solution sur~- Com-

----
mentaire?

- •
Résolution d'une équation différentielle non linéaire par changement de
fonction ou de variable

a) Résoudre sur~~ (Ea) : x2 y''+ 4xy1 - (x2 - 2) y= 0, en posant z(x)


b) Résoudre sur~~ (Ebl : y' (x) =y ( i ), en posant z(t) =y(et).
=x2 y(x).

---- - Etude d'une fonction définie par une intégrale


On considère la fonction f définie par f (x) = f;x t+s~n(t) dt.
a) Préciser l'ensemble de définition de/.
b) Vérifier que f est paire.
c) Tvlontrer que f est dérivable sur son ensemble de définition et donner f' (x) .
d) A l'aide du théorème des gendarmes, déterminer la limite de f en +oo.

---- -
u
0
C:
::J
0 Convergence d'une suite d'intégrales
lfl
,-i J
0
N
On considère la suite (un) n;::, 1 définie par : V n;::: l, Un = (JJ (1 !'~)" dt) ri .
@ a) Calculer UJ .
...., , . . 1
.c b) Etablir que. V n;::: l, 0 ~ Un ~ 2 .
Ol
·;:: 1
>- c) Soit a E (0, 1). Montrer que : \f n;::: l, un;::: (1 - a) ii J~a.
a.
0 d) A l'aide d'une suite (an) n;::, 1 à valeurs dans (0, 1) judicieusement choisie, montrer
u
que la suite (un) n;::, 1 converge vers !.

187
Chapitre8 Intégration, équations différentielles

---- - Calcul d'une familles d'intégrales


Pour a E: [- 1, 1), on considère la fonction fa définie par: fa (x)
a) Pour tout a E:) - 1, l [ et tout x E: [0, n) vérifier les propriétés suivantes:
• (1 - la1) ~ fa(x) ~ (1 + lal)
2 2
=11 - aeix 1 .
2

• fa(1T- x) = f-a(x)
• f~2(x) = fa(~) f-a ( ~)
On pose, pour tout a E: ] - 1, 1[ : g(a) =J011 1n (fa (x)) dx.
b) Montrer que g est une fonction paire.
c) Montrer que : 'v'ae:) - 1,l[, g(a2 ) = 2g(a).
d) Montrer que g est continue en O.
e) En déduire que: 'v' a E:] - 1, 1[, g(a) =O.

---- - Calcul d' une d'intégrale grâce à une sym étrie axiale
Soit f une fonction continue sur un segment [a, b).
a) Vérifier que : fr~ f (x) dx = J! f (a+ b - x) dx.
b) On suppose que, pour tout x e: [a, b] : f(a+ b - x) = f (x). Exprimer J}: xf(x) dx en
fonction de J! f (x) dx.
c) Calculer rn tsin(tl dt.
JO l+cos 2(tl

---- - Calcul d' une suite d'intégrales (d'après Agro-Véto 2006)


._,
P our ne:,..,, . _ f11 cos(n0) dS
on pose. 1 - - n s+ cos(eJ .
11 4
a) Vérifier que: 'v' n E: 1\1, 111 ~ 2n. Calculer 11 + j Io.
b) Montrer qu'il existe une constante réelle ex (à déterminer) telle que: 'v' n E: 1\1,
111+2 + In = al 11+l ·
c) Donner la valeur de T11 en fonction de n E: 1\1 et de Io. Calculer alors 111 en fonction
den.

u
0

0
C:
::J

lfl
,-i
0
---- - Résolution d'une équation différentielle à paramètre (d'après Agro-Véto
2006)

a) Soit u une fonction C2 sur~ telle que u(O) = -u(l) = 1. Calculer:


N
@
...., fo 1 ( u" (t) + n 2 u(t)) sin(n t) dt
.c
Ol
·;::
>-
a. u" (t) + n 2 u(t) = -bsin(nt)
0 b) Résoud,e sm • le pmblème de Cauchy { , ern fonc-
u u(O) = -u(l) = 1
tian du paramètre b E: ~-

188
Intégration, équations d ifférentielles Chap,itre 8

---- - Intégrales de Wallis


Pour tout n EN, on pose: In = Il" sinn tdt.
a) Vérifier que, pour tout n EN: In> 0 et In = Il" cosn(t).
b) Donner une relation de récurrence entre ln+2 et In. En déduire l'expression de In
en fonction de la parité de n.
c) Vérifier que pom tout n EN, on a : (n + lHn+ 1In =; et In+2 ~ In+ 1 ~ In.
d) En déduire un équivalent de In, puis de (2,;1), lorsque n -+ +oo.

---- - Inégalité de Jensen pour le logarithme

a) Vérifier que : 'v' t > 0, ln(t) ~ t - 1.


b) Soient x1, x2, ... , x 11 n réels strictement positifs. En utilisant ak = i, où x est la
moyenne des Xk, établir que:

-
ln [ ln(xk) ~
(1 n
ln - [ Xk
)
n k=I n k= I

c) Soit f une fonction continue sur [0, 11, à valeurs strictement positives. Montrer
que:

---- - Résolution d'une équation intégrale (d'après Agro-Véto 2006)

a) Résoudre sur lm l'équation différentielle (E 1 ): y''+ y = - sin(x).


b) Soit f une fonction continue sur~. Montrer que la fonction

F: x - fo\x - t)f(t)dt
u
0
C:
::J est C 1 sur lm.
0
lfl c) On considère dans cette question l'équation intégrale (E2) :
,-i
0
N
'v'xe:lm, f(x) = sin(x) - Lx tf(x - t)dt
@
....,
.c
Ol
·;::
où f es une fonction continue sur~-
>-
a. Soit f une solution de (E2). Montrer que f est C2 sur lm et donner une relation
0
u entre f" et f.
d) Résoudre alors (E2).

189
Chapitre8 Intégration, équations différentielles

---- - •
Une formule de calcul intégral (d'après Agro-Véto 2006)
Dans cet exercice, a est un réel strictement positif, f : (0, a) - ~ une fonction conti-
nue et strictement croissante sur [O, a], dérivable sur JO, a[, nulle en O. La fonction f
est alors bijective de (0, a) sur [O,f(a)]. de réciproque notée g. On veut montrer que,
pour tout réel te: (0, a) :

rt
Jo f(x)dx+ J o
rrui g(y)dy = tf(t) (1).

a) Vérifier la relation (1) dans le cas où: f (x) = xP, p e: 1\1*.


Pour tout te: (0, a). on note <.p(t) la quantité:

<.p(t) =Jortf(x)dx+ Jrof(t) g(y)dy-tf(t).


b) Montrer que <.p est définie et continue sur (0, a). dérivable sur JO, a[.

---- -
c) En déduire l'égalité (1).

Inégalité de la moyenne améliorée


On considère deux fonctions f et g définies sur un intervalle I = [a, bl de~- On sup-
pose que f est de classe C1, positive et décroissante sur I et que g est continue sur I.
On considère le fonction G définie sur I par G(x) =J; g(t) dt.
a) Justifier que G est de classe C 1 sur I.
b) Montrer qu'il existe deux réels met M tels que : G ([a,b)) = [m,MJ.
c) Montrer que :
b lb
l a f(t)g(t) dt = f(b)G(b) - a .f' (t)G(t) dt

d) En déduire que:
m.f(a) ~ lb f(t)g(t)dt ~ Mf(a).
e) Montrer qu'il existe ce: [a, b) tel que:

la
b f(t)g(t) dt = f(a) le a
g(t) dt.

f) On suppose que a> 0; montrer que: J/: l - ~ost dt~ 2+ ~- a.


g) Montrer que: lim ...!..2 J} sin2 t dt= O.
u x-+oox x t
0
C:
::J
0
lfl
,-i
0
N
@
....,
.c
Ol
·;::
>-
a.
0
u

190
Intégration, équations différentielles Chap,itre 8

Du mal à démarrer ?

Ill a) Utiliser une formule de t rigonométrie. 1111 a) Facile.


b) Uti liser le changemen t de variable donné. b) Uti liser un changement de variable.
c) Procéder par intégrations par parties successives. c) Utiliser les résultats du a) et u n changement de variable.
d) Utiliser le changement de variable donné. d) Encadrer g(a) à l'a ide du a).
e) et f) Décomposer la fraction rationnelle en é léments simp les. e) Remarquer que g( a2" ) = 2n g(a) , pour tout n E 1\1.
g) Faire apparaître un terme en ::g/
1
et un autre en
1
:~<tJi 2 .
h) Déte rm iner une équation vérifiée par ls. G9 a) Utiliser un changement de variable.
b) Reprendre la formule du a).
l9 a) et b) Procéder par intégration par parties. c) Pren dre a= 0 et b = rr.
c) Uti liser le changement de variable donné.
d) Faire un calcu l dans C 1111 a) Encadrer l' intégrande.
b) Ut iliser une formule de trigonomét rie.
Ill a), b) et c) Utiliser les résul tats du cours sur les équations c) Reconnaître une suite récurren te linéai re d'ordre 2 puis déter-
différentiel les du premier ordre. miner lim In.
11-+oo
d), e) et f ) Util iser les résultats d u cours sur les équations d ifféren-
t ielles du second ordre.
1111 a) Procéder par intégration par parties.
b) Procéder par analyse-synthèse et utiliser le a).
. . Uti liser le théorème de la valeu r m oyenne.

Ill a) et b) Faciles.
11D a) Utiliser un changement de variable.
b) Procéder par intégration par parties.
c) Remarquer que f est solution particulière su r ~ . c) Reprendre la relat ion du b).
d) Comm encer par montrer que In - In+l ·
11111 a)Faci le.
+oo

b) Faire apparaître le groupement du a). 11111 a) Facile.


b) Appliquer le a) avec t = ak .
Ill a) Faci le. c) Penser au théorème de la valeur moyenne.
b) Procéder par analyse-synt hèse et utiliser le a).
1111 a) Classiq ue.
.. Procéder par analyse-syn th èse e t déterminer une équation b) Exprimer F à l'aide de pri m itives de fonctions continues.
différentiel le vérifiée par z. c) Vérifier que f est solution de (Ei).
d) Procéder par analyse-synthèse.

Ill a) Étud ier la con tinuité de l'intégrande.


b) Ut iliser un changement de variable. 11111 a) L'application réciproque de f est x- o/x.
c) Introduire une primi tive de l' intégrande. b) Exprimer <p à l'ai de de primi tives de fonctions contin ues.
d) Commencer par encadrer l' intégrande. c) Calculer la dérivée de <p.

u
0
111111 a) Remarquer que 1 ~ 1 = l - 1 1 . l 11D a) et b) Appliquer les résul tats du cours.
b) Commencer par encadrer l'intégrande. c) Procéder par intégration par parties.
C:

0
::J c) Ut iliser le fait que si f est positive alors fJ /(t) dt ;;, .fJf U) dt, d) Encadrer : / 1 (t)G(t).
pou r a E [O, l]. e) Diviser l'inégalité du d) par f (a).
lfl
,-i d) Choisir an = 1- t,. f) et g) Ut iliser le résu ltat du e).
0
N
@
....,
.c
Ol
·;::
>-
a.
0
u

191
Chapitre8 Intégration, équations différentielles

Corrigés des exercices

-
a) Sachant que :

'vtE~, cos(t)+l =2cos


2
{1)
c) Effectuons une intégration par parties :

u(x) = x 2 + x + 1
1
u (x) = 2x+ 1
v' (x) = ex
v(x) = ex

On obtient successivement : Les fonctions u et v étant de classe C1 sur [0, l] on a:

1
[(x2 +x+ l)ex ]~ - fo (2x+ l)é dx

3e - 1 - L 1
(2x + l)ex dx

Mais si t E [o, ~] on a cos(t) ~ 0 donc si t E (0, n) on a


Effectuons une nouvelle intégration par parties du second
cos(~)~ 0, donc:
terme :

Jï fo\os(i)dt
u(x) = 2x + 1 v' (x) = ex

vz[2sm(i)J: u'(x) = 2 v(x) = ex

= 2V2 Les fonctions u et v étant de classe C 1 sur [O, 1) on a:

b) Effectuons le changement de variable C1


1
3e-1- [ (2x+ l)ex ]~ + fo 2ex dx
J t=ln(x)<=>x=et dx=etdt

l t1 = ln(I) =0 t2 = ln(elT) = n 3e - 1 - 3e + 1 + [ 2ex] ~


2(e-l)
Alors :

folT cos(t)et dt d) Effectuons le changement de variable C1

folT ~e(eO+i)t) dt Jx = cos(t) dx = - sin(t)dt

= ~e(folT eCl+i)t dt) 1 =;


t1 t2 =0
Ona:
u Oron a:
0

0
C:
::J eCI+i'.t ]lT
[ 1+ z
4 f~
2
0
J1 - cos2(t)x( - sin(t))dt
lfl 0
,-i
0
N = _1__ (e(l+i)lT - 1) fo if Jsin2 (t) x sin(t) dt
l+t
@
....,
.c
Ol
_1_ X l - i X (elT
l+i 1-i
X eÎlT - 1) = f rr
2
lsin(t)I x sin(t) dt
·;:: 1- i
>- -2-(-elT -1)
a. Mais si t E [ 0, i ]on a sin(t) ~ 0, donc:
0
u Donc :
1- i lT ) 1 + elT
I 2 =~e ( - - (-e -1) =- - -
2 2

19 2
Intégration, équations différentielles Ch ap,itre 8

Or, pour tout te: IJll :


=
l - 2sin 2(t) cos(2t) doncsin 2(t) = l-c~s(2t) .
Ainsi:
ls -1
3
folo (-x+2-1 + -x-2
1- ) dx

~ (f (l - cos(2t))dt i [21n(lx+ l i) +ln(lx-21) ]~


2 Jo
n
1
~ [ t _ sin(2t) 12 ln(2)
2 2 o 3
1t

4 f) Décomposons [6(x) = x2:1~+4 en éléments simples.


On aurait pu aussi remarquer que la courbe d'équation SachantqueX2 +4X+4 =(X+2)2, on sait qu'il existe (.a,b) e:
y = Vl - x 2 pour x e: [O; l] représente le quart de cercle Dll 2 tel que :
unité supérieur droit :

a b ax + (2a + b)
\:/xe:Oll\{-2), Js(x)= - - + - - = - - - -
x + 2 (x+2) 2 x 2 +4x+4

On a donc:

Ja=l
0 X
l 2a+b = 1

ce qui nous donne: a = 1 et b = - 1.

Donc:

Le quart de disque unité supérieur droit a comme aire: 1 1


\:/xe: Oll\{-2}, fs(x) = - - --
l4=!xnxl = l 2 x+2 (x+2)2

e) Décomposons / 5 (x) = x/--x~ 2 en éléments simples.


On en déduit que :
Sachant que X2 - X - 2 = (X - 2)(X + 1), on sait qu'il existe
(a, b) e: Dll 2 tel que :

\:/xe: Oll\{-1,2}, F.s(x)---a + --b___ (a+b)x+(b - 2a)


1:
T6 = fol{x:2 - (x:2)2 ) dx
x+ 1 x - 2 x2 - x - 2 1 1
[ln (lx+21)+ -x+2
- ]
"O 0
0 On a donc:
1 1
C:
::J
} a+b = l
= ln(3) - ln(2) + - - -
3 2
0
lfl
,--i
0
N
1b - 2a= - l lnG)-i
@ ce qui nous donne : a = j et b = k.
....,
.c Donc: g) Sachant que X2 +X+ l n'a pas de racine réelle, on ne peut
Ol pas utiliser ici de décomposition en éléments simples.
·;::
>-
a.
2 1
\:/xe:Oll\{- 1,2}, fs(x) = - - + - - On sait qu'il faut mettre en évidence un terme de la forme
0 3(x + 1) 3(x - 2)
u
11{- et une autre de la forme .;vz, qui seront facilement in-
1
Ainsi on obtient que : tégrables. Pour tout x e: IJll :

193
Chapitre8 Intégration, équations différentielles

De plus, pour (a, b) E: lo, f [, on a:


tan(a)-tan(b)
h(x) tan(a-b) =- -----
1 + tan(a) tan(b)

On obtient donc :

la = {i ln {i + 1 - tan(u)) du
= Jo 1 + tan(u)

Li
o
ln ( 2
1 + tan(u)
) du

L"î (tn(2)-ln (1 + tan(u))) du


n
On en déduit que :
fo 4 ln(2) du - la
1
17=- L 1
2x + 1 dx+-
2 1 1 L dx nln(2) - la
2
2 0 x + X+ 1 3 0 l +( 2/r )2 = 4

On connaît une primitive de l'intégrande dans la première On en déduit que:


intégrale. Dans la seconde intégrale on effectue le change- la = n ln(2)
ment de variable C1 : 8

On obtient:
t .=

t1 =
2Jil ~x = 0~-1
'1 t2 = V3 -
a) Il suffit d'écrire que F1 (x)
parties :
= J 1 x ln(x) dx et d'intégrer par

u(x) = ln(x) v' (x) = 1


u'(x) =~ v(x) =x
Les fontions uet v étant de classeC 1 sur~:, on obtient:

'v'xE:~:. F1(x) = xln(x) -J ldx = xln(x) - x

b) Utilisons la même méthode que précédemment :

"O h) Effectuons Je changement de variable C1


0
C:
u(t) =arctan(t) v'(t) = l
::J
0
u= i - x ~ x= i - u dx =-du u'(t) = 11r2 v(t) =t
lfl
,--i
{
0
N
u1 = î u2 = O Les foutions u et v étant de classe C1 sur~. on obtient :
@ Alors:
....,
.c
Ol
·;::
tarctan(t)- J _!_2 dt
l+t

0
u
>-
a. la - J !!°in( 1 +tanG- u)) du
4
tarctan(t) - 1 J
2 1 + t 2 dt
2t

Lrn

ln( 1 +tan G- u)) du tarctan(t)- ~ ln( 1 + t 2 )

194

www.bibliomath.com
In tégration, é quations d iffére ntielles Chap,itre 8

c) Mettons tout d'abord e- t en évidence :


S~ = {xe:~ ...... ke- ! Ln(x2+1) = k , ke:~}
Jx 2 +1

JJ-1 1 J -t
'efte:~. F3(t) =
-, (e - 1)2
-1
dt = e
J1 - (e- t)2
dt • Cherchons à présent une solution particulière de (E1) en
utilisant la méthode de la variation de la constante.
On cherche une solution de (E1) sur~ sous la forme :
k(x)
Effectuons à présent le changement de variable C1 f (x) = ~
V x2 + l
x = e- t e:JO, l[ ~ t = - ln(x) e: ~ : avec k une fonction dérivable sur~- On obtient :
{ dt= _dx
X k'(x)
'efxe: lli,
Alors : Jx 2 + 1
donc:

J -l dx 1
'ef X E: ~. k (X) = 1
Vl-x2
On en déduit qu'une solution particulière de (E1) est défi-
arccos(x)
nie sur ~ par :
arccos(e- t)
X
f(x)= ~
d) On remarque: V l + x2
• rensemble des solutions de (E1) sur~ est donc :

k+x }
S 1 = { x e: ~ - ~ · k e: ~
V x2 + l
Oron a:
b) • Commençons par résoudre sur~ l'équation homogène
'efxe:~. f e(l+i)xdx _l_e(l+i)x
l+i
associée :

1 1- i ,
(E;) : y'+y=O
= - - x - - x ex x e 1 x
l+i 1-i
ex , qui admet pour ensemble de solutions :
= - (1 - i)e 1 x
2
S~ = { X E ~ ,..... ke - X, k E ~}
Finalement :
Pour trouver une solution particulière de (E2) sur Ili, on va
X utiliser la méthode de superposition des solutions, car Je
'ef x e: ~. F4(x) = e ( cos(x) + sin(x)) second membre est somme de trois termes de types diffé-
2
rents.
• On cherche f solution particulière sur ~ de y' + y = ex
'O
0
C:
l;SI sous la forme :
::J a) Comme x 2 + 1 ne s'annule pas sur Ili on a : f(x) = k1(x)e- x
0
lfl où k1 est une fonction dérivable sur~-On obtient :
,--i I 1 2X l
0 (E1)<==> y + - , -2- y = - - - 'ef x e: ~. k~ (x)e-x = ex
N 2 x +1 Jx2+l
@ donc:
,µ • Résolvons l'équation homogène associée :
.c 'ef x e: ~. k~(x) = e2x
Ol
·;:: I I 1 2X
>- (E 1)<==> y + 2, x2 + 1 y = 0 On en déduit qu'une solution particulière de y'+ y = ex est
a.
0 définie sur~ par :
u
Elle est de la forme y'+ a(x)y = 0 et admet donc pour en-
semble de solutions :

195

www.bibliomath.com
Chapitre8 Intégration, équations différentielles

• On cherche g solution particulière sur IR de y' + y = x2 On en déduit qu'une solution particulière de y' + y =
sous la forme : - cos(x) est définie sur IR par :

1 1
2 cos(x) - 2sin(x)
h(x) = -
où k2 est une fonction dérivable sur IR. On obtient :
• I.;ensemble des solutions de (E2) sur IR est donc :

donc : -x ex - cos(x) - sin(x)


S2 = { x ...... ke + + x 2 -2x+2,
2

et donc: c) Sur JO;n[,sin3(x) nes'annulepas.Onadonc:

J 2
x ex dx (E3) = y'
2cos(x)
= sm
. 3 (x) y
x2é-2 J xé dx Or une primitive de x- 2c0;(<xl.) est x-
sm x smx
(on recon--+()
x2 é - 2 ( xé - Jé dx) 1
naît u (x) x u(xf' avec u(x) = sin(x) et a = 3).
I.;ensemble des solutions de (E3) sur JO, ni est donc :
= x2é - 2xex + 2ex
1
S3 ={X e:JO, 7T [...... ke - sin2(x), k e: jR}
grâce à deux intégrations par parties successives.
d) • Résolvons tout d'abord sur IR l'équation homogène asso-
On en déduit qu'une solution particulière de y'+ y = x2 est
ciée :
définie sur IR par :
(E~) : y" + y' + y = 0
g(x) =x2-2x+2
dont l'équation caractéristique est :
• On cherche h solution particulière sur IR de y' + y =
- cos(x) sous la forme: r 2 + r+ 1 = 0

g(x) = k3(x)e-x de solutions complexes _ l2 - i v3


2
et _ l2 + i v3
2 ·
I.;ensemble des solutions de (E~) sur IR est donc :
où k3 est une fonction dérivable sur IR. On obtient:

'v' x e: IR, k~ (x)e- x = -cos(x)


s~ ={XE: iR-.. ( Acos ( X)+ Bsin ( V: V: X)) 2
e- !x, (A, B) E: iR }

donc :
'v' x e: IR, '4 (x) = - cos(x)ex • Pour déterminer une solution particulière f de (E4 ) sur IR,
on va déterminer une solution particulière z complexe de
et donc: (E~):
y" + y'+ y= e2ix
"O
0
C:
-J cos(x)ex dx
Comme 2i n'est pas racine de l'équation caractéristique
0
::J

lfl
-J ~e(e(l+i)x) dx (donc racine d'ordre 0), on cherchez sous la forme:
,--i z(x) = kxo e2ix = ke2ix
0
N -~e(f e(l+i)x dx)
@ avec k e: C.
...., e(l +i)x 1 - i) Alors z'(x) = 2ike 2ix et z"(x) = - 4ke2ix_
.c -~e ( ---x--
Ol
·;::
l+i 1- i
Comme z est solution de (E~) sur IR, on doit avoir:
>-
a. . ix ex)
0
u
= - ~e ( (1 - z)e
2 _ 4 ke2ix + Zi ke2ix + ke2ix =e2ix
ex
- 2 (cos(x) + sin(x)) pour tout x e: IR.

196

www.bibliomath.com
Intégration, équations différentielles Ch ap,itre 8

k vérifie donc : pour tout x E IR.


(2i - 3)k =1 (a, b) vérifie donc:
d'où:
-2a= 1
1 1 - 2i - 3 - 3 - 2i
k =-- = - - x ---=--- {
2i - 3 2i - 3 - 2i-3 13 2ia - 2b = O
On a alors: ·
0 no btlenta=- 1 b i
2 et =- 2 .
-3-2i 2· On a alors:
z(x) = - - - e ,x
13 x +i ·
Vx e: ~, z(x) = - - - e 1x
2
et finalement :
et finalement:
Vx e: IR, f(x) = ~e(z(x)) =
1
~ ( - 3cos(2x) + 2sin(2x)) Vx E IR, f (x) = ..5P m(z(x)) = -~ cos(x) -1 sin(x)

• En conclusion, l'ensemble des solutions de (E 4 ) sur IR est • Onen déduit l'ensemble des solutions de (Es) sur IR :
donc:

Ss ={x- Aex + Be- x - ~(cos(x) + xsin(x)), (A,B) e: im 2 }

S4 = {xe:~-(Acos( V: x) +Bsin( V: x))e-!x f) • Commençons par résoudre sur~ l'équation homogène


associée :
2sin(2x) -3cos(2x) }
+ (A,B) E IR 2
13
(E~): y'' - 2y1 + y = O
e) • Commençons par résoudre sur IR l'équation homogène dont l'équation caractéristique est :
associée :
r 2 - 2r + 1 =(r - 1J2 =0
de racine double 1.
dont l'équation caractéristique est :
I.:ensemble des solutions sur IR de (E~) est donc :
2
r =1 S~ ={x E IR- (Ax+ B)ex, (A,B) E IR 2}
de solutions - 1 et 1.
• On va déterminer une solution particulière f de (E5)
I;ensemble des solutions de (E;) sur IR est donc : sur IR. Comme 1 est racine double de l'équation caractéris-
tique (donc racine d'ordre 2), on cherche f sous la forme:
S~ = { x E IR - Aé + Be- x, (A, B) E !R2 }
f(x) = Cx2 ex
• Pour déterminer une solution particulière f de (Es) sur IR,
avec CE~-
on va déterminer une solution particulière z complexe de
(E~): Alors f'(x) = (2Cx+ Cx2 )ex et f"(x) = (2C+4Cx+Cx2 )ex.
u
0
C:
y" - y= xeix Comme f est solution de (E5) sur IR, on doit avoir:
::J
0 Comme i n'est pas racine de l'équation caractéristique (2C + 4Cx + cx2 Jé - 2(2Cx + Cx2 )ex + cx2 ex =ex
lfl
,--i (donc racine d'ordre 0), on cherche z sous la forme:
0 pour tout x E IR.
N
@
z(x) = (ax+ b)x0 eix = (ax + b)eix C vérifie donc 2C = l, d'où C = !.
...., On a alors:
.c avec (a, b) e: <C 2 .
Ol Vx e: IR, f(x) = 1 x 2 e X
·;::
>- Alors z' (x) = (iax +a+ ib)eix et z" (x) = (2ia - b- ax)eix. 2
a. • Onen déduit l'ensemble des solutions de (E6) sur IR :
0 Comme z est solution de (E~) sur IR, on doit avoir:
u
(2ia- b-ax)eix - (ax + b)eix =xeix S5 ={x E IR- (Ax+ B+ ~x2) ex, (A,B) E ~ 2}

197

www.bibliomath.com
Cha pitre8 Intégration, équations différentielles

-
a) On rappelle le résultat de cours concernant le théorème de
la valeur moyenne :
Finalement :

r
n~Too Un
existe 1T
= 6

-
Si f est une fonction continue sur [O; 1] alors

lim - I:
n-+oo
1 n- l
n k=O
t ( -k)
n
=Llo tcri dt a) Étudions rapidement la fonction g sur [ij+ . Elle est déri-
vable sur cet intervalle (comme composées et produits de
. 1 n .(
Ic1 : un= - I:/ -
k) avec/:. X>-+cos(x)sin(x)= -sin(2x)
- -,
fonctions dérivables) et :
n k=I n 2
la fonction f étant continue sur [O; 1].
Afin de pouvoir utiliser le théorème rappelé plus haut, il 'v'xe:[ij + ' g'. (x) = - -
1 - - -1- = - -X- - ~o
2
(1 + x) 1+ x (1 + x)2
suffit de remarquer que :
Or g(O) = 0 donc on en déduit le tableau suivant :
Un = -
1 n-
Ll f ( -k) - -.f(O)
1 1
+-.f(l)
n k=O n n n X 0 +oo
On obtient donc par somme de limites : Signe de 1

g'(x)
©
1
-

k) 1sin(2t)
lim -1 n
n-+oo n t' f 1
(-
n
existe
L
O
- - d t - 0 +0
2 Variations de 0~

= [-i cos(2t)
l - cos(2)
t On en déduit que:
g
- oo

= 4
\f X E [ij+, g(x) ~ 0
Finalement :
b) • La fonction f est définie et dérivable sur ~ (comme
. existe 1 - cos(2)
11m u 11
n-+oo
= 4
somme de fonctions dérivables) et :

b) On remarque que :
X
n-1 1 1 n-1 1 \f XE [ij, f' (X) -e- xln(l +ex) +e- x · _e_
1 +eX
Un= L = - L
-;::===
k=O, ./
2ny 1 - 4
I(k)2
n Zn k=O . / 1(k)2
y 1- 4 n : e-XL:XeX - ln(l + é))

Introduisons la fonction f : x ....... J1 x qui est continue


= e - xg(é)
2
1-T
sur [O; l] alors : Or g est négative sur [ij+, et ex e: [ij+ pour tout x e: [ij_ On en
"O déduit que:
0

0
C:
::J Un=~._!.
2
ïl f (~)n
n k=O
1
\f X E [ij, / (X) ~ 0

lfl
• Remarquons que f(x) = ln(~+ex) .
,--i D'après le théorème de la valeur moyenne, on a donc : Or d'après les résultats du cours limé 0 et
0
N x--oo
ln(l + u)
@ lim 1. =
-k)
....,
.c lim -
1 n- l
.f ( existe r1 --;::=1== d r u-0 u
r /()existe!
Ol
·;::
n-+oo n kz;O n Jo J1 - (fl2 O n en d'd " que x2~oo
e utt x ..
= .. par compos1tt0n de
>-
a.
0
u [2arcsin ( ~) J: limites.
De plus f(x) = e-x(ln(ex) + ln(e-x + ll) = x e -x +
2rn-o) e- x ln(e- x + 1).

198

www.bibliomath.com
Intégration, équations différentielles Chap,itre 8

Par croissances comparées lim xe - x = O.


x- +oo 1
(E') : y'+ - - - y=O
Par produit de limites lim e - x ln(e- x + 1) = 0 x O= O. x(x+ 1)
x-+oo
· · · de x - x(x+
U ne prurutive 1
lJ = x1 - x+1 1 est
Donc, finalement, lim f (x) ex[pe O par somme de li-
x- +oo
mites. x-. ln(x) - ln(x+ l) =ln(_:_)
x+l
• On en déduit le tableau suivant :
Donc:
X - oo +oo
Signe de
- s / = { XE [ij+* ...... ke -!n(....L.) X+l
x +I = k--;-, ke: [ij}
J'(x)
• Pour déterminer une solution particulière d e (E)
sur JO, +oo[ utilisons la méthode de la variation de la
Variations de 1~ constante.
f
0 Cherchons une solution particulière f définie par

c) • Commençons par résoudre sur [ij l'équation homogène f (x) = k(x) · -x+l
X
associée :
avec k une fonction dérivable sur ]O; +=[.
1
(E ) : y'+ y= 0 Alors .f est solution de (E) si, et seulement si :

qui admet pour ensemble de solutions sur [ij :


, x + 1 arctan(x) , 1
k (x) · - - = <> k (x) = - - - · arctan(x)
S' = {xe:[ij-. ke-x, ke:[ij} x x(x + 1) (x + l) 2

On a donc:
• D'après la question b), .f vérifie :

X
'rlx>O, k(x) = f -- 1
-
(x+ 1) 2
· arctan(x) dx

/"(x) + f(x) -e- x ln (1 + é) + e-x · _ e_ Utilisons une intégration par parties :


l + ex
+e-xln(l +é)
1 u(x) = arctan(x) v' (x) = (x.; 1)2
1 +ex u'(x) = -x21+-1 v(x) = - xl1

Donc f est une solution particulière de (E) sur [ij, Comme u et v sont de classe CI sur~:, on a:
• On en déduit que l'ensemble des solutions de
est :
(E) sur [ij
k(x) = - arctan(x) · -1- +
x+ 1
f l
(x + l)(x2 + 1)
dx

Or on a vu au a) que pour tout x > 0 :


'O
0

0
C:
::J l;IM (x + l)(x2 + 1)

lfl
a) Pour tout t > 0 : =
,--i
0 2 2
N 1( 1 1 - t) 1 t +1 +1- t 1 Donc, pour tout x > 0 :
@ 2 1 + t + t2 + l = 2 (l + t)(l + t2) = (1 + t)(l + t2)
....,
.c
Ol b) Sur JO,+oo(, x(x+ 1) ne s'annule pas donc k(x)
·;::
>-
a. arctan(x) 1 { 1 :> )
, 1 arctan(x) - + - ln ll+xl+arctan(x)- - ln lr+ll
0 (E)<>y + - - - y = - - - x+ l 2 2
u x(x+ l) x(x+ 1)
• Résolvons sur JO,+=[ l'équation homogène associée : -x-l
- - arctan(x) + ln F J+x
g
2(x + l) \ J1+x2

199

www.bibliomath.com
Chapitre8 Intégration, équations différentielles

On en déduit que : b) Déterminons ensuite les éventuelles solutions de (E} sur llt
Pour cela, procédons par analys-synthèse.

x- 1 x + l { J+x
g Analyse (=conditions nécessaires). On suppose qu'il
'v' x > 0, f (x) = - - arctan(x) + - - ln ~
existe au moins une solution .f de (E) sur IR1.
2x x v 1 +x2
Alors f est aussi solution de (E) sur 11 et 12. D'après ce qui
• Donc l'ensemble des solutions de (E) sur !RI~ est :
précède, on sait donc qu'il ex.iste deux constantes réelles
k1 et k2 telles que :
x --1 arctan(x)+x-+-l ( k +ln {Jg+x
S = {x - - ~),

l
2x x vl+x2 k.1 + l _ arctan(x) si x < 0
.f (x) = xz x xz
b. + 1. _ arctan(x) si x > 0
l;fl
a) Résolvons l'équation différentielle sur chacun des inter-
x2° X xZ

valles 11 =I -oo;O[ et 12 =JO; +oo[. De plus, f doit être dérivable sur IR1 (par définition d'une
solution) et donc en particulier continue en O.
Sur I1 et I2, x ne s'annule pas donc :
3
Comme : arctan(x) = x - x3 + o(x3)
(E) ç:>
2
y' + - y = - -
1 x- o
x 1 +x2 on a: 1. - arct~x = K + o(x)
X X x- 0 3
• Résolvons tout d'abord l'équation homogène associée : donc: 1. _ arctanx _ K
X x2 x-0 3
2 et donc :
(E'): y'+ -y=O lim ! _ arctanx exgte 0
X
x- o x x2
Une primitive de x ...... f
est x ...... 2lnlxl = ln(x2) donc
l'ensemble des solutions de (E') est. sur chacun des inter-
f n'a donc une limite finie en o+ ou o- que pour
valles :
k1 = k2 = o.

S / ={x- 2k . On a donc:
X

l
• Pour chercher des solutions particulières de (E) sur 11
x - arctan(x) six> 0
et 12, nous allons appliquer la méthode de variation de la f (x) =
constante. x-arctan(x) six< 0
Posons [j(X) = k:~x ) avec kj une fonction dérivable sur Ij,
pour j e: 11,2}. Ma.is alors limf (x) = lim f (x) = 0 - arctan(O) o.
x-o+ x- o-
Alors [j est une solution de (E) sur I j si, et seulement si : Comme f est continue en Oon a donc f (0) = O.

k'.(x) 1 x2 1 Ainsi:
x2
1
- - =- -
l+x2
ç:> k 1-(x)
1
=- - =1 -
l+x2
--
l+x2 'v' x e: !RI, f (x) = x- arctan(x)
'O
0 pour tout x e: 1j.
C:
::J
On obtient donc : Synthèse (=condition suffisantes).
0
lfl
,-i 'v'xe:lj, kj(x) = x - arctan(x) On vérifie facilement que x- x-arctan(x) est solution sur
0 !RI de (E).
N
@ • Les solutions sur 11 sont donc de la forme :
...., Conclusion. (E) admet une unique solution sur !RI qui est
.c k1 1 arctan(x) x- x - arctan(x) .
Ol x ...... - + - - -----,--
·;:: x2 x x2
>-
a. Commentaire: Si l'équation avait été sous forme résolue, on
0
u aurait dû trouver une infinité de solutions! Mais ici ce n'est
k2 1 arctan(x)
pas le cas et on voit donc que le cours ne s'applique plus, dès
X ...... - + - - -----,,--- que l'équation difjërentielle n'est pas sous forme résolue.
x2 x x2

200

www.bibliomath.com
Intégration, équations d ifférentielles Chap,itre 8

-
a) Procédons par analyse-synthèse.
Analyse (=conditions nécessaires). Soit y une solution de
(Ea) sur lll1: (on suppose qu'il en existe au moins une).
Ainsi z est deux fois dérivables sur IR1 (comme composée de
fonctions deux fois dérivables).
On a alors pour tout t e: IR1 :

z' (t) =e' y' (e')


On pose:
et
V X > o. Z(X) =x 2y(x)
Par définition d'une solution y est dérivable sur lll1:, donc z" (t) e2t y'' (e') + e' y' (e')
z est aussi dérivable sur !RI:. eu y" (e') + z' (t)
On a pour tout x > 0 :
D'autre part et> 0 pour tout te: IR1 donc on peut substituer
et à x dans (Ea) :
z1(x) = 2xy(x) + x 2 y' (x)

et
z" (x) = 2 y(x) + 2x y' (x) + x2 y" (x) + 2xy' (x) et alors, en dérivant cette relation :
Et donc, puisque y est solution de (Ea) sur !RI: :

Vx > 0, z" (x) - z(x) =0 En réinjectant dans l'équation précédente on obtient


donc:
On obtient donc qu'il existe (A, B) e: lll12 tel que :
\f t E 1Jl1, z11 ( t) - y'(e-t) + z'(t)
Vx>O, z(x)=Aé+Be-x
- y(e') + z' (t) (cary' (e') = y(e- t))
Finalement : = - z(t) + z' (t)
z(x) ex e-x
Vx>O, y(x)=- =A-+B- I.:équation caractéristique r 2 - r + 1 = 0 ayant pour solu-
x2 x2 x2
tions r = 1±~/3, on sait donc qu'il existe (A, B) E IR1 2 tel que:
Synthèse (=conditions suffisantes). Soit (A, B) e: Ol1 2 .

Il est clair que la fonction x - AxA e: +Be-~, est deux fois Vte:lll1,
( J3t) +Bsm
z(t) = Acos T
( . ( TJ3 t )) e2L
X X
dérivable sur lll1: et de simples calculs permettent de véri-
fier qu'eUe est bien solution de (E 0 ). Mais on a vu que :
Conclusion. I.:ensemble des solutions de (Ea) sur !RI: est
Vte:IRI, z 1 (t) = et y 1 (er) = ety(e-t) = et z(-t)
donc:
Or:

"O b) Procédons par analyse-synthèse. Vte:IRI, z'(t) = e1 (-A~ sin(~\) +B ~cos(~\))


0
C:

~e~ (Acos( ~ r) +Bsin( ~ t))


::J Analyse (=condition s n écessaires). Soit y une solution de
0 (Eb) sur !RI: (on suppose qu'il en existe au moins une). +
lfl
,--i
0
On pose :
N V te: !RI, z(t) = y(et) donc:
@
....,
.c
Par définition d'une solution y est dérivable sur !RI:. Mais z '()
t = e 1.(A+Bh
2 cos -
2
(v'3)
2 2
sm (v'
t + B - Av3. - 3 t ))
2
Ol ona:
·;::
>-
a. Vx>O, y'(x) = r(;)
0
Et pour tout t e: IR1 :
u
ce qui prouve que y' est eUe aussi dérivable surlRI: (comme
composée de fonctions dérivables). J3t) -Bsm
etz(- t) = e2L ( Acos ( T . (T
J3t))

201

www.bibliomath.com
Cha pitre8 Intégration, équations différentielles

On doit donc avoir, pour tout t e: IR : (j)(t) > 0 sur JO, +=[ ;
4>(t) < 0 sur J -=,01 ;
B\.13- Acos (\.13 t ) = AV3 - 3B sin
. (\.13 t ) 4>(0) = O.
2 2 2 2
Et donc la fonction t ...... (l)~t) est définie et continue sm IR*.
En prenant t = 0, on obtient que A= B\.13.
• Revenons à f :
Ainsi:
six> 0, alors l'intervalle d'intégration [x,2x) est inclus

t
V e: IR, z(t) = Be ~ ( V3 cos ( ~\) + sin ( ~ t)) dans JO, +=[ donc t - (1)/tl est continue sur [x, 2x] et f
est définie en x;
six< 0, le raisonnement est similaire ;
et finalement:
six= 0, alors la fonction à intégrer n'est pas définie et
Vx>O, y(x) par suite f non plus.
z( ln(x)) On en déduit que f est définie sur IR*.

= Bvx( V3cos( ~ ln(x)) +sin(~ m(xi)) b) Pour tout réel non nul x :

f(-x)=J-2x dt
Synthèse (=conditions suffisantes). Soit B e: Dt -x t + sin(t)
Il est clair que la fonction
Effectuons le changement de variable u = - t :
:!,}
f: x ...... BJx ( V3 cos ( 1n(x)) + sin ( 1n(x))) :!,}
est deux fois dérivables sur IR:, et de simples calculs per- 2x - du l2x du
mettent de vérifier qu'on a bien: f(-x) =
lx - u - sin(u)
=
x u + sin(u)
= f(x)

Comme ~· est de plus symétrique par rapport à 0, on en


VX> 0, f' (X) =f ( ~) déduit que f est paire.
c) • Soit F une primitive de t - t+sfu(t) sur l'intervalle Ili:,
Donc f est solution de (Ebl sur IR:.
alors :
Conclusion. I:ensemble des solutions de (Eb) sur~: est:
Vx > 0, f (x) = F(2x) - F(x)

De plus Fest C1 sur~:. cari est continue.


Sb= {xe:!R:-.. svx( V3cos( ~ ln(x))
On en déduit que f est C1 donc dérivable sur IR: et que :

+sin ( ~ m(x)) ), Be: IR}


2
Vx > 0, / 1 (x) = 2F 1(2x) - F' (x) = . - ---
2x + sm(2x) 1 + sin(x)

u
0
l;fl • Le même raisonnement sur l'intervalle llf'.. donne que f
C: a) • Notons (j) : t ...... t + sin(t). est aussi dérivable sur IR'.'.. et :
::J
0 Cette fonction est dérivable sur~ (somme de fonctions dé-
lfl rivables) et : I 2
,-i Vx<O, f (x) = - - - -
0 . 2x+sin(2x) 1 +sin(x)
N
Vt e: ~. 4>1 (t) = 1 + cos(t) ç 0
@
..., • En conclusion, f est dérivable sur IR* et:
.c et 4>1 (t) > 0 sauf en des points isolés : les tk = (2k + l)n,
Ol 2
ï:::: k e: Z.
>-
a. Vx f. O, f' (x) = 2x + sin(2x) 1 + sin(x)
0 Comme (j) est continue sur IR, on peut en déduire qu'elle
u est strictement croissante sur~- Attention : on ne peut pas se donner une primitive de f sur
Or 4>(0) = 0 donc : Ili* car~* n'est pas un intervalle.

202

www.bibliomath.com
Intégration, équations différentielles Chap,itre 8

d) Sachant que pour tout te: llll, -1 ~ sin(t) ~ l, on en déduit En multipliant par -1 :
que, pour tout te: llll, t - 1 ~ t + sin( t) ~ t + 1 puis que, pour
tout t > 1 : 1 1
- - - S::. --
l + t"' 2
1 1 1
--S::. S::.--
t+ 1"' t+sin(t)"' t - 1 Finalement :
Si x > 1 alors 1 < x < 2x. I.:intégrale étant croissante et les
bornes dans le bon sens, on a pour tout x > 1 : t 1 1 1
OS::.--=l-x--S::.1--=-
"' l+t l+t"' 2 2
2x dt.. 12x dt
1x - - ~f(x)~
t+J X
--
t-1 Donc, pour tout te: [0, 1) :

[1n(t+ l)J:x ~f(x) ~ [1n(t-1)J:x OS : .tn


--& ( -
"' (1 + t)I! "' 2
1) 11

2x +
In ( - 2x --
-1 ) ~f(x) ~In { - l) I.;intégrale étant croissante et les bornes dans le bon sens
x+l x-1 (0 < 1):
2x +l
Or lim - - =2.
x-+oo x+ 1
o= Jo rl Odt~ u~ ~ Jorl(1)1! (l)n
2 dt= 2
2x+
Donc lim In ( - - 1) = ln(2) par continuité de In au
x-+oo x+ 1
point 2. On en déduit que, pour tout entier naturel n :

De même lim ln ( - -
x-+oo x- 1
2x - l) existe
= ln(2). 1
Ü< Un~ -
On en déduit, d'après le théorème des gendarmes, que : 2
puisque la fonction x ...... (1x est strictement croissante sur
lim f(x) exgte ln(2) llll:.

-
x-+oo
c) Soit n;;:: 1.
La fonction t - i!r est croissante sur [O, l] (sa dérivée est
a) Pour n = l positive).
Donc :
UJ
ri_ t _ dt
Jo 1 + t
1 l+t-l
= Lo 1+ t
dt et donc :

= ri
Jo
1 - _ 1_ dt
1+ t
pour tout te: [a, 1).
u
0
C:
[t - ln(l+ t)]~ Alors, l'intégrale étant croissante et les bornes dans le bon
::J l - ln(2) sens (a~ 1):
0
lfl
,-i b) Encadrons i! t sur (0, l) )n dt= (1- a) (-a- )n > 0
0
N
@
u~ ;;::
1
a
1 ( -a-
l+a l+a
....,
.c
Ol
ï::::
Donc: Finalement, par croissance stricte de X ...... rx sur llll:. :
O<l+t~2
>-
a. 1 a
0 En passant à l'inverse: (1-a)ii - - ~un
u l+a
1 1
-2 ~--
l+t
pour tout n ;;:: 1.

203

www.bibliomath.com
Cha pitre8 Intégration, équations différentielles

d) Si a e: (0, 11, on a donc pour tout n;;:: 1 : Ona:

a 1 2
(1 - a) iï
L
+a ~ un~ .
fa(11 - x) = 1 - aeme- . 1
ix =1l+ae- ix. 12
1 2 1

On va donc chercher une suite (an) à valeurs dans (0, 1] Or :


2
telle que (1 - an) iï
l
1~.. converge vers !.
1
l1- aeix 1 = (1- acos(x)) 2 + (asin(x))2
Par exemple, on peut essayer de trouver une suite (an) telle et:
1
que (1 - a,i) ïï converge vers 1 et 1 z~. converge vers !.
l1 - ae- ix 1
2
= (l - acos(- x)) 2 + (asin(- x)) 2
On essaye an = 1 - fi ...et ça marche !
En effet, lim an =1 donc n-+oo
lim -3:!!.._
1 = (1 - acos(xli2 + (- asin(x)i2
n-+oo 1 + an 2 (1- acos(x))2 + (asin(x)) 2

De plus ln ( (1 - an) fr) = i ln( i) = _ln~n) qui tend vers donc:


0 (croissances comparées), donc (1 - an) ri co nverge vers
1
fa(11 - x) =f - a(X)
eO = 1.
pour tout a e: (-1, l] et x e: (0, 11).
Par produit de limites, on a donc :
• Soientae: (-1,1) etxe: [0,TC).
. l an 1
hm (1 - a,i) 11 - - =- Ona:
n-+oo l + an 2

De plus, comme (an) est à valeurs dans (0, l] : fa(~)!-a(~) 1 - a · e1• ïX 12 · 11 + a · e1•ïX 12
1
l an 1 ï ( 1 + a · e1•Xï )12
1(1 - a · e •X)
1
Vn~ l, (1 - an)" - - ~ Un ~ -
1 +an 2
l1-a2-eix12
donc, d'après le théorème des gendarmes:
[ 0 2 (x)

. existe 1
1un Un = -
n-+oo 2 b) D'après le a) :

-
a) • D'après l'inégalité triangulaire, pour tout a e: (- 1,1] et
xe:[0,11] :
Vae:) - 1,1(, Vxe:[0,11),

= =
[a(x) ~(l- lal) 2 >0

donc la fonction g: x - ln (.fa(x)) est définie sur [O, Tli).


Or g(x) ln (ta (x)) ln ( (1 - a cos(x) )2 + (a sin (xll 2) donc
la fonction g est continue sur (0, TC], comme somme et com-
posée de fonctions continues sur cet intervalle.
or leixl = 1 donc : La fonction g est donc bien définie sur l'intervalle) - 1, 1[,
qui est symétrique par rapport à O.
u
0
C: De plus, pour tout a e:J -1, 1[ :
::J
0 etcommea e: [- 1,1) :
lfl
,-i g(-a) = L1T ln U-a (x)) dx = L1T ln (fa (TC - x)) dx
0
N
0 ~ 1- la l ~ l1- aeix l ~ 1 + lal
@
....,
Effectuons le changement de variable t =11 - x, dt = - dx :
.c De plus la fonction x - x2 est croissante sur im+, donc :
Ol
ï::::
>- (l - lal) 2 ~ fa(x) ~ (1 + la i/
0
a. g(-a) = - Lo ln (fa (t)) dt= L1t ln (fa (t)) d t= g(a)
u pourtoutae: (-1,ll etx e: [0,11).
• Soient a e: (-1, 1] et x e: (0, 11]. On en déduit que g est paire sur ] - 1, 1[.

204

www.bibliomath.com
Intégration, équations différentielles Chap,itre 8

c) Soitae:J -1,1(.0na: e) Soit a e:J -1, l [.


D'après la question c), g(a) =!
g( a2) :l, g (a 4 ) = =....
11
Par une récurrence immédiate on montre que, pour tout
fo ln(/adx)) dx entier naturel n :
11
fo ln(fa(~)f-a(~)) dx g(a) = }ng(a2" )
11
fo [ln(ia (~)) +ln(.f- a(~))] dx Or lal < 1 donc lim a 211 ex!Jte O (sous-suite de la suite
n-+oo
11 11 géométrique (a 11 )).
fo ln(.fa (~)) dx+ fo ln(!- a (~)) dx De plus, g est continue en Odonc

fo ln (fa (~)) dx + {T ln (!a (7r-i)) dx


11
lim g a
n-+oo ( 2") existe lim a 2")
= =g ( n-+oo · =g(O) =0

donc par quotient de limites :


Dans la première intégrale on pose t = f, et dans la se- lim ~ (a2") ex!Jte 0
conde t =7r - f On obtient: n-+oo 2 11 g

Mais on aussi :
g(a2) 2{
n
2
1n(/0 Ctl) dt-2 f n
2
1n(/aUl) dt
lim -1g { a
n-+oo 2 11
2") existe
= lim g(a) = g(a)
n -+oo
2 fo\n(/aCtl) dt Par unicité de la limite, on en déduit que:
2g(a)
Vae:J - 1,1[, g(a) = O

grâce à la relation de Chasles.

d) Puisque la fonction ln
est croissante sur Ili~, la première
double inégalité de la question a) nous donne, pour tout
a e:]- l ,ll etxe:[0,7r):
-
a) Effectuons le changement de variable t

b f(x) dx = - ra
=a+ b - x

f(a+ b- t) dt = lb f(a+ b - t) dt
:

la Jb a
b) Comme f(a + b - x) =.f(x) pour tout x e: [a, bl, alors :
I.;intégrale étant croissante et les bornes dans le bon sens
(0 < 7r), on obtient :
I= Lbxf(x)dx = i \ r f(a+b - x)dx
7r ln {(l - lal) 2) ~ g(a) ~ 7r ln {(1 + lal) 2) Effectuons le changement de variable t = a+ b- x :

pour tout a e:J - l, l [.


l= i \a+b-t)f(t)dt=(a+b) ib f(t)dt-l
u
0
Or lim
a-o
ln ((l - lall2) = 0 et lim ln (c1 + lall )
a-o
2
0 donc,
C: On en déduit que
::J d'après le théorème des gendarmes
0
lfl
b xf(x) dx a+blb f (x) dx
=--
,--i lim g(a) ex!Jte 0 la 2 a
0
N a-o
@ c) Notons f(t) = 1+cos
sin(9
2 (t)
.
....,
.c
Ol
Or g(O) = fo 11
ln (1) dx = O. On a donc : Alors:
ï:::: sin(t)
>-
a. Vte: [0, 7r]. f(O + 7r - t) = + (- cos(t)) 2 = f(t)
0 fun g(a) exgte g(O) 1
u a-o
Nous pouvons donc utiliser le résultat de la question pré-
Par définition, on en déduit que g est continue en O. cédente :

205

www.bibliomath.com
Chapitre8 Intégration, équations différentielles

On en déduit que, pour tout entier naturel n :


n sin(t) d ~ rn
Lo
t.
1 + cos2(t)
t = 2
TC
sin(t) dt
Jo
l +cos2(t)
Lrr
-sin(t) ln+2 + ln 2
1 n cos(0) cos( (n + 1)0) dt
-n 5 + 4cos(0)
-- dt
2 o 1 + cos 2 (t) 11n 4cos(0) cos( (n + 1)0)
- dt
-~ [arctan(cos(t))jn 2 -n 5 + 4cos(0)
2
-~(-~-~)
0

=
1
-
ln
(s+4cos(0)-s)cos((n+ 1)0)
dt
2 -n 5 + 4cos(0)
,r2
= 4
!l
2
rr cos((n+ 1)0) dt- ~ln+l
2

-
-1(

5
0- 2ln+I
5
a) • Sachant que pour tout réel t, - 1 ~ cos(t) ~ 1, on a donc -2ln+l
pour tout 0 e: [-TC, TC) et ne: N :
c) • La suite (ln) est donc une suite récurrente linéaire d'ordre
cos(n0) ~ 1 et 5 +4cos(0) ~ 1> 0 2 d'équation caractéristique :

donc: 5
1 r2 + - r +1 =0
cos(n0) ~ 1 et O< e ~1
5+4cos( )
2

et alors par produit par une inégalité positive :


Les racines sont -! et -2 donc, on sait qu'il existe (A, B) e:
llll 2 tel que :
cos(n0)
----~l
5+4cos(0) 'v'n e: N, In=A(-~r +B(-2)n
{;intégrale étant croissante et les bornes dans le bon sens
(-,r < ,r), on en déduit que :
Or Io = A+ B et I 1 = ~ - i Io = -!A- 2B donc A et B véri-
fient :

ln ~ ln
-1(
d0 =2,r
A+ B = Io
pour tout n E: N. {
A+4B = - TC+ ~lo
Comme lo = l-rrr
5 + 4 cos(0)
1
d0 et = On obtient :
n cos(0)
l-rr 5 + 4cos( e)
d0 on en déduit que :

5 n cos(0) + i
"O
0
C:
l1 + - lo
4 l! lrr
----d0
- n 5+4cos(8)
4 cos(0) + 5 de
d'où :
::J
0 4 - n 5+4cos(0)
lfl
,--i 1
0 : - X 27T
N 4
@ TC • Il reste à calculer To. Puisque rien de «simple» ne saute
...., 2 aux yeux et que Je programme de BCPST interdit trop de
.c
Ol technicité dans Je calcul intégral, il va falloir être astucieux!
·;::
b) Sachant que cos(p) + cos(q) = 2 cos ( p;q) cos ( p;q) on en
0
>-
a.
déduit que :
Dans la formule précédente, puisque 1-11< l, on a :
u
cos((n + 2)8) + cos( n0) =2cos( (n + 1)0) cos(0) lim (~+!10)(-!)n
n-+oo 3 2 2
=O
206

www.bibliomath.com
Intégration, équations d ifférentielles Chap,itre 8

Mais alors, puisque lim (-2) 2n = lim 4n = +oo, on a, Effectuons une seconde intégration par parties :
n-+oo n- +oo
par somme de limites :
/z (t) = COS(7T t) ~ (t) = U
1
(t)
· rr 1
+oo SI - 3 + 2 10 > 0 f~(t) = - ,rsin(,rt) gz(t) = u(t)
lim I2n
n-+oo = Osi - !!.
3
+ .l2 Io =O h et g2 étant de classe e1 sur [0, l]. on obtient donc :
{
-OO SI. - 3
rr + 2tr0 < o

La sous-suite (lz11) est nécessaire car la suite (( - 2) n) n'a pas


de limite en +oo (divergente de seconde espèce), par contre
= - ,r{[u(t)cos(,rt) ]~ + ,r LI u(t)sin(,rt)dt)

( (- 2) 211 ) diverge vers +oo (divergente de première espèce). ,r(u(l) + u(O))-,r 2 fo 1


u(t)sin(,r t) dt
Mais on a vu au a) que:
Ainsi :

ce qui empêche d'avoir : lim I2n = +oo.


n-+oo
L 1
( u" (t) + ,r 2 u(t)) sin(,r t) dt= ,r(u(l) + u(O)) = 0
On a donc nécessairement :
b) Procédons par analyse-synthèse.
1
7T
- - + - To ~ 0 Analyse (= conditions nécessaires). Soient be: [ij et u une
3 2 "' solution du problème de Cauchy associé à b.
Le même raisonnement sur la suite (12 11+ 1) donne que: • D'après Je résultat du a) :
7T
--+-Io>.:O
3 7
1
2 L 1
( u" (t) + ,r 2 u(t)) sin(,r t) dt = 0

Ainsi:
7T 1 donc:
--+-Io=O 1
3 2 -b fo sin2(,rt)dt=O
ce qui donne :
2
Io = - ,r Comme la fonction t -. sin2(,r t) est continue, positive, et
3 non constante égale à 0, sm [O, l]. on sait par théorème
• En conclusion : que:
1
_ 2,r ( 1 )n fo
sin 2(,rt)dt>O
'v'ne:N, In-
3 -2

-
On obtient donc b = O.
Si b f. 0, on vient de prouver que le problème de Cauchy n'a
pas de solution.
1

"O
a) Calculons tout d'abord I = fo u" (t)sin(,rt) dt. • Pour b = 0, u est alors solution de l'équation homogène
y" + 7T2 y =o.
0 Effectuons une intégration par parties :
C:
::J Par les méthodes habituelles, on obtient donc qu'il existe
0 (A, B) e: [ij 2 tel que :
lfl
,-i
/1 (t) = sin (,r t) g~ (t) = il' (t)
0
N f{ (t) = 7T COS(7T t) g1 (t) = il (t) u : t -. Acos(m) + Bsin (t7T)
@
...., Comme fi et g1 sont de classe e 1 sm [O, l]. on a: • On donne ensuite les conditions initiales: u(O) = - u(l) =
.c 1.
Ol
·;::
>- 1 (A, B) est donc solution de :
0
u
a.
[ u'(t)sin(,rt)j~ -,r fo u'(t)cos(,r t) dt
1
= - ,r fo u'(t)cos(,rt) dt
A=l
{ -A=-1
207

www.bibliomath.com
Cha pitre8 Intégration, équations différentielles

donc : A= 1. Or cos 2 t = l -sin2 t, d'où :


• En conclusion, pour b = 0 on a :
In+2 = (n + llln - (n + 1Hn+2
u: t ...... cos (tn) + Bsin (tn)
et donc :
Synthèse (=conditions suffisantes). Soit Be: IJll. n +1
Il est clair que la fonction u: t ...... cos(tJT) +Bsin(tJT) est
Vn e: 1\1, In+2 = --In
n+2
deux fois dérivable sur Ili et qu'elle est solution du pro-
u" ( t) + n 2 u( t) =0 • On obtient rapidement que To =; et 11 =1.
blème de Cauchy :
{
* Pour n = 2p (p e: 1\1), on a en appliquant plusieurs fois la
u(O) = - u(l) = 1 formule de récurrence :
Conclusion . Si b i: 0, le problème de Cauchy
2p-l
u" (t) + n 2 u(t) = -bsin(n t) - p-r.,; .,...<,,qr~J
i
n'a aucune solution sur !RI. 2
{
u(O) =- u(l) =1 ~ =
2p - 3
2 (p - 1).!zt,r-2r

u"(t) +n 2 u(t) = 2p-5


- - { . - ,, _
Si b = 0, le problème de Cauchy a 2(p - 2i=-.,,
{
u(O) = -u(l) = 1
pour solutions sur Ili les fonctions :

u: t ...... cos (tn) + Bsin (tJT)

-
où Be: !RI.

En multipliant ces égalités on obtient:


a) • Sur [O; ; ], sin est continue à valeurs positives et n'est pas
identiquement nulle. Il en est donc de même pour la fonc-
tion t ...... sinn (t), pour tout ne: 1\1. (2p - 1) X(2p - 3) X (2p - 5) X• ·· X 3 X 1
bp = (2p) X 2(p - l) X 2(p - 2) X•• • X 4 X 2
~
Par théorème, on sait donc que:

Vn e: 1\1, 111 >0 On multiplie cette fraction par :

• Effectuons le changement de variable x = ; - t. On ob- (2p) x2(p - 1) x 2(p - 2) x · ·· x 4 x 2 2Pp!


1- ---
tient : - (2p)x2(p - l)x2(p - 2)X·· · x4x2 - 2Pp!

ce qui donne:
Vn e: 1\1, In = -
(0
h. sin 2 11
(
1T r !!
2
- x) dx =Jo cosn(x) dx I - (Zp)! I
u 2 2p - z2P(p!)2 0
0
C:
::J b) • Soit ne: 1\1.
0 l! Or Io =; donc :
lfl On remarque que 111+2 = J02 sin(t) x sin11 +1 (t) dt. On va
,-i
0 donc effectuer une intégration par parties :
N (2p)!
Vp e: 1\1, I2 - 1l
@ P - 2 2p+ 1 (p!l2
...., u(t) =sin 11
+1 (t) =sin(t)
v' (t)
.c
Ol u (t) =(n+ l)cos(t)sinn(t)
1
v(t) =- cos(t) * pour n = 2p + 1 (p e: 1\1), nous obtenons avec les mêmes
ï::::
>-
a. techniques que :
0 u et v étantC 1 on a:
u
n l!.
In+2 = [-cos tsinn+l t]! + (n+ 1) fo 2
sinn tcos 2 t dt Vp e: l\l, l2p+l
z2P(p!)2
= (2p + l)!
208

www.bibliomath.com
Intégration, équations différentielles Chap,itre 8

c) • Del' égalité

nous déduisons que :


VnE.N, (n+2)In+2=(n+l)In -
a) Soit g la fonction définie sur JO; +oo( par: g(t) = ln(t}- (t-
1). Elle est dérivable sur JO; +oo[ comme somme de fonc-
tions dérivables et :

Vne:N, (n + 2lin+2In+l = (n + llin+Jin Vt>O, g1 (t) = l-t


t
La suite (( n + llI 11 +J In) m,N est donc stationnaire: On en déduit le tableau suivant :

t 0 1 +CX)
Vn e: N,
Signe de 1

g' (t) + (i)


1
-
• Soit n E. N. On a :

In+I - In = {
n
2
sinn(t)(sin(tl - 1) dt ~ 0
Variations de
g /0~
car sur [o.; ], 0 ~ sin(t) ~ 1.
Donc: Ainsi g est à valeurs négatives sur JO; +oo[ donc
VnE.N, ln+l ~ ln
Vt>O, ln(t) ~ t-1
Ainsi :
b) On pose:
1 n
Vn e: N, In+2 ~ In+l ~ In x=- E xj
n j=l
d) De la double inégalité précédente nous déduisons :
Soit k e: [1, n]. On applique l'inégalité du a) à t = ak = i :
Vn e: N,

car In> 0 d'après Je a) .


c'est-à-dire:
Xk
0 r I n+2 = + 1 I n, d one 1a sm·te ( --ç
n+
11 2
111+2 ) converge vers 1. ln(xk)-ln(x) ~ -=- -1
X
Alors, d'après l'inégalité précédente et Je théorème des pour tout k e: [l, n].
gendarmes, la suite ( 11j;, 1 ) coverge elle aussi vers 1.
On somme ces inégalités :
Ceci prouve que In+ 1 - In.
+oc
Or (n + 1)1 11 +J In = ~ donc I~ - .2:. .
Î: (ln(xk) - ln(x)) ~ bÎ:l (~X - 1)
bl
+oc 2n
On en déduit que donc:
n 1 n
u
0
C:
ln -
+oc V~
{n L 1n(xk) -
k=l
nln(x) ~
X
=k=l
L Xk - n
::J
'--....-'
0 Or on a montré à la question b) que =O
lfl
,-i
0
N _ (2n)! n _ (2:) n Ainsi après multiplication par ti :
Vne:N, I2 , 1 - - - - · - - - · -
@ (2n n!)2 2 22n 2 1 n
...., -L ln(xk) ~ ln(x)
.c
Ol
·;:: Finalement : n k=l
>-
a.
0 Finalement :
u 2n) - 22nx -2 {[;
- - - 22n-
( n +oc
-1Ln (1 L
1T 4 n - ,/ITTl
n )
ln(xk) ~ ln - xk
n k=l n k=I
209

www.bibliomath.com
Chapitre8 Intégration, équations différentielles

c) Soit n;,,: 1. Pour tout k e: [1, nil, on pose: avec k e: C.

(k-1) >
Alors z' (x) = (k + i kx)eix et z" (x) = (2i k- kx)eix.
Xk =f ---;;- Q
Comme z est solution de (Ef) sur~. on a :
puisque f est à valeurs dans~~-
(2i k - kx)eix + kxeix =- eix
On applique le b) :
pour tout x € ~-

~n k=l
f ln(t(~))
n
~ ln(~n k=l
[ t(~))
n
k vérifie donc :
2ik = -1
et après Je changement d'indice k' = k - 1 : d'où:
1 i
k=- - = -
-L
111- 1 ln (f (k)) - L l f (k))
- ~ In (111- - 2i 2
n k=O n n k=O n On a alors:
pour tout n;,,: 1. Vxe:~. z(x) =-ix2 e ·x
1

De plus, puisque les fonction f et ln of sont continues sur et finalement :


(0, l], Je théorème de 1a valeur moyenne nous donne :
X
Vx e:~. f (x) = JP m(z(x)) = cos(x)
L
lim -1 n-1 f ( - · fol f (t) dt
k) ex!Jte 2
n-+oo n k =O n o
• En conclusion, l'ensemble des solutions de (E1) sur~
et: est :
lim -1 1\=,
n-+oo n k =O
1
L. ln ( f ( -k )) existe
n
= Lol ln(/Ct)} dt
S1 = {xe: ~-Acos(x) +Bsin(x) + ~ cos(x), (A,B) e: ~ 2}
Mais comme f est continue, positive et n'est pas la fonc-
JJ
tion nulle sur [O, l], on sait que f(t) dt> O. Alors la conti-
b) Remarquons que :
nuité de la fonction In sur~: nous donne :

Vx e: ~, F(x) = x L x f(t)dt - L x tf(t)dt


lim 1n(~nf t(~))ex~te1n(flf(t)dt)
n-+oo n k =O n Jo
La fonction g: t - tf(t) est continue sur~ (comme pro-
Finalement, par passage à la limite dans une inégalité: duit de fonctions continues). Elle admet donc une primi-
tive G de classe C1 sur~- De même, f admet une primitive
H de classe C 1 sur~-

-
Alors:

Vx e: ~. F(x) = x(H(x) - H(O)) - (G(x) -G(O))


a) • Les méthodes habituelles donnent que l'équation homo-
"O
0 gène (E;) admet pour ensemble des solutions sur~ : Donc Fest elle-même de classe C1 sur~-
C:

0
::J
c) • Soit x e: ~ - En effectuant Je changement de variable C1
{ x-. Acos(x) + Bsin(x), (A, B) e: ~ 2 } u =x- ton obtient :
lfl
,--i
0 • Pour déterminer une solution particulière f de (E 1) sur~.
N
on va déterminer une solution particulière complexez de f(x) = sin(x) - Lx (x - u)f (u) du = sin(x) - F(x)
@
...., (Ef) :
.c y"+ y= -eix Or sin est de classe C00 sur~ et F de classe C1 donc f est
Ol
·;::
>- de classe C1 sur ~-
a. Comme i est racine simple (= d'ordre 1) de l'équation ca-
0
ractéristique on sait qu'il faut chercher z sous la forme : La fonction g est donc de classe C 1 sur~- Une de ses primi-
u
tives Gest donc de classe C2 sur~ et H primitive de f l'est
z(x) =kx 1 eix =kxeix également.

210

www.bibliomath.com
Intégration, équations différentielles Chap,itre 8

On en déduit que F est de classe C2 sm IJll et par suite f Synthèse(= conditions s uffisantes). Pour tout x e: IR :
aussi.
d
• On sait que :
x f
t (x- t) t =Lx t (x- t)cos(x- t) +sin(x- t) d
t
LO O 2
On effectue la changement de variable C1 y = x - t :
Vx e:Oll, f(x) = sin(x) - x Lx f(t)dt + Lx tf(t)dt
Lx tf(x - t) dt
sin(x) - x(H(x) - H(O)) + ( G(x) - G(O))
o ycos(y) + sin(y)
En dérivant on obtient : lX (X- y) 2 (-dy)

~ r x ycos(y) dy +:: rx sin(y) dy


VXE: Oll, f 1(X) 2 Jo 2 Jo
= cos(x) - H(x) + H(O) - xH' (x) + G' (x) -~2 Jorx y2cos(y)dy-~2 Jorx ysin(y)dy
= cos(x) - Lx f(t)dt - xf(x)+xf(x) Puis après quatre simples intégrations par parties que l'on
ne détaillera pas ici :
cos(x) - Lx f(t) dt
Lx tf(x - t) dt = sin(x) - f(x)
cos(x) - H(x) + H(O)
pour tout x e: Oll.
En dérivant à nouveau :
Donc f est bien solution de (E2).
Vx e: IR, f" (x) = -sin(x) - f (x) Conclusion. (E2) admet pour unique solution sur Oll la fonc-
tion:
c'est-à-dire que f est solution sur IJll de (E 1). : x e: IR- xcos(x) + sin(x)
.f

-
2
d) Procédons encore une fois par analyse-synthèse.
Analyse (=conditions n écessaires). Soit f une solution de
(E2) sur IR (on suppose qu'il en existe au moins une).
.
a) Si f(x) = xP, f réalise une bijection de [O. a] sur lü, aP J car
On vient de voir qu'alors f est aussi solution de (E 1) sm!R. p e: N* et alors sa réciproque est bien définie de [O,aP ] sm
l
D'après le a), il existe donc deux réels A et B tels que : [O,a] par g(y) = yri.
De plus, pour tout t e: IO, a) :
Vx e: Oll, f(x) = Acos(x) + B sin(x) + ~ cos(x)
/if __E_ l
2
rt rt(t) xP+1 1t + [ tP
Jo f(x) dx+ Jo g(y) dy
Or f doit vérifier l'équation (E2) donc: [p+l O p+l 0

tP ptP
f(O) = sin(O) - Lo tf (x- t) dt= O t·--+t · - -
p+l p +l

et d'après le c):
1
t x tP ( - - +
p+l p+l
__E_)
"O
0 tf(t)
C: f' (O) = cos(O) - Lo f(t) dt= 1
::J
0 b) Les fonctions f et g étant continues sur leurs intervalles
lfl de définitions, elles admettent des primitives dérivables F
,-i Les réels A et B vérifient donc :
0 etG. Alors:
N
@
...., A= O
Vt e: [O, a l, 1.p(t) =F(t) - F(O) + G(f (t)) - G(O) - t.f (t)
.c
Ol
·;::
{ B+!=l
>- La fonction '-Pest donc définie et continue sm [O, a] comme
a. Finalement: le sont F, Go f et f.
0
u
Elle est de plus dérivable sur JO, a[ comme le sont Go f et
Vxe:Oll, f(x) = xcos(x)+sin(x)
f, la fonction F étant elle dérivable sur [O, a].
2

211

www.bibliomath.com
Chapitre8 Intégration, équations différentielles

c) On peut donc calculer <p1(t) pour tout te: JO, a [ : e) Si f (a) = 0, f est la fonction nulle et Je résultat est banal.
Sinon f (a)> 0, et on obtient :
1
<p' ( t) f(t) + / (t) X g O f(t)- f(t) - tf' (t)
tf' (t)- tf' (t) - 1 lb f(t)g(t) dt e: [m,MJ = G([a, b])
1 (a) a
0
Par suite, il existe bien ce: [a, b] tel que :
Comme <p est continue sur [O, al, on sait donc par théo-
rème qu'elle est constante sur [O,a]. Il existe donc un réel
k tel que : - 1 lb f (t)g(t) dt = G(c)
1 (a) a
V te: [O, a), <p(t) =k c'est-à-dire:
Or <p(O) = 0 donc <p est identiquement nulle sur (0, a).
Lb f(t)g(t) dt= f (a) L e g(t) dt

-
I.;égalité (1) est vérifiée pour tout te: (0, a).

f) On vérifie facilement que l'on est dans les conditions d'ap-


a) Gest une primitive de la fonction g. Comme g est continue f
plication de e) avec f (t) = et g(t) = 1 - cos(t). Il existe
sur [a, bl, Gest de classe C1 sur ce segment. donc un point c de [a, b) tel que :
b) La fonction G étant continue sur un segment, J'image par
b 1- cos(t) 11c 1
G de ce segment est un segment [m,M] de~. d'après le
théorème de continuité sur un segment. la
---dt= -
t a a
(l-cos(t))dt= -[c-a+sina-sinc]
a
c) En intégrant par parties sachant que G(a) = 0 et que les
d'où:
fonctions f et G sont C1 sur [a, b) :
b 1 - cos(t) dt~ .! (b - a + 2)
la t a
Lb f(t)g(t) dt [f(t)G(t) ]! - lb f' (t)G(t) dt g) Soitx> 1.
a
On est encore dans les conditions d'application de laques-
f(b)G(b)- Lb f' (t)G(t) d t f i,
tion e) avec f (t) = et g( t) = sin,<tJ . Il existe donc c e: [ l i
tel que :
d) Soit te: [a, b).
On am~ G(t) ~ M donc en multipliant par - f' (t) qui est
positif(/ est décroissante) :
D'où:
-f' (t)m ~ - f' (t)G(t) ~ - f' (t) 1 fil sin(t) d 1 ficsin(t) d
o~-
x2lt2
- - t~-x l
-- t
t
pour tout te: [a, b]. X X

Alors, en intégrant (les bornes sont dans l'ordre croissant) : Mais sur IR?+, on a sin(t) ~ t donc:

u
0 - m(f(b) - f(a)) ~ - Lb t'(t)G(t)dt ~ - M(f(b) - f(a)) 1 fi csin(t)
-
1 fic
1 1 1 1
- - d t ~ - 1 dt = -(c - -)~-(1 - -)
C:
::J
X -1
X
t X-
X
X X X X
0 et en remplaçant dans la formule obtenue en c) :
lfl Ajnsi, pour tout x > 1 :
,-i
0
N
@ mf(a) + f(b)(G(b)-m) ~ Lb f(t)g(t)dt 0~ -
l fil sin(t) 1 1
- - d t ~ -(1 - -)
...., x2lt2
X
X X
.c
Ol ~ Mf(a) + f(b)(G(b) -M)
ï:::: Alors par le théorème des gendarmes :
>-
a. Comme f(b) ;;,: 0 et G(b) - m ;;,: 0, G(b) - M ~ 0, il vient
0
u bien : . 1 fil sin(t) d ex{§te
~ L\(t)g(t) dt~ Mf(a)
11m - -- t - 0
mf(a) x-+oo x2 1 t2
X

212

www.bibliomath.com
CHAPITRE

Déno1nbre1nent et calcul des proba-


bilités

'Tlièmes a6ordés dans {es e~ercices


- Dénombrement
- Règles de calcul des probabilités
- Probabilités conditionnelles
- Indépendance

Points essentiefs du cours pour {a réso{ution


des e~ercices
- Arrangement, combinaisons et permutations
- Dénombrement des applications d'un ensemble fini dans un autre
- Formule des probabilités composées (ou du conditionnement multiple)
- Formule des probabilités totales
u
0 - Formule de Bayes
C:
::J
0 - Indépendance de deux évènements
lfl
,-i - Indépendance mutuelle d'une famille d'évènements
0
N
@
....,
.c
Ol
·;::
>-
a.
0
u

213

www.bibliomath.com
Chapitre 9 Dénombrement, probabi Iités

Les méthodes à retenir

Le nombre d'applications d'un ensemble à p éléments dans un


ensemble à n éléments est égal à nP. C'est aussi le nombre de p-
uplets (x1, . . . , Xp) d'éléments pris dans un ensemble à n éléments.
~ Exercices 9.1, 9.2, et 9.18

Le nombre d'applications injectives d'un ensemble à p éléments


dans un ensemble à n éléments est égal au nombre d' arrange-
ments de p éléments parmi n: A~= (n~~l! = n x (n - 1) x · · · x (n -
p + 1). C'est aussi le nombre de p-uplets (x 1, • • • ,xp) d'éléments
deux à deux distincts pris dans un ensemble à n éléments.
Pour reconnaître les dénombrements ~ Exercices 9.1 et 9.18
usuels
Le nombre d'applications bijectives d'un ensemble à n éléments
dans un ensemble à n éléments est égal au nombre de permuta-
tions de n éléments : n!.
~ Exercices 9.2, 9.6 et 9.18

Le nombre de parties à k éléments d'un ensemble à n éléments


est égal au nombre de combinaisons de k éléments parmi n :
(~) = c~ = (n-'r)!k!. Le nombre total de parties d'un ensemble à
n éléments est égal à zn.
~ Exercices 9.1, 9.2, 9.6, et 9.20

On choisit p éléments parmi n. S'ils ne sont pas deux à deux dis-


tincts on a nP possibilités. Sinon, soit on tient compte de l'ordre
dans lequel on les choisit : A~ possibilités, soit on ne tient pas
compte de cet ordre : (;) possibilités.
~ Exercices 9.1, 9.2, 9.3, 9.6, 9.8, 9.9 et 9.12

- Si on impose un ordre final lorsque l'on tire p éléments parmi


n (ordre croissant etc...), on commence par choisir les éléments
"O sans tenir compte de l'ordre des choix, puis on dénomb re le
0
C: Pour dénombrer des situations plus nombre de façons de mettre ces éléments dans l'ordre imposé.
::J
0 complexes ~ Exercices 9.2, 9.6 et 9.18
lfl
,-i
0 - Si on prend p éléments distincts parmi n, sans tenir compte de
N
@
l'ordre des choix, le nombre de tirages de telle sorte qu'on en ait
...., exactement Pl dans un sous-ensemble à n1 éléments, et P2 dans
.c
Ol
·;::
un sous-ensemble à n2 éléments (p1 + p2 = pet n1 + n2 = n), est
>-
a.
égal à: (;~H;!)- Cette formu le se généralise facilement à k sous-
0 ensembles deux à deux disjoints:(;;) x · · · x (;:).
u
~ Exercices 9.3, 9.8 et 9.9

214

www.bibliomath.com
Dénombrement, probabi Iités Chap,itre 9

- Si on prend p éléments distincts parmi n, en tenant compte de


l'ordre des choix, le nombre de tirages de telle sorte qu'on en ait
exactement p 1 dans un sous-ensemble à n 1 éléments, et p2 dans
un sous-ensemble à n2 éléments (p1 + p2 = pet n1 + n2 = n), est
;t
égal à: (P1 2 )A~; A~; . Si on impose les tirages qui ont lieu dans le
premier sous-ensemble (et donc aussi ceux du deuxième) la for-
m ul e est pus · lement.· APIAP2
l srmp 11 11 •
1 2
~ Exercice 9.3

- Si on raisonne étape par étape dans l'ordre chronologique de l'ex-


périence, on multiplie les résultats de dénombrement obtenus à
chaque étape.
~ Exercice 9.8

- Si les évènements sont deux à deux incompatibles, on additionne


simplement la probabilité de chaque terme de l'union. Si de plus
on an termes, et si la probabilité de chacun de ces termes est égale
à une constante p, alors la probabilité de l'union est n x p.
~ Exercices 9.7, 9.9, 9.13 et 9.15

Pour calculer la probabilité d'une - Si les évènements sont mutuellement indépendants, on multiplie
union/intersection d'évènements simplement entre elles les probabilités associées à chaque terme
de l'intersection.
~ Exercices 9.7, 9.10, 9.12, 9.14, 9.15 et 9.19

Dans le cas contraire on applique la formule des probabilités com-


posées.
~ Exercices 9.3 et 9.13

- Pour traduire sur les probabilités un raisonnement par disjonc-


tion de cas incompatibles, on utilise la formule des probabilités
totales.
'O
0 ~ Exercices 9.4, 9.5, 9.17 et 9.19
C:
::J
0 - Si les différentes étapes sont indépendantes dans leur réalisation,
lfl
,-i
Pour étudier une expérience qui se alors il en est de même pour les évènements associés.
0
N déroule en plusieurs parties ~ Exercices 9.12, 9.13, 9.14, 9.15 et 9.19
@
...., - Si l'expérience comporte plusieurs étapes dans le temps, et si on
.c
Ol
·;:: doit calculer la probabilité d'un évènement couvrant plusieurs
>-
a. étapes, on peut utiliser la formule des probablités composées (en
0 respectant la chronologie).
u
~ Exercices 9.3 et 9.13

215
Chapitre 9 Dénombrement, probabi Iités

On utilise la définition. On a alors deux probabilités à calculer.


~ Exercices9.llet9.14

Pour calculer une probabilité - On utilise la formule de Bayes.


conditionnelle ~ Exercices 9.4 et 9.5

On fait un calcul direct en utilisant l'énoncé.


~ Exercices 9.4, 9.5, 9.11 et 9.19

1 tirage simultané de k boules dans une urne de n boules. 0 =


ensemble des com binaisons de k boules parmi n. 10 1=(~).
~ Exercice 9.3

k tirages successifs sans remise d'une boule dans une urne de


n boules. n =
ensemble des arrangements de k boules parmi n.
IOl =A~.
Pour les histoires d'urnes ~ Exercice 9.3

k tirages successifs avec remise de k boules dans une urne de n


boules. 0 =ensemble des k-uplets de [1, n]. 10 1= nk. Les diffé-
rents tirages forment une famille d'expériences mutuellements in-
dépendantes.
~ Exercice 9.12

"O
0
C:
::J
0
lfl
,-i
0
N
@
....,
.c
Ol
ï::::
>-
a.
0
u

216
Dénombrement, probabi Iités Chap,itre 9

Énoncés des exercices

---- - Coloriages
Un étudiant en BCPST veut colorier ses notes de cours en attribuant la même cou-
leur pour chaque matière: biologie, sciences physiques, chimie, mathématiques, in-
formatique, français et LVl. Il dispose de 10 couleurs différentes.
a) Combien y a-t-il de coloriages possibles?
b) Combien y a-t-il de coloriages, de sorte que chaque matière ait une couleur diffé-
rente des autres?
c) On choisit autant de couleurs différentes qu'il y a de matières. Combien y a t-il
de coloriages possibles en utilisant seulement ces couleurs? De sorte que ch aque
matière ait une couleur différente des autres?
d) Combien y a-t-iJ de coloriages. de sorte qu'au moins deux matières aient la même
couleur?
e) Combien y a-t -il de coloriages, de sorte qu'exactement deux matières aient la
même couleur?

---- - Anagrammes et palindromes

a) Combien y a-t-il d'anagrammes du mot BCPST? du mot ANAGRAMME? du mot


CELLULE?
b) Combien y a-t-il de mots composés de 5 lettres? de 5 lettres distinctes? de 5 lettres
distinctes dans l'ordre alphabétique? de 5 lettres et de sorte qu'il soit un palin-
drome?

u
0

0
C:
::J

lfl
---- - Histoire d'urnes
On dispose d'une urne avec 8 boules blanches, 7 boules noires et 5 boules vertes.
a) On effectue un tirage simultané de 5 boules dans l'urne. Quelle est la probabilité
d'obtenir 2 blanches, 1 noire et 2 vertes?
,-i
0 b) On effectue 5 tirages successifs et sans remise d'une boule dans l'urne. Quelle
N est la probabilité d'obtenir 2 blanches, 1 noire et 2 vertes? Quelle est la pro babi-
@ lité d'obtenir 2 blanches, 1 noire et 2 vertes dans cet ordre? (on proposera deux
....,
.c méthodes différentes)
Ol
·;:: c) On effectue 5 tirages successifs et avec remise d'une boule dans l'urne. Quelle
>-
a. est la probabilité d'obtenir 2 blanches, 1 noire et 2 vertes? Quelle est la probabi-
0
u lité d'obtenir 2 blanches. 1 noire et 2 vertes dans cet ordre? (on proposera deux
méthodes différentes)

217
Chapitre 9 Dénombrement, probabi Iités

---- - Test d'une maladie rare


Un laboratoire propose un test de dépistage de la maladie de la grippe porcine H5Nl.
Des études randomisées ont permis d'établir les statistiques suivantes :
• sil'animal est sain, le test est négatif dans 99.8 % des cas;
• si l'animal est malade, le test est positif dans 99.9% des cas.
On sait d'autre part qu'il y a un animal malade sur 10 000. Peut-on avoir confiance
en ce test ? Pour cela, on déterminera :
a) la probabilité que l'animal soit malade, sachant que Je test est positif;
b) la probabilité que l'animal soit sain, sachant que Je test est négatif.

---- - Elevage de canard


Un éleveur de canard élève trois races différentes: le canard de Barbarie (30%). Je ca-
nard Nantais (20%) et le canard Tvlulard (50%). Suite à divers traitements hormonaux
accélérateurs de croissance (utilisés sur plusieurs générations de volailles), certains
animaux n'ont qu'nne seule patte: 10% des canards de Barbarie, 2% des canards Nan-
tais et 25% des canards Mulard. On choisit un canard au hasard.
a) Quelle est la probabilité qu'il n'ait qu'une seule patte?
b) Sachant qu'il n'a qu'une seule patte, quelle est la probabilité que ce soit un canard
Mulard? un canard Nantais?

---- - k-uplets
Soit E un ensemble fini non vide de cardinal n.
a) Combien y-a-t-il de parties de E formées de k éléments?
b) Combien y-a-t-il de k-uplets d'éléments de E?
c) Combien y-a-t-il de k-uplets d'éléments deux à deux distincts de E?
d) Combien y-a-t-iJ de k-uplets d'éléments deux à deux distincts de E, tel que le
premier élément est le plus petit et le dernier élément est Je plus grand?
e) Combien y-a-t-il de k-uplets d'éléments de E ordonnés dans l'ordre strictement
croissant?

u
0

---- -
C:
::J
0
lfl
,-i Problèmes célèbres
0
N Résoudre les problèmes historiques suivants :
@ a) Le Prince de Toscane demande à Galilée pourquoi, en lançant trois dés, on ob-
....,
.c tient plus souvent llI1 total de 10 qu'un total de 9, alors qu'il y a dans les deux cas
Ol
·;:: exactement 6 façons d'obtenir ces résultats.
>-
a. b) Le Chevalier de Méré soutient à Pascal que les deux jeux suivants sont favorables
0
u au joueur: obtenir au moins un 6 en lançant 4 fois de suite un dé, et obtenir au
moins un double 6 en lançant 24 fois de suite 2 dés. Qu'en est-il vraiment?

218
Dénombrement, probabi Iités Chap,itre 9

---- - Poker
Un joueur de poker reçoit une "main" de 5 cartes d'un jeu de 32 cartes (sans joker).
Quelle est la probabilité que sa main contienne :
a) une seule paire?
b) deux paires?
c) un brelan?
d) un carré?
e) un full?

---- - Rangement des chaussettes


On dispose de 8 paires de chaussettes distinctes et distinguables (dans une même
paire). On prend simultanément 4 chaussettes au hasard.
a) Quelle est la probabilité d'avoir reconstitué deux paires?
b) au moins une paire?
c) exactement une paire?

---- - Expérience de Mendel


On dipose de deux populations de la plante lvlirabilis : dans la première population
les fleurs sont blanches. dans la seconde elles sont rouges. En croisant les plantes
de ces deux populations, on obtient des plantes à fleurs rouges, blanches ou roses.
Ensuite on effectue de nouveaux croisements entre deux plantes à fleurs roses : on
obtient alors à nouveau des plantes à fleurs rouges dans 25% des cas, blanches dans
25% des cas et roses dans 50% des cas.
Le gène qui détermine la couleur de la fleur a deux allèles : R ou B. Si la plante donne
des fleurs rouges c'est qu'elle a les deux gènes R. si elle donne des fleurs blanches
c'est qu'elle a les deux gènes B. si elle donne des fleurs roses c'est qu'elle a un gène
R et un gène B. Lors de la reproduction sexuée un des deux gènes est transmis au
hasard, par chaque parent, à l'enfant. A l'aide de ce mécanisme biologique, donner
une explication probabiliste de l'expérience de Mendel.

u
---- - Monty Hall
Le Monty Hall est un jeu télévisé dont la règle est la suivante. Le candidat est de-
vant trois portes: derrière se trouvent une voiture et deux chèvres. Le candidat peut
d'abord éliminer une porte sur les trois. Le présentateur ouvre alors, parmi les deux
0
portes restantes, une porte cachant une chèvre. Le candidat peut alors ouvrir la porte
C: restante ou revenir sur son premier choix. Quelle est la meilleure stratégie?

---- -
::J
0
lfl
,-i
0 Tirages avec remise
N
@ Dans une urne, on place 7 boules blanches et 3 boules noires. On tire successivement
...., avec remise quatre boules de l'urne.
.c
Ol
·;:: a) Quelle est la probabilité pour que l'on obtienne, dans cet ordre, deux noires puis
>- deux blanches?
a.
0 b) Quelle est la probabilité d'obtenir deux boules blanches exactement?
u
c) Quelle est la probabilité d'obtenir au moins une boule blanche?

21 9
Chapitre 9 Dénombrement, probabi Iités

---- - Jeu de tir


Alphonse et Bernard tirent au pistolet sur une cible suivant les règles suivantes:
• Ils tirent chacun leur tour. Le premier qui atteint la cible a gagné.
• Lorsqu'il tire, Alphonse atteint la cible avec la probabilité a (0 < a< 1) et il la rate
avec la probabilité a = 1 - a.
• Lorsqu'il tire, Be12_1ard atteint la cible avec la probabilité b (0 < b < 1) et il la rate
avec la probabilité b = 1 - b.
• Alphonse tire Je premier.
Ainsi, Alphonse (resp. Bernard) lieffectue que des tirs de rang impair (resp. pair).
On considère, pour tout entier n ~ l, les évènements A2n- l : "Alphonse gagne à
l'issue du tir numéro 2n - l ", B2n : "Bernard gagne à l'issue du tir numéro 2n".
a) Calculez. en fonction de a et b, les probabilités des évènements A1, 82 et A3. Plus
généralement, calculez IJ»(A2n-d et IJ»(B211).
b) Pour n ~ 1, on note Cn (resp. Dn) l'évènement : "Alphonse (resp. Bernard) gagne
à un tir dont le numéro est entre 1 et 2n - 1 (resp. entre 2 et 2n)." Calculer lfl>(C11)
et lfl>(Dn).
c) Calculer a= litnn-+00 1P(Cn) et 13 = limn- +00 1P(Dn). Vérifier que a+ 13 = 1.

---- - Jeu de dés

On dispose de 2 dés A et B. Le dé A a 4 faces noires et 2 faces blanches. Le dé B a 2


faces noires et 4 faces blanches. On lance une pièce de monnaie truquée telle que la
probabilité d'obtenir« pile» soit p e:JO, l [.

- Si on obtient« pile» on décide de jouer uniquement avec Je dé A;

- Si on obtient« face» on décide de jouer uniquement avec le dé B.


a) Calculer la probabilité d'obtenir noir au premier lancer de dé.
b) Calculer la probabilité d'obtenir noir aux deux premiers lancers. Les évènements
« obtenir noir au premier lancer» et « obtenir noir au second lancer» sont-ils in-
dépendants ?
c) On a obtenu noir aux n premiers coups (n e: N*). Déterminer la probabilité
d'avoir utilisé le dé A. Déterminer sa limite quand n - +oo. Interprétation?

u
0

0
C:
::J

lfl
,-i
0
N
---- - Pile ou face
On considère une suite den lancers indépendants d'une pièce truquée pour laquelle
la probabilité d'obtenir« pile» est p et la probabilité d'obtenir« face» est q 1 - p
! ).
(p e:J 0, 1 [, p i «pile» (resp. «face») sera noté en abrégé P (resp. F).
=
@ a) Soit k e: [2, nil. On note Ak l'évènement : « La séquence PF apparaît pour la pre-
...., mière fois aux lancers (k - 1) et k. » Calculer IP(Akl-
.c
Ol
·;:: b) Quelle est la probabilité de l'évènement A: « La séquence PF apparaît au moins
>-
a. une fois».
0
u c) Soit k e: [o, nil. On note Bk l'évènement : «On a obtenu exactement k fois P ». Cal-
culer IJl>(Bk).

220
Dénombrement, probabi Iités Chap,itre 9

---- - Marche aléatoire sur l


On considère une marche aléatoire sur Z. Partant de 0, on se déplace de + 1 avec
=
probabilité p €)0, l [ ou de - 1 avec probabilité q 1 - p. On effectue ainsi n € I\J sauts
indépendants.
a) Déterminer la probabilité d'avoir atteint le point x € Z après li sauts.
b) En déduire la probabilité de retour en O après n sauts.

---- - Marche aléatoire sur les sommets d'un carré


On considère une marche aléatoire sur les sommets d'un carré ABCD. Lorsqu'on est
sur un sommet, on se déplace sur un des deux sommets placés sur la même arête (i.e.
!
Bou D si on est en A), avec probabilité dans chaque cas, ou on reste sur place avec
!.
probabilité On effectue ainsi n € 1\1 déplacements indépendants, toujours selon le
même processus. On suppose qu'initialement on se trouve en A.
On pose An = « après li déplacements on se trouve sur le sommet A», et an IJl>(An). =
On définit de même Bn, C11 , D 11 , b 11 , c11 et d 11 •
a) Donner un système de relations de récurrence vérifiées par les quatre suites
réelles (anlnEN, (bnlnEN, (cnlnEN et (dnlnEN·
b) Ecrire ce système de relation à l'aide d'une relation matricielle.

1
On con,;dè,e les matdce, M = k(!
0
0

il" = 3M - J,, J =
1
1
1
1
1
1
1
1
1
1
1

-1
-1 -1
etK =
-1 ~-111
-1 -1 1
c) Vérifier que, pour tout n 2: 1 :

En déduire à l'aide de la formule du binôme que, pour tout n 2: 1 :

Tvl n -114 + -1(


=3n . 1- - 1) 1((-l)n
J+ -. - - (l)n)
- K
4 3n 4 3 3
u
0
C:
::J
d) En déduire l'expression de a 11 , b 11 , Cn et d 11 en fonction den.
0 e) Déterminer fun an, lim b11 , fun Cn et fun dn. Interprétation?
lfl n -+oo n-+oo n -+oo n - +oo
,-i

---- -
0
N
@
...., Dénombrements d'applications
.c
Ol
·;::
Soient E et F deux ensembles finis non vides de cardinal respectif n et m.
>-
a. a) Combien y-a-t-il d'applications de E dans F?
0
u b) Combien y-a-t-il d'applications injectives de E dans F?
c) Combien y-a-t-il d'applications bijectives de E dans E?

221
Chapitre 9 Dénombrement, probabi Iités

d) Combien y-a-t-il d'applications strictements croissantes de E dans F?

---- -
e) Combien y-a-t-il d'applications croissantes de E dans F?

La loi de la jungle
Les lionnes chassent des gazelles et des zèbres, pour Je lion. La probabilité pour que
les lionnes rapportent une gazelle est de 2 /3, celle pour qu'elles rapportent un zèbre
est de 1/ 3. On suppose que les compositions des repas sont indépendantes.
a) Quelle est la probabilité que le lion ait mangé deux gazelles, lors des 2 premiers
repas? un zèbre puis deux gazelles, lors des 3 premiers repas?
b) On considère l'évènement E = «il a mangé une gazelle deux fois de suite, pour la
première fois, aux troisième et quatrième repas ». Déterminer lfl>(E).
c) Pour tout entier n::: 2 on note Yn = «à son neme repas le lion a, pour la première
fois, mangé deux gazelles consécutives ». On note un = lfl' (Yn) pour tout n::: 2, et
=
on pose u 1 O. Établir que, pour tout n::: 2, on a: Un+2 =} Un+ 1 + ~ Un.

---- -
En déduire u 11 en fonction den e: N*

Sommes et cardinaux
Soit E un ensemble fini de cardinal n. Déterminer les sommes suivantes en fonction
den:
a) I: CardA.
A€.9'(E)

b) L Card(AnB).
(A,B) €&> {E) 2

c) I: Card(AuB).
(A,B)€.9'(E)2

u
0
C:
::J
0
lfl
,-i
0
N
@
....,
.c
Ol
·;::
>-
a.
0
u

222
Dénombrement, probabi Iités Ch ap,itre 9

Du mal à démarrer ?

. . Modéliser un coloriage par un sept-uplet. 11111 Faire un arbre et supposer que les choix de porte se font de
façon équiprobable.
IIJI Modéliser un mot par un n-uplet.
ll9 Util iser l'i ndépendance des tirages.
Ill a) Modéliser un tirage simultané par une combinaison.
1111 Les tirs ne sont pas indépendants: utiliser la formu le des
b) Modéliser les tirages successifs sans remise par un arrangement,
ou utiliser la formule des probabilités totales. probabi lités composées.
c) Modéliser les tirages successifs avec remise par un n-up let, ou
util iser l'i ndépendance des tirages.
1111 Les lancers ne sont pas indépendants, mais ils le sont sachant
avec quel dé on joue.
Ill Uti liser la formule de Bayes.
ll9 Util iser l'i ndépendance des lancers.
Ill Uti liser la formule de Bayes.
111111 Compter le nombre de sauts+ 1 effectués par la particu le.
1111 Pour les dénombrements avec ordre final imposé, commen- 1111 Utiliser la formule des probabilités totales.
cer par chosir les éléments de façon non ordonnée.

Ill Dénombrer tous les cas possibles.


11111 d) Raisonner comme pour des n-uplets.
e) Si f est croissante, poser + k - 1.
g(k) = /(k)
1111 Choisir une carte c'est choisir sa hauteur et sa cou leur. 111D Pour a) et b), uti liser un arbre.
Ill Faire une analogie avec le jeu de poker.
c) Utiliser la formule des probabilités totales, puis reconnaître une
suite récurrente linéaire d'ordre 2.
11111 Dessiner un arbre pour modéliser la reproduction. 11111 Compter le nombre de parties qui ont le même card inal.

u
0
C:
::J
0
lfl
,-i
0
N
@
....,
.c
Ol
·;::
>-
a.
0
u

223
Chapitre 9 Dénombrement, probabi Iités

Corrigés des exercices

- Un coloriage peut être modélisé par un sept-uplet de


choix de couleurs (bio,phys,chi,math,info,fran,lvl).
a) Il y a 10 possibilités pour choisir la couleur de chacune des
7 matières. Il y a donc autant de coloriages possibles de
b) - On dispose de 26 lettres : il y a autant de mots de 5
lettres que de cinq-uplets d'un ensemble à 26 éléments,
donc 265 = 11881376 mots de 5 lettres ;
Cette fois, il ne peut y avoir répétition: il y a donc au-
tant de mots de 5 lettres sans répétition que d'arran-
sept-uplets d'un ensemble à 10 éléments, c'est-à-dire 107 . gements de 5 éléments d'un ensemble à 26 éléments,
b) Le nombre de coloriages de sorte que chaque matière ait doncA~ 6 = 7893600 mots de 5 lettres distinctes.
une couleur différente est le nombre de sept-uplets d'élé- On commence par choisir (sans ordre) les 5 lettres qui
ments deux à deux distincts d'un ensemble à 10 éléments, composeront le mot: on a (256) = 65 780 choix possibles.
c'est-à-dire le nombre d'arrangements de 7 éléments de Ensuite on compte le nombre de mots de 5 lettres
l'ensemble des couleurs : que l'on peut former à partir de ces 5 lettres, de sorte
qu'elles soient dans l'ordre alphabétique: il n'y en a
Afo = 10 x 9 x 8 x7x 6x 5 x 4 = 604800 qu'un!
Il y a donc au total 65 780 mots de 5 lettres dans l'ordre
c) Cette fois il y a 7 couleurs. Tl y a donc 77 = 823543 colo- alphabétique.
riages possibles et~ = 7! = 5040 coloriages de sorte que Un mot palindrome est un mot dont les lettres gardent
chaque matière ait une couleur différente des autres (cela le même ordre, qu'on le lise de gauche à droite ou de
revient en fait à permuter les couleurs). droite à gauche. Pour un mot de 5 lettres, les deux der-
d) I.:ensemble des coloriages de sorte qu'au moins deux ma- nières lettres sont les mêmes que les deux premières
tières aient la même couleur, est le complémentaire de (mais dans l'ordre inverse).
l'ensemble des coloriages de sorte chaque matière ait Pour construire un palindrome de 5 lettres, on choisit
une couleur différente des autres. Le nombre cherché est donc les deux premières lettres : 262 choix, puis la lettre
donc: du milieu (la troisième) : 26 choix.
107 -A 710 = 9395200
Au total : 263 = 17 576 palindromes de 5 lettres.
e) Pour un coloriage ayant exactement deux matières de la
même couleur, il faut : 811
• choisir deux matières parmi 7 (sans ordre car elles seront a) Ici, on ne tient pas compte de l'ordre. Un tirage simultané
coloriées de la même couleur) : il y a
sibles,
m = 21 choix pos- de 5 boules parmi 20 correspond à une combinaison de 5
boules parmi 20. Donc l'univers n est l'ensemble des com-
• ensuite choisir une couleur parmi 10 pour ces deux ma- binaisons de 5 boules parmi 20 et Card(ü) = (25°).
tières : il y a 10 choix possibles, On note A l'évènement: "le tirage donne 2 blanches, 1
• puis choisir une couleur différente parmi 9 pour chacune noire et 2 vertes". Pour que A se réalise, on am choix de
des 5 matières restantes : il y a~ choix possibles. deux boules blanches parmi 8, (i) choix d'une boule noire
Le nombre cherché est donc : parmi 7 et @ choix de deux boules vertes parmi 5. Donc
au total: m X (il X @tirages qui réalisent A.
21 x 10x9x8x7x6x5 =3175200
u Comme on est dans le cas de l'équiprobabilité, la probabi-
0
C: lité cherchée est donc:
0
::J

lfl
818 8
~(Al _ ( 2) X (7)l X (
5
2) _ 245
__
a) - Pour BCPST, il s'agit de permuter 5 lettres toutes dis-
,-i
0 tinctes : il y a 5! manières de le faire ;
- (250) - 1 938
N
@ Pour ANAGRAMME, il y a a priori 9! anagrammes mais b) Ici, on tient compte de l'ordre des tirages. Cinq tirages suc-
...., pour un mot donné, des permutations des 3 «A>• ou cessifs sans remise de 5 boules parmi 20 correspondent à
.c 9!
Ol un arrangement de 5 boules parmi 20. Donc l'univers n
ï:::: des 2 « M >> ne changent pas ce mot. Il y a donc ! x ! =
>- 3 2 est l'ensemble des arrangements de 5 boules parmi 20 et
a. 30240 anagrammes d'ANAGRAMME.
0 Card(ü) = A~ 0 .
u 71
Pour CELLULE, le problème est le même : - ·- = 420 • On note B l'évènement: "le tirage donne 2 blanches, 1
3! X 2!
anagrammes. noire et 2 vertes". Pour que B se réalise, on a A~ choix de

224
Dénombrement, probabi Iités Chap,itre 9

deux boules blanches parmi 8, ~ choix d'une boule noire lfl'B1 (82) = i79
parmi 7 et A§ choix de deux boules vertes parmi 5; puis @ ifDB1 nB2(N3)= ?a
choix des 2 tirages qui donnent une blanche, (512) choix
du tirage qui donne 1 noire et (5- ~- 1) choix des 2 tirages
ifDB1nB2nN3(V4) = ?1
qui donnent une verte (on choisit d'abord les boules puis lfl'B1 nB2nN3nV4 (V5) = 1~
les tirages). Donc au total : A~ X A~ X A§ x@ X X @tirages m On obtient: lfl'(81 n82 n N3 nV4 nVs) = - - .
49
qui réalisent B. 11628
Comme on est dans le cas de l'équiprobabilité, la probabi- c) Ici, on tient compte del' ordre des tirages. Cmq tirages suc-
lité cherchée est donc: cessifs avec remise de 5 boules parmi 20 correspondent
à un cinq-uplet de l'ensemble des 20 boules. Donc, en
2 2 5 3 2
A xA1 xA x ( } x ( } x ( } 245 numérotant les boules, l'univers n est égal à [1, 20] 5 et
lfl>(B) = 8 7 s 2 1 2 = __
Card(D) = 205.
A~o 1938
• On note D l'évènement: "le tirage donne 2 blanches, l
Remarquons qu'on obtient le même résultat que dans le noire et 2 vertes". Pour que D se réalise, on a@ choix des
cas du tirage simultané des 5 boules. Cette propriété aurait tirages qui donnent une blanche, (512) choix du tirage qui
pu être "prédite" grâce à la loi hypergéométrique, mais ce
donne une noire et (5- ~- 1) choix des tirages qui donnent
n'est pas le sujet de ce chapitre...
une verte; puis 8 choix de la première boule blanche et
• On note C l'évènement: "le tirage donne, dans cet ordre, aussi 8 choix de la seconde, 7 choix de la boule noire, 5
2 blanches, 1 noire et 2 vertes". Pour que C se réalise, on a choix de la première boule verte et aussi 5 choix de la se-
~ choix de deux boules blanches parmi 8, aj choix d'une conde (on choisit d'abord les tirages puis les boules). Donc
boule noire parmi 7 et A§ choix de deux boules vertes
parmi 5. Donc au total: A~ x A~ x A~ tirages qui réalisent
au total: @x m
x @x 82 x 7 x 52 tirages qui réalisent D.

Comme on est dans le cas de l'équiprobabilité, la probabi-


C.
lité cherchée est donc :
Comme on est dans le cas de l'équiprobabilité, la probabi-
3) 2) 8 2 X 7 X 52
lité cherchée est donc: (5) X ( X ( X 21
lfl>(D) = 2 2 2
205 200
A2 x A! x A2 49
lfl>(C) _ 8 ..7 5 _ __
- 5 - 11628 • On note E l'évènement: "le tirage donne, dans cet ordre,
A20
2 blanches, 1 noire et 2 vertes". Pour que Ese réalise, on a
Autre méthode : on utilise la formule des probabilités com- 8 choix de la première boule blanche et aussi 8 choix de la
posées. seconde, 7 choix de la boule noire, 5 choix de la première
Pour i € [1, 5], on considère les évènements : boule verte et aussi 5 choix de la seconde. Donc au total :
82 x 7 x 52 tirages qui réalisent E.
- Bi : "obtenir une blanche au tirage numéro i";
Comme on est dans le cas de l'équiprobabilité, la probabi-
- Ni : "obtenir une noire au tirage nwnéro i";
lité cherchée est donc:
- Vi : "obtenir une verte au tirage numéro i".
On a C = B1 n B2 n N3 n V4 n Vs. On cherche donc à calculer 82 X 7 X 52 7
1?(81 n 82 n N3 n V4 n V5). D'après la formule des probabili- lfl'(D) = 205 = -20_0_0
'O tés composées, cette probabilité est égale à :
0 Autre méthode : on utilise l'mdépendance (mutuelle) des
C:
::J résultats de chaque tirage.
0
lfl lfl>(81) x lfl'B1(82) x lfl'B1 nB2 (N3) x lfl'B1 nB2nN3 (V4) En reprenant les notations du b), on a: E = 8 1 n 82 n N3 n
,--i
0 V4 n V5. Les évènements 81, B2, N3, V4 et Vs étant (mutuel-
N x lfl'B1 nB2nN3nV4 (V5) lement) indépendants (comme conséquence de l'indépen-
@ dance des résultats des tirages), on a:
...., Comme à chaque tirage, on est dans le cas de l'équipro-
.c
Ol babilité, la probabilité d'obtenir une couleur donnée est ifD(E) ifD(81) x ifD(B2) x ifD(N3) x ifD(V4) x ifD(V5)
·;::
>- égale au nombre de boules de cette couleur, divisé par le
0
u
a. nombre total de boules dans l'urne au moment du tirage.
Donc :
csQr ;Q (:Qr X X

7
- lfl'(B1) = fo 2000

225
Chapitre 9 Dénombrement, probabi Iités

-
O n peut modéliser les données par l'arbre suivant en notant :
M l'évènement : «l'animal est malade» ;
P l'évènement : «le test est positif>>.
B l'évènement : « l'animal est un canard de Barbarie» ;
N l'évènement : «l'animal est un canard Nantais» ;
M l'évènement : « l'animal est un canard Mulard» ;
D l'évènement : « l'animal a une seule patte»;

M----
999/1000 p 1/10 D

----- 1/1000 p

.Œ-~~~N----
3/10 ~ 9/10
-D

-----
-----
499/500 p
l/500

a) La probabilité que l'animal soit malade sachant que le test


p

----- 1/5
1150

49/50
D

-----
est positif est l?p (M). 1/2
~D
D'après la formule de Bayes, appliquée avec le système
complet d'évènements (M, M), on a:
3/4 D
IJDM(P)i?(M) a) (B,N,M) est un système complet d'évènements. On a donc,
IJDp(M)
l?M (P)l?(M) + l?M(P)l?(M) d'après la formule des probabilités totales:
999 1 IJD(D) = IJD(Bn D) +IJD(N nD) +IJD(M n D)
1000 X 10000
999 1 1 9999 =IJD(B) x IJD5(D)+IJD(N) x IJDN(D) +IJD(M) x IJDM(D)
1000 X 10000 + 500 X 10000
lll 3 1 1 1 1 l
=- x- +- x- +- x-
2333 10 10 5 50 2 4
4,76% 159
1000
b) La probabilité que l'animal soit sain sachant que le test est b) Sachant qu'il n'a qu'une seule patte, la probabilité que ce
négatif est IJDp(M). soit
D'après la formule de Bayes, appliquée avec le système - un canard Mulard est IJDo(M);
complet d'évènements (M,M), on a: - un canard Nantais est l?o(N).
D'après la formule de Bayes, appliquée avec le système
complet d'évènements (M,N,B), on a:
IJDM(P)IJD(M) + IJDM (P)i?(M) l?M (D)l?(M)
499 9999 i?o(M)
500 X 10000 l?M(D)l?(M) + IJDN(D)l?(N) + l?B(D)l?(B)
= 499 9999 1 1 lxl
500 X 10000 + 1000 X 10000 4 2
9979002 1 1 1 1 1 3
4 2+ 50 X 5 + 10 10
u
= 9979003 125
X X

0 99,99%
C: 159
::J
0 "" 78,62%
Si le test est positif, la probabilité que l'animal soit vraiment
lfl
,-i malade est très faible. Avec ce test, les animaux déclarés Demême:
0
N malades sont en majorité sains, on ne peut donc pas avoir
@ confiance en ce test pour déterminer si un animal est malade. i?o(N) = IJDM(D)l?(NI) + i?N(D)IJD(N) + IJDB(D)l?(B)
....,
.c Par contre, si un animal est déclaré sain, on peut être sûr 1 1
Ol 50 X 5
ï:::: qu'il l'est avec une probabilité de 99,99%. On peut donc avoir = 1 1 1 1 1 3

-
>-
a. confiance en ce test pour déterminer si un animal est sain. 4x 2+ 50 x 5 + 10 x JO
0 4
u
159
O n peut modéliser les données par l'arbre suivant en notant : 2,52%

226
Dénombrement, probabi Iités Ch ap,itre 9

-
a) D'après le cours, il y am parties de E formées de k élé-
ments.
b) D'après le cours, iJ y a nk k-uplets de E.
Listing 9.2 - Effectifs des différentes sommes obtenues
def effectif(l) :
sortie=[]
for k in range(3, 19)
compteur=O
for i in l :
c) D'après le cours, il y a A~= (n~!k)! k-uplets d'éléments de
if i ==k :
E deux à deux distincts. compteur+=1
d) On commence par choisir les k éléments deux à deux dis- sortie+=[compteur]
tincts, sans tenir compte de l'ordre des choix: (i) choix return sortie
possibles.
sortie=effectif(liste)
Ensuite on choisit le plus petit de ces éléments: 1 seul
choix possible (car les k éléments sont 2 à 2 distincts), et n=3
on le place en première position du k-uplet. Puis on choisit for x in sortie :
le plus grand de ces éléments : 1 seul choix possible, et on print n,x
le place en dernière position du k-uplet. Dernière étape: 11+=1
on place les k-2 éléments restants dans les k-2 positions 3 457
restantes du k-uplet: (k - 2)! choix possibles. 4 1386
Donc au total, il y a m x (k- 2)! = k(k-{{in-k)! k-uplets
d'éléments de E deux à deux distincts, tels que le premier
5 2840
6 4622
7 6971
élément est le plus petit et le dernier le plus grand. 8 9756
e) On commence là aussi par choisir les k éléments deux à 9 11578
deux distincts, sans tenir compte de l'ordre des choix: (~) 10 12438
choix possibles. 11 12496
12 11526
Ensuite on les met dans l'ordre strictement croissant: 1 13 9686
seule possibilité. 14 7063
15 4587
Donc au total, iJ y am X l = m k-uplets d'éléments de E 16 2778

-
dans l'ordre strictement croissant. 17 1342
18 474

a) Simulons à l'aide de Python le lancement de trois dés. On observe que« 10» sort plus que« 9» (12438>11578). Et
expérimentalement, la fréquence d'obtenir « 10 » serait de
La commande randrange (a, b) du module random
renvoie un entier aléatoirement choisi dans [ a, b - 1Il.
/Jo1lo = 0, 12438, et la fréquence d'obtenir « 9 » serait de
On construit une liste contenant 100000 séries de la somme /~SJ!o = 0, 11578.
des chiffres obtenus lors du lancer de trois dés. Pour cela on Pourtant, il y a six façons d'obtenir« 9 » :
utilise avec une boucle for. {6,2, 1}
{5,3, 1}
Listing 9.1 - Expérience du Duc de Toscane
{5,2, 2}
import random as rd
"O
0 {4,4, l}
C:
::J liste= [] {4,3,2}
0 for k in range(lOOOOO) :
lfl a=rd.randrange(l,7) {3,3,3}
,--i
0 b=rd.randrange(l,7) et six façons d'obtenir « 10 » :
N c=rd.randrange(l,7)
@ liste+=[a+b+c] {6,3,l}
...., {6,2,2}
.c
Ol On crée ensuite une fonctioneffectif ( l) qui dresse la
·;:: {5,4, l}
>- liste des effectifs de chacune des valeurs prises, dans une liste
a. {5,3,2}
0 l d'entiers de [3, 18Il.
u {4,4,2}
On applique cette fonction à liste, puis on affiche le ré-
sultat de manière« esthétique». {4,3,3}

227
Chapitre 9 Dénombrement, probabi Iités

Pour modéliser la situation, numérotons les dés. Un lancer Par indépendance des résultats obtenus à chacun des 4
correspond donc à un triplet de chiffres entre 1 et 6 et l'uni-
vers n est : n = Œl,6n2.
lancers, la probabilité de « n'obtenir aucun six » est ( i J4,
et donc la probabilité« d'obtenir au moins un six» est :
Au total, il y a donc 63 lancers possibles.
Tl y a 3! triplets donnant les chiffres {6,2, J}, et autant pour 1 - (~r :::::0.5177
les chiffres {5,3, 1} et {4,3,2}. Pour {5,2,2), le chiffre 2 appa-
f!
raît deux fois : il y a donc triplets donnant les chiffres
(ce qui correspond à notre simulation).
{5,2,2).et autant pour les chiffres {4,4, I}. Enfin il y a un
unique triplet donnat les chiffres {3,3,3). Le premier jeu du Chevalier de Méré est donc favorable au
joueur.
=
Au total, on a a donc 6 + 6 + 6 + 3 + 3 + 1 25 triplets don-
nant un total de« 9 ». Comme tous les triplets sont équipro-
bables, la probabilité d'obtenir un total« 9» est~ ::::: 0, 1157 Adaptons la méthode précédente pour le deuxième jeu :
(remarquons qu'on retrouve à peu près la valeur expéri-
mentale). Listing 9.5 - deuxième jeu du Chevalier de Méré
Un calcul similaire permet d'obtenir que la probabilité import random as rd

6 6~
=
d'obtenir un total de« 10 » est 5 +5 +5 +3 +3 +3 rl : : : 0, 125
compteur=O
(remarquons encore une fois qu'on retrouve à peu près la
valeur expérimentale). for k in range(lOOOO)
Comme 27 > 25, on a donc bien démontré qu'on obtient lancers=[]
for i in range(24) :
plus souvent un total de« 10 » qu'un total de« 9 >>.
a =rd.randrange(l,7)
b) On créé 10000 simulations de 4 lancers de dés. Si on trouve b =rd.randrange(l,7)
au moins un 6 dans cette série on ajoute 1 à notre compteur, lancers+= [a+b]
sinon rien. if 12 in lancers :
compteur+= 1
Listing 9.3 - premier jeu du Chevalier de Méré
On obtient le résultat suivant :
import random as rd

compteur=O
Listing 9.6 - nombre de double six dans 10000 séries de
24 lancers
for k in range(lOOOO) print c~pteu,
a =rd.randrange(l,7)
b=rd.randrange(l,7) _4913
c =rd.randrange(1,7) 1

d=rd.randrange(l,7)
if a==6 or b==6 or c==6 o r ~ On a donc une fréquence expérimentale de 0,4913, qui est
compteur+= 1 plus petite que la valeur d'équilibre 0, 5. Cette fois, le jeu ne
semble pas être favorable.
On obtient le résultat suivant : Pour le vérifier, calculons la probabilité de l'évènement
contraire de « obtenir au moins un double six». II s'agit de
'O
0 Listing 9.4 - Nombres de séries de 4 lancers contenant au l'évènement« n'obtenir aucun double six».
C:
::J moins un 6 sur 10000 séries
0 Par indépendance des résultats obtenus à chacun des 4
lfl print compteur lancers, la probabilité de «n'obtenir aucun double six» est
,-i
0
N . 5136 (~ J4 donc la probabilité« d'obtenir au moins un six» est :
1

@
...., On a donc une .fréquence expérimentale de 0, 5136, qui est 35 )24 ::::: 0,4914
.c 1- ( 36
Ol
·;::
plus grande que la valeur d'équilibre 0, 5. Le jeu semble donc
>- être favorable.
a.
0 Pour le vérifier, calculons la probabilité de l'évènement (ce qui correspond à notre simulation).
u
contraire de « obtenir au moins un six». Il s'agit de l'évè- Le second jeu du Chevalier de Méré est donc défavorable
nement« n'obtenir aucun six». au joueur.

228
Dénombrement, probabi Iités Chap,itre 9

- On modélise la distribution d'une main de 5 cartes Comme toutes les mains sont équiprobables on a donc:
par le choix d'une combinaison de 5 cartes parmi 32
(car l'ordre dans lequel les cartes sont distribuées n'a pas G»(B) = (~) x (~/ x (~) x (i) = 108 "" O 12
d'influence sur le jeu du joueur). I.:univers n est donc (3s2) 899 '
l'ensemble des combinaisons de 5 cartes parmi 32, et
Card(n) = (3s2).
c) On note C l'évènement: "obtenir un brelan"="obtenir
Remarquons ensuite que dans un jeu de 32 cartes, il y a 4 trois cartes de la même hauteur, et deux autres cartes de
couleurs (carreau, coeur, pique et trèfle) et 8 hauteurs dans hauteurs distinctes et d'une hauteur différente de celle des
chaque couleur (7, 8, 9, 10, valet, dame, roi, as). trois premières cartes".
Toutes les figures considérées dans cet exercice dépendent de Il faut choisir la hauteur des trois premières cartes: (~) pos-
la hauteur des 5 cartes de la main du joueur. Pour les dénom-
brements, nous allons donc commencer par choisir la hauteur
sibilités, les hauteurs des deux autres cartes: possibili-
tés (car "roi,7" donne la même main que "7,roi"), la cou-
m
des 5 cartes, puis leur couleur.
leur des trois premières cartes: (;) possibilités, la couleur
a) On note A l'évènement: "obtenir une seule de la quatrième carte : (1} possibilités, et la couleur de la
paire"="obtenir deux cartes de la même hauteur, et trois cinquième carte : (1) possibilités.
cartes de hauteurs distinctes et différentes de celle de la
paire".

X G) X G)
Au total:

Card(C) (n
= X(:) XG) X
Comme toutes les mains sont équiprobables on a donc:
(~r
w1e 2 cartes 3 hauteurs une carte
hauteur dans dans les 7 dans
parmi 8 cette restantes chaque
hauteur hauteur
Remarquons que le choix des deux cartes de la paire se fait
2
de (1) façons différentes, et pas de (1) façons, car obtenir d) On note D l'évènement : "obtenir un carré"="olbtenir
"roi de coeur, roi de trèfle" donne la même paire qu'obte- quatre cartes de la même hauteur, et une autre carte d'une
nir "roi de trèfle, roi de coeur". Il ne faut donc le compter hauteur différente de celle des quatre premières cartes".
qu'une seule fois.
Il faut choisir la hauteur des quatre premières cartes : (T}
Comme toutes les mains sont équiprobables on a donc :
possibilités, la hauteur de la cinquième carte: (i) possibili-
tés, la couleur des quatres premières cartes:(!) possibilité,
et la couleur de la cinquième carte : (1) possibilités.
Au total:
b) On note B l'évènement: "obtenir deux paires"="obtenir
deux cartes de la même hauteur, deux autres cartes aussi
Card(D) = (~) x (:) x G) x (;)

de la même hauteur mais différente de celle de la première


Comme toutes les mains sont équiprobables on a donc:
paire, et une carte d'une hauteur différente de celle des
u deux paires".
0
C: On adopte la même méthode. Il faut choisir la hauteur de
Q'.»(D) = (~) x (!) x (i) (f)
X = _l_ "°' 0 001
::J (352) 899 •
0 chacune des 2 paires: @ possibilités (car "paire de roi,
lfl paire de 7" donne la même main que "paire de 7, paire de
,-i
0 roi"), la hauteur de la cinquième carte: (~) possibilités, la e) On note E l'évènement: "obtenir un full"="obtenir trois
N cartes de la même hauteur, et deux autres cartes de la
couleur des deux cartes de la première paire : (~) possibili-
@
....,
.c tés, la couleur des deux cartes de la seconde paire :m pos-
même hauteur et d'une hauteur différente de celle des
trois premières cartes" .
Ol
ï:::: sibilités, et la couleur de la cinquième carte: (f) possibili-
Il faut choisir la hauteur des trois premières cartes: (~) pos-
>-
a. tés.
0 sibilités, la hauteur des deux autres cartes: (i) possibilités,
u Au total:
la couleur des trois premières cartes: m
possibilités, et la
Card(B) =(:)X (!f X(~) X(~) couleur des deux cartes de la paire : (~) possibilités.

229
Chapitre 9 Dénombrement, probabi Iités

Au total: c) On note C l'évènement: "obtenir exactement une paire" .

Card(E) = (~) x (!) G) (!)


x x
Pour simplifier s'intéresse ici encore à son contraire C="ne
pas obtenir de paire ou obtenir deux paires"=Au B.
Comme A et B sont incompatibles, on a:
Comme toutes les mains sont équiprobables on a donc:
- - 1 8 41
IP(C) = IP(A) + IP(B) = - + - = -
65 13 65
donc:
- 24
IP(Cl = 1 - IP(Cl = - = 0,37
65

Mil
C'est un peu comme le jeu de poker: il y a 8 «couleurs>>
Autre méthode: on a C="obtenir deux chaussettes d'une
même paire, et deux chaussettes appartenant à deux
(les 8 paires de chaussettes) et pour chacune 2 « hauteurs» paires distinctes (nécessairement différentes de la pre-
(les 2 chaussettes d'une même paire) ... I:univers n est donc mière paire)".
l'ensemble des combinaisons de 4 chaussettes parmi 16, et On choisit la première paire : (~) possibilités, les deux
Card(D) = (~6) = 1820. paires distinctes :mpossibilités, les deux chaussettes de
2
a) On note A l'évènement: "obtenir deux paires"="obtenir la paire: @possibilité, et les deux autres chaussettes: (î)
deux chaussettes de la même paire, et deux autres chaus- possibilités.
settes aussi de la même paire (nécessairement différente Au total:
de la première paire)".
On choisit les 2 paires : m
possibilités, les deux chaus-
settes de la première paire : @ possibilités, et les deux
chaussettes de la seconde paire: @possibilités. Comme toutes les situations sont équiprobables on a

(~r
donc:
Au total:

Card(A) = (:) x

Comme toutes les situations sont équiprobables on a


donc:
@811• On peut modéliser les données par l'arbre suivant
ennotant :
(8) X (2)2 1 R1 l'évènement : « le gène transmis par le premier parent
IP(A)= 2 2
= - :::;0015
(146) 65 . de la plante a l'allèle R » ;
B1 l'évènement : « le gène transmis par le premier parent
b) On note B l'évènement: "obtenir au moins une paire". de la plante a l'allèle B » ;
Pour simplifier on va d'abord s'intéresser à son contraire
R2 l'évènement : « Je gène transmis par le second parent
B="ne pas obtenir de paire"="quatre chaussettes apparte-
de la plante al' allèle R » ;
nant à quatre paires différentes".
B2 l'évènement : « le gène transmis par le second parent
On choisit les 4 paires : (:) possibilités, puis une chaussette de la plante al' allèle B ».
4
u par paire : (i) possibilités.

----
0 1/2 R2
C:
Au total:

ur
::J
0 R1----
lfl
,-i
0
Card(B) = (!) x = 1120 1/2 B2
N
@ donc:

----
...., 1/2 R2
.c Card(B) = Card(n) - Card(B) = 1820 - 1120 = 700
Ol B1----
ï::::
>-
a. 1/2 B2
0 Comme toutes les situations sont équiprobables on a
u donc: Si on note Al' évènement : « les fleurs de la plante sont roses»,
1P B) = Card(B) = 2_ "" 0 38 alors A = {R1 B2, B1 R2l et, d'après la formule des probabilités
( Card(.û) 13 '

230
Dénombrement, probabi Iités Chap,itre 9

totales avec le système complet d'évènements (R1 ,B1), et l'in- if porte_choi=='Voiture'


dépendance des gènes transmis par chacun des parents : porte_rest = 'Chevre'
else:
1 1 1 1 1 porte_rest= 'Voiture'
IP(A) = - X - +- X - =- # int(False) == 0 et int(True) == 1 en Python
2 2 2 2 2
vic_sans_chg+=int(porte_rest=='Voiture')
D'autre part, si on note B l'évènement: «les fleurs de la plante vic_en_chg+=int(porte_choi=='Voiture')
sont rouges», alors B = {R1 R2} et, d'après l'indépendance des return vic_sans_chg,vic_en_chg
gènes transmis par chacun des parents:

1 1 1 On obtient alors, pour 10 000 000 de simulations :


IP(B) = - X - =-
2 2 4
De même, si on note C l'évènement : « les fleurs de la plante print monty(lOOOO)
=
sont blanches », alors C {B 1 B2J et, d'après l'indépendance
des gènes transmis par chacun des parents: (6683 , 3317)

IJl>(C) =-21 X -
1
2
=-41
La deuxième option semble la bonne... il reste à le prouver !
Malheureusement, au xixème siècle, Gregor Mendel ne dis- Dressons un arbre en trois étapes (après avoir numérotées nos
posait pas encore de la théorie des probabilités (apparue deux chèvres) :
après) et il n'a pas pu apporter un cadre mathématique à ses

-
théories.
Le candidat choisit d'abord une porte de manière équipro-
O n peut avoir deux idées contradictoires : bable;
le présentateur ayant ouvert une porte, le candidat a une
chance sur deux de tomber sur la voiture ; Le présentateur, sachant où se trouve la voiture, choisit
une porte cachant une chèvre ;
Au départ, le candidat a une probabiJité de de tomber !
sur la voiture. Ensuite, s'il garde son choix initial, il a tou-
f
jours la même probabilité de gagner donc s'il change Le candidat prend ensuite la décision de garder ou de
changer son choix initial.
d'avis, la probabilité qu'il gagne est donc 1 - = ! i.
Qui a raison ?
Simulons la situation à l'aide de Python.
La première branche correspond au choix du joueur, la se-
conde au choix du présentateur, et la troisième au choix du
Listing 9.7 -simulation du j eu de Monty Hall
joueur.
import random as rd

rd . randrange(l,7) Notons A l'évènement « le joueur gagne en conservant son


# entier aléatoire entre 1 et 6 choix initial». Il n'y a que deux déroulements de l'expérience
qui réalisent A :
u def monty(n) : # 11 parties
0 - le joueur choisit la porte cachant la voiture (avec probabilité
C:
::J
vic_sans_chg=O
vic_en_chg=O
f ), le présentateur choisit la porte cachant la chèvre 1 (avec
0
portes = ['Voiture', 'Chevre', 'Chevre'] probabilité ! ), le joueur choisit de garder son choix initial
lfl
,-i
0 for k in range(n) : (avec probabilité!);
N # Le joueur prend une porte au hasard - le joueur choisit la porte cachant la voiture (avec probabilité
@
....,
porte_choi = portes[rd . randrange(0,3)] k), le présentateur choisit la porte cachant la chèvre 2 (avec
.c
Ol
# Le présentateur ouvre w1e des deux
# autres porte et c'est toujours une
probabilité ! ),
le joueur choisit de garder son choix initial
·;:: # chèvre derrière celle- ci. (avec probabilité!).
>-
a. On a donc:
# La dernière porte
0
u # contient donc w1e voiture
# si le candidat a 1 1 1 1 1
# choisit une chèvre, et inversement. IP(A) = - X - +- X -=-
3 2 3 2 3

231
Chapitre 9 Dénombrement, probabi Iités

Nous avons ici supposé que le joueur garde/change son choix


initial de manière équiprobable, ce qui peut être discuté ...
Change 1/2
iiti
Pour [l,4Il, on considère les évènements:
i E
Chèvre 1 1 - - - - l Chèvre 2 - N;= "obtenir une noire au lme tirage"
- B;= "obtenir une blanche au ième tirage"
Garde l /2 Chèvre 1
a) Le tirage se fait sans remise. Une simple application de l'in-
dépendance des résultats obtenus à chaque tirage, permet
d'obtenir que :
Change 1/2 IP(N 1W(N2)IP(B3)1P(B4)
3 3 7 7 441
- x - x - x - = --
Chèvre 2 ,___ ____. Chèvre 1 10 10 10 10 10000
b) Les tirages de deux boules exactement sont les ana-
Garde 1/2 Chèvre 2 grammes du "mot" NNBB. Ils sont au nombre de 2 2! =6. ,!'
L'évènement A="obtenir exactement deux blanches" est
donc l'union de 6 évènements incompatibles et de même
probabilité p, calculée à la première question.
On a donc:
1P 441 2646
(A)= 6 x 10000 = 10000
c) Pour B="Obtenir au moins une boule blanche",
Change 1/2 Chèvre 2 on considère son contraire B="ne pas obtenir de
blanches"="obtenir que des noires"=N1 n N2 n N3 n N4.
Par indépendance des résultats obtenus à chaque tirage :

Donc:
4
- = 1 - ( -3 ) = -
IP(B) = 1 - IP(B) 9919
-
10 10000
Change 1/2 Chèvre 1
Simulons le tirage des quatre boules pour vérifier les deux der-
niers résultats.
De même, notons B l'évènement « le joueur gagne en chan-
geant son choix initial». Il n'y a que deux déroulements de l'ex- Listing 9.8 - Tirages de 4 boules avec remise
périence qui réalisent B:
import random as rd
- le joueur choisit la porte cachant la chèvre 1 (avec probabilité
!), le présentateur choisit la porte cachant la chèvre 2 (avec def boule(n, nb) :
probabilité 1), le joueur choisit de changer son choix initial X=O
u (avec probabilité i); # compteur à zéro
0
C:
::J - le joueur choisit la porte cachant la chèvre 2 (avec probabilité for i in range(n):
0 } ), le présentateur choisit la porte cachant la chèvre 1 (avec # n simulations
lfl
,-i probabilité !), le joueur choisit de changer son choix initial
B=O
# boule blanche au départ
0
N (avec probabiJité ! ). for k in range(4) :
@ On a donc : #011 tire 4 boules
...., l 1 2 i f rd. randrange( 0, 10)<7
.c IP(B) = - x 1 + - x l = -
Ol 3 3 3 # les boules blanches sont les numéros
·;::
Il vaut donc mieux pour le candidat changer son choix initial. # de O à 6 en tirant un numéro de O à 9
>-
a. B+=l
0 Pour avoir une idée de la controverse passée et présente au-
u if B==nb:
tour de ce problème voir: # nb est le nombre de boules
http ://fr.wikipedia.org/wiki/Problème_ de_ Monty_ Hall # blanches que l'on désire

232
Dénombrement, probabi Iités Chap,itre 9

X+= l
return float(X)/n
# proportion de tirages favorables n
# en Python 2. 7 on utilise float() IP'(Cn) L (1- a)k- l (l - bl- l a
# pour que/ soit la division des flottants k=I
n- 1
Comptons les tirages contenant 2 boules blanches exactement :
= a [ (l-a)k(l-b)k
k=O
boule(l000000,2) l - (l - a) 11 (1-bf1
= a-------
1 - (1 - a)(I - b)
.l 0.264013
car (1 - a)(l - b) i 1.
Pour avoir au moins une boule blanche :
On obtient de même que, pour tout entier naturel n non
1-boule(IOOOOOO,O) nul:

. 0 . 991983
1

n
On constate que ces résultats expérimentaux sont bien en ac- E ci - al(l - bl- 1b

-
cord avec nos résultats théoriques. k=I

= -
b f k
L (1 - a) (1 - b)
k
1- b k=l
a) Dressons un arbre : 1-(1-a)n(l-b)n
: b(l - a) X -------
1- (l - a)(l - b)

c) Comme 11 - al < 1 et 11 - h l < 1 on obtient :

a
ex = lim IP'(Cn) = - - - - - -
n-+oo 1 - (1 - a)(l - b)

13 = lim IP'(Dn) = b(l - a)


n- +oo 1 - (1 - a)(l - b)
Remarquons que les tirs ne sont pas indépendants, car ils
n'ont lieu que si personne n'a gagné au tour précédent... Onvérifiequecx+R= a+b(l-a) =a+b- ab=l.
1-' 1-(1-a)(l-b) a+b- ab
D'après la formule des probabilités composées : On peut interpréter ce résultat de la façon suivante : au
bout d'un très grand nombre de tours, il y a toujours un
IP'(A1) = a des deux joueurs qui gagne.

-
IP'(B2) = IP'(B2 nA1) = IP'(A1 )IP'(B2 IA1) = (1- a)b
IP'(A3) = IP'(A3nB2 nA1) = IP'(A1)IP'(B2IA1)IP'(A3IB2 nAi) =
(1 - a)(l - b)a
a) Dressons un arbre :
"O De manière générale, la formule des probabilités compo-
0
C: sées (trop longue à écrire ici...) donne que, pour tout entier 2/3 N1
::J
0 naturel n non nul :
lfl A~
,-i IP'(A2n- 1) = (1 - a)n- l (1 - b)n- l a 1/3 811
0
N IP'(B2n) = (l-a)n(l-b)n- 1b
@
...., b) C 11 est une réunion d'évènements bien sûr deux à deux in- 1/3 N1

~
.c
Ol compatibles
ï::::
>-
a. 81 1
0 2/3
u
D'après la formule des probabilités totales avec le système
On en déduit que, pour to ut entier naturel n non nul : complet d'évènements (A,B):

233
Chapitre 9 D énombrement, probabi Iités

tion:

IP(N1) = IP(A)xlPA(N1)+1P(B)xiPB(N1) 1Jl>(N2) IP(A) x 1Jl>A(N2) +IP(B) x lfl'B(N2)


2 1 2 1
3p + 3(l- p) = 3P+3(l - p)
1
1
= 3(p+ 1)
3(p + l)
= IP(N1l

b) Complétons l'arbre précédent :


On obtient donc que IP(N 1) x 1Jl>(N2) = ~ (p 2 + 2p + 1) et
1Jl>(N1 nN2) = ~(3p+ l).
2/3 N2
Les évènement N 1 et N2 so nt donc indépendants si, et
2/3 N1~ seulement si :
3p+l = p 2 + 2p+l ~ p(p - 1) = O.
/ 113 Bl2
Ils ne sont do nc pas indépendants pour IJl> (mais ils le sont

~ pour IJl>A et IPB),

----
2/3 N2
c) On veut calculer lfl'N 1nN2 n .. ·nN,. (A). D'après la formule de
11~,1----- Bayes, appliquée avec le système complet d'évènements
11 3 Bl2 (A,B):

-----
1----
1/3 N2

De même qu'à la question précédente, les évènements N 1,


1- p 213 Bl2 N2, ..., N 11 sont indépendants pour les probabilités IJl>A et
IP5. La formule des probabilit és totales nous donne donc:

-----
Bl1----
213 Bl2
1/3

On veut calculer IP(N 1n N2). Comme précédemment, la for-


N2

=
1Jl>(N1 nN2 n -.. nNn)
IJl>(AJIPA(N1 nN2 n -.. nN11)
+IJl>(B)IJl>B(N1 nN2 n· .. nN 11 )
IP(A)IPA(N1)1Jl>A(N2) x ... x IP'A(Nnl
+IP(B)IPB(N1liPB(N2) x ... x IPB(Nn)

(~rp+ ur
mu le des probabilités totales donne:

= (l- p)

D'autre part :
Or, une fois le dé choisi, les lancers se font de manière in-
dépendante. On peut le traduire par le fait que les évène- IPA(N1 nN2 n .. · nN11)
ments N 1 et N2 sont indépendants pour les probabilités
IPA(N1)1Jl>A(N2) x ... x 1Jl>A(N 11 )
'O IPAetlJl>B,

rnr
0
C:
::J On obtient:
0
lfl
,-i
IJl>(A) x 1Jl>A(N1) x 1Jl>A(N2) On a donc:
0
N
+IJl>(B) x lfl'B(N1) x lfl'B(N2)
@
...., 2 2 1 1
.c p X - X - + (1- p) X - X -
Ol 3 3 3 3
·;::
>-
0
a. Hp +~) Étudio ns la limite en +oo de cette expressio n. Il s'agit d'une
u !
forme indéterminée car 1 1 < 1 et I i 1 < 1 donc numérateur
Calculons maintenant IP(N2), comme a la première ques- et dénominateur tendent vers O.

234
Dénombrement, probabi Iités Chap,itre 9

n
Factorisons Je dénominateur par Je nwnérateur : b) On remarque que A = LJ Ak. Or les Ak, 2 :5 k :5 n, sont
(ir p deux à deux incompatibles, donc:
k=2

(~rp+(!t(l-p) 1 +(!t· 1?·


Or i < 1 donc : n
I 1
lfl>(A) L lfl>(Ak)
k=2

L~ . ,pq
-- (p k-1- q.k- 1)
On peut interpréter ce résultat de la façon suivante: si k=2 p - q
on obtient un très grand nombre de fois des faces noires
consécutives, on peut être presque sûr qu'on joue avec Je .J!!:L (
p-q k=2
f. pk-1 _ f.
k=2
qk-1)

-
dé A.
pq ( l-pn - 1 l - qn- 1)
- -- p - q
O<p<l p- q 1- p 1- q
a) Si le premier lancer est «pile>> alors la série de lancers sera pn+l - qn+l
du type 1- - - - - -
p-q
pp ... pf
-...,.._,
k- 1 fois c) Bk est l'évènement "on obtient k fois Pet (n-k) fois F". Le
«mot»
Sinon, elle sera du type
PP·· ·P FF· ··F
-...,.._, ' - - v - '
FF··· F pp ... p F k fois n-k fois
' - - v - ' -...,.._,
j fois k-1-j fois est de probabilité pk qn- k (par indépendance des résultats
obtenus aux différents lancers). De plus Bk est l'union des
On peut considérer que Je premier cas est un cas particu- 1
k!(:i ~k)! = (i:) anagrammes de ce mot, et ils ont deux à deux
lier du second avec j O.= incompatibles et de même probabilité p. On a donc :
Soit E1· l'évènement« lancer FF · · · F PP··· P F ».
' - - v - ' -...,.._,
j fois k-1-j fois
k-2
Alors Ak = LJ Ej. Or les Ej sont deux à deux incompa-
j=O
tibles. - On modélise les n sauts par une séquence de +l
Finalement, on obtient grâce à l'indépendance des résul- et de -1. c'est-à-dire par un n-uplet de {-1, 1). On prend
tats obtenus à chaque lancer que : donc pour univers n = {-1, ll, et alors Card(ü) =zn.

a) On considère l'évènement Rx="la particule arrive en x


k- 2 après n saut". Notons d Je nombre de pas« +l >> effectués
lfl>(Ak) Llfl>(Ej) par la particule pour arriver en x; elle effectue alors n - d
j=O
pas « -1 » et on a d e: ŒO, n]. Les entiers x, n et d vérifient
k- 2 donc la relation :
"O
0
I: qi Pk-1-1 q
C: j=O
::J
d 1 + (Il - d) (-1) =X
r_ (1)j
X X
0 2
lfl pk-lq
,--i
j=O p C'est-à-dire n + x = 2d.
0
N
1-(g_)k-l - Sin et x n'ont pas la même parité alors Rx = <t> et donc:
@
...., k-1 P lfl>(Rx) O;=
.c
Ol
i,!J p q 1 - (i) sinon, d = n;x et la probabilité d'atteindre x en n pas
ï::::
>- est donc:
a.
0
u
pk p~q (1-(~r-1)
.J!!:L (Pk-l _ qk- 1)
p- q

235
Chapitre 9 Dénombrement, probabi Iités

b) 0 est un entier pair. On procède de même pour calculer bn+l = IP'(Bn+1),


Ainsi, si n est impair, la probabilité d'atteindre O en n pas en+l = IP'(Cn+1l et dn+l = IP'(Dn+1l.
est nulle. Au final, on a donc le système de relations de récurrence
Si n est pair alors n = 2k avec k e: Net: suivant, pour tout ne: 1\1:

(n)
n "
IP'(Ro) = qpïqï = (2k)
kpqk k an+I = ian +ibn+ !dn

- Résumons la situation à l'aide d'un graphe probabi-


j bn+l =kan+ ibn+ ken
en+l = ibn + ien + !dn
dn+I = kan + ken+ kdn
liste pondéré :

n
1/3 1 0 1
1 0
b) Pour ne: 1\1, on noteXn = bn et M =}
en Û 1
A

~
dn 1 0
1/3 Le système précédents'écrit alors: V ne: 1\1, Xn+ 1 = Mx Xn.

c) • Vérifions la première formule par récurrence.


1/3
i
On a: B = (J - K), donc la formule est vraie pour n = l.
Supposons la formule vraie à un rang n ~ 1 fixé. On a:
l/3c B D .:)113
sn+l B x B11

t2
1/3
= !2 (J - K) x [2 11 - 2] + (- 1) 11 2 11 - 2 K]
1/3

C Des calculs simples donnent que: ]2 = 41, K2 = 4K et JK =

u l/3
KJ =O. D'où:

8 n+l =
=
2 n- lJ - (- l)n 2 n- lK
2 (n+l)-2J+(- l)n+l 2 (n+l)-2K
a) Soit n e: N fixé. Après n déplacements, on se trouve en A,
B, Cou D, et il est évident qu'on ne peut pas se trouver à La formule est donc vraie au rang n + 1.
deux positions différentes simultanément. Autrement dit D'après le principe de récurrence, la formule est donc vraie
les évènements An, B11 , Cn et Dn forment un système com- pour tout n ~ l.
plet d'évènements.
=
• On remarque que: M !(B + 14 ). Alors, comme B et 14
La formule des probabilités totales donne donc : commutent, la formule du binôme donne que, pour tout
n~ 1:
'O 1P'A 11 (An+ 1)lll'(An) + IP'B 11 (An+ 1)IP'(Bn)
0
C:
::J
+IP'c,, (An+1llfl'(C11) + IP'o,, (An+1llfl'(Dnl
0
lfl Or l'énoncé nous donne:
,-i
0
N
IP'A 11 (A,a1)=};
@ IP'B,, (An+1) = k;
....,
.c IP'c (An+1l =0;
11
Ol
IP'o (An+1) = !-
·;::
>-
a.
11

0 On obtient donc la relation :


u
1
an+l = - (an +bn +dn)
3

236
Dénombrement, probabi Iités Chap,itre 9

Finalement, pour tout n::: 1 : c) D'après le cours, il y a ( Card(E) )! = n! applications bijec-


tives de E dans lui-même.
Mn = 3n Lt +
1 41( l - 31) J+ 41((-l)n
3 - (1)
11 3
11
)
K d) • Une application strictement croissante est m1ective.
Donc si Card(E) > Card(F), il n'existe pas d'application
d) Une rapide récurrence permet de montrer que : rln e: N, strictement croissante de E dans F.
11
Xn = M Xo.
• Si Card(E) s Card(F), se donner f : E - F stric-
tement croissante revient à se donner le n-uplet

De phIB Xo = t] puisqu'on esten A ,u dép"t.


(fO).f (2), ... ,f(n)) d'éléments de F dans l'ordre stricte-
ment croissant. Or on a vu dans l'exercice 9.6 que ces
n-uplets sont au nombre de(:~).

On obtient : Il y a donc ('::) =(g:~m) fonctions strictement croissantes


de E dans F.
1 1( 1) 1((-1)
311 + 4 l - 311 + 4 3 - 31)
11
11
e) • Remarquons que si f : [l, n] - [l, m] est croissante
41(1 - 3"
1)- 41((-1
3 )n. - 31) 11 =
alors la fonction g définie par g(k) f (k) + k - 1 est stric-
v'n:::l, Xn.=
41(l - 31)+ 41((-1) 1) tement croissante de [l, n] dans [1, n+ m - 1]: si k 1 < k2
11 3 n. - 3"
alorsf(k1) sf(k2) et k1 -1 < k2 -1 doncg(k1) < g(k2).
41 ( l - 3l11 ) - 41 (( 3- J ) n - 3"
J )
• Réciproquement si g est strictement croissante de [l, n]
Finalement : dans [1, n + m - 1], alors la fonction f définie par /(k) =
g(k) - k + 1 est croissante de de [1, n] dans [1, m]. En effet
an = 4 } 311(3 11 +2+(- lf1) on a k < k + 1 donc g(k) < g(k + 1) et comme ce sont des

\in::: 1,
bn. = 4 x\n (3 11 - (-1)
11
}
nombres entiers, on a donc g(k + 1) ::: g(k) + 1 pour tout
k e: [1,n-1]. Alors si k1 s k2, on pose p = k2 -k1 e: Net
{ Cn.= 4x\,,(3n-2+(-l)n) on a: g(k2) =g(k1 + p). On itère plusieurs fois le raisonne-
dn = 4}3,.(3n+(-l)n) ment précédent :

et tl{) = 1, ho = co = do = O.
g(k2) =g(k1 +p) ::: ~ +1
e) Pour (a 11 )n:ë!:1' on remarque que, pour tout n::: 1:
~ : : : ~+ l

an =
1(l+ 2+ (-1)
4 3 3n
11
) ~ : : : ~+ l

Comme -31 < l, on a donc : lim an = ! . ~ ::: _g..(.k.rfTf+ 1


n-+oo 4
Le même raisonnement donne que : lim bn
n.- +oo
= ~ :':: g(k1)+l
1
lim
n-+oo
Cn =n-+oo
lim dn. =-.
4 et en additionnant ces inégalités :
On peut en donner l'interprétation suivante : au bout d'un
"O
0
C:
très grand nombre de déplacements, on se trouve de façon
'--..--'
=
g(k2)::: g(k1) + 1 + 1 + .. . + 1 g(k1) + k2 - k1
::J équiprobable sur n'importe quel sommet du carré.
0 p termes

'"''
lfl
,--i
0 On en déduit alors aisément que: f (k2)::: f(k1).
N
@
=
a) D'après le cours, il y a Card(Ffard(EJ m 11 applications de
...., E dans F. • On en déduit que compter les fonctions croissantes de
.c [l, n] dans [1, m] revient à compter les fonctions stricte-
Card(F) = A m = ---2:!!L.
b) • D'après le cours• il y a A(;,'lfd(E) (m- n.)! appli-
11
Ol
·;:: ment croissantes de [1, n] dans [1; n + m -1], ce que nous
>- cations injectives de E dans F, dans le cas où Card(E) s
a. avons fait dans la question précédente.
0 Card(F).
u
• Si Card(E) > Card(F), il n'existe pas d'application injec- Il y a donc (11 +i:,.7 - 1) fonctions croissantes de E dans F
tive de E dans F. lorsque n = Card(E) et m = Card(F).

237
Chapitre 9 Dénombrement, probabi Iités

-
a) Schématisons deux repas avec des notations naturelles :

~G2
- U4

0 n ven
= IJl>(E) =
, "fie que u3
~Î-
= 31 u2 + 92 u1.
• Les évènements (Z1,G1) forment un système complet
d'évènements. La formu le des probabilités totales nous
2~G1------- donne donc que, pour tout n::: 2 :
/ 1/3 Z2 IP>(Yn+2) = 1Jl>(Z1Wz 1(Yn+2) + 1Jl>(G1W'G 1(Yn+2l

~
1/3 z1 <: 213

1/3
G2

Z2
c'est-à-dire:

De plus: IP>z 1 (Yn+2l = IJl>(Yn+l) = Un+l· En effet, compter le


Alors, par indépendance de la composition des repas : nombre de déroulement possibles de la composition des
repas, en imposant que le premier soit un zèbre et que
deux gazelles consécutives arrivent pour la première fois
aux repas n+ 1 et n+2, revient en fait à compter le nombre
De même avec trois repas : de déroulement possibles de la composition des repas, en
imposant que deux gazelles consécutives arrivent pour la
première fois aux repas n et n + 1.
D'autre part, comme (Z2,~) est aussi un système complet
d'évènements, une nouvelle applications de la form ule des
probabilités totales donne :

1Jl>(Z2)1Jl>G1 nZ2 (Yn+2) + 1Jl>(G2)1Jl>G1 nG2 (Yn+2)


'-...--'
=O
1
= 31P>G1 n Z2 (Yn+2l
Le meme raisonnement que ci-dessus donne alors que:
~G3 lfDG1nz2 (Yn+2l =lfD(Yn) = Un.
2~G2------- En conclusion, on a :
1/3 / 1/3 Z3 1 2 1 l 2
Vn:::2, Un+2 = - Un+ 1 + - x - x Un = - Un+ 1 + - x Un
1 3 3 3 3 9
~
1/3 z2<: 2/3

1/3
G3

Z3
De plus, on a vu que cette formule est vraie pour n = 1.
Donc:
1 2
Vn::: 1, Un+2 = - Un+l + - x Un
3 9
Alors. toujours grâce à l'indépendance de la composition • La suite (un)n;;,2 est donc une suite récurrente lin éaire
u
0
des repas : d'ordre 2, d'équation caractéristique : r 2 r - = O. Cette -1 i
C:
::J
équation a deux solutions réelles : {-!, j}.
0
Il existe donc (À,µ) E ~ 2 tel que :

r rnr
lfl
,-i
b) I.;évènement cherché est :
~
0
N
E = (G1 nZ2 nG3 nG4) u (Z1 nZ2 nG3 nG4) V n::: l, Un = À(- +µ
@
...., dont la probabilité est :
.c
Ol IJl>(E) = j l j j + l l j j =if.
X X X X X X
En utilisant que u1 =0 et u2 = i. on obtient que : À = ! et
·;:: - 2
>- c) • Dans les questions précédentes, on a calculé : µ- 3·
a.
0 Finalement:
u u2 = 1Jl>(G1 nG2) = !;
_
u3 = 1Jl>(Z1 nG1 nG2) = .J, ; Vn::: l, Un. - -4 ( - -1)n+1 + (2)n.+1
-
3 3

238
Dénombrement, probabi Iités Ch ap,itre 9

-
a) Remarquons que lorsque A parcourt.9>(E), alors E \ A aussi.
Ainsi:

S1 = L Card(A) = L Card(E \ A)
L
B€91'(E)
Card(AnB) L
X€91'(A)
zn- Card(AJ Card(X)

211-Card(A) L Card(X)
AE&'(E) A€&(E) X€91'(A)
211- Card(A) X Card(A) X zCard(A)- 1
Donc:
211- 1 Card(A)

S1 = L
AE&'(E)
(Card(E) - Card(A)) = n( LAE&'(E)
1) - s1 = n2n - S1 l'avant dernière égalité découlant du a).
Il reste à faire la somme pour toutes les parties A de E :
car il y a zn parties de E.
Finalement: S 1 = nzn-I.
Autre méthode : on peut aussi considérer qu'il y a (1) par- L L Card(AnB)
ties de E de cardinal k. AE&'(E) BE&(E)

AinsiS 1 = [ k(n)k = 0+ [ k(~)- L zn-l Card(A)


A€&(E)
k =O k=I
Or k(i) = n(t::D pour k ;;:: 1. 2n-l L Card(A)
A€&'(E)
AinsiS1 = n [ (n-l) = nnI:l (n~ 1) = nzn-I_ = 211-l X n X 211-l
k=l k-l j=O J li x z2(n- l)
b) Soient A une partie de E et X une partie de A.
Choisir une partie B de E telle que An B = X revient à choi- c) Il suffit de se souvenir que :
sir une partie de E \ A à laquelle on rajoute X. Il y a donc
Card(A u B) = Card(A) + Card(B) - Card(A n B).
zn-Card(A) parties B de E telles que An B = X (autant que
de parties de E \ A). Donc

S3 L Card(AuB)
(A,B)E(&'(E)) 2

= L Card(A) + Card(B) - Card(A n B)


(A,B)E(&'(E)) 2

A 2x L Card(A) - L Card(AnB)
B\X (A,B)E(&'(E)) 2 (A,B)E(&'(E)) 2

= 2x L n x 211- 1 - (n2 2 Cn-l ))


B€9i'(E)

2 x 211 x ( li x 2n-l ) - ( n2 2 (n-l))


u 3n X z2(n-l )
0
C:
Donc pour A fixée : =
::J
0
lfl
,--i
0
N
@
....,
.c
Ol
·;::
>-
a.
0
u

239
Variables aléatoires

'Tlièmes a6ordés dans {es e~ercices


- Calculs des probabilités
- Loi d'une variable aléatoire
- Lois usuelles
- Moments d'une variable aléatoire
- Transfert de loi

Points essentiefs du cours pour {a réso{ution


des e~ercices
- Loi d'une variable aléatoire
- Lois usuelles
- Moments d'une variable aléatoire
u - Règles de calcul de l'espérance
0
C:
::J - Transfert de loi : théorème de transfert
0
lfl
,-i
- Règles de calcul de la variance et de l'écart-type
0
N - Inégalité de Bienaymé-Tchebytchev
@ - approximation de la loi hypergéométrique par la loi binomiale
....,
.c
Ol
·;::
>-
a.
0
u

240
Va riables a léatoires Chapitre 10

Les méthodes à retenir

- On identifie dans l'énoncé de l'exercice une ou plusieurs variables


aléatoires dont la loi est l'une des lois usuelles
~ Exercices 10.l, 10.2, 10.10, 10.10 et 10.12
Pour déterminer la loi d'une variable -Par le calcul. On commence par déterminer à l'aide de l'énoncé
aléatoire l'ensemble X(O ) d es valeurs prises par la variable aléatoire X que
l'on étudie. On détermine ensuite les probabilités lfP(X = k) pour
tout kEX(D).
~ Exercices 10.2, 10.3, 10.4, 10.6, 10.7, 10.8, 10.9, 10.10 et 10.11

- Si on a reconnu une loi usuelle, on utilise la formule du cours.


~ Exercices 10.l et 10.13

- Si on connaît la loi de la variable aléatoire, on revient à la d éfini-


Pour déterminer l'espérance d'une tion de l'espérance.
variable aléatoire ~ Exercices 10.2, 10.3, 10.4, 10.6, 10.7, 10.8 et 10.11

- Si la variable aléatoire est de la forme f(X) , où la loi de X est


connue, on utilise le théorème de transfert.
~ Exercice 10.13

- Si on connaît la loi de la variable aléatoire, on utilise simplement


Pour déterminer les moments d'une le théorème de transfert.
variable aléatoire ~ Exercices 10.l, 10.2 et 10.3

- On utilise la formule de Koenig-Huygens: on doit d'abord déter-


miner l'espérance de X et son moment d'ordre 2.
~ Exercices 10.2, 10.3 et 10.8
Pour déterminer la variance d'une
variable aléatoire - On calcule IE(X(X - 1)). La linéarité de l'espérance donne alors:
V(X) = IE(X(X - 1)) + IE(X) - (IE(X) )2.
~ Exercice 10.3

- Si e> 0, on peut majorer IJl>(IX - IE(X)I > e) grâce à l'inégalité de


Bienaymé-Tchebychev.
"O
0 ~ Exercice 10.5
C: Pour faire des calculs approchés
::J
0 - Si N > 10n, Jé'(N, n, p) peut-être approximer par ~(n, p).
lfl
,-i ~ Exercice 10.5
0
N
@
....,
.c
Ol
·;::
>-
a.
0
u

241
Chapitre 10 Variables a léatoires

Énoncés des exercices

---- - Lois usuelles


Dans chacune des expériences aléatoires ci-dessous, reconnaître une loi usuelle pour
la variable aléatoire X. Donner alors son espérance.
a) On choisit une carte au hasard dans un jeu de 32 cartes numérotées. X= "hauteur
de la carte choisie"
b) Un lion mange à chaque repas une gazelle (avec probabilité 2/3) ou un zèbre
(avec probabilité 1/3). On suppose que la composition d'un repas est indépen-
dante de celle des autres. X= "nombre de zèbres mangés sur 10 repas consécu-
tifs".
c) Dans un enclos se trouvent 15 gazelles et 30 zèbres. Un lion mange à chaque re-
pas une gazelle ou un zèbre, pris parmi les animaux de l'enclos. X = "nombre de
zèbres mangés sur 10 repas consécutifs".
d) On tire une à une les cartes d'un jeu de 32 cartes, jusqu'à obtenir le valet de pique.
X = "nombre de tirages effectués".
e) On lance un dé à 6 faces. X= "chiffre obtenu".
f) Bruno a dans sa poche 7 allumettes et 12 pièces de monnaie. Il met sa main dans
se poche et en sort 5 objets. X= "nombre de pièces obtenues".
g) A l'oral de mathématiques du concours Agro-Véto, 3000 élèves passent une
épreuve de mathématiques. Chaque élève réussit son oral avec probabilité 2/3.
X= "nombre d'élèves qui ont réussi leur oral".
h) Bruno dort chez son amie Justine qui lui a prêté son appartement. Il dispose d'un
trousseau de 7 clés, et ne sait pas laquelle ouvre la porte de l'appartement. Il les
essaye une par une, en mettant de côté chaque clé essayée. X = "nombre d'essais
nécessaires pour trouver la bonne clé".

u
0
---- - Le dé truqué
On considère 6 dés, cinq étant équilibrés. Le dernier est pipé de manière à ce que
lorsque l'on lance ce dé, chacun des chiffres apparal't avec une probabilité propor-
tionnelle à ce chiffre.
C: a) Donner la loi, l'espérance et la variance de la variable aléatoire égale au chiffre
::J
0 donné par le dé truqué lorsqu'on le lance.
lfl
,-i On effectue n choix successifs et indépendants d'un dé parmi les six.
0
N b) Quelle est la loi suivie par la variable aléatoire égale au nombre de fois où on a
@ tiré le dé truqué? Combien de tirages doit-on effectuer pour que la probabilité
...., d'avoir obtenu le dé truqué parmi ceux tirés soit supérieure ou égale à l /2?
.c
Ol
·;:: On effectue n tirages (n :S 6) successifs sans remise d'un dé parmi les six.
>-
a. c) Quelle est la loi suivie par la variable aléatoire égale au nombre de fois où est tiré
0
u le dé truqué? Combien de tirages doit-on effectuer pour que la probabilité d'avoir
obtenu le dé truqué parmi ceux tirés soit supérieure ou égale à 1/2?

242
Variables aléatoires Chapitre 10

---- - Lancer de jetons


Soit n un entier naturel non nul. Une boîte contient 2n + 1 jetons bicolores (une face
est blanche, l'autre est noire) et non truqués. Les jetons sont numérotés de 1 à 2n + 1
sur leur face blanche, les faces noires ne portant pas de numéro.On lance simultané-
ment tous les jetons et on observe leur face supérieure. Une et une seulement des
deux couleurs apparaît un nombre impair de fois. Soit X la variable associée à ce
nombre.
a) Déterminer la loi de X.
n n
b) On rappelle que: I: (~i) = 22 n-l et I: (~i=D = 22 n-2 (voir l'exercice 1.14). Cal-
k=O k=l
culer l'espérance et la variance de X.

---- - Un troisième problème historique


Deux joueurs jouent à un jeu équitable, se déroulant en plusieurs manches. Ils ont
misé chacun 32 pistoles. Le premier qui gagne trois manches remporte la partiie et le
pot des 64 pistoles mises en jeu.
Un évènement extérieur provoque l'interruption de la partie alors qu'un des joueurs
a gagné deux manches et l'autre en a gagné une. Comment répartir de manière équi-
table le pot des 64 pistoles misées?
Pour répondre à cette question, on calculera l'espérance du gain du joueur ayant
déjà remporté deux manches, dans le cas où les joueurs reprendraient la partie après
l'interruption.

---- - Calculs approchés

a) On effectue n lancers indépendants d'une pièce équilibrée. Al' aide de l'inégalité


de Bienaymé-Tchebychev, déterminer le nombre de lancers nécessaires pour que
la moyenne empirique du nombre de "Piles" obtenus se situe dans l'intervalle
[n/2 - 0.01, n/2 + 0.01), avec une probabilité supérieure ou égale à 95%.
b) Une urne contient 10000 bouJes noires et 90000 boules blanches. On tire simul-
tanément li boules, avec li ~ 1000. Quelle est la valeur minimale de n pour que
la probabilité d'obtenir au moins une blanche soit supérieure ou égale à 0,9? (on
utilisera l'approximation de la loi hypergéométrique par la loi binomiale)
u
0
C:
::J

---- -
0
lfl
,-i
0
N Plus grand numéro
@ Une urne contient N boules numérotées de 1 à N. On effectue un tirage simultané de
....,
.c n boules (ne: Œl. Nil). On note X le plus grand des numéros obtenus.
Ol
·;:: a) Déterminer la loi de X.
>-
a. N
0
u b) On rappelle que: I: (~) = (~:
k=n
f) (voir l'exercice 1.11). Déterminer l'espérance de
X.

243
Chapitre 10 Variables a léatoires

---- - Tirages avec remise avec condition d'arrêt


Une urne contient n boules numérotées de 1 à n. On effectue des tirages avec re-
mise. On arrête les tirages dès que le dernier numéro obtenu est supérieur ou égal au
numéro obtenu lors du précédent tirage. On appelle X la variable aléatoire égale au
nombre de tirages effectués.
a) Déterminer X(ü).
b) Déterminer la probabilité des évènements IX?: 2]. [X?: 31 et [X = 2).
c) Pour tout k E X(fl) déterminer la probabilité de l'évènement (X?: k]. En déduire
la loi de X.
d) En déduire l'espérance de X, puis la limite de cette dernière lorsque n tend vers

---- -
l'infini.

Dispersion des numéros lors de tirages avec remise


Une urne contient n boules numérotées de 1 à n. On effectue trois tirages d'une
boule avec remise.
On désigne par Z la variable aléatoire égale à la différence entre le plus grand numéro
obtenu et le plus petit numéro obtenu.
a) Déterminer Z(fl).
b) Déterminer la loi de Z.
c) Déterminer l'espérance de Z.
d) Donner un polynôme P de !m(XJ tel que: P(X + 1) - P(X) =nX3 - X4 . En déduire

---- -
V(Z).

Rang du kème succès lors de tirages sans remise


Une sac den bonbons contient r bonbons roses (r e: l"\J*) et b = n - r bonbons bleus
(b E l"\J*). Un enfant tire un par un tous les bonbons et les mange au fur et à mesure.
On note X la variable aléatoire égale au rang d'apparition du premier bonbon bleu.
a) Reconnaître la loi de X dans le cas b =1.
b) Déterminer la loi de X dans le cas général.
Pour k E [l, b], on note Zk le rang d'apparition du kème bonbon bleu.

---- -
c) Déterminer la loi de Zk .

Jeu aléatoire
Une urne contient b boules blanches (be: l"\J*), n boules noires (ne: l"\J*), r b oules
rouges (r E N).
u Le joueur tire une boule. Si elle est blanche, il gagne; si elle est noire, il perd; si elle est
0
C:
::J rouge, il la met de côté et effectue un autre tirage. Dans ce cas, si la boule est blanche,
0 il gagne; si elle est noire, il perd; si elle est rouge, il la met de côté et effectue un autre
lfl
,-i tirage etc...
0 La parties' achève lorsque le joueur a gagné ou perdu. On note X,. la variable aléatoire
N
@ égale au nombre de tirages nécessaires pour qu'une partie s'achève, l'urne contenant
...., au départ r boules rouges .
.c
Ol a) Calculer les espérances IE(Xo), IE(X1) et IE(X2).
·;::
>- b) Trouver une relation entre IP(X,. = k) et IP(Xr-1 = k- l), pour r?: 1 et k?: 2. En
a.
0
u déduire une relation entre IE(X,.) et IE(Xr- 1).
c) En déduire IE(X,.) .

244
Variables aléatoires Chapitre 10

---- - Tirages sans remise avec condition d'arrêt (d'après Agro-Véto)


Une urne contient n boules. On y effectue des tirages sans remise.
a) Quelle est la probabilité d'obtenir p boules données en k tirages sans remise?
I.:urne contient p boules rouges et n - p boules blanches. On note T la variable aléa-
toire égale au nombre de tirages nécessaires à l'obtention des p boules rouges.
b) Déterminer T(ü) puis lfl>(T ~ k) pour k e: T(ü). Vérifier que cette probabilités'ex-
• C • d (!)
pnme en ,onction e (;).

c) Déterminer la loi de T.
r
d) On rappelle que, pour tout r e: N, pour tout q e: [O, ril: [ (~) = (~!D (voir l'exer-
k=q

---- -
cice 1.11). Calculer IE(T).

Entropie de la loi d'une variable aléatoire discrète

On considère la fonction f définie sur llll+ par f (x) = { - xln (x) si x > O
0 si X= 0
Soit n un univers fini et X une variable aléatoire définie sur n et à valeurs dans llll+.
On appelle entropie de la loi de X le réel, noté H(X), déifni par:

H(X) = L f (lfl>(X = X))


XE:X(fl)

On pose aussi N =Card (X(ü)).


a) Déterminer H(X) lorsque X suit la loi uniforme. Même question lorsque X suit la
loi certaine.
b) Dans le cas général, déterminer le signe de H(X).
c) Etudier le signe sur llll+ de la fonction h(x) = f (x) - l + x. En déduire le signe du
réel: [ f(N x lfl>(X = x)), puis l'inégalité : H(X) ~ lnN.
xE:X(fl)
d) Etablir quel' entropie est minimale si et seulement si X suit la loi certaine.

---- -
e) Etablir que l'entropie est maximale si et seulement si X suit la loi uniforme.

Fonction génératrice d'une variable aléatoire discrète


Soit X une variable aléatoire sur un univers n supposé fini, et à valeurs dans N. On
appelle fonction génératrice de X la fonction Gx: llll - llll définie par:

u
0
'v'te:llll, Gx(t)= L lfl>(X = k)rk
keX(fl)
C:
::J
0
a) Pour tout te: llll, exprimer IE(tx) en fonction de Gx.
lfl
,-i
0 b) Pour tout ne: N*, exprimer IE(X(X - 1) ... (X - n + 2) (X - n + 1)) en fonction de Gx
N ou de ses dérivées.
@
...., c) Montrer que la fonction génératrice de X caractérise la loi de X.
.c
Ol d) Relier IE(X) et Var(X) à Gx et G~.
·;::
>-
a. e) Déterminer Gx dans les cas suivants: X<-+ flJ(p), X<-+ aJ(N, p) et X<-+ OZt([l, N]).
0
u

245
Chapitre 10 Variables a léato ires

Du mal à démarrer ?

IJIII Etudier l' indépendance des répétit ions pour dist inguer loi bi- IJD a) Reconnaître une des lois usuelles.
nomiale et loi hypergéométrique. b) et c) Disti nguer le /me tirage des tirages 1 à (j - 1).

C!D a) Noter p la probabil ité d 'obtenir « 1 » avec le dé pipé.


b) Rem arquer q ue les ti rages sont indépendants. CIIIII) a) Procéder par calcul direct.
c) Remarquer que les tirages ne sont pas indépendants b) Util iser la formule des probabilités totales pour obteni r la rela-
tion. Ensu ite sommer, pour obten ir une relation sur les espérances.
IJIII a) Distinguer deux cas suivant la couleur des jetons. c) Reconnaître une suite arith mé tico-géomé t riq ue.
b) Pour l'espérance, revenir à la définition. Pour la variance, calcu-
ler IE{X(X - 1)}.
111111 a) Introdu ire la variable a léatoire égale au nombre de boules
"données" obtenues.
11!11 Uti liser un arbre pour visualiser toutes les issues possibles. b) Utiliser la question a), puis travailler la formule e n revenant aux
factoriel les.
IJID a) Considérer la variable aléatoire ~ où X est égale au c) Remarq uer que: [T :s k) = [T = k) u [T :s k - 1).
d) Reveni r à la définit ion et reconnaître des sommes télesco piques.
nombre de boules bl anches obtenues. b) Considérer la variable
aléatoire X égale a u nombre de bou les blanches obtenues. 1111D a) Faire un calcul direct.
b) Commencer par le signe de la fonction f.
111111 a) Utiliser un calcul direct.
c) ~t udier le signe de h'. Calcu ler ens uite L =
Nl?(X x) en fonc-
b) Reven ir à la défin ition. xtaX(fll
tion de M(X) .
IJIII a), b) et c) Remarquer que IX 2: k) = [X= k] u [X 2: k + l J. d) Utiliser la propriété : s i une somme de termes positifs est nulle,
d) Revenir à la défi nition et utilise r judicieuseme nt la formule du c'est que chaq ue terme est nu l.
binôme. e) Reprendre les calculs du c).

C!Z!I a) et b) Fai re un calcu l d irect. 11111 a) Utiliser le théorème de transfert .


c) Revenir à la défini ti on et util iser les formules :

k=O
t k2- = n(n + 1)(2n + 1) et
6
f
k=O
k 3 = ( n(n + 1)
z
)2.
b) Déterminer l'expression de la dérivée nème de Gx .
c) Uti liser la formule de Taylor pour les polynômes.
d) Reprendre les résu ltats du b) pour n = 1 et n = 2.
d) C hercher P sous la forme d' un polynôme de degré 5. e) Fai re un calcul d irect.

u
0
C:
::J
0
lfl
,-i
0
N
@
....,
.c
Ol
·;::
>-
a.
0
u

246
Variables aléatoires Chapitre 10

Corrigés des exercices

-
a) Toutes les cartes peuvent être choisies de manière équipro-
bable, donc chacune des huit hauteurs également. On en
déduit que :
- X<-+ o/L(Œl,8Il) ;
-
- IE(X) = 7 ; 1 = 4.

a) Notons p la probabilité d'obtenir« 1 » avec le dé pipé alors


la probabilité d'obtenir« 2 » est 2p et de manière générale,
- [(X) -_ 8 +2 1 -_ ~2· la probabilité d'obtenir« k » est kp.
b) On répète dix fois dans les mêmes conditions la même ex- Or il n'y a que 6 faces donc:
périence qui n'a que deux issues contraires, et ceci dema-
6 6 6(6+ 1)
nière indépendante. On en déduit que :
[ kp = 1 = p [ k = p - ,- = 21p
k=l k= l 2
- X '- BlJ (10, i) ;
- [(X)= 10 X i = \Q. =
On a donc: p ,A. Finalement, si on note X la variable
aléatoire égale au chiffre donné par le dé pipé :
c) On effectue 10 tirages successfis sans remise parmi 45 ani-
maux. La proportion de zèbres est ~g = i.
On en déduit k
que : IP(X = k) = -
21
- X'- &(45, 10, i) ; Représentons cette loi dans un tableau :
- IE(X) =10 xi =f.
k 1 2 3 4 5 6
d) Toutes les cartes peuvent être choisies de manière équipro- 1 2 3 5
lfl>(X = k) 21 21 21
4
21 21
6
21
bable. On en déduit que :
- X<-+ o/L(Œl,32]) ; On en déduit que :
_ [(X)= 322+1 = 3z3.
6 k2 13
[(X)= [ - = -
e) Les 6 sorties possibles sont équiprobables. On en déduit k= l 21 3
que :
6 k3 441
- X'-+o/L(Œl,6Il); Puis: IE(X2) = [ - = - = 21, et donc:
- [(X) _ 6+1 _ 1
- 2 - 2·
k= l 21 21

f) On effectue 5 tirages successifs sans remise parmi 19 ob- 2


13) 20
jets. La proportion de pièces est rn.
On en déduit que : V(X)=21- (
3 =g

- X._..?t-'(19,5, i~); b) On répète n fois dans les mêmes conditions et de manière


-
[(X) -_ 5 X
12 _ 60
19 - 19"
indépendante la même expérience qui n'a que deux issues
"O
0 contraires : « tirer ou ne pas tirer le dé truqué» dont l'issue

0
C:
::J
g) On répète 3000 fois dans les mêmes conditions et de ma-
nière indépendante la même expérience qui n'a que deux
favorable a pour probabilité i.
lfl issues contraires. On en déduit que : On en déduit que X<-+ BlJ ( n, l, ).
,-i
0
N
- x...... BiJ(3000, j);
On cherche ensuite à déterminer n tel que p(X;;:: 1) ;;:: f.
@ Or p(X ;;:: 1) = 1 - p(X =0) donc
...., - [(X) = 3000 X ~ = 2000.
.c
Ol 1 1
·;:: h) On effectue des tirages successifs sans remise d'une clé p(X ;,:: 1) ç - <==> p(X =O) ~ -
>- 2 2
0
u
a. parmi 7. Il n'y a qu'une seule clé sur les 7 qui ouvre la porte.
On en déduit que : Or p(X = 0) = ( 1 - i r
On doit donc résoudre l'inéquation :

247
Chapitre 10 Variables a léatoires

b) • Par définition de l'espérance, on a :

(~r ~ ~ ~ ntn(~) ~ ln(~) IE(X)


n
= L (2k + l)IP(X =2k + 1)
k=O
ln(!)
n~ -ln ()6~ =
11
(2n +
kz;O (2k+ 1) 2k + 1 22n
1) 1
On doit donc effectuer au moins 4 tirages. = 1 Ln (Zk+ l) (2n2k ++ 1)1
z2n
c) Cette fois les tirages s'effectuent sans remise. k =O

On en déduit que X<-+.?/: (6, n, à).


Cette fois encore on cherche les valeurs de n telles que
= +f.
2 n k=O
(2n + 1i( 211
2k
) (formule de factorisation)

!
IP(X ~ 1) ~ c'est-à-dire lfD(X = 0) ~ !. 2n+ 1 f.
(2n)
On a, pour n E [0,5] : = 22n k=O 2k

(à) X (~) 2n+ l 2n- l


- -2 (formule de l'énoncé)
IP(X = 0) 2211
(~)
lx
5! n!(6 - n)!
x ---
On obtient: IE(X) 2 1 . = nt
n!(5 - n)! 6! • Pour simplifier le calcul, on commence par déterminer
6- n IE(X(X - 1)) grâce au théorème de transfert:
= 6
n
et on remarque que cette formule est aussi valable pour IE(X(X - 1)) L (2k + 1)(2k + 1 - l)IP(X = 2k + 1)
n =6 (car dans ce cas X est certaine égale à 1). k=O
6-n,... l (théorème de transfert)
0r 6 ~ 2 ~3~n.
On doit donc effectuer au moins 3 tirages. 2
o+ + [,(2k+1)(2k)( n+l)
2 n k=l 2k+I
81111
a) X prend comme valeurs tous les entiers impairs entre 1 et : tl (2n+ 1)(2k)(!:)
2 11
2n + 1. On a donc:
(formule de factorisation)
= {2k + 1/ k e: [0, n]}
X(Q)
+
2 11
f.
k= l
2
(2n+ 1)(2n)( n-
2k-l
l)
Soit k E [O. n] fixé quelconque.
(formule de factorisation)
On considère les évènements Bk « on obtient exactement
2k+ 1 faces blanches» et Nk «on obtient exactement 2k+ I 2n(2n+ 1) L
z2n
.n (~n-
k= l 2k-l
l)
faces noires>>. Ces deux évènements sont incompatibles et
[X= 2k+ 1] = Bk u Nk. 2
n(Zn + l) 2 2n - 2 (formule de l'énoncé)
De plus la variable aléatoire égale au nombre de faces 2211
"O
0 blanches obtenues suit une loi binomiale ~ (2n + l, !) 2n(2n+ 1)
C:
::J (par indépendance de la face sur laquelle tombe chaque 4
0 jeton). On a donc :
lfl D'après la formule de Koenig-Huygens et la linéarité de
1P B = (2n2k+l
+ 1) (!)2k+l (!)2n+l-(2k+l) = (2n+ l) _l_
,--i
0
l'espérance:
N
( k) 2 2 2k+l z2n+l 2
@ V(X) IE(X2 ) - (IE(X))
....,
.c Le même type de raisonnement donne : IE(X(X - 1)) + IE(X) - (IE(X))
2
Ol
·;:: ""N
ir( k) =
(2n+l) 1
2k +l 22»+! · 2
>-
a. = 2n(2n + 1) + 2n + 1 - { 2n + 1 )
0 On a donc : 4 2 2
u
2 1 1 2 1 1
'vkE [O,n], IJD(X=2k+l) = 2x( n+ ) - - = ( n+ ) 2 Après simplifications: V(X) = 2 n4+ 1 .
2k+ 1 22n +l 2k+ 1 2

248
Variables aléatoires Chapitre 10

- On note A le joueur qui a gagné les deux premières On cherche donc n tel que :
manches, B le joueur qui n'en a gagné qu'une.
On imagine que la partie reprend après l'interruption, et on 1- -
1
4n
104 ~ 0, 95 = -
1 4
4n
10 ~0.05
note alors X la variable aléatoire égale au gain (en pistoles) du
joueur A. A gagne la partie dès qu'il remporte une manche; B = n~ 50000

gagne la partie dès qu'il a remporté deux manches. Résumons


la situtation à l'aide d'un arbre: Il faut donc effectuer au moins 50000 lancers.

1/2 A b) On note X la variable aléatoire égale au nombre de boules


~ 1/2 A blanches obtenus. On sait que X '-" 100000, n, .le( à).
~B~ On cherche n tel que IJl>(X ~ 1)
0, l (en effet [X~ 1) = (X= 0)).
~ 0,9 ou encore IJl>(X = O) ~

1/2 B Comme N ~ lOn avec ici N = 100000 et n ~ 10000, on peut


X ne prend que deux valeurs: {O, 64).
D'après l'arbre : n
approximer la loi hypergéométrique de X par la loi bino-
miale f,8 ( n,

1 1 1
1Jl>(X = 64) = IJl>(« A gagne») = - + - x - = -
2 2 2
3
4
Donc: IJl>(X = 0) = (~) ( ~ )
On cherche donc n tel que:
0
r r·
(R = (R

et donc :
3
IJl>(X = 0) = 1 - - = -
4
l
4
rnr ~0.1 = n1nŒ)~ln(O,l)=-ln(lO)

On a donc: - ln(lO)
n> - - -
3 1
IE(X) = - x 64 + - x O= 48 - ln(~)
4 4
La répartition équitable consisterait donc à donner 48 pistoles
On donc effectuer au moins 20 tirages.
au joueur A et 16 pistoles au joueur B.
Ce problème, connu sous le nom de « problème des partis»,
a été résolu par Blaise Pascal. Sans le savoir, cette résolution
l'a poussée à être le premier à utiliser la notion d'espérance
mathématique.

+1111
-
a) • L'universn est l'ensemble des combinaisons den boules
parmi N, donc Card(D) = (~).
Le tirage amenant les plus petits numéros possibles est ce-
lui donnant les numéros: 1, 2, . .. , n. Dans ce cas X = n.
a) On note X le nombre de «piles» obtenus. On sait que
X '-" f,ô ( n,!), J
IE(X) = et V(X) = 1i} . Le tirage amenant les plus grands numéros possibles est
celui donnant les numéros: N - n+ l,N - n+2, ... ,N. Dans
La variable aléatoire égale à la moyenne empirique du ce cas X = N.
nombre de piles obtenus est Y=~, et on a IE(Y) = -fïiE(X) =
! et V(Y) = ~ V(X) = in· Donc X va prendre comme valeurs tous les entiers entre n
et N, c'est-à-dire: X(Q) = [n, N].
'O D'après l'inégalité de Bienaymé-Tchebychev:
0
C: • Soit k e: Œn, N] fixé.
::J
0 L'évènement [X= kl correspond à l'évènement« on tiré la
lfl
,-i
boule numéro k et (n - 1) boules portant des numéros in-
0 férieurs ou égaux à k - 1 ». Donc:
N Donc poure = 0,01, on a:
@

(1) (k- 1) (k-1)


....,
.c Card([X = k)) = x =
Ol 1 n-1 n-1
·;::
>-
a.
0 d'où: Ainsi:
u
(k 1)
IJl>(X = k) = n~l
(Il)
249
Chapitre 10 Variables a léatoires

b) Ona: k
c) • Soit E [3, n+ 1n. L:évènement IX~ k) correspond à l'évè-
N nement « aux (k - 1) premiers tirages on a obtenu des nu-
[(X) = L klfl>(X = k) méros dans l'ordre strictement décroissant».
k=n
Comme iJ y a (k~l) (k-1)-uplets de [2, n + Ill dans l'ordre
N (k-1)
I: k n~i strictement décroissant, et nk-l (k-1)-uplets au total. On
k=n (,J a donc:

_l f k(k -l)
(~) k = n n- l
lfl>(X;;,: k) = (k~1)
nk- 1

et on remarque que cette formule est aussi valable pour


_l f
(~) k=n n
n(k) k=2.
• Soit k E [ 2, n + 1Il. On a : [X ~ kl = IX = kl u IX ~ k + 1) .
(formule de factorisation) Comme les évènements [X = k) et [X ~ k + 1) sont incom-

= (~) JJ:) patibles, on a donc :

IJl>(X = k) = IJl>(X ~ k) - IJl>(X ~ k + 1)

(~)(::~) Donc, si k E [2, n] :


(formule de l'énoncé)
N+l IJl>(X = k) = (k~ 1) _ (i;)
n--
n+l nk-1 11 k
(formule de factorisation)
et cette formule est aussi valable pour k = n + 1 grâce à la

-
On a donc obtenu: [(X) = n~~l). 1
convention : (11 1) = O.

d) • On a:
a) Au minimum, on effectue deux tirages.
Au maximum, le numéro obtenu est toujours inférieur au n+ 1
précédent, donc on a obtenu dans cet ordre au n premiers
[(X) I: klfl>(X= k)
k=2
tirages : n, n - l, . .. , 2, 1. Le numéro suivant étant toujours
supérieur ou égal à l, on s'arrête. Donc au maximum on
effectue n + 1 tirages.
= 't\( (k~l) _ m)
k=2 nk- 1 nk
Ainsi X prend toutes les valeurs entières entre 2 et n + 1 : n+ 1 ( n ) n+ 1 (n)
X(D) = [2, n + Ill. I: k k~1 - I:
k..l:_
k=2 nk-1 k =2 nk
b) • L'évènement [X~ 2) est donc toujours réalisé. Donc:
lfl>(X~ 2) = 1. n (11) n+l (n)
j=k- 1
I: (j + 1) -4 - I: k kk
• L:évènement [X~ 3] correspond à l'évènement« au se- j=l nl k=2 n
cond tirage on a obtenu un nombre strictement inférieur à n .('}) n CJ) n+ l (k)
u
celui obtenu au premier tirage>>. I: 1-. + I: - . - I: k,z
0 Pour deux tirages on a au total n2 possibilités (n pour j=l nJ j=l nl k=2 n
C:
::J chaque tirage). Compter le nombre de tirages qui réalisent
0 [X~ 3) revient à compter le nombre de couples (i,j) de
lfl Mais par télescopage :
,--i [1, nil 2 tels que i > j: il y en a G). Comme toutes les possi-
0
N bilités sont équiprobables :
@
....,
.c
Ol
lfl>(X~3) = (~1
n
ï::::
>- • On a: IX~ 2] = [X = 21u IX~ 3). Comme les évènements et d'après la formule du binôme:
a.
0 [X = 2) et [X~ 3) sont incompatibles, on a donc :
u
lfl>(X = 2) = lfl>(X ~ 2) - lfl>(X ~ 3) = 1 - (~1
n
.[
j=l nl
(n) (
n -CJ).= .[n . -1 )j 1nJ_l=l+
j=O .J
_.
n
( -1 )n -1
n

250
Variables aléatoires Chapitre 10

On a donc: c) Ona:

n- 1
IE(Z) L klP(Z = k)
• Comme (1 +l)n = enln(I+i) et1n(1+l) - l, on k=O
n n n- +oo n n- 1
obtient par composition de limites: O+ L klP(Z= k)
k=I
lim IE(X) = e n - l 6k(n - k)
n-+oo
= I: k=l
k
n
3

iH:i 62
n k=I
[,k2- II63 [,k3
k=I
a) Comme les tirages se font avec remise on peut tirer 3 fois
de suite le même numéro. Dans ce cas: Z = O. ~ x n(n+ 1)(2n+ 1) _ ~ (n(n+ 1) )2
n2 6 n3 2
Inversement si un tirage amène le numéro 1 et un autre le
numéro n alors: Z = n - 1.
Après simplifications :
Z prend donc toute valeur entière entre Oet n - 1 :
Z(fl) = [O, n- lil. n 2 -1
IE(Z) = - -
b) • Comme dit ci-dessus [Z = O] est l'évènement« on obtient 2n
3 fois le même numéro». n y a n numéros possibles donc:
d) • Comme P(X + 1) et P(X) ont le même terme de plus haut

IJl'(Z = 0) = Ln ( - 1)3 = 3n =-
1 degré, les termes dominants vont s'annuler lorsqu'on cal-
cule P(X + 1) - P(X).
k=l n n n2
On cherche donc P(X) sous la forme d'un polynôme de
• Soit k e: [1, n - Ill fixé. L'évènement [Z = k] correspond à degré 5: P(X) = ax5 + bX4 + cx3 + dX2 + ex+ f avec
l'union des évènements Ai=« on a obtenu la boule numéro (a, b, c, d, e,f) e: ~ 6 .
i, la boule numéro i + k et une autre boule portant un nu- Alors, après calculs:
méro entre i et i + k » pour i e: [1. n - k]. Comme ils sont 2
à 2 incompatibles, on a : P(X + 1) ax5 + (Sa+ b)X4 + (10a + 4b + c)X3
+(lOa +6b+ 3c + d)X2
n- k
IJl'(Z = k) = L IJl'(Aj) +(5a+4b+3c+2d +e)X
i=l +(a+ b+ c+ d + e + f)

De plus, si i E [l, n-kll : l'évènement A1 peut être réalisé de


Donc :
3 façons différentes. Avec 3 numéros distincts, avec 2 nu-
méros égaux au minimum, avec 2 numéros égaux au maxi- 5aX4 + (10a + 4b)X3 + (10a + 6b + 3c)X2
mum:
P(X + 1) - P(X) =
+(5a+4b+3c+2d)X+ (a+b+ c+d +e)
le premier cas donne (k - 1) x 3! possibilités (on choisit
un numéro entre i + 1 et i + k- l) puis on choisit à quel
Par unicité des coefficients d'un polynôme, (a,b,c,d,e,f)
u tirage on a obtenu chaque numéro);
0 Sa -1
C: le second donne 3 possibilités (on choisit à quel tirage
::J
0 on a obtenu le maximum); 10a+4b n
lfl
,-i le troisième donne 3 possibilités (on choisit à quel ti- est solution du système : 1Oa + 6b + 3c 0
0 rage on a obtenu le minimum).
N 5a +4b+3c+2d 0
@ Au total: Card(Ai) = 3(2(k - l) + 1 + 1) = 6k. a+b+c+d+e 0
....,
.c On a donc: · 1 b n +2 3n+2 d n 1.
Ol 0 no b tienta=- 5 , = - 4- ,c=- - 6 - , = 4 ete= 30
ï::::
>- fpeut être choisi comme on le souhaite: on prend f = O.
a. IJl'(Z = k) = [ k 6k = (n _ k) x 6k = 6k(n - k)
0 Donc:
u i=l n3 n3 n3
5
(on remarque que cette formule est fausse pour k = 0) P(X) = _ x + n+2x4 _ 3n+2x3 + .::x2 + ~X
5 4 6 4 30

251
Chapitre 10 Variables a léatoires

• Ona : Ainsi:
n- 1 b C~1l
L k211"(Z = k) ll"(X = j) = .
n- ; +i
X - n-
C-1l
k=O
n-l 6k(n-k) (on remarque que cette formule reste valable pour .i = 1).
= o+ [ k2
k=I
3
n • On trouve souvent une autre formule dans la littérature.
Soit j E [l, r + 1]:
= 63 I\nk3 - k4)
n k=l ll"(X = j)
6 n- 1 b r! .!j..,--itf(n - .i + l)!
[ P(k+ 1)-P(k) --- X X --'-----''---
3n k=l n- j +1 Jj..--itr(r - .i + l)! n!
6 b r!(n - j + 1)! b!
-
télescopage 3n (P(n) - P(l))
---X
n - j + 1 (r - j + l)!n! b!
X-

r!b! (n - j)!
6 - X -----'----

3n P(n) n! (b - l)!(r - .i + l)!


PO )=O
n- j)
( b-1
La formule de Koenig-Huygens donne alors:
2
(i)
V(Z) =IE(Z ) -
2
(IE(Z) )
2
=: 11
3 P(n) - ( : : l) c) • Le kème bonbon bleu peut apparaître au minimum au
kème tirage (on tire successivement k bonbons bleUJs).
Après simplifications :
Inversement, si on ne tire que des bonbons roses, cela ne
n4 - 1 peut arriver que r fois (tirages sans remise), donc au maxi-

,,.,, V(Z) = - -
20n2

a) On reconnaît une loi usuelle: X'- °tt([l, n]).


mum le kème bonbon bleu apparaît au (r + k)ème tirage.
Donc Zk prend toute valeur entière entre k et r + k:
Zk(ü) = [k, r + k].
• [Zk = kl correspond à l'évènement« on a o btenu que des
b) • Le premier bonbon bleu peut apparaître au premier ti- bonbons bleus lors des k premiers tirages». Si on note Y' le
rage. nombre de bonbons bleus obtenus au k premiers tirages,
Inversement, si on ne tire que des bonbons roses, cela ne alors [Zk = k) =[Y'= k] et Y''- .lf(n, k, fi). Donc:
peut arriver que r fois (tirages sans remise), donc au maxi-
mwn le premier bonbon bleu apparaît au (r + l)ème tirage.
ll"(Z = k) = ll"(Y1 = k) = (b(~) = (%)
Donc X prend toute valeur entière entre 1 et r + 1 :
X(ü)=[l,r+l] .
k m m
• [X= 1) correspond à l'évènement« on a obtenu un bon-
• Soit je: [k + l, r + k]. I.:évènement [Zk = j ) correspond à
bon bleu au premier tirage». On a donc:
1'évènement « on a obtenu le kème bonbon bleu au _;ème
p(X = 1) = l tirage et (j - k) bonbons roses ainsi que (k - 1) bonbons
• Soit j E [2, r + l]. I.:évènement [X= j] correspond à l'évè-
u bleus aux tirages 1 à (j - 1) ».
0 nement« on a obtenu un bonbon bleu au /me tirage et
C: On pose A=« obtenir un bonbon bleu _;ème tirage» et
::J que des bonbons roses aux tirages 1 à j - 1 ».
0 B=« obtenir (j - k) bonbons roses et (k - 1) bonbons bleus
lfl
On pose A=« obtenir un bonbon bleu _;ème tirage » et aux tirages 1 à (j - l) ». On a donc: [Zk = j] = An B et
,--i
0
B=« n'obtenir que des bonbons roses aux tirages 1 à j - 1 ». ll"(Zk = j) = ll"(B)il"B(A).
N On a donc: [X = .il = An B et ll"(X = j) = ll"(B)ll"a(A).
@ Immédiatement : il"B(A) = n-~+l.
Immédiatement: 11"5(A) = ~::.J!f.
...., Et si Y est la variable aléatoire égale au nombre de bon-
.c Et si Y est la variable aléatoire égale au nombre de bon-
Ol bons roses obtenus au (.i - 1) premiers tirages, alors B =
·;:: bons roses obtenus au (j - 1) premiers tirages, alors B =
>-
a. [Y= j-1] et Y'-' Jf(n,j-1, fi). Donc: [Y = j - kl et Y'- Jf ( n, j - 1, fi). Donc :
0
u
. (j~l)(g)
ll"(B) = ll"(Y = J - 1) = ( n)
J-1

252
Variables aléatoires Chapitre 10

Ainsi: Et donc
. b - k + l (j~k)(k~l)
lfl>(Zk = ]) = . x ( n ) IP(X2 = 3) =
n - J+l j- 1
b+n 2(b+ n)
(on remarque que cette formule reste valable pour j = k). l----
b+n+2 (b+ n+ l)(b+ n+ 2)
• On trouve souvent une autre formule dans la littérature.
Soit j E ll k, r + kil : Ainsi:
lfl>(Zk = j)
IE(X2) = l x IP(X2 = 1) + 2 x IP(X2 = 2) + 3 x IP(X2 = 3)
: . . ; . ._l)!(n-
b- k+ 1 X - - - - . . r!b!(j- j +_1)!
_ _...;.... _ __
n - j + 1 (j - k)!(r- j + k)!(k- l)!(b - k+ l)!n!
Après simplifications :
(j - 1)! (n - j)! r!b!
: X X -
(k- l)!(j - k)! (b- k)!(r - j + k)! n!
IE(X )
2
= n+ b+3
j-l)(n-j)
(k- n+b+l
1 b- k
(i) b) • Pour r quelconque, la partie peut s'arrêter au minimum
au premier coup, et au maximum au (r + l)ème coup (si le
- Dans tout l'exercice, on note Bi (respectivement Ni, joueur tire successivement les r boules rouges). X,. prend
Ri) l'évènement« le joueur tire une boule blanche (respec- donc toutes valeurs entières entre 1 et r + 1 :
tivement noire, rouge) au ième tirage», pour i EN.
a) • Si r = 0, Je joueur obtient au premier tirage une boule X,.(!1) =lll, r + Ill
noire ou une blanche. La partie s'arrête donc dès le pre-
mier tirage : • Les évènements R1, N1 et 81 forment un système com-
Xo(D) = {l} et donc IE(Xo) = 1. plet d'évènements. La formule des probabilités totales
nous donne donc, pour tout k E lll, r + 1] :
• Si r = l, la partie s'arrête au premier tirage ou au second
(si la joueur a tiré la boule rouge au premier coup). Donc:
IP(X,. = k) 1Jl>(R1)1PR1 (X,.= k) +IP(N1)1PN1 (X,.= k)
=
X1(D) {1,2}.
Ona [X1 = 1) = R1, donc: +1Jl>(B1)1PB1 (X,. = k)

l b+n
lfl>(X1 = 1) = l-lP(R1) = 1- = --- Supposons que k ~ 2. On a alors lfDN 1 (X,. = k) = IPB 1 (X,. =
b+ n + l b+n+l k) = 0 (car si N1 ou B1 se réalise alors la partie s'arrête au
et donc: premier coup) et:
1
IP(X1 = 2) = 1-IP(X.i = 1) = - - - IPR1 (X,. = k) = IJl>(X.,._ 1 = k- 1)
b+ n + l
Ainsi: car compter le nombre de déroulements en partant avec
b+n 1 b+n+2 r bouJes rouges, et en imposant de tirer une boule rouge
IE(X1)=lx +2x =--- au premier coup et que la partie s'arrête au kème coup, re-
b+n+l b+n + l b+n + l
vient en fait à compter Je nombre de déroulements en par-
"O • Sir= 2, la partie s'arrête au premier tirage, au deuxième
0 tant avec r - l boules rouges, et en imposant que la partie
C:
::J
ou au troisième (si la joueur a tiré successivement les deux s'arrête au (k - l)ème coup.
0 boules rouges aux deux premiers coups). Donc : X2 (.Q) =
Donc pour k E ll2, r + 1Il :
lfl
,-i
{l,2,3}.
0 On a [X2 = 1) = R1, donc: r
N
l?(Xr = k) = - - - x IP(X,._1 = k - 1)+0
b+n+r
@ 2 b+n
...., IP(X2 = 1) = l-lP(R1) = 1- = --- r
.c b+ n +2 b+n+2 ---lfDCX,--1 = k-1)
Ol b+n+r
·;::
>-
a.
De même [X2 = 2) = R1 n R2, donc d'après la formule des
0 probabilités composées : et si k = l, alors [X,. = 1) = R1 donc:
u
r b+ n
lfl>(X,. = l) = l-lfl>(Ri) = l- b+ n+ r = _b_+_n_+
_r

253
Chapitre 10 Variables a léatoires

• On en déduit que : b) • Pour obtenir toutes les boules rouges, on effectue au


minimum p tirages sans remise (on tire successivement
r+l toutes les rouges), et au maximwn on effectue n tirages
IE(X,) [ klfl'OCr = k)
(on tire une à une toutes les boules et la dernière est une
k= l
rouge).
r+l
lfl'(Xr = 1) + L
klfl'(X, = k)
Donc T prend toute valeur entière entre p et n: T(Q) =
k=2
[p,n].
b +n r+l r
= --- + L k lfl'(Xr- 1 =k-1) • Soit k e: T(û). I.:évènement IT :5 k] correspond à l'évène-
b + n + r k= 2 b + n + r
ment « on a obtenu les p boules rouges lors de k tirages
b+ n r
,.
-
j =k-1
---+
b+n+r
L (i + l)lfl'(X,._1 =i)
b+n+rj=l · ·
sans remise ». D'après la première question :

b+ n r r
n-p)
(k-p
b+n+r + b+n+r [Jlfl'(Xr- l = ))
J=l lfl'(T :5 k) = (k)
=IE<Xr-il
(n - p)! p!
r r ----- x kt(il---k)f x -
+
b+n+r j = l
L lfl'(X,._1 =j) (k - p)L(il---k)f n! p!

'--.,.-" (;)
=l
(;)
Finalement, pour tout r ~ 1:
r
IE(Xr) = IEOC,-- 1) + 1 c) • On remarque: [T :5 p) = [T = p). Donc:
b+ n+ r

c) Les résultats de la question a) nous permettent de conjec-


turer que: (~) 1
lfl'(T = p) = lfl'(T :5 p) = (;) = (;)
n + b+r + l
\;fr~ l, IE(X,.) = b
n + +r

Cette formule se vérifie alors très simplement par récur-


rence grâce à la relation du b). • Si k e: [p + l, n], on a [T :5 k ) = [T = k) u [T :5 k - 1) et par

-
incompatibilité de [T = k ) avec [T :5 k- 1) :

a) Pour simplifier supposons que les p boules données soient lfl'(T = k) lfl'(T :5 k)-lfl'(T :5 k-1)
blanches et que toutes les autres soient noires. On veut
donc calculer la probabilité d'avoir tirer les p boules
(i) (k~l)
blanches en k tirages sans remise. (;) - (;)
u Cette probabilité est nulle si k < p.
0
C: Si k ~ p on note X la variable aléatoire égale au nombre
::J
0 de boules blanches obtenues en k tirages successifs sans
d) Ona:
lfl =
remise. On veut calculer lfl'(X p), et on sait que X ' -
,--i
0
N
..1e(n, k, fi). donc :
n
@
...., (pP)(''.-p) (n-p) IE(T) L kffD(T = k)
.c k-p k-p k=p
Ol
ï::::
lfl'(X = p) = ('.D = (k)
>-
a.
0 (on remarque qu'avec les conventions habituelles sur les
u
coefficients binômiaux, cette formule est vraie aussi si k <
p).

254
Variables aléatoires Chapitre 10

Donc H(X) est une somme de nombre réels positifs et


donc : H(X) ~ O.
IE(T) c) • Remarquons tout d'abord que le fonction f est dérivable
sur im:. comme produit de fonctions dérivables. De plus :

lim f(x)
x - o+·
=xlim
- o+
- xln(x) =0 (croissances comparées)
-
j=k- 1
On a donc f (0) = 0 = lim f (x), et donc on peut dire que f
x-o+
est continue sur im+.
• On en déduit que h est elle aussi continue sur llll+ et déri-
vable sur llll: . De plus, pour tout x > 0 :

/ / 1
h (x) = f (x) + 1 = - ln(x) - x- + 1 = - ln(x)
X

P P (t)
n-1 On en dé duit que h est croissante sur [O, l] et décroissante
télescopage n)
(p + n - (n)
p
- ~ (n)
j =p p sur (1, +oo[. Elle admet donc un maximum global en x = 1.
=
Mais h(l) O. On a donc: Vx E im+. h(x) s O.
= n- -1 n-I:1 ( 1·) • Onen déduit irrunédiatement que:
(;) J= p p 'v X;?; 0, f (X) S l - X
(p~i) Et donc, pour tout x E X(n) :
énoncé n- (;) f (N x IP'(X = x)) s 1 - N x IP'(X = x)

Si p -:/, n on trouve plus simplement : En a dditionnant ces inégalités, on o btient :

[(T) = n-
n! p!(n- p)!
x---
L f(N x IJl>(X = x)) s L (1 - N x IJl>(X = x))
(p + l)!(n - p - 1)! n! X€X(O) X€X(O)
n-p = N - Nxl
n---
p+l
c'est-à-dire:

-
et on remarque que cette formule reste valable si p = n.
L J (N x IP'(X = x)) s 0
x€X(O)

a) • Si X suit la loi uniforme, alors pour tout x E X(n) : • Soit x E X(O). Si IJl>(X =x)-:/, o:
IP(X = x) = ~- f(N x lfD(X = x)) -N x IJl>(X = x) ln (N x IJl>(X = x))
On a donc :
= - Nln(N)IJl>(X = x)

I: 1( N )
1
H(X) - NIJl>(X =x)ln (IJl>(X =x))
X€X(û)
- Nln(N)IP'(X = x) + N/(IP'(X = x))

'O
= Nt(~) Si IP(X = x) = 0 alors alors on a aussi :
0 f(N x IP(X = x)) = - Nln(N)IP'(X = x) +Nf(IP'(X = x))
0
C:
::J -10(~) On en déduit que :
lfl
,--i
lnN 0 ;?; L f(NxlP'(X = x))
0 X€X(fl)
N • Si X suit la loi certaine alors X(û) ={al et IP(X =a) = 1,
@
...., pour a E llll fixé. = L (-Nln(N}IJl>(X=x)+N/(lfD(X=xl))
X€X(O)
.c On a donc:
Ol
·;::
>-
H(X) = f (1) = 0 =-Nln(N) L IJl>(X=x)+N L f(IJl>(X=xl))
a. X€X(O) X€X(O)
0 b) Pour tout x E X(n). on a: 0 s IP'(X = x) s let donc: = - Nln(N) x 1 + N x H(X)
u
/(IP'(X = x)) ~ 0 (il est immédiat que la fonction f est posi-
tive sur [O, l]). Finalement: H(X) s ln(N).

255
Chapitre 10 Variable s a léatoires

d) • On a vu que H(X) ~ 0 et que H(X) = 0 lorsque X est b) Une nouvelle application du théorème de transfert nous
constante. donne:
• Réciproquement, montrons que si H(X) = 0 alors X est
IE(X(X - 1) · · · (X - n + 2)(X - n + 1))
constante.
Comme H(X) est une somme de termes positifs, H(X) 0 = = L k(k - 1)· ·· (k - n + 2)(k - n+ l)lfD(X =k)
keX(n)
donne que chacun des termes de la somme est nulle, c'est-
à-dire : Mais la fonction polynôme Gx est C00 sur~ et en particu-
'v'xe:X(D), f(lfD(X = x)) = O lier, pour tout te:~· :
ce qui donne : Gt1l(t) = L k(k - l)· ··(k- n+2)(k - n + l)IJ»(X = k)tk-n
kEX(O)
'v'x e:X(ü), IJ»(X = x) = 0 ou lfD(X = x) = 1
On a donc:
=
Or six e: X(û). on a lfD(X x) f. 0 donc lfD(X x) 1. = = IE(X(X-1) ···(X- n+ 2)(X- n + 1)) = Gt1l (1)
Comme: LlfD(X = x) = Card (X(ü)} x 1 = 1 on a néces-
xe:X(O)
c) D'après la formule de Taylor pour les polynômes, appli-
sairement Card (X(D)) = 1.
quée à Gx en 0, on a:
Donc X est constante.
• On a donc: H(X) est minimale si et seulement si X est dkl(O)
constante. 'v' k e: X(ü), lfD(X = k) = _ x_ _
k!
e) • On a vu que H(X) s ln(N) et que H(X) = ln(N) lorsque X
On en déduit que Gx donne la loi de X.
suit une loi uniforme. I
• Réciproquement, montrons que si H(X) = ln(N) alors X d) • D'après le b), on a: IE(X) = Gx (1).
Il
suit une loi uniforme. • Toujours d'après le b) : IE(X(X - 1)) = Gx (1).
On a vu à la question c) que : Et d'après la formule de Koenig- Huygens et la linéarité de
l'espérance, o n a :
L f(N x IJ»(X = x)) = -Nln(N) + N x H(X)
XEX(O) V(X) = IE(X(X - ll)+IE(Xl - (IE(XJ) 2

Donc H(X) = ln(N) donne: L f (N x lfD(X = x)} = O. G; (1) + G~ (1) - ( G~(l) )2


XEX(O)
Comme cette somme est composée de termes positifs, on e) • Si X <.-. g&(p) alors, pour tout te:~:
peut en déduire que chacun des termes est nul : Gx (t) = lfD(X = 0) t 0 +lfD(X = l)t1 = lx (1-p)+tx p = (1-p)+ pt

'v' x e: X(!1), f(N x lfD(X = x)} = 0 • Si X'- p;&(N, p) alo rs, pour to ut te: Ili:

et donc :
Gx.Ul = f
k=O k
(~)pk(l - p)N- ktk
'v' x e: X(!1), N x IJ»(X = x) = O ou N x lfD(X = x) = 1

Mais comme: 'v' x e: X(!1), lfD(X = x) f. 0, on a donc :


f
k=O k
(N)(pt/(1- p)N- k
u
0
C: 1 (pt+ 1- p)N (d'après la formule du binôme)
::J 'v'xe:X(D), lfD(X=x)= -
0 N
lfl • Si X'- OZ?Œl, Nil) alors, pour tout te: Ili:
,-i
0 On a donc prouvé que X suit une loi uniforme.
N
• On a donc: H(X) est maximale si et seulement si X suit Gx(t) ~ 1
L, - t
k
@

-
...., une loi uniforme . k=l N
.c
Ol 1 ~ tk
·;::
>-
= - L,
N k=l
a. a) So it te:~- D'après le théorème de transfert:
0
u
IE( i-X) = L tklfD(X = k) = Gx(t)
Donc si t f. l: Gx(t) = ~ x \-_r;,
k€X(O) et Gx(l)=l.

256
CHAPITRE

Vecteurs aléatoires

'Tlièmes a6ordés dans {es e~ercices


- Calculs des probabilités
- Loi conjointe d'un vecteur aléatoire
- Transfert de loi
- Calculs d'espérance, de variance et de covariance
- Indépendance

Points essentiefs du cours pour {a réso{ution


des e~ercices
- Loi conjointe d'un vecteur aléatoire, lois marginales
- Transfert de loi : théorème de transfert
- Règles de calculs de l'espérance, de la variance
u - Covariance, coefficient de corrélation linéaire
0
C:
::J - Indépendance
0
lfl
,-i
0
N
@
....,
.c
Ol
·;::
>-
a.
0
u

257
Chapitre 11 Vecteurs a léat oires

Les méthodes à retenir

- Pour le couple (X, Y) on commence par déterminer X(O) et Y(O).


On calcul alors IJl>(X = i, Y= j) pour tout (i,j) e: X(D) x Y(D).
.____. Exercices 11.1, 11.4, 11.8, 11.9, 11.11 et 11.12

- Si on connaît la loi de X sachant Y alors on a :


Pour déterminer les lois conjointe et IJl>(X= i,Y= j) = IJl>(Y= j) X [P>Y=j(X = i) .
marginales d'un couple de variables .____. Exercices 11.4, 11.9, 11.11, 11.12, 11.13 et 11.14
aléatoires
- Pour déterminer les lois marginales de X et Y, on peut procé-
der par un calcul direct, ou utiliser la loi conjointe et la formule
des probabilités totales associée aux systèmes complets d'évène-
ments associés à X et Y .
.____. Exercices 11.1, 11.4, 11.5, 11.8 et 11.13

- Pour la variable aléatoire Z = /(X, Y) on a Z(D) = f(X(D), Y(O )) et


IJl>(Z= z)= :[ IJl>(X=i,Y=j).
(i,j)EX (fl)xY(fl) / f(i,j )=z
.____. Exercices 11.1, 11.6, 11.8, 11.15 et 11.16

- PourZ=X + Yonobtient :IJl>(Z =z)= :[ IJl>(X=i,Y=z - i).


iE:X(fl)
.____. Exercices 11.6, 11.14, 11.15 et 11.16
Pour déterminer la loi d'une variable
aléatoire fonction d'un couple de - Pour déterminer l'espérance de Z = f (X, Y) on peut soit utiliser le
variables aléatoires théorème de transfert, soit commencer par déterminer la loi de Z,
soit utiliser la linéarité de l'espérance.
.____. Exercices 11.1, 11.8 et 11.14

- Pour déterminer la variance de Z, on utilise la formule de Koenig-


Huyghens ou encore les règles de calculs de la variance.
.____. Exercices 11.8 etl 1.14

- Pour la covariance de Z = f(X, Y) on a Cov(X, Y)= [(XY) - [(X)[(Y) .


u .____. Exercices 11.2 11.4 et 11.8
0
C:
::J - Pour déterminer [ (XY) on détermine la loi de XY ou on utilise le
0
lfl
théorème de transfert.
,-i
0 Pour déterminer la covariance et le .____. Exercices 11.2, 11.4 et 11.8
N
coefficient de corrélation d'un couple
@ - Une autre méthode consiste à utiliser les régies de calculs de la
...., de variables aléatoires
.c covariance.
Ol
·;:: .____. Exercice 11.3
>-
a.
0 - Le coefficient de corrélation s'en déduit alors facilement.
u
.____. Exercice 11.4

258
Vecteu rs alé atoires Chapitre l i

- La plupart du temps c'est une hypothèse de l'énoncé.


<.......+ Exercices 11.1, 11.3, 11.7, 11.10 et 11.16

- Si on a affaire à un couple (X, Y) le calcul des lois conjointes et


Pour étudier l'indépendance de marginales permet de dire avec certitude s'ils elles sont indépen-
variables aléatoires dantes ou non.
<.......+ Exercices 11.1, 11.2 et 11.4

- Si Cov(X, Y) # 0 alors X et Y ne sont pas indépendantes.


<.......+ Exercices 11.2 et 11.4

n
- Il s'agit d'une variable aléatoire Z = L X; .
i=l
<.......+ Exercices ll.7,ll.15etll.17
Pour étudier la somme de n variables
aléatoires n.
Son espérance est donnée par linéarité: IE(Z) = L IE(Xj).
i =l
<.......+ Exercices 11.7, 11.15 etll.17

u
0
C:
::J
0
lfl
,-i
0
N
@
....,
.c
Ol
ï::::
>-
a.
0
u

2 59
Chapitre 11 Vecteurs aléatoires

Énoncés des exercices

---- - Loi du Min et du Max


Une urne contient 5 boules numérotées de 1 à 5. On en tire 2 au hasard. On note X le
plus petit numéro obtenu, et Y le plus grand numéro obtenu.
a) Déterminer la loi de (X, Y), puis la loi de X et celle de Y, dans les cas suivants: ti-
rage simultané de 2 boules, 2 tirages successifs d'une boule avec remise, 2 tirages
successifs d'une boule sans remise.
b) Dans chacun des cas, déterminer si X et Y sont indépendantes.

---- -
c) Dans chacun des cas, déterminer la loi et l'espérance de Z =Y-X.

Indépendance de deux variables aléatoires de Bernoulli


Soient X et Y deux variables aléatoires de loi de Bernoulli. Montrer qu'elles sont indé-

---- -
pendantes si et seulement si Cov(X, Y) O. =
Exemple de transfert de loi
Soient X et Y deux variables aléatoires réelles ne prenant qu'un nombre fini de va-
leurs. On note IE(X) et IE(Y) leur espérance et V(X) et V(Y) leur variance.
On pose Z =X + Y et T =X - Y.
a) Exprimer Cov(X,Y) en fonction de V(Z) et V(T).
b) Montrer que si Z et T sont indépendantes alors V(X) =V(\').
On suppose que X et Y sont indépendantes, suivant toutes deux la loi 0/L ({1,2,3}).
c) Donner la loi de probabilités de Z et de T puis calculer leur espérance et leur
variance.
d) Z et T sont-elles indépendantes ?

---- -
e) Calculer Cov(Z, T).

Un autre exemple de transfert de loi


Soit une variable aléatoire de loi 0/L(U- n, nll) (n E: N*). On pose Y = X2.
a) Déterminer la loi du couple (X, Y). En déduire la loi de Y.
b) Retrouver ce résultat par un calcul direct.
c) Calculer Cov(X, Y) puis p(X, Y) (coefficient de corrélation).

---- -
d) X et Y sont-elles indépendantes?
u
0
C:
::J Egalité de variables binomiales
0
lfl
Soient X et Y deux variables aléatoires indépendantes suivant la même loi binomiale
!).
~ (n, Déterminer lll'(X =Y).

---- -
,-i
0
N
@
...., Somme de variables aléatoires indépendantes de loi binomiale
.c
Ol
·;::
Soit X une variable aléatoire de loi ~(n1,P) et Y une variable aléatoire de loi
>- [!S(n2, p).On suppose X et Y indépendantes.
a.
0 a) Déterminer la loi de la variable aléatoire Z =X+ Y. Interpréter ce résultat.
u
b) Soit r E: llü, n 1 + n2Il. Déterminer la loi de Xsachant que Z =r.

260
Vecteu rs alé atoires Chapitre l i

---- - Nombre de séries unicolores


Une urne contient des boules blanches en proportion p et des boules noires en pro-
portion q. On procède à une suite den tirages avec remise (n ~ 3). On constitue ainsi
des séries unicolores, au gré des couleurs obtenues dans la suite des n tirages. Par
exemple, pour n = 9, la suite de résultats BBNBNNNBB fournit 5 séries unicolores :
BB, puis N, puis B, puis NNN et enfin BB.
Soit X la variable aléatoire qui compte le nombre de séries unicolores obtenues à
l'issue des n tirages.
a) Calculer ~(X = 1) et ~(X = n).
Pour i ? 2, on désigne par Y; la variable aléatoire égale à 1 si on a obtenu une boule
blanche au (i - l)ème tirage et une boule noire au ième tirage, et égale à Osinon.
Symétriquement pour j ? 2, on désigne par Z j la variable aléatoire égale à 1 si on a
obtenu une boule noire au (j - l)ème tirage et une boule blanche au _;ème tirage, et
égale à Osinon.
b) Exprimer X en fonction des Y; et des Zj· En déduire l'espérance de X.

---- - Rouge ou Bleu


Soit n e: 1\1*. Une urne contient des jetons à deux faces portant chacun, su r une
des faces un numéro bleu, et sur l'autre face un numéro rouge. On sait que, sur
l'ensemble des jetons de l'urne, k exactement portent le numéro bleu k, ceci pour
k e: [1, nil, et que, parmi les k jetons portant Je numéro bleu k, un et un seul porte Je
numéro rouge i, ceci pour i e: [l, k].
a) Déterminer, en fonction den, le nombre de jetons contenus dans l'urne.
On tire au hasard un jeton de l'urne. On désigne par B la variable aléatoire égale à
son numéro b leu, et par R la variable aléatoire égale à son numéro rouge. On pose,
d'autre part, G = 8- R.
b) Déterminer la loi du couple (B, R). En déduire les lois de B et de R, ainsi que leur
espérance et leur variance.
c) Suite au tirage d'un jeton, on gagne G. Préciser l'espérance de G et calculer la
variance de G.

u
0

0
C:
::J

lfl
---- - Jeu d'enfants
Deux enfants jouent avec un sac de n jetons numérotés de 1 à n. Le premier
enfant effectue des tirages successifs et sans remise d'un jeton dans Je sac, jusqu'à
obtention du jeton portant le plus grand numéro. On note X1 le nombre de tirages
,-i nécessaires.
0 S'il reste des jetons dans Je sac, le second enfant effectue la même expérience. On
N
@ note X2 le nombre de tirages nécessaires (nombre qui vaut éventuellement zéro si le
...., sac est vidé par Je premier enfant) .
.c
Ol
·;::
>- a) Donner la loi, l'espérance et la variance de X1.
a.
0 b) Déterminer la loi conditionneHe de X2 sachant X1, puis la loi de X2.
u
c) Déterminer l'espérance de X2.

261
Chapitre 11 Vecteurs aléatoires

---- - Système linéaire aléatoire (d'après Agro-Véto)

Soit (S) le système d'inconnues (x,y) : {


2
x- Y
ax-by
3
c
Afin de déterminer les entiers a, b etc, on réalise un lancer simultané de 3 dés équi-
librés à 6 faces : a, b et c sont égaux respectivement aux numéros amenés par Je 1er,
le zème et le 3ème dé. Déterminer la probabilité pour que (S) ait:
a) Une infinité de solutions.
b) Aucune solution.
c) Une unique solution.
d) La solution unique (3,0).
d) Mêmes questions si les dés sont pipés de telle sorte que la probabilité de sortie
d'une face soit proportionnelle au numéro porté sur cette face.

---- - Urne à composition changeante (d'après Agro-Véto)


On dispose de n jetons numérotés. On effectue une succession de tirages de la ma-
nière suivante : on tire un jeton au hasard, si son numéro est k on le remet alors dans
l'urne avec k autres jetons portant Je numéro k. On note: a 11 = .[
n
nt k . Soit X le
k=l
numéro du premier jeton sorti, Y celui du second.
a) Déterminer la loi et l'espérance de X.
b) Déterminer la loi de Y.
c) Calculer IE(Y).

---- - Justine
On dispose den pièces non équilibrées (n 2: 2). Chaque pièce peut amener Pile avec
=
probabilité p €]0, 1 [ et Face avec probabilité q 1 - p. On lance simultanément les n
pièces.
u a) Soit X la variable aléatoire égale au nombre de pièces ayant amené Pile. Détermi-
0
C:
::J ner la loi de X.
0
lfl
b) Justine a les yeux bandés et n'a pas assisté au lancer. Elle choisit au hasard k
,-i pièces parmi les n, k étant un nombre fixé à priori avec O < k < n. Elle gagne
0
N alors parmi les k pièces celles qui présentent Pile. Déterminer la loi de la variable
@ aléatoire Y égale au nombre de pièces gagnées par Justine.
....,
.c On soupçonne Justine d'avoir triché en ayant essayé de deviner au toucher les pièces
Ol
·;:: ayant amené Pile. On lui demande donc de relancer les k pièces qu'elle a choisies:
>-
a. eUe gagnera ceUes, parmi les k, qui amèneront Pile. Soit Z la variable aléatoire repré-
0 sentant ce nouveau gain.
u
c) Déterminer la loi de Z. Que peut-on en déduire?

262
Vecteu rs alé atoires Chapitre l i

---- - Succession d'expériences


On dispose d'une pièce qui donne "Pile" avec probabilité p e:JO, 1[. On la lance n fois
(ne: N*) et on note X la variable aléatoire égale au nombre de "Piles" obtenus. On
définit alors une variable aléatoire Y par :

\:/w e: n, Y(w) =J X(w) siX(w) i 0


l un nombre choisi au hasard entre 1 et n siX(w) = 0

Déterminer la loi de Y, ainsi que son espérance.

---- - Les cartons d'anniversaire (d'après Agro-Véto)


Un enfant envoie n invitations pour son anniversaire. Chaque invité lui répond in-
dépendamment des autres avec une probabilité p . Il fait alors un deuxième envoi
à ceux qui n'ont pas répondu la première fois. Encore une fois, chaque invité lui ré-
pond indépendamment des autres avec une probabilité p.
On désigne par X la variable aléatoire égale au nombre total de réponses.
a) Calculer IP'(X = 0) et IP'(X = 1).
b) Déterminer la loi de X, son espérance et sa variance.

---- - IIanimalerie
Une animalerie possède N animaux de k espèces différentes(3 $ k $ N). Pour tout
i e: U1, kil, on note ni le nombre d'animaux de l'espèce numéro i, et on pose Pi = ';J.
Un dimanche, n clients achètent un animal au hasard et de façons indépendantes.
Pour tout i e: lll, kil, on note Ni le nombre d'animaux achetés de l'espèce i. On note
aussi X le nombre d'espèces qui n'ont connu aucun achat.
a) Pour i e: lll, kil, déterminer la loi de Ni, son espérance et sa variance. Pour (i, j) e:
lll, kD2 tel que i i j, déterminer la loi de Ni + N j, son espérance et sa variance,
puis en déduire la covariance de Ni et N j.
b) Exprimer X comme une somme de variables aléatoires de Bernoulli. En déduire
IE(X) puis calculer lim IE(X). Interpréter ce résultat.
n-+oo
c) Comparer IP'(X ~ 1) et IE(X). En déduire lim IP'(X
n- +oo
=0). Interpréter ce résultat.
u
0
C:
::J

---- -
0
lfl
,-i
0
N Variables aléatoires d'ordre associées à des tirages sans remise
@ Une urne contient N boules numérotées de 1 à N. On tire, au hasard et sans remise,
....,
.c n boules de l'urne (n $ N). On note Xi la variable aléatoire associée au numéro de la
Ol
·;:: boule obtenue au ième tirage, pour i € lll, nn.
>-
a. a) Déterminer la loi de la variable aléatoire Xi pour i e: lll, nil. Préciser son espérance
0
u et sa variance.
b) Déterminer la loi conjointe de (X1,X2, ... ,Xr,.).

263
Chapitre 11 Vecteurs aléatoires

A l'issue du nème tirage les nwnéros obtenus sont classés par ordre strictement crois-
sant. On note Yj la variable aléatoire associée au /me numéro, classé par ordre stric-
tement croissant, obtenu parmi les n numéros.
Ainsi Y1 est la variable aléatoire min(X1 ,X2, . .. ,Xn), Y2 est le nombre aléatoire venant
juste après Y1 par ordre strictement croissant, etc... et Yn est la variable aléatoire
max(X1 ,X2, ... ,Xn).
c) Déterminer la loi conjointe du n-uplet (Y1, Y2, . .. , Yn).

---- -
d) Déterminer la loi de Yj pour j E: Œl, n].

IIascenceur (d'après Escp-Eap)


Un immeuble de p étages est équipé d'un ascenseur; n personnes montent dans l'as-
censeur au rez de chaussée et descendent chacune à un étage au hasard et de façon
indépendante. On note X la variable aléatoire égale au nombre d'arrêts de l'ascen-
sew·.
a) Déterminer la loi et l'espérance de X dans les cas suivants:
• dans le cas p =2 et n ~ 2;
dans le cas n = 2 et p ~ 2.

On revient au cas général.


Pour 1 s i s pet 1 s j s n, on note Yi,j la variable aléatoire prenant la valeur 1 si le
/me passager descend au ième étage et la valeur Osinon.
Pour 1 sis p. on note Xi la variable aléatoire prenant la valeur 1 si l'ascenseur s'ar-
rête au ième étage et la valeur Osinon.
b) Déterminer les lois des variables Yi,j et X; .
c) Calculer IE(X).
On note sg le nombre de surjections d'un ensemble fini de cardinal a sur un en-
semble fini de cardinal b.
d) Donner la loi de X en fonction des nombres sg.
min(n,p) _ 1 _
e) Endéduire: :[ (L)S~=pn-(p-l)n.

---- -
1
k=l

Étude du nombre de descendants d'une plante


Soit p E:]0, 1(.
On considère une plante qui peut donner naissance à deux descendants avec la pro-
babilité p, ou à aucun descendant avec la probabilité 1- p. Pour n E: N, on note Xn le
nombre de descendants issus de la nème génération, c'est-à-dire le nombre de des-
u cendants de no tre plante à la (n + l)ème génération.
0
C: On note aussi fla fonction définie sur (0, 1] par: f (x) = px2 + (1 - p).
::J
0 a) Montrer que f est strictement croissante sur (0, 1].
lfl
,-i
0
b) On définit une suite (un)nE:N par uo = 1- pet:
N
@ VnE:N, Un+I = /(un)
....,
.c
Ol
·;::
Montrer qu'elle est bien définie, puis étudier sa monotonie et sa convergence.
>-
a. c) Pour n E: N, donner une relation entre ifDOCn+l = 0) et ifD(Xn = 0).
0
u d) En déduire lim ifD(Xn = 0). Interpréter ce résultat.
n~+oo

264
Vecteurs aléatoires Chapitre li

Du mal à démarrer ?

11D a) Procéder à l'aide de dénombrements. e) Commencer par déterminer la probabilité d'obtenir « 1 » avec
le dé.
b) Ut iliser la loi conjointe de (X, Y).
c) Utiliser le système complet d 'évèneme nts associé à X.

GID a) Reconnaître une loi usue lle.


11D Remarquer que si Cov(X,Y) = 0 a lors XY su it une loi de ber- b) Ut iliser la loi cond itionnelle de Y sachant X.
nou lli de paramètre IE(X)IE(Y). c) Revenir à la définition de l'espérance.

1111 a) Utiliser les règles de calculs de la covariance.


Cll9 a) Reconnaître une loi usue lle.
b) On a Cov(Z, T) = O.
b) Uti liser la loi cond ition nelle de Y sachant X.
c) Uiliser le système complet d 'évènements associé à X.
c) Recon naître une loi usuelle.
d) Uti liser la loi conjointe de (Z, T).
e) Reprendre le calcul du b).
Gl9 Util iser la loi conditionnelle de Y sac han t X.
1111 a) Décrypter l' évènement [X= i] n [Y= j ].
b) Remarquer q ue [Y= i 2 J = [X= i l u [X= - i ).
c) Remarquer que Cov(X, Y) = IE(X3 )- IE(X)[(X2).
11111 a) et b) In trodui re les variables aléatoires égales au nombre
de réponses reçues à chaq ue envoi et bien reprendre son souffle
d) Uti liser la loi conjointe de (X, Y). avant de se lancer dans les calculs.

11D Uti liser le système comp let d'évènements associé à X.


1111 a) Reconnaître p lusieurs fois une loi usuelle.

1111 a) Utiliser le système com plet d'évènements associé à X puis b) Interpréter la variable a léatoire
k
L Y;.
reconnaître une loi usuelle. i =I
b) Revenir à la défi nition d'u ne probabilité conditionnelle puis c) Revenir à la définition de l'espérance puis appeler les gen-
simplifier sauvagement la formule. darmes.

1111 a) Décrypter les évènements [X= 1) et [X= n ). Pour ce der-


nier, ne pas hésiter à discuter su ivant la parité de n.
am a) Utiliser la formu le des probabilités composées puis recon-
naître une loi usuel le.b) c) et d) Procéder à l' aide de dénombre-
n n
b) Interpréter les variables aléatoires L X; et L Xj, ments.
i=2 j=2

1111 a) Il y a k jetons portant le nu méro k et k varie de l à n.


b) Décrypter l'évènemen t [B = il n [R = j ). Utiliser ensuite les sys-
am a) Décrypter les évènements.
b) Reconnaître plusieurs fois une loi usuelle.
tèmes complets d 'évènemen ts associés à B et R. p
c) Relier G à B et R.
c) Rel ier X et L X;.
i=l
d) Procéder à l'aide de dénombrements.
1111 a) Reconnaître une loi usuel le. e) Calculer IE(X) de deux façons différentes.
b) Reconnaître une loi usuel le puis utiliser le système complet

u
0
C:
d 'évènements associé à X1 .
c) Revenir à la défini tion de l'espérance.
am a) Facile!
b) Procéder avec les méthodes classiques. Pour déterminer la li-
0
::J

lfl
,-i
am a), b) etc) Mettre le système sous forme rédu ite de Gauss.
d) Utiliser le c).
mi te, discuter les valeurs de p par rapport à
c) Utiliser la formule des probabilités totales.
d) Utiliser les résu ltats du b).
!.
0
N
@
....,
.c
Ol
·;::
>-
a.
0
u

265
Chapitre 11 Vecteurs aléatoires

Corrigés des exercices

-
a) • Tirage simultané. I.;univers n est l'ensemble des combi-
naisons de 2 boules parmi 5, et Card(û) =
On a immédiatement X(û) = Y(û) = [1, 5].
= 10. m
* si i < j alors :
[X= il n [Y= jl=« on a obtenu les numéros i et j »
et donc: Card([X = iln [Y = jl) = 2 (ce sont les couples (i,j)
et (i, i)).
* si i = j alors :
Pour tout (i,j) e: [1,5Il 2 : [X= il n [Y = il =« on a obtenu deux fois le numéro i »
[X = il n [Y = j] =« Je plus petit numéro obtenu est le i et Je et donc: Card ([X = il n [Y = il) = 1 (c'est le couple (i, i)).
plus grand numéro est le j » Ainsi, pour tout (i,j) e: [1.5] 2 :
Donc:
*
*
si i?:. j : alors (X= il n [Y= j l = 0,
si i < j alors :
[X= il n [Y= jl=« on a obtenu les numéros i et j »
et donc: Card (rx = il n [Y= jl) = (DG)@= 1
P(X = i, Y = j) = l 0
2
25
1
25
si i > j
si i < j
si i = j
(on choisit la boule i, puis la boule j, puis O boule portant
Résumons ces résultats sous forme d'un tableau à double
un autre numéro).
entrée:
Ainsi,pourtout(i,j)e:[1,sn 2 :

~ 1 2 3 4 5 loi de X

ifD(X.= i,Y= j) = { ~ si i?:. j


si i < j
1 1
25
2
25
2
25
2
25
2
25
9
25
10
1 2 2 2 7
2 0 25 25 25 25 25
Résumons ces résultats sous forme d'un tableau à double
3 0 0 1 2 2 5
entrée (qui va nous permettre de calculer les lois margi- 25 25 25 25
nales): 1 2 3
4 0 0 0 25 25 25

~ 1 2 3 4 5 loi de X 5 0
1
0 0 0
7
1
25
1
25

1 0 1
10 10
1 1
10
1
10
4
10
loi de Y 25 ls }5 25 i5 total= 1

2 0 0 1 1 1 3 • Tirages sans remise. I:univers n est l'ensemble des arr-


10 JO 10 10
nagements de 2 éléments de [1,5], et Card(ü) = A~ = 20.
3 0 0 0 1 1 2
10 10 10 On a immédiatement X(ü) = Y(ü) = [1,5].
4 0 0 0 0 1 1 Pour tout (i, j) e: [1,5]2 :
10 10
[X= il n [Y= jl=« le plus petit numéro obtenu est le i et le
5 0 0 0 0 0 0 plus grand numéro est Je j >>
loi de Y 0 1 2 3 4
total= 1 Donc:
10 10 10 10
u
0 * si i ?:. j alors [X = il n [Y= jl = 0;
C:
::J
En sommant sur chaque ligne du tableau, on obtient la loi * si i < j alors :
0 de X. Le même procédé sur les colonnes donne la loi de Y. [X= il n [Y= j l=« on a obtenu les numéros i et j »
lfl
,-i • Tirages avec remise. I:univers n est l'ensemble des et donc: Card (rx = iln [Y= jl) = 2 (ce sont les couples (i,j)
0
couples de 2 éléments de [1,5], et Card(û) = 52 = 25. et(j,i)).
N
@ On a immédiatement X(û) = Y(ü) = [1, 5]. Ainsi, pour tout (i,j) e: [1,5] 2 :
....,
.c Pour tout (i, j) e: [1, sf :
Ol
ï::::
>-
a.
[X= il n [Y = jl =« le plus petit numéro obtenu est le i et le IP(X = i, Y = j) = { ~ si i ;;: j
si i < j
0 plus grand numéro est Je j » 20
u
Donc: Résumons ces résultats sous forme d'un tableau à double
*si i > j alors [X = il n (Y= j l = 0; entrée:

266
Vecteu rs aléatoires Chapitre l i

~
Et donc :
1 2 3 4 5 loi de X
4 3 2 1
IE(Z) = 0 x O+ 1 x - + 2 x - + 3 x - + 4 x - = 2
1 0 1 1 1 1 4 10 10 10 10
10 10 10 10 10
1 1 Tirages avec remise.
2 0 0 1 3
10 JO 10 10
1 l 2
z 0 1 2 3 4
3 0 0 0 10 10 10
IJl>(Z = z) 5 8 6 4 2
1 1
Z5 Z5 Z5 Z5 Z5
4 0 0 0 0 10 10
Et donc:
5 0 0 0 0 0 0
1 2 3 4
IE(Z) = 0 x 2_ + 1 x J!_ + 2 x ~ + 3 x _!_ + 4 x 2_ = ~
loi de Y 0 10 10 10 10 total= 1 25 25 25 25 25 5
Tirages san s remise.
b) Tirage simultané. On a:
z 0 1 2 3 4
IJl>(Z = z) 4 3 2 l
0 Io 10 10 Io
Donc X et Y ne sont pas indépendantes. Et donc:

Tirages avec remise. On a : 4 3 2 1


IE(Z) = 0 x O+ 1 x - + 2 x - + 3 x - + 4 x - = 2
10 10 10 10
1Jl>(X = 2,Y= 1) = 0 ;,.! 1Jl>(X = 2) x!Jl>(Y = 1)
i•fj 1 ===> 1Par théorème, si X et Y sont indépendantes
Donc X et Y ne sont pas indépendantes. alors : Cov(X, Y) = O.

Tirage sans remise. On a :


1 = 1On suppose que Cov(X, Y) = 0, c'est-à-dire que : IE(XY) =
IE(X)IE(Y).
• On pose p = IJl>(X = 1), q = IJl>(Y = 1) et on a donc X.._ $(p) et
Y'- &8(q). De plus p = IE(X) et q = IE(Y).

Donc X et Y ne sont pas indépendantes. Comme (XY)(D) = {O, l}, on sait donc queXY suit aussi une loi
de Bernoulli de paramètre IJl>(XY = 1). Mais alors :
c) On a: Z(D) c uo.4]. IP(X = l, Y= 1) = IP(XY = l) = IE(XY) = IE(X)IE(Y) = pq
Si z e: Z(D), on a, en appliquant la formule des probabi-
donc :
lités totales avec le système complet d'évènements (IX =
IJl>(X = l,Y = 1) = IJl>(X = 1) x !Jl>(Y= 1)
klh€[l,5Il :
• D'après la formule des probabilités totales avec le système
5 complet d'évènements ([Y = 0]. [Y = 11) :
IJl>(Z = Z) = L IJl>(X = k, y= k + Z)
k=l IP(X = 1) = lfD(X = l,Y= 0) +IP(X = l,Y= 1)

Donc: et donc:
"O
0 IJl>(X = l, Y= O) IJl>(X = 1)-IJl>(X = l,Y = 1)
C:
::J IJl>(Z = 4) 0 + IJl>(X = l, Y= 5)
0 IJl>(X = 1) - IJl>(X = 1) x IJl>(Y = 1)
lfl 1Jl>(Z = 3) = 0 + 1Jl>(X = l,Y = 4) +1Jl>(X= 2,Y = 5)
,-i lfD(X = l)x(l - lfD(Y = l))
0 1Jl>(Z = 2) = 0 + 1Jl>(X = l,Y = 3) +1Jl>(X= 2,Y = 4)
N lfD(X = 1) X lfD(Y = 0)
@ +IJl>(X = 3,Y = 5)
...., etc .. . • De même:
.c
Ol
ï:::: IP(X = 0, Y= l) lfD(Y = 1) - lfD(X = l, Y= 1)
>-
a. Tirage simultané. lfD(Y = 1) - lfD(X = l) X lfD(Y = l)
0
u = IJl>(Y=l)x(l-lJl>(X=l))
z 0 1 2 3 4
IJl>(Y = 1) x IJl>(X = O)
4 3 2 l
IP(Z= Z) 0 10 10 10 10
267
Chapitre 11 Vecteurs aléatoires

Et : z 2 3 4 5 6
l 2 2 l
IJl'(X=O,Y= 0) IJl'(Y = 0) - IJl'(X = 1, Y= 0) IJl'(Z = z) 9 9 i 9 9
= IJl'(Y = O) - IJl'(X = 1) x IJl'(Y = 0)
On sait que: [(X) = [(Y) = 3~ 1 = 2. Donc par linéarité de
IJl'(Y = O) x (1- IJl'(X = 1)) l'espérance :
IJl'(Y = 0) X IJl'(X = 0)
[(Z) = [(X) +[(Y)= 2+2 = 4
• Ces quatres formules prouvent que X et Y sont indépen-
dantes. • On a T(D) c [ - 2,2] .
En conclusion, on a démontré que : De plus, en appliquant la formule de des probabilités to-

-
X et Y sont indépendantes si et seulement si Cov(X, Y) = O. tales avec Je système complet d'évènements ([X= li, [X=
2J, [X=3J), on a:

Z+T Z- T 'v' t E T(D), IJl'(T = t) IJl'(X = l)IJl'x=1 (Y = 1 - t)


a) On a immédiatement: X= - - et Y= - - .
2 2
+IJl'(X = 2)1Jl'X=2 (Y = 2 - t)
D'après les règles de calcul sur la covariance:
+IJl'(X = 3Wx=3CY = 3- t)

Cov(X,Y) Cov(Z+T Z-T)


2 ' 2 et comme X et Y sont indépendantes, on a donc, pour tout
t e:T(D):
1
- ( Cov(Z, Z) + Cov(T, Z) - Cov(Z, T)
4
=O IJl'(T = t) = IJl'(X = l)IJl'(Y = 1 - t) + IJl'(X = 2)1Jl'(Y = 2 - t)
- Cov(T,T)) + IJl'(X = 3)1Jl'(Y = 3 - t)

~(V(Z)-V(T))
4 On en déduit la loi de T :

b) Si z et T sont indépendantes a lors Cov(Z, T) = O. t -2 -1 0 1 2


IJl'(T = t) l 2 3 2 l
Or: 9 9 9 9 9
Cov(Z, T) = Cov(X + Y, X - Y) Par linéarité de l'espérance:
= Cov(X, X) + Cov(Y, X) - Cov(X, Y) - Cov(Y, Y)
[(Z) =[(X) - [(Y) =0
=O
= V(X) - V(Y) De plus X et Y sont indépendantes, on a donc Cov(X, Y) =
0, et donc d'après Je a): V(T) = V(Z). De simples calculs
Donc si z et T sont indépendantes, on a V(X) = V(Y). donnent:
C) • On a Z(fl) c [2,6]. 4
V(T) = V(Z) =
De plus, en appliquant la formule de des probabilités to- 3
tales avec Je système complet d'évènements (IX = l i, [X = d) Ona: [T = 1J n[Z = 3] = [X = 2] n[Y = 1). Donc:
21, [X = 31), on a:
u 1
0 IJl'(T = l,Z = 3) = IJl'(X = 2,Y = 1) =lfl'(X= 2) X IJl'(Y = 1) = 9'
C: 'v'z E Z(D), IJl'(Z = z) IJl'(X = l)IJl'x= 1 (Y = z - 1)
::J
0 +IJl'(X = 2)1Jl'x=2 (Y= z - 2) par indépendance de X et Y.
lfl
,-i +IJl'(X = 3)1Px=3 (Y= z - 3)
0 Et IJl'(T = 1) = IJl'(Z = 3) = ~ donc:
N
@ et comme X et Y sont indépendantes, on a donc, pour tout
...., IJl'(T = l,Z = 3)-:j.1Jl'(T = 1) x IJl'(Z = 3)
ze:Z(D):
.c
Ol
ï:::: Ceci prouve que Z et T ne sont pas indépendantes.
>- IJl'(Z = z) = IJl'(X = l)IJl'(Y = z - 1) + IJl'(X = 2)1Jl'(Y = z - 2)
a. e) On a vu au b) que: Cov(Z, T) = V(X) - V(Y). Or ici X et Y ont
0 + IJl'(X = 3)1Jl'(Y = z-3)
u la même loi donc V(X) = V(Y).
On en déduit la loi de Z : On a donc : Cov(Z, T) = O.

268
Vecteu rs aléatoires Chapitre li

-
a) • On aX(!l) = [-n, nil donc: Y(!l) = {k2/ k e: [O, nil} .
Soit (i, j) e: X(!l) x Y(û). On a:
c) Par définition: Cov(X, Y) = IE(XY) - IE(X)IE(Y) et donc :

si j ;é i 2
[X = i) n [Y = JI = [X = il n [X2 = j] = J <l>
l [X= il si j =i 2 Ona:
et donc:

f(X =i, Y =j) = l 2n+l


0
1
si j ;é i 2
si j = i2
IE(X)
n
I:
k=- n
klfl>(X = kl

• Onen déduit la loi de Y. Si k e: [o, n], la formule des pro- n


babilités totales, appliquée avec le système complet d'évè- I: k
2n+ 1 k=-n
nements ([X = il);e:[-n,n], nous donne: 1
- - ( ~ +~ + ... + J.,,,,-16 + ... +__c,..-rf
n 2n+ 1
lfl>(Y=k2)= L lfl>([X=i)n[Y=k2J) +o + j + ... + /t + ... +1n-1T+ f)
i= - n
0
Donc si k ;é O:
[P>(Y= k2)
De même :
=IX=k l =IX=- kl
11
2
+ ~" lfl>(fX = il n.,iY = k 1)
1
i#et1-,1- 1.; =~
IE(X3)
1 1
- - + --+0 n
2n + l 2n+l
2
L k3 1P'(X= k)
k= - n
= 2n+ 1 1 n
et de même:
= I: k3
2n+ 1 k= - n
11

[P>(Y = 0) lfl>(fX = O) ~[Y= OJ) + ;~,, i?(fX = il~ [Y= OI) _ l _ (vnf+ ~ + · · · +.Htf+···+~
2n+ 1
=IX=OJ =~
+o3 +/+···+Y+ ·· · + ~+/ )
i,'O

__1. -+0
2n+l = 0
1
2n + l
On a donc: Cov(X, Y) = 0 - 0 x IE(X2 ) = O.
u b) OnaY(!l) = {k2/ke:[O,nil}.
0
C:
::J Si k e: [O, n] : 0 n en de'd mt
· . (X Y)_ Cov{X,Y) _
0 que. p , - v'V{X)V(Y) - 0 .
lfl
,--i
0
N
@
2
IV = k21 = [)( = k21 = l [X = OJ
[X = k) u [X = - k]
si k = 0
si k ;é 0
d) On a trouvé une covariance nulle donc on ne peut rien en
conclure sur l'indépendance de X et Y.
...., et donc pour k ;é O:
.c
Ol Par contre, d'après les résultats du a):
[P>(X = 0, Y= 1) = 0, lfl>(X = O) = 2111
ï::::
>- l?(Y = k2) = lfl>(X = k) +l?(X =-k) = _ 1_ + _
1_ = _ 2 _ + 1et lfl>(Y = 1) = 2,?H .
a. 2n+l 2n+l 2n+l
0
u et: Donc : lfl>(X = 0, Y= 1) ;é lfl>(X = O)lfl>(Y = 1). Ceci prouve que
1
lfl>(Y = O) = lfl>(X = 0) = - - X et Y ne sont pas indépendantes.
2n + 1

269
Chapitre 11 Vecteurs aléatoires

- Onsaitquelafamille d'évènements ([X = kJ)kE[O,n]


associée à X est un système complet d'évènements. La for-
IJ»(Z = k)
mule des probabilités totales nous donne alors :
n1
= L IJ»(X = i,X+Y = k)
n i=O
IJ»(X = Y) L o:»x= k(X = Y)IJ»(X = k) n1
E lfD(X = i, Y = k - i)
k=O
n i=O
= L o:»([X = Yl n [X = kl)
-
Il]

L lfD(X = i)lfD(Y = k- i)
k=O X et Y
n indépendantes i=O
= L o:»([Y = kJ n IX = kJ)
k
k=O L IJ»(X = i)lfD(Y= k-i)
(car [X = Y] n [X = k] = [Y = k] n [X = k]) i=O
n
E lfD(Y = kW<X = k) +. E
I!]

o:»(X = i)lfD~~- iJ
k=O 1=k+l =~
(car X et Y indépendantes)
1 1 1
k
L lfD(X = i)lfD(Y = k- i) + 0
tJ:)2k211 -k(;);k 211 -k i=O
2 donc:
n (n)
E k
1
z2n
k=O
2
n (n)
= 1
z2n t'o k

Mais, on a: {Z) = (n~k). La formule de Vandermonde nous


donne donc que:

La formule de Vandermonde nous donne alors :

lfD(Z = k) = ( n1: n2 )pk(l - p)ni +n2-k

Finalement : Onadonc:Z = X+Y '- 8$(n1 +n2,p).


On peut interpréter ce résultat de la façon suivante : X est

(2n)
le nombre de piles obtenus sur n 1 lancers,, Y est le nombre
1
lfD(X = Y) = Il z2n de piles obtenus sur n2 lancers effectués après les n 1 pré-
'O cedents et Z est le nombre de piles obtenus sur ces n 1 + n2
0
C:
::J
lancers au total. On sait que X'- @(ni, p), Y._ @(n2, p), X
0 et Y indépendantes, Z '- 8$(11 1+ n2, p) et on a bien Z =X+ Y.

-
lfl
,--i
On retrouve donc le résultat donné par les calculs précé-
0 dents.
N
@ b) Soit k e: [o, 111 + n2]. On a:
....,
.c IJD(X= k,X+ Y= r)
Ol
·;::
a) On a :X(ü) = [o, 111Il etY(û) = [o, n2], donc: Z(ü) c [o, 111 + o:»z=r(X = k) IJD(Z = r )
>- n2].
a. IJD(X = k, Y= r - k)
0
u = lfD(Z = r)
Soit k e: [O, n1 + n2]. D'après la formule des probabilités lfD(X = k)lfD(Y = r - k)
totales, appliquée avec le système complet d'évènements
x et v lfD(Z = r)
indépendantes
270
Vecteu rs aléatoires Chapitre li

Donc si k > r on a IPz= ,.(X = k) = 0 et sinon : de couleurs différentes, c'est-à-dire au nombre de "chan-
gements".
D'autre part, le nombre de séries unicolores est égale au
nombre de "changements" additionné de 1 (à cause de
la première série unicolore qui n'a pas été comptabilisée
corrune un changement). On a donc :
n n
X = l +L Yi+ L Zj
i =2 j =2

• Par linéarité de l'espérance, on a donc:


et, avec les conventions habituelles sur les coefficients bi- n n
nômiaux, cette formule est aussi valable si k > r. IE(X) = 1+ L IE(Yi) + L IE(Zj)
i =2 j =2
On a donc obtenu que : X1z= r .._ ..?t'(ni + n2, r, 11 /~ 112 ).
Or Yi (11) = {0, l}, donc Yi suit une loi de Bernoulli de para-

i•H
a) • I.:évènement[X = li correspond à l'évènement« on a tiré
mètre ifD(Yi = 1) et donc : IE(Yi) = ifD(Y; = 1).
Le même raisonnement donne que: IE(ZJ) = ifD(ZJ = 1).
que des blanches ou que des noires ». Donc: Donc:

IP(X = 1) = pn + qn n n
IE(X) = 1 + L IP(Yi = 1) + L ifD(Z j = 1)
i=2 j=2
• I.:évènement [X= n] correspond à l'évènement« on a tiré
à chaque coup une couleur différente de la précédente». Mais: ifD(Y; = 1) = pq et ifD(Z j = 1) = qp. Donc:
Pour savoir à combien de boules blanches ou noires cela
correspond, distinguons deux cas suivant la parité de n. n n
IE(X) = 1 + L pq+ L qp = 1 +(n - l)pq + (n - l)qp
*Sin est pair : alors n = 2k avec k e: 1\1. L'évènement [X = n]
i =2 } =2
correspond aux tirages BNBN · · · BN (avec k fois B et k fois
N) et NBNB ···NB (avec k fois Net k fois B). Donc: = l + 2(n -

-
Finalement: IE(X) l)pq.

a) Il y a 1 jeton portant le numéro bleu 1, 2 jetons por1tant le


* Si n est impair: alors n = 2k + l avec k e: N. I.:évènement numéro bleu 2, ... , k jetons portant le numéro bleu k, ... ,
[X= n ] correspond aux tirages BNBN · · · BNB (avec k+ 1 fois
li jetons portant le numéro bleu li.
B et k fois N) et NBNB · · · NBN (avec k + 1 fois N et k fois B).
Donc: n
Donc au total le nombre de jetons est égal à : k = L
k= l
n(n + 1)
H-1
= (pq) T 2
"O
0
C: b) • On a B(fl) = [1, n] et R(fl) = [1, n].
::J
n
0 Pour (i, .i) e: [1, n]2, l'évènement IB = il n [R = .il corres-
lfl
b) • La variable aléatoire L Yi représente le nombre de fois pond à 1' évènement « on a tiré le jeton portant le numéro
,-i i=2
0 où on a obtenu une blanche puis une noire. bleu i et le numéro rouge j ».
N
@ n Donc si i < .i on a [B = il n [R = fJ = f/> et si i 2:: .i : Card ( [B =
....,
.c
De même la variable aléatoire L Z J représente le nombre il n [R = jl) = 1 (car il y a un seul jeton portant le numéro
} =2
Ol bleu i et le numéro rouge j) .
·;:: de fois où on a obtenu une noire puis une blanche.
>-
a. n n On a donc:
0
u Donc la variable aléatoire L Yi + L Z J représente le
i =2 } =2
nombre de fois où on a obtenu deux boules consécutives •UB = ;J n [R = jl) = j 2
0

n(n+l)
si i < j
si i 2:: j

271
Chapitre 11 Vecteurs aléatoires

• Soit k E [1, n]. En appliquant la formule des probabili- iJL€[l,n] • on obtient :


tés totales, avec le système complet d'évènements ([R =
jJ) J€[l,n], on obtient: n
IP'(R = k) L IP'(B = i, R = k)
i= l
n
k- 1 n
IP'(B = k) L IP'(B = k, R = j) [IP'(B = i,R = k) + [IP'(B = i,R = k)
J= I i= l ~ i=k
k n
=O
L IP'(B = k, R = j) + L IP'(B = k, R = j)
=
n
o+ [ - -
2
J= I J=k+I' - . . - " i = k n(n + 1)
=O
k 2 2(n-k+l)
I: +o = n(n+ 1)
J= I n(n+ 1)
2k On a ensuite :
n(n + 1) n
[(R) L klP'(R = k)
k=I
On a ensuite :
[. 2k(n- k+ 1)
n k=I n(n + 1)
IE(B) L klP'(B = k)
k= I 2 n
Il 2k2
- - I: (en+ 1)k-k2)
n(n + l) k=I
t'i n(n+ 1)
~ f. k - 2 f. k2
2 Il n k=I n(n+ 1) k=I
= n(n+ 1) k= I
[k2
2n + 1
= (n + l) - - -
3
2
~(2n+l)
: ~X 6 n+2
2n+l 3
3 Le théorème de transfert donne :
n
Le théorème de transfert donne : IE(R2 ) = L k21P'(R =k)
k= l
n
[(8 2 ) L k21P'(B = k) [. 2k2 (n-k+l)
k=l k= I n(n + 1)
n 2k:' 2 n
= k~l n(n+ 1) - - I: (en+ 1)k2 -
n(n+l) k= I
k3 )
2
- - [k3
n(n + 1) k=l
n
~f.k2_ 2
nk=l n(n+l)k=I
[.k3
u 2
0 2 x { n(n + 1) ) (n+1)(2n+l) n(n+l)
C:
::J n(n + 1) 2 3 2
0 n(n + 1) (n+ l)(n+2)
lfl
,-i 2 6
0
N
et alors, d'après la formule de Koenig-Huyghens: et alors, d'après la formule de Koenig-Huyghens:
@
....,
.c
Ol
2 V(R) = (n+l)(n+2) - {n + 2)
2
= (n+2)(n - l)
·;:: V(B) = IE(B2 )-(IE(B)) 2 = n(n + 1) -( 2n + 1 ) = (n + 2)(n - 1) 6 3 18
>- 2 3 18
a.
0 c) Par linéarité de l'espérance:
u
• Soit k E [1,n]. En appliquant la formule des probabili-
2n+l n+2 n-1
tés totales, avec le système complet d'évènements ([B = [(G) = [(B) - IE(R) = - - - - = -
3 3 3

272
Vecteu rs aléatoires Chapitre li

Et d'après les propriétés de la variance: V(G) = V(B) + et donc, d'après la formule de Koenig-Huyghens:
V(Rl - 2Cov(B, R). 2 2
Or:
2)
V(X1) = IE(X1 -
(IE
CX1)
)2 = (n + 1)(2n + 1) ( n + 1)
-- = --
n - 1
6 2 12

IE(BR) i[t 1
ij(J)(B=i_'~~j)l
= OSI l<J
b) • OnaX2(ûl = [O,n - l].
* Si X1 = n alors il n'y a plus de jetons dans le sac et X2 = O.
On a donc:

t ([ ij(l)(B
i=l j =l
= i,R = j))
(J)x 1=n(Xz = 0) = 1 et (J)x 1 =n(Xz = k) = 0 si k E [l, n - 1]

* Soit j E [1, n-1] . Si CX1 =j ] est réalisé alors il reste n- j


t(t
i=l j =l n(n + 1)
2ij )
jetons dans Je sac : Je second enfant les tire un par un jus-
qu'à obtenir un jeton donné (le plus grand numéro), et
alors X2 <-+ Olt ([ l, n - J]) . On a donc :
f 2
i
i=l n(n + l) j =l
(t 1) si kE [1,n - j]
f 2i i(i+l) si k E [ n - j + 1, n - 1Il
= L, - - - x - -
i=l n(n+ l) 2
• Soit k E [O,n - 1]. La formule des probabilités totales,
=
1
n(n+l) f:i 11 .3 ·2
(i +i)
appliquée avec Je système complet d'évènements ( IX1
Jl) jE:[l,n]' nous donne:
n
1 ( f i3 + f i2)
(J)(Xz = k) = L (J)(X1 = JJ(J)x1 = j (X2 = k)
n(n + 1) i~ i~
j =l
2
1 ( n (n + 1)2 + n(n + 1)(211 + 1) )
n(n+ 1) 4 6
* Si k i Oon a donc :

n(n+ 1) 2n+ 1
---+--
4 6
et donc:
Cov(B,R) IE(BR) - IE(B)IE(R)
n(n+l) 2n+l (2n+l)(n+2)
--- +-- - -----
4 6 9
Après simplifications, on a donc :
V(G) = (n- l)(n + 2)

-
18

a) On tire un à un les jetons jusqu'à obtenir un jeton donné.


"O D'après le cours, on sait donc que: X1 <-+ Olt([I. n]).
0
On a donc : IE(X1) = 11 ; 1 .
C:
::J i = n-j
0
De plus, d'après le théorème de transfert :
lfl
,-i
n
* De plus :
0
N IE(XI) L k 2(J)(Xl = k) n 1
@ k=l L -(J)x1=jCX2 =Ol
...., j=l Il
.c t .!.
0
u
Ol
·;::
>-
a.
k=l
k2
n
n(n + 1)(2n + 1)
-x-----
n 6
= .!_
n
('t 1
(J)x1 = j (X2
=O
=0)
+ (J)x1= n (X2
j=I ' - . . - ' ....___...__,
=l
=0)]
(n + 1) (2n + 1) 1
= 6 n

273
Chapitre 11 Vecteurs aléatoires

c) Par définition de l'espérance: alors:


lfl>(A) IP([2a = bl n [c = 3a))
n-1 6
L k1P(X2 = k) L lfl>([a = k) n [b = 2a) n [c = 3a)}
k =O k= l
n- 1 6
O+ L, klP(X2 = k) L lfl>([a = k) n [b = 2k) n [c = 3kl)
k=l k=l
n-1
L,k
k=I
(
-1n r. -1):-
n-1

i=k 1
a, b etc
6
L lfl>(a = k)lfl>(b = 2k)IP(c = 3k)
k=l
indépendantes
= -r.
1 n- l(n- 1
L,-:-1
k) IP(a =l)lfl'(b =2)1P(c =3)
n k= l i=k
+IP(a = 2)1P(b = 4)1fl>(c = 6) + 0
-n r.
1 n - 1( i
L, -:-1
k) 1 1 1 1 1 1
- x - x - + - x - x - +O
permutation i = l k= l 6 6 6 6 6 6
des 1:
Donc : IP(A) = TÔ1î.
= -r.-:-
n
r. k
1 n- l 1 ( i
i=l I k=l
)
b) Soit B l'évènement« (S) n'a aucune solution ». On a: (S) n'a
'-v-'
aucune solution si et seulement si 2a - b = 0 et c - 3a ~ 0,
- i (i +l)
- -2-
c'est-à-dire: B = [2a = bl n [c ~ 3a).
On remarque que A et B sont incompatibles et que : Au B =
-1 n-1r.
2n i= 1
u+1) [2a= b).
La même technique qu'au a) donne :

= -1 (n-r,1ï + cn - 1i ) IP(AuB) IP(2a = b)


2n i = l 6

- 1 ((n - l)n +(n - 1)) = L IP([a = kl n lb= 2al)


= 2n 2 k=l
6
L IP([a = kl n l b = 2kJ)
k= l
Après simplifications:
6
L IP(a = k)lfl>(b = 2k)
aet b k=l
(n - 1) (n + 2) indépendantes
IE(X2 ) = - - - - -
4n 1 1 1 1 1 l
- x -+- x - + - x - +0
6 6 6 6 6 6
1
12
- Remarquons tout d'abord que les variables aléa- Donc: lfl>(B) = lfl>(2a = b) - lfl>(A) = }7 .
toires a, b et c sont indépendantes et de même loi
u c) Soit C l'évènement « (S) a une unique solution>>. On sait
0
"2t([l,6Il),
C:
qu'il n'y a que trois cas possibles (et incompatibles) : (S) a
::J une infinité de solution ou aucune solution ou une unique
0
lfl
,--i
0
N
@
De plus : (S) <==>
l fxl
~
- 2y
(2a-b)y=c-3a
=3 solution, c'est-à-dire que les évènements A,B et C forment
un système complet d'évènements.
On a donc:
....,
.c a) Soit A l'évènement« (S) a une infinité de solutions». On a : lfl>(C) = 1 - IP(A) - IP(B) = ~~
Ol (S) a une infinité de solutions si et seulement si 2a - b = 0
·;::
>- et c-3a = 0, c'est-à-dire: A= [2a = b) n [c = 3a). d) Soit D l'évènement « (S) a pour unique solution (3,0) ».
a. On a : (S) a pour unique solution (3, 0) si et seulement si
0
u 2a - b -t, 0 et c - 3a = 0, c'est-à-dire: D = [2a -t, b] n le = 3aJ.
La formule des probabilités totales, appliquées avec le sys-
tème complet d'évènements (l a = kJ) kElll,6] , nous donne On a donc D et A incompatibles et C uA = [c = 3a].

274
Vecteu rs aléatoires Chapitre li

La même technique qu'au a) donne:


11111
6 a) On a immédiatement X<-+ "21([1, n]) et donc: IE(X) = ~ .
IP'(c = 3a) L IP'([a = k] n [c = 3aJ) b) On a Y(Q) = [l, n].
k=l
6 Soit k e: [1, nil. La formule des probabilités totales, ap-
L IP'([a = k] n [c = 3kJ) pliquée avec le système complet d'évènements (IX =
k=l
jl) jE[l,n]' nous donne :
6
-
a etc
L IP'(a =k)IP'(c =3k) n
indépendantes k=l IJl>(Y = k> = I: IJl>(X = jWx=1<Y = k>
J=l
1 1 1 1
-x-+-x- + 0
6 6 6 6 Or, sachant que « X=j » est réalisé, on a donc n + j jetons
1 dans l'urne, dont exactement (j + 1) portent le numéro j,
= 18 et exactement 1 porte le numéro k, pour k e: [2, n] et k i j.
C'est-à-dire :
Donc : IJl>(D) = IP'(c = 3a) - IP'(A) = 1g8 .
si j i k
e) • Notons p la probabilité d'obtenir« 1 » avec un dé. On a
donc, pour tout k e: [1,6Il: IJl>(a = k) = kp.
\;/ 1. e: [1 'n] ' lfl'x-- . (Y= k) = {
J
nl
k+l
J
n+k
si j = k
6
{;égalitéL IJl>(a = k) = 1 nous donne alors p = 2\. On a donc:
k=l
• On reprend le calcul du a) : IJl>(Y = k)
n

IP'(A) =
6
L IP'(a = k)lfl'(b = 2k)IP'(c = 3k) lfl'(X = k)lfl>x=k(Y = k) + L lfl'(X = jllfl>x= j<Y =k)
j=I
k=l j#k

1 2 3 2 4 6 1 k+ 1 n 1 1
= - x - x - + - x - x - +O
21 21 21 21 21 21
-x--+[-x--
n n+k i= 1 n n+j
j#
Donc : IP'(A) = 3~3 .
• On reprend le calcul du b) :

IP'(AuB)
6
I: IP'(a = k)lfl>(b =2k>
= ~(n:k + n:k + t n:j]
j#

k=l
1 2 2 4 3 6
-x- + -x-+-x-+0
21 21 21 21 21 21
4 On obtient donc, pour tout k e: [1, n] :
63
k an
IP'(Y= k) = +-
Donc : IP'(B) = 1Jl>(2a = b) - IP'(A) = li887 . n(n + k) n
• On reprend le calcul du c) : c) On a par définition de l'espérance:
u n
0 IJl>(C) = 1 - lfl>(A) - lfl>(B) = ::
C:
::J
IE(Y) L k IP'(Y = k)
0 k=l
• On reprend le calcul du d) :
lfl
,-i
0 6
f ( k kan)
k=l n(n+k) +-;--
2

N
@
IP'(c = 3a) L lfl>(a = k)IP'(c = 3k)
n k2 n
k=l
....,
.c 1 3 2 6
= -1 [ - - + an L k
Ol
ï::::
= -x-+-x-+O
21 21 21 21
n k= J n + k n k= l
>- 1 n k2 an n(n + 1)
a. 5 -[-- + - x ---
0 n k= l n+ k n 2
u 147
11 2
Donc: IJl>(D) = IP'(c = 3a) - IP'(A) = 1~~ 9 . = -n1 k=I k n+l
[ - - + - - an
n+ k 2

275
Chapitre 11 Vecteurs aléatoires

-
a) Chaque pièce donne «pile» avec la même probabilité p,
de plus les pièces sont lancées simultanément donc cha-
cune est lancée de manière indépendante des autre. On
sait donc que: X<-+ @(n, p).
En réinjectant dans Je calcul précédent, on obtient donc :

=
IJD(Y = j)
k)
o+ . n-k+
.L j (n. -
(J
k). pi(l - p)n-i
i=j l-J
= [O, k].
(1k) n~k (n -€ k) p e+ j (1 - p) n-e- j
b) On a Y(n)
-
f=i-j
. L,
Soit je: [o, k]. D'après la formule des probabilités totales, f=O
appliquée avec le système complet d'évènements ([X =
il) i€[O,n], on a : =
n
IJD(Y = J) = .[ IJD(X = i)IJDx=i (Y= J) -
formule
i=O du binôme

Or sachant que [X = il est réalisé, Justine ne peut obtenir


au maximum que i «piles», et donc: IJDx=i (Y = j) = 0 si
j > i. Si j si, on effectue un tirage simultané de k pièces,
la variable aléatoire Z égale au nombre de« piles» obtenus Finalement, on trouve que : Y<-+ @(k, p) .
suit donc la loi ..1e(n, k, fi).
et donc:
c) On lance k pièces simultanément et on compte le nombre
de «pile». Le même raisonnement qu'au a) donne Z <-+
@(k,p).

On remarque que Y et Z ont la même loi, c'est-à-dire que


la gain de Justine est Je même dans les deux e:Kpériences. Il
On a donc :
est donc équivalent de compter le nombre de «piles» ob-
tenus, parmi k pièces choisies au hasard parmi n lancées
IJD(Y = j)
simultanément, ou parmi k pièces lancées simultanément
j-1 n après les avoir choisies au hasard parmi n pièces.
= .[ IJD(X = i)IJDx=i(Y = j) + .[ IJD(X = i)IJDx=i(Y = j)
i =O ' - - v - - - ' i =j
=O
- On sait que X c.... @(n,p). De plus, ona Y(ü) = [l, n].
~ (n). p i (1- p )n- i (1Ht)
= 0 + iL, (n) X
=j l k • Soit k e: [l, n]. La formule des probabilités totales, appliquée
avec le système complet d'évènements ([X = Jl) j€[O,n]. nous
donne:
On remarque alors que :
I!
IJD(Y = k) = .L IJD(X = .i)IJDx= j(Y = k)
(7) (}) (t::J) j=O
'O
0 (i)
C: Or, sachant que [X = 01 la variable aléatoire Y suit la loi
0
::J
fi
--- X
;! X -------
J,n.-i1f
qL([l, n]), c'est-à-dire:
lfl fJ,n.--iif J!(i - })! (k- j)!(n - i - k + j)!
,-i
0 k!(n- k)!
1
N X fi 'v'ke:[1.nll, IJDx=oCY=k)=-
@ n
....,
.c
Ol
·;::
et sachant que [X = Jl avec j 1- 0, la variable aléatoire Y est
>- constante (ou certaine) égale à .i, c'est-à-dire:
a.
0 mais attention ce calcul n'est valable que si k- j s n - i,
u c'est-à-dire : i s n-k+ j . Si i > n-k+ j, on a toutsimple-
n- Ji) = O. 0 sikc/j
ment ( k- 'v'Je:[1,n],'v'ke:[1,n], IJDx=j(Y = k) =
{ 1 si k= j
276
Vecteu rs aléatoires Chapitre li

On a donc, pour tout k E [1, nil : La variable aléatoire X est égale au nombre de réponses ob-
tenues, donc: X = Y1 + Y2.
lfl'(Y= k) lfl>(X = O)lfl'x=oCY = k) +lfl'(X = k)lfl'x=k(Y = k)
'-'"V--' '-v-" • On a, d'après les résultats du cours sur les sommes de
: l,, =l variables aléatoires :
n
+ [ lfl'(X = j)lfl'x= j (Y= k)
lfl>(X = 0) lfl>(Y1 +Y2 =0)
j=l '-'"V--'
j# =O n
1 L lfl'y1=k(Yl + Y2 = O)IJl'(Y1 = k)
- lfl'(X = O)+lfl'(X = k) k=O
Il n

; (; )po(l - p)n-0 + (: )pk(l - p)n-k


I: IP'v1=kCY2 = - kllfl'CY1 = k)
k=O
lfl>y1=o(Y2 = O)IJl'(Y1 = 0)
11
(l - p) n
n + (n)
k p k (l- p) n-k + L lfl'y,=k(Y2 = - k)lfl'(Y1 = k)
k =l ' - - - . . - - - '
=O
• Par définition de l'espérance:
n (;)po(l-p)n- o x ( ;)ro(l-p)n- o +o
IE(Y) =L klfl'(Y = k)
k= I (l _ p)2n

donc:
• De même:
IE(Y) = f(
k=I
(1- p)n
n
+ (n)pk(l - p)n-k)
k
lfl'(X=l) lfl>(Y1+Y2=l)

= (1 - p)n f k+ f (n)pk(l - p)n- k n


L IP'y1=k(Yl +Y2 = l)IP'(Y1 = k)
n k=I k=l k
k=O
+ fL, k(n) p k (l-p) n-k
(1 - p)n n(n + 1) Il
--- x I: lfl>v,=kCY2 = 1- kllfl'CY1 = kl
n 2 k =l k
k=O
On remarque la somme de droite est égale à l'espérance d'une lfl>y1 =0CY2 = l)lfl'(Y1 = 0) + lfl'y1 =1 CY2 = O)lfl'(Y1 = 1)
variable aléatoire de loi binomiale @(n,p), c'est-à-dfre à np. n

Donc:
+ I: IP'v1 =kCY2 = 1- kllfl'CY1 = k)
n+l k=2' - - - - - - _ _ ;
=
IE(Y) - - ( 1 - p) 11 + np =O

-
2
(7)PI (1 - p)n-I X ( ; )po(l - p)n-0

a) • Commençons par traduire les hypothèses de l'énoncé.


On note Y1 la variable aléatoire égale au nombre de ré-
+( n; 1)po(l - p)n-1-0 x ( 7)pl (l - p)n- l

ponses obtenues suite au premier envoi, et Y2 la variable np(l - p)2n-I + np(l - p)2n-2
u
0 aléatoire égale au nombre de réponses obtenues suite au
C: np(l - p)2n-2((1- p) + 1)
::J second envoi.
0 np(l - p)2n-2 (2 - p)
lfl
Pour le premier envoi: chaque personne répond indépen-
,--i damment des autres, chacune avec la même probabilité p,
0
N et il y an personnes. On a donc Y1 <-+ @(n,p).
@
b) • On passe au cas général.
Pour le second envoi, on pourrait faire le même raisonne-
...., Au minimum, l'enfant peut ne recevoir aucune réponse, et
.c ment, le problème étant que le nombre de personnes à qui
Ol
·;:: on envoie une invitation est aléatoire (égale à n-Y1). Pour au maximum, il peut recevoir n réponses.
>-
a. contourner ce problème on va utiliser une loi condition- Donc X prend toute valeur entière entre Oet n :
0 neUe. On fixe donc k E Y1 (Q) = [O, nil. Alors on peut dire xcn) =[o,nl
u
que la loi conditionnelle de Y2 sachant que [Y1 = k) est la
loi@(n-k,p). • Soit j E [o, nil fixé. D'après les résultats du cours sur les

277
Chapitre 11 Vecteurs aléatoires

sommes de variables aléatoires : • On sait donc que:

IJD(X= j) [(X) = np(2 - p) et V(X) = np(2- p)(l - p) 2

-
IJD(Y1 + Y2 = j)
11
L [IDY1=k(Yl + Y2 = j)IJD(Y1 = k)
k=O
n a) • Chaque client achète un animal de l'espèce i avec la
= [
k=O
IIDy, = k(Y2 = j - k)IJD(Y1 = k) n
même probabilité Pi, et les clients font leur achat indé-
pendamment les uns des autres. On sait donc que Ni <-+
j ~(n,pil-
L [IDY1=k(Y2=j-k)IJD(Y1 =k)
k=O En utilisant les formules habituelles :
n
+ [ 1JDv1=k(Y2=j-k)IJD(Y1=k)
k =j+l'--~~.,-~--'
=O

= t (~=
k=O J
k)pj-k(l - p)(n-k)-(j-k) x (n)pk(l - p)n-k
k k
• Ni + N j représente le nombre de clients qui achètent un
animal de l'espèce i ou j. Or la probabilité qu'un client
donné achète une de ces deux espèces est Pi+ p j. Comme
t (r~ - k)(n)pj(l-p)2n-j-k
k=O J - k k
les n clients font leur achat de manière indépendante, on
a Ni+ Nj <-+ ~(n, Pi+ p_; ).
Et d'après les formules habituelles:
·
pl(l-p) 2 n- 1· [ j (n._-k)(n) (1-p) -- k
k=o J k k

On remarque alors que :


• On a donc:

(n - k) (n) .1ft--fc'5f
j - k k = (n - j)!(j - k)!
n!
k[f.Jl.--fc)! X Yij! = (n)j (j)
k
l
2(V(N; + N _;)-V(N;) -V(N _;))
X

1
On obtient donc :
2(n(pi + Pj)(l-pi -pj)-npi(l -pi)
ifD(X= j) pj (l - p)2n-j t (~) ( j) (l - p)-k
k=O 1 k = - np;p_;
-npj(l - Pj ))

= pj(l - p)2n-j(~) t
1 k=O k
(j)(l - p)-k b) • Pour tout i e: [l, kil, on note Yi la variable aléatoire de
Bernoulli égale à 1 si l'espèce i n'a connu aucun achat et 0

p.Ï(l-p)2n-j(~) t
.l k=O k
(j)(~)k 1.i-k
1 p
sinon .
On remarque que [Yi = 1) = [N; = O]. On a donc: lfP(Y; =
1) = (1 - Pi ) 11 et donc, Y; suit la loi de Bernoulli de para-

'O = pl· (l - p) 2n- J· j (n) (1 + _1 P )j mètre (1- p;) 11 •


1
0
C: Ona:
::J
0 Pj (l - p)2n-j (;)(2- p)j (l - p)-j
lfl k
,--i
0
[ Y; nombre d'indices i pour lesquels Y; =1
N Finalement, pour tout j e: [O, nil : i=l
@ nombre d'espèces qui n'ont pas connu d'achat
....,
.c
Ol
IJD(X = j) = (;) (p(2 - p) )j ((1 - p)2) (n - jJ = X
·;::
>-
a. • On a donc, par linéarité de l'espérance:
0 Cela ressemble à une loi binômiale... On remarque en effet
u
que: 1 - p(2 - p) = 1 - 2p + p 2 = (l - p)2. On peut donc dire k k
que X<-+~( n, p(2- p)) . [(X)= L [(Yi)= L (l- p;)n
i =l i=l

278
Vecteurs aléatoires Chapitre li

Comme O < 1- Pi < l, on a lim (1- Pi) n = 0 et donc, par La formule des probabilités composées nous donne donc :
n- +oo
somme de limites :
lf"(X1 = k) = lf"(X1 t k)lf"x 1 ;t!k(X2 i k) x ···
lim IE(X) = O
n- +oo X If"n/:~[Xpik] (Xi- 1 i k) X If"nj:\ lX1#l (X; = k)

• On peut donner de ce résultat l'interprétation suivante: et donc:


pour un très grand nombre de clients, chaque espèce va
connaître au moins un achat, en moyenne. IP(Xi = k)
.w.-r .w--z
- - X - - X•• · X
l>!---r-Fr 1
X ---
N .N--r .N-r-FZ l>!---r-Fr
c) On a X(fl) = [O, kil et donc, par définition de l'espérance: 1
N
k k
IE(X) = L j lf"(X = j) =0 + L jlf"(X = j)
On a donc obtenu que X;'- Olt{[l,Nil).
j=O j=l
• Ona donc:
Mais sij E [1,kil, on a: N+l
IE(Xj) = - -
j;?; 1 donc j lf"(X = j) ;?; IP(X = j) 2
et donc, en additionnant ces inégalités : De plus, d'après le théorème de transfert:

r.. .i
k

j =l
lf"(X = j);?;
k
L lf"(X = j).
j =l IE(X7) = f k2 lf"(Xi = k) = ~ f k2 = (N + 1)(2N + 1)
k= l N k=l 6
Mais les évènenements [X= j l, j E [1; kil sont deux à deux
incompatibles, donc :
La formule de Koenig-Huyghens nous donne alors:

IE(X7)-(IE(X;)) 2
~ LJ
k lf"(X = j) = If" ( k [X= j] ) = IP(X;?; 1)
J =l

On obtient donc :
1=1
V(Xi)

(N+l)~2N+l) - (N;l r
N2 - l
IE(X) ;?; lf"(X;?; 1) ;?; 0
= 12

b) Soit (k1 , .. . , kn) E X1 (fl) x · · · x Xn(fl).


• Le théorème des gendarmes nous donne alors que n
lim lf"(X ;?; 1) existe et est égale à O.
n - +oo
I.;évènement n[Xi = kd correspond à l'évènement« au ti-
i= l
Mais comme lf"(X = 0) = 1 - lf"(X;?; 1) on a donc: rage 1 on a obtenu la boule numéro k1, ... , au tirage non a
obtenu la boule numéro kn ».
lim lf"(X=O) = 1
n-+oo Donc: Card ( fl [Xi =k; 1) = 1.
t= l
• On peut interpréter ce résultat de la façon suivante : pour Mais Je nombre de déroulements possibles est égal au
u un très grand nombre de clients on peut être certain qu'au- nombre d'arrangements de n boules parmi N (car les ti-
0 cune espèce ne connaîtra aucun achat.

-
C: rages sont effectués sans remise).
::J
0 On a donc:
lfl
,-i
0
N a) • On aXi(fl) = [1,Nil.
@
...., Soit k E [l,Nil. I.;évènement [Xi = k] correspond à l'évène-
.c ment « on a tiré la boule numéro k au ième tirage et une c) Pour tout i E [l, nil, on a: Yj(fl) c [1, Nil.
Ol
ï:::: boule d'un autre nwnéro aux tirages 1 à i - 1 ». Donc on a
>-
a. Soit (k1, ... , kn) E [1, Niln fixé.
0
la relation :
u S~ k1 < k2 < · ·· < kn n'est pas( ~~riflé alors):
n[Y; = ki] = ~.et donc: n
i= l
If"
i=l
[Yi = kj] = O.

2 79
Chapitre 11 Vecteurs aléatoires

n
Si k1 < k2 < · · · < kn est vérifié alors l'évènement n [Yi =
i=l
choisissent un étage au hasard et de façons indépend antes,
on a:
11
ki J correspond à l'évènement« les n tirages ont amené les
f(X = 1) = ( -1)11 + (1)
- = -1-
numéros k1,k2, ... ,kn ». Ces déroulements des n tirages 2 2 2n- l
sont au nombre de n! donc: Et donc:
1
n ) n! ,n( 1
f ( O [Yi = kj] = An = ....rfN) = (N)
1- l N n
/''l11
f(X = 2) = 1 - f(X = 1) = 1 - -
2n-l

Ainsi:
d) Au minimum Yj prend la valeur j (si on tire les nwnéros 1 X 1 2
à n), et au maximum Yj prend la valeur N - n + j (si on tire
f(X = X) l l
les numéros N - n + 1 à N). 211- I 1 - 2ri- i
Donc Yj prend toute valeur entière entre j et N - n + j: Calculons l'espérance de X:
Y1(!1) = [j,N - n+ j].
IE(X) = 1 x -1- + 2 x ( 1 - -1- ) = 2 - _ 1.-
Soit k e: V, N - n + jn. =
I.;évènement [Yj kl correspond à 211- l 211- l 2n- l
l'évènement« on obtenu (j - 1) numéros entre 1 et k-1, le
numéro k, et (n - j) numéros entre k + 1 et N ». Le nombre son moment d'ordre 2, grâce au théorème de transfert :
de déroulements réalisant cet évènement est : 2 2 1 2(
[(X ) = 1 x - - + 2 x 1 - - - = 4 - - -
2n-l
1)
2n-l 2n-l
3
At~ xAl xA~-=-{k+l) +l xC:l)x(n-(i-1)) et sa variance, grâce à la formule de Koenig-Huyghens:
choix des numéros . . . 2
cholX des trrages qui donnent 4 3 (2 1 ) 1 1
les numéros entre l et k- l V(X)= - 2n-1 - - 2n- 1 = 2n-1 - 22n- 2
et de celui qui donne le numéro k
• Si n = 2, il n'y a que deux personnes et donc encore une
On a donc: fois X(D) = {l, 2}.
I.;évènement [X= 1) correspond à l'union de p évènements
Aj- 1 An- j ( .n )(n- j+l)
k-1 N-k J - l 1 incompatibles:« les 2 personnes descendent à l'étage i »,
f(Yj = k) i e: [l,pil. Comme les deux personnes font leur choix de
manière indépendante :

= f(X = l) = ( t r +(if + .. · +(tr =px(if = i


p termes

Et donc : f(X = 2) = 1 - f(X = 1) = 1 - -f,.


Finalement, pour tout k e: [j, N - n + jll :
X 1 2
1 1 - lp
f(X = x) p
'O
0
C:
::J Calculons l'espérance de X:

-
0
lfl
,--i
1
IE(X)=lx -+2x ( 1- -1) =2--
1
0 P p p
N
@ a) • Si p = 2, l'immeuble n'a que deux étages. On a donc son moment d'ordre 2, grâce au théorème de transfert:
...., X(û) =11, 2}.
.c
Ol
·;:: I.:évènement [X= l i correspond à l'évènement« les n per- IE(X2) = 12 x ~p+ 22 x (1 - p
~) = 4 - ~p
>-
a. sonnes descendent au même étage». [X = 1) est donc
0 l'union de deux évènements incompatibles: « les n per- et sa variance, grâce à la formule de Koenig-Huyghens:
u
sonnes descendent au premier étage» et « les n personnes 2
descendent au second étage ». Comme les n personnes 3 (
V(X) = 4 - p - 2-p1 ) p- 1
=y

280
Vecteu rs aléatoires Chapitre li

b) • Yij ne prend que deux valeurs: 0 ou 1. Elle suit donc une e) On a vu que :
loi de Bernoulli de paramètre fP(Yij = 1).
[(X)= p ( 1-(1- ~) n)
D'après l'énoncé: IP'CYij = 1) = 75.
Et par définition de l'espérance :
DoncYij "-[J(J( ).
75 m in(n,p)
• X; ne prend que deux valeurs : 0 ou 1. Elle suit donc une [(X) = L kfP(X =k)
loi de Bernoulli de paramètre IP(Xi = 1). k=l
min(n.,p) (P)sk
L évènement IXi = O) correspond à l'évènement « les n per- I: k..!.......!!.
11
sonnes ne descendent pas à l'étage i ». Or chaque per- k=l P
sonne ne descend pas à l'étage i avec probabilité 1 - , et min(n,p) (p- l)sk
75 , k-1 n
elles agissent de manière indépendante, donc : = L,
k=I
p n
p .

( pl)n. et donc IP'(X; = 1) = 1 - (1 - pl)n.


IP'(X; = 0) = 1 - _ l _ min(n,p) (p - l)sk
p
n-1 L
k=l
k- 1 n

Ainsi: Xi <-->[J(J{1-(1-i )'l Comme les deux calculs doivent donner Je même résultat :
c) • On a

-
nombre d'indices i pour lesquels Xi = 1

nombre d'arrêts de l'ascenceur


X a) f est une fonction polynôme. Elle est donc dérivable sur
[O, l] et:
• Donc par linéarité de l'espérance: 'ri x E (0, 11, f' (x) = 2px
On a: 'r/0 < x::: l, f'(x) > 0 et donc, par théorème, f est
strictement croissante sur [O, l).
b) • Comme f est strictement croissante sur [O, l] :
d) • Au minimum, X prend la valeur 1 (toutes les personnes f([O, l i) = If (0)./(1)) = 11 - p, ll c (0, 1).
descendent au même étage). Une récurrence immédiate donne alors que (u 11 )ne:N est
Au maximum, X prend la valeur rnin(n, p) : (n si n ::: p et bien définie et à valeurs dans (0, JJ.
toutes les personnes descendent à un étage différent, psi • Ona:
p < net au moins une personne descend à chaque étage).
u1 - uo = p(l - p) 2 + (1 - p) - (1 - p) = p(l - p)2 2: 0
X prend donc toute valeur entière entre 1 et min(n, p) :
X(fl) = [1,min(n,p)Il.
Comme f est croissante sur (0, li, une récurrence immé-
• Soit k E [l,min(n,p)]. diate donne que (unlne:N est croissante.
u • La suite (u 11 ) 11e:N est donc croissante majorée (elle est à
0
C:
I.:évènement [X= kl correspond à l'évènement« les n per-
::J sonnes sont descendues à k étages différents». Au total: valeurs dans [0, 11). D'après le théorème de la limite mono-
0
il y a p 11 déroulements possibles (chaque personne choi- tone, elle converge vers un réel e.
lfl
,-i sit un étage parmi p). Le nombre de déroulements réali- Comme f est continue sur [O, l]. un passage à la limite
0
N sants [X = kl est égal à (f)S~: on choisit les k étages, puis quand n - +oo dans la relation: Un + l = /(un) nous
@
....,
compter le nombre de déroulements pour lesquels les n e
donne : /(€) = €, c'est-à-dire que est un point fixe de/.
.c personnes descendent à ces k étages ,revient à compter le
Mais on a:
Ol
·;:: nombre de surjections de l'ensemble des n personnes sur
>-
a.
l'ensemble des k étages. f (X)= X = px2 + (1- p) = X
0
u Ainsi:
(P)sk
= px2 - x + l - p = O
1- p
fP(X =k) =..!.......!!.
pn. = x= 1 oux= - -
p

281
Chapitre 11 Vecteurs aléatoires

On a toujours: 1 e: (0, 1]. Par contre : Mais: 1Jl>Xo =o(X11 +J = O) = l , carsilapremière plante n'a au-
1-p l cun descendant alors toutes les générations suivantes sont
- - E (0, 1) <==> 1 - p S p <==> p:::: - d'effectif nul.
p 2
* Si p s !,fa un unique point fixe: 1. Et: l?Xo=2CXn+1 = 0) =l?CXn = 0) 2. En effet, sachant que la
plante« mère » a eu deux descendants, on calcule la proba-
X 0 1 bilité qu'elle n'ait pas de descendant de (n + 2lme géné-
1- p ration, c'est-à-dire que chacun de ses deux« enfants » n'ait
Variations de pas de descendant de (n + l)ème génération: cela se pro-
f(x) - x duit pour chacun avec la probabilité IP(X 11 =0) et leur deux
~o descendances évoluent de manière indépendante.

Donc (un)neN converge vers 1. On obtient donc :

* Si p > ! , 1
fa deux points fixes: ;P < 1 et 1.
l?CXn+l = 0) = (1 - p) + p(l?CXn = 0) ) 2
1-p 1
X 0 p 'Ip 1

Variations d e
f (x ) - X
1- p
---o ---1-p-i, __..o d) Ainsi:

1
lfl>CXn+ l =0) = f (lfl>CXn = 0))
Mais U<J =1- p < ;P et par récurrence immédiate (en uti-
lisant que f est strictement croissante) :
• Comme on a U<J = lfl>(Xo = 0) et pour tout n EN :
1-p
v'nEN, U 11 < -- = 0) = f (lfl>(Xn = 0)), une récurrence immédiate
IP'CXn+ 1
p nous donne:
et en passant à la limite quand n - +oo: es 1
;P < 1.
Ainsi : e = l ;p . Vn e: N, lfl>(Xn = O) = Un
1-p
Dans ce cas (u 11 ) n eN converge vers P.
c) • La variable aléatoire Xo représente le nombre de descen- On peut donc utiliser les résultats du b) :
dants de la plante de départ. Elle ne peut donc prendre
que deux valeurs : 2 avec probabilité p et O avec probabi- • Si p s !, on a: n-+oo
lim lfl>(X 11 = 0) = l. C'est-à-dire qu'au
lité 1 - p. bout d' un très grand nombre de générations, la population
Donc: IP(Xo = 0) = 1- p = U<J. va s'éteindre.
• Soit n e: N. La formule des probabilité totales, appliquée
avec le système complet d'évènements (IXo = O], lXo = 21), • Si p > !, on a: n-+oo
lim 1Jl>(X11 = 0) = l - p. C'est-à-dire
p
nous donne: qu'au bout d'un très grand nombre de générations, l'ex-
lfl>D<n+l =0) IP(Xo = O)[P>Xo =o(Xn+l = 0) tinction est possible mais pas certaine.

'O
+IP(Xo =2)1Jl>Xo=2 (X 11 +1 =0) Ce résultat n'est pas conforme à l'intuitition car on pourrait
0 = (1 - p)lfl>x 0 =o(Xn+l = 0)
C: penser que si cette espèce a tendance à ne pas avoir de des-
::J
0 +p1Px 0 =2CX11+ 1 =0) cendance, elle va s'éteindre avec probabilité 1...
lfl
,--i
0
N
@
....,
.c
Ol
·;::
>-
a.
0
u

2 82
CHAPITRE

Géo1nétrie

'Tlièmes a6ordés dans {es e~ercices


- Géométrie dans le plan
- Géométrie dans l'espace

Points essentiefs du cours pour {a réso{ution


des e~ercices
- Coordonnées dans un repère cartésien
- Produit scalaire
- Orthogonalité
- Vecteurs normaux
- Projection orthogonale
- Mesure des angles
u
0 - Distance de deux objets géométriques
C:
::J
0 - Produit vectoriel
lfl
,-i - Droites
0
N - Plans
@
...., - Cercles
.c
Ol
·;::
>-
a.
0
u

283
Chapitre 12 Géométrie

Les méthodes à retenir

- On a la relation de Chasles, pour A, B et C trois points du plan :


AB +OC =AC:. De plus: BA= -AB etM= Ô.
<---+ Exercices 12.1, 12.3, 12.14 et 12.16

- Dans un repère orthonormé, si A(xA; YA), B(xB; J'B), (x; y) et u


Pour faire du calcul vectoriel dans le v(x';y') alors: AB(XB - XA;YB - YA), u.v = xx' + yy' et l ü l =
plan Jx2+y2=~.
<---+ Exercices 12.1, 12.2, 12.3, 12.14 et 12.16

On a: l ïI + v l = l ïill2 + l v l 2 +2Û.v. Donc si û et v sont


2
-
orthogonaux : Il Û + v 2 = Û 2 + v 2 (théorème de Pythagore) .
11 Il 11 Il 11

<---+ Exercices 12.2, 12.3, 12.14 et 12.16

-
- -o.
Une droite peut être définie par un point A et un vecteur directeur
u #
<---+ Exercice 12.4

- Dans le plan muni d'un repère orthonormé: l'ensemble des


M(x; y) tels que ax + by + c = 0 avec (a, b) # (0, 0), est une droite
dirigée par Û( - b; a) dont un vecteur normal est ri (a; b) .
<---+ Exercices 12.4 et 12.8

- Dans le plan muni d'un repère orthonormé: l'ensemble des

M(x;y) tels que 3t e: IIV


X = À + at
avec (a,~)# (O,O), est
{
y
µ+~t =
la droite passant par A(,\;µ), dirigée par Û(a;~) dont un vecteur
normal est n(- ~;a).
<---+ Exercice 12.4
Pour étudier une droite dans le plan
- Si~ est la droite passant par A et dirigée par û, alors M appartient
à ~ si et seulement si AM et û sont colinéaires. On obtient ainsi
une représentation paramétrique de~-
<---+ Exercice 12.4
"O
0
C:
::J - Si~ est la droite passant par A et dont un vecteur normal est n,
0
alors M appartient à ~ si et seulement si AM et n sont orthogo-
lfl
,-i
0
naux. On obtient ainsi une équation cartésienne de~-
N <---+ Exercice 12.4
@
...., - Si ~ 1 et ~ 2 sont deux droites du plan dirigées respectivement par
.c
Ol
ï:::: v,
û et alors une mesure de l'angle formé par ces deux droites est
>-
0
a. l' umque
. , l a e: [0, 1T .l te1que.. cos(a) --
ree 2 l u1uli .ïil v1 .
u 1
<---+ Exercice 12.4

284
Géomét rie Chapitre 12

- Un cercle peut être définie par son centre n et son rayon R > O.
<.......+ Exercice 12.5

- Dans le plan muni d'un repère orthonormé: l'ensemble des


M(x; y) tels que (x - a) 2 + (y - b) 2 = R2 avec R > O, est le cercle
de centre ü(a; b) et de rayon R.
<.......+ Exercices 12.5, 12.8 et 12.16

- Dans le plan muni d'un repère orthonormé : l'ensemble des


Pour étudier un cercle dans le plan x = À+ Rcos(t)
M(x;y) tels que 3t E IIV
{ y = µ + Rsin(t) avec R > 0, est le
cercle de centre ü(,\; µ) et de rayon R.
<.......+ Exercice 12.5

- Si ~ est le cercle de centre n et de rayon R, alors M appartient


à~ si et seulement si IDMII = R. On obtient ainsi une équation
cartésienne de ~ .
<.......+ Exercices 12.5 et 12.16

- Si M est un point du plan et ~ une droite, dirigée par û et de vec-


teur normal n,alors le projeté orthogonal de M sur~ est l'unique
point H de~ tel que MH et û sont orthogonaux, ou encore tel que
MA et ri sont colinéaires.
<.......+ Exercice 12.4

- Si M est un point du plan et ~ une droite, alors la distance de M à


~ est d onnée par : d(M, ~) = Il MH Il où H est le projeté orthogonal
Pour utiliser la projection orthogonale deMsur~.
sur une droite dans le plan <.......+ Exercice 12.4

- Dans le plan muni d'un repère orthonormé, si ~: ax + by + c = 0


et M(~1;.Y1',1), alors:
u
0
C: d M ~ = laxtv1+h}'M+cl
::J (') Ja2 + b2
0
lfl
,-i
0
<.......+ Exercice 12.4
N
@
....,
.c
Ol
ï::::
>-
a.
0
u

285
Chapitre 12 Géométrie

- On a la relation de Chasles, pour A, B et C trois points de l'espace :


AB + OC =AC. De plus: BA= - AB etM= Ô .
<---+ Exercices 12.6, 12.9, 12.11 et 12.17

- Dans un repère orthonormé, si A(xA;J A ; ZA), B(xB;YB; ZB),


Ü (x; y; z) et ïi (x'; y'; z') alors : AB(XB - XA; YB - JA; ZB - ZA),
ü.ïi = xx' + yy' + zz' et l ï1 1 = Jx 2 + y 2 + z 2 = Jü.ü.
<---+ Exercices 12.6, 12.9 et 12.11
2
Pour faire du calcul vectoriel dans
- On a: l ï1 + ïi l = lï1112 + l ïi l 2 + 2Ü .v. Donc si Ü et v sont
2 2 2
l'es pace
orthogonaux: Il Ü + v 11 = Il Ü 11 + Il v 11 (th éorème de Pythagore) .

<---+ Exercices 12.6, 12.9 et 12.11

- Dans un repère orth on ormé direct, si Ü (x; y; z) et ïi (x'; y'; z')


alors leur produit vectoriel ü /\ ïi a pour coordonnées (yz' -
zy'; zx' - xz'; xy' - x' y).
<---+ Exercices 12.13 et 12.17

- Ü /\ v est orthogonal à Ü et à v. De plus : ü et v sont colinéaires


si et seulement si Ü /\ v = Ô.
<---+ Exercices 12.13 et 12.17

- Un plan peut être défini par un point A et deux vecteurs généra-


teurs Ü et ïi (non colinéaires). Un vecteur normal est = Ü /\ ïi . n
<---+ Exercices 12.6, 12.9 et 12.13

- Dans l'espace muni d'un repère orthonormé: l'ensemble des


M(x; y; z) tels que ax + by + cz + d = O avec (a, b, c) t (O, 0, O), est
un plan dont un vecteur normal est (a; b;C). n
<---+ Exercices 12.6, 12.9, 12.10 et 12.11

- Dans l'espace plan mun i d'un rep ère orthonormé : l'ensemble


X - À+ 0:1 S + 0:2 t
"O Pour étudier un plan dans l'espace
0
C:
des M(x;y;z) telsque3(s,t)e: ~ 2/ y = µ+ l3 1s+l32t avec
::J {
0 Z = v+ns + y2t
lfl
,-i (0:1,131, Yi) et (0:2, 132, Y2) non proportionnels, est le plan passant
0
N par A(À; µ;v), engendré par Ü (o:1;131; yi) et (o:2;132; Y2). v
@ <---+ Exercices 12.6 et 12.9
....,
.c
Ol - Si [ïJJ est le plan passant par A et engendré par Ü et ïi, alors M
ï::::
>-
a. appartient à [ïJJ si et seulement si AM est comb inaison linéaire de
0
u Ü et v. On obtient ainsi une rep résentation paramétrique d e [iJJ.
<---+ Exercices 12.6 et 12.9

286
Géométrie Chapitre 12

- Si 9 est le plan passant par A et dont un vecteur normal est ri,


alors M appartient à 9 si et seulement si AM et n sont orthogo-
naux. On obtient ainsi une équation cartésienne de 9.
~ Exercices 12.6, 12.9, 12.11, 12.12 et 12.13

- Si 91 et 92 sont deux plans de vecteurs normaux respectifs ni


et n 2 , alors une mesure de l'angle formé par ces deux plans est
, · [ n] .
I unique réel ex E 0, 2 . tel que. cos(cx) -
_ Il nilni-îiil
li li ni .
11
~ Exercices 12.6 et 12.ll

- Une droite peut être définie par un point A et un vecteur directeur


û # Ô, ou comme l'intersection de deux plans non parallèles.
~ Exercices 12.6, 12.7, 12.9, 12.10, 12.ll et 12.13

- Dans l'espace muni d'un repère orthonormé direct: l'en-

) que a1x+b1y+c1z+d1 = 0
semble des M (x; y; z) tes
{
a2x +b2y +c2z +d2 = 0
avec (a1,b 1,ci) et (a2,b2,c2) non proportionnels, est une droite
n
dirigée par = (a1; b1; ci)/\ (a2; b2; C2).
~ Exercices 12.6, 12.7, 12.9, 12.10, 12.ll et 12.13

- Dans l'espace muni d'un repère orthonormé : l'ensemble des


Pour étudier une droite dans l'espace
M(x;y;z) tels que :lt E ~/ { : : ::;; avec (<X,~,y) #
z = v+yt
(0, 0, 0), est la droite passant par A(À; µ; v) et dirigée par û (a ; ~; y).
~ Exercices 12.10 et 12.13

- Si~ est la droite passant par A et dirigée par û, alors M appartient


à~ si et seulement si AM et û sont colinéaires (on obtient ainsi
u
0
une représentation paramétrique de ~) si et seulement si AM A
C:
::J û = Ô (on obtient ainsi un système d'équations cartésiennes de
0 ~).
lfl
,-i
0
~ Exercices 12.10, 12.l l et 12.13
N
@
....,
.c
Ol
·;::
>-
a.
0
u

287
Chapitre 12 Géométrie

- Si M est un point de l'espace et [!J) un plan, de vecteur normal n,


alors le projeté orthogonal de M sur [!J) est l'unique point H de [!J)
tel que MH et ri sont colinéaires.
<.......+ Exercice 12.12

- Si M est un point de l'espace et [!J) un plan, alors la distance de M


à [!J) est donnée par : d(M, [!J)) = IlMH Il où H est le projeté orthogo-
Pour utiliser la projection orthogonale nal de M sur [!J).
sur un plan dans l'espace <.......+ Exercice 12.12

- Dans l'espace muni d'un repère orthonormé, si [!J) : ax + by + cz +


d =0 et M(XM; JM; ZM), alors:

d(M,Si') = laxM + byM + CZM + dl


Ja2 + b2 +c2

<.......+ Exercice 12.12

u
0
C:
::J
0
lfl
,-i
0
N
@
....,
.c
Ol
·;::
>-
a.
0
u

288
Géomét rie Chapitre 12

Énoncés des exercices

---- - Aire d'un triangle


Le plan est muni d'un repère orthonormé (O; 7,
sommetsA(O;l), B(l;O) etC(-1;0).
Calculer l'aire de ABC.
7). On considère le triangle ABC de

---- - Une caractérisation des rectangles


Soient Ü et
a)
v deux vecteurs du plan.
Établir que: Ü et v sont orthogonaux<==> llû + v 1
1=llû - v 11-
b) En déduire une condition nécessaire et suffisante pour qu'un parallélogramme
ABCD soit un rectangle.

---- - Orthocentre d'un triangle


On considère trois points A, B et C formant un triangle non plat du plan (i.e. que AB
et AC ne sont pas colinéaires).
a) Vérifier que, si M est un point quelconque du plan :

-- -- ~-
MA.BC + MB.CA + MC.AB = 0

b) On note H le point d'intersection des hauteurs issues de B et de C. Montrer que:


HA.Be = O. En déduire que les trois hauteurs du triangle sont sécantes en H.

u
0
C:
---- - Droites dans le plan
Le plan est muni d'un repère orthonormé direct (O; 7, 7).
a) Déterminer le point d'intersection de la droite
1
@1 de représentation paramé-
::J
0 trique { xy + t avec la droite ~2 passant par O et dirigée par Ü (I; 2). ~l
lfl 2- t
,-i
0 et 0:>2 sont-elles paraUèles? orthogonales? Donner une mesure de l'angle formé
N par ces deux droites.
@
...., =
b) Soit@ la droite d'équation cartésienne: x + y - 2 O. Donner un point de cette
.c droite, un vecteur directeur et un vecteur normal. Déterminer une représentation
Ol
·;:: paramétrique de@.
>-
a.
0 c) Soit!!,. la droite d'équation cartésienne: 2x- y+ 1 = O. Déterminer la distance du
u point M(2; 1) à la droite!!,.. Donner ensuite les coordonnées du projeté orthogonal
HdeMsurt!,..

289
Chapitre 12 Géométrie

---- - Cercles dans le plan


Le plan est muni d'un repère orthonormé (0; 7, 7).
a) Déterminer, en fonction de m E ~'l'ensemble$ des points M(x;y) du plan tels
=
que: x 2 + y 2 - 4x+ y+m O. Dans le cas d'un cercle, en donner une représentation
paramétrique.
b) Soit<(; le cercle de centre A(3;0) et de rayon 1. Déterminer ses éventuels points
d'intersection avec la droite~ d'équation cartésienne : x - y+ 1 =O.
c) Pour m > 0, on note <(;m le cercle d'équation cartésienne: x 2 + (y - 4) 2 m. Déter- =
miner les éventuels points d'intersection de <(;m avec <~, en fonction des valeurs
dem.

---- - Droites et plans dans l'espace


r.: espace est muni d'un repère orthonormé direct (O; 7, 7, k).
=
a) Soit f?l> le plan d'équation cartésienne: x + y+ z 2. Donner un point de ce plan,
un vecteur normal, une famille génératrice et déterminer une représentation pa-
ramétrique de f?l>. Déterminer la distance du point M(l; 2; 1) à f?l>, puis donner les
coordonnées de son projeté orthogonal H sur f?l>.
b) Soit .0'>1 le plan passant par les points A(l; 1;0), B(O; 1;2) et C(1;3; - 2). Soit .01>2 le

l
X s+t
plan de représentation paramétrique yz s- t . Ces plans sont-ils
1 +2s+ t
parallèles? orthogonaux? Tvlontrer que leur intersection est une droite !Z) dont
on donnera un système d'équations cartésiennes ainsi qu'un vecteur directeur.
Donner une mesure de l'angle formé par les plans f?l>1 et f?l>2.

---- - Droites coplanaires


I.:espace est muni d'un repère orthonormé (O; 7, j, k). Donner une condition né-
cessaire et suffisante sur a E Ili pour que les droites suivantes soient coplanaires:

@ J x - az+ 1 0
et
@' J x - z+2 0
l y - 2z - 3 0 l y - 3z+ 1 0

u
0

0
C:
::J

lfl
,-i
0
N
---- - Droites tangentes à un cercle
Le plan est muni d'un repère orthonormé (0; 7,
nées (- 1;0).
7). On note A le point de coordon-
a) Déterminer l'ensemble<(; des points M(x; y) du plan tels que: x2 + y2 -4x- 2y+
@ 4=0.
....,
.c b) Pour m E ~. on note @m la droite passant par A et de coefficient directeur m .
Ol
·;:: Déterminer, suivant les valeurs de m, le nombre de points d'intersection de !Z)m
>-
a. avec<(;.
0
u c) Donner l'équation des éventuelles tangentes à<(; passant par A ainsi que les co-
ordonnées de leur point d'intersection avec<(;.

290
Géomét rie Chapitre 12

---- - Famille de plans dans l'espace dépendants d'un paramètre (d'après Agro-
Véto 2007)
I.:espace est muni d'un repère orthonormé direct (O; 7.
pointsA(O;l; - 1), B(- 1;1; - 1) etC(0;2; - 2).
7. k). On considère les trois
a) Montrer que ces trois points définissent un plan I!2, dont on donnera une équation
cartésienne.
b) Soit me: Ili. Déterminer, en fonction de m, un vecteur normal au plan &>m d'équa-
tion: x - my+ mz 1. =
c) On note@ la droite passant par O et dirigée par Û (l; l; 1). Montrer qu'il n'existe
pas de valeur de m, pour laquelle@ serait orthogonal à &>m. Pour quelles valeurs
de m, la droite@ est-elle parallèle à fY>m?
d) On note &lm le plan contenant@ et perpendiculaire à &>m. Pour tout me: IR?, mon-
trer que les trois plans fY>m, ~ et &lm sont sécants en un point lm dont on déter-

---- -
minera les coordonnées.

Calculs de représentation paramétrique de droites dans l'espace (d'après


Agro-Véto 2007)
I.:espace est muni d'un repère orthonormé direct (0; 7, 7, k). On considère le point
X 1 +2À
A(l;2;3), la droite@ de représentation paramétrique yz 3À et les plans

&>1 et &>2 d'équation cartésienne respective: x +y+ z


1= = - 1 +À
3 et 2x - y+ z =2.
a) Trouver le plan fY>{ parallèle à f:1>1 et passant par A. Déterminer la distance de A à
f:1>1.
b) On note 6. l'intersection des plans &>1 et &>2. Déterminer une représentation pa-
ramétrique de la droite 6 .
c) Donner une représentation paramétrique de la droite 6. 1 parallèle à 6., contenue

---- -
dans &>1 et sécante avec@.

Quatre points définissent deux plans... (d'après Agro-Véto 2008)


I.:espace est muni d'un repère orthonormé direct (O; 7. 7. Î). On considère les
quatre points A(l; l; 1), B(0;2;2), C(-1;2;0) et D(0;-3; 1).
a) Montrer que les points A, B et C définissent un plan f!1> dont on donnera une
équation cartésienne.
b) Donner l'équation paramétrique de la droite passant par D et orthogonale à&>.
u
0 c) Déterminer les coordonnées du point D' symétrique de D par rapport à f:1>.
C:
::J
d) Calculer l'angle des deux plans (ABC) et (ABD).

---- -
0
lfl
,-i
0
N Inéquations définissant un tétraèdre (d'après Agro-Véto 2007)
@
...., I.:espace est muni d'un repère orthonormé direct (O; 7. 7. Î). On considère les trois
.c pointsA(3;0;0), B(0;2;0) et S(0;0;4) .
Ol
·;:: a) Donner l'équation cartésienne du plan SAB.
>-
a. b) Déterminer la distance du point O à ce plan.
0
u
c) Donner un système d'inéquations que doivent respecter les coordonnées de
M(x; y; z), pour être strictement à l'intérieur du tétraèdre SOAB.

291
Chapitre 12 Géométrie

---- - Intersection de droites et de plans dans l'espace (d'après Agro-Véto 2005)


J,
I.:espace est muni d'un repère orthonormé direct (O; 7. Î). On considère les
droites@ 1 = {(3+ t, -2-t, l + t)/ t E [ij} et @2 = {(x,y,z) E [ij 3/ x+2y+z = -x+2y-z =
o}.
a) Donner un point A1 de@1 et un vecteur directeur iii de @1. De même donner A2
et Vz pour @2.
b) Déterminer un vecteur Ü orthogonal à iii et Vz.
c) Pour i E {l, 2}, on note ~i le plan passant par A; et parallèle au plan engendré par
Ü et Vf. Montrer que 02'1 n 02'2 est une droite@, que@ et @1 sont sécantes ainsi
que@ et @2, et que@ est perpendiculaire à@1 et @2.

---- - Un peu de mouvement... (d'après Agro-Véto 2008)


On considère un carré ABCD tel que: (AB,AD) = ; (2n). Si M est un point de la dia-
gonale [AC). on note P (respectivement Q) son projeté orthogonal sur [AB) (respecti-
vement [BC)).
a) On note I le milieu du segment [PQJ. Déterminer l'ensemble décrit par I lorsque
M décrit IACJ. et en donner une équation cartésienne dans le repère (A, AB. AD).
b) Donner une équation cartésienne de la médiatrice de [PQ] dans le repère
(A.AB.AD).
c) Vérifier que, lorsque M décrit [AC). les médiatrices des segments [PQJ sont sé-
cantes en un même p oint.

u
0

0
C:
::J
---- - Formule du double produit vectoriel
r.: espace est muni d'un repère orthonormé direct (O; 7, j, Î ). On pose IE =[ij3 et on
se donne Ü, v et wtrois vecteurs de IE.
a) A-t-on: Ü /\ (v /\ w) =(Ü /\ v) /\ w?
lfl
,-i
0
b) Ecrire la matrice dans la base canonique de IE des applications linéaires sui-
N vantes:
@
....,
.c t: x E IE - (ü.x)ü -
g:xEIE - (- -
u.v)x -
Ol
·;:: hv : X E IE .......... Ü /\ X s: x E IE- ü /\ (ü /\ x)
>-
a.
0
u En déduire que pour tout XE (E: û /\ (ü /\ x) =(û .x)v - (û .v)x.
C) A quelle condition a-t-on Û /\ (v /\ w) = (u /\ v) /\ w?

292
Géomét rie Chapitre 12

---- - Distance d'un point à un disque (d'après Agro-Véto 2006)


Le plan est muni d'un repère orthonormé (O;

notée d(M,D), par : d(M,D) = inf 1 ~ 11-


i, J).
On note D le disque de centre
0 et de rayon 1. Si M est un point du plan, on définit la distance de M au disque D,

M' e:D
a) On suppose que M (x; y) est un point extérieur à D. On note alors Mo le point d'in-
tersection du cercle de centre O et de rayon 1 avec la demi-droite [OM). Établir
que: d(M, D) = 11~11-En déduire une expression de d(NI, D) en fonction de x
ety.
b) On note t!,. la droite d'équation y = - 2. Déterminer l'ensemble if: des points M tels
que d(NI, D) = 2d(M, t!,.). Le représenter graphiquement.

---- - Produit vectoriel et tétraèdre (inspiré de Agro-Véto 2005)


Le plan est muni d'un repère orthonormé direct (O; 7, j, k). On considère un tétra-
èdre ABCD. On pose :

s;
a) Vérifier que : s"; + s"; + = ~-
b) On suppose dans cette question que le tétraèdre est rectangle en A. Vérifier que
le carré de l'aire du triangle BCD est égale à la somme des carrés des aires des
triangles ABC, ABD et ACD (théorème de Descartes).
c) Soient (a, 13, y) e: ~ 3 tel que : 0 <a< 13 < y :S l. On définit les points M1, M2 et M3
par:

---
et on pose: T = M1M2 AM1M3.
Exprimer T comme une combinaison linéaire des";, s"; et~. et en déduire que:

1r 11s 1:51:54
m!U( Ils; Il

(On remarquera que: '<l(a, b,c) e: ~ 3 , a(b+ c) - be = a 2 - (a - b)(a - c))


d) Donner une interprétation géométrique du résultat de la question précédente.

u
0
C:
::J
0
lfl
,-i
0
N
@
....,
.c
Ol
·;::
>-
a.
0
u

293
Chapitre 12 Géométrie

Du mal à démarrer ?

ID Niveau collège. CIIII) a) Déterm iner un vecteur normal à 91!1.


b) Utiliser u ne résol ution de système à partir des éq ua ti ons carté-
IEII a) Utiliser les iden tités remarquables du produit scalaire.
sien nes.
c) Déterminer un vecteur direc teur de e/.
b) u + v et ü - v représen tent les d iagonales du paral lélogramme.
1111 a) Utiliser la relation de C hasles.
mD a) Calcu ler ABAAC:.
b) Déterm iner un vecteu r di recteur de cette droite.
b) Parti r de la relation du a) avec M = H. c) Déterminer les coordonnées de M projeté ort hogonal de D sur
91!.
ID a) Déterminer une représentation paramét rique de ~2- d) Utiliser les vec teu rs normaux.
b) Résoud re l'équation cartésienne définissant ~-
c) Utiliser la formu le du cou rs, puis remarquer que
vecteur normal à~.
HM est un ma a) Calcu ler SAASB.
b) Uti liser la form u le d u cours.
c) Si SAB a pour vecteur normal ri, il coupe l'espace en deux demi -
1111 a) Mettre les trinômes du second degré sous leur forme ca- espaces selon le signe de SM· ri.
non ique.
b) et c) Uti liser une résolut ion de système à partir des équations cm, a) Facile.
cartésiennes. b) Penser au prod uit vectoriel.
c) Uti liser une résol ution de système à part ir des éq ua tions carté-
1111 a) Résoudre l'équation cartésien ne.
siennes.

b) Déterm iner l'équation cartésienne de 91!1 et de 9i'z.


cm, a) Donner u ne représentation paramétrique de cet ensembl e
en notant M(t; t), t € [O, 1).
. . Deux d roites non parallèles sont coplanaires si et seulement
b) Ut iliser le paramét rage du a).
si elles sont sécantes. c) Déterminer l'i ntersection des médiatrices obtenues pou r M = A
e t M=C.
Ill a) Mettre les trinômes d u second degré sous leur forme ca-
non ique.
b) Util iser une résolu ti on de système à partir des équations carté-
CIII a) Vér ifier avec les vecteurs 7, j et k.
b) Remarquer que 0 = hu o hv.
siennes. c) Utiliser le b).
c) Reprendre les calculs du b).

. . a) Calculer AB A AC.
ml a) Déterm iner les coordon nées de Mo.
b) Commencer par don ner une équation cartésienne de /J.
b) Facile.
c) Déterm iner un vecteur normal à~-
d) Uti liser une résolution de système à parti r des équations carté- mD a) Utiliser la relation de Chasles.
siennes. b) et c) Utiliser le a).
d) ! ll'f 11 représente l'aire du triangle M1M2M3.
u
0
C:
::J
0
lfl
,-i
0
N
@
....,
.c
Ol
·;::
>-
a.
0
u

294

www.bibliomath.com
Géométrie Chapitre 12

Corrigés des exercices

- Notons d l'aire cherchée.


On peut commencer par utiliser des outils de Collège...
A
b) Considérons un parallélogramme formé à partir de deux
vecteurs non colinéaires :

I
I
I I
C 0 B
/
I I

I.;aire de ABC vaut deux fois l'aire de AOB : I ------------

l
d =2x - x OA x OB= 1
2
Alors IlÜ + Ü Il et IlÜ - Ü Il sont les longueurs des diago-
Dans des cas plus compliqués, il pourra être utile d'utiliser nales.
d'autres outils si on travaille dans un repère orthonormé.
Appelons f) une mesure de l'angle géométrique ACB. De plus le parallélogramme ci-dessus est un rectangle si, et
Alors d = ! CB x CAsin9 seulement si, Ü .l Ü .
Si on« gonfle » la figure en dimension 3 en rajoutant une troi-
sième coordonnée nulle à chaque point. Ajnsi un parallélogramme est un rectangle si, et seulement
Dans !CB x CAsin8 on retrouve la moitié de la norme de si, ses diagonales sont isométriques.
CBACA.

.çl/ = ~ 11a" ü ll
Avec CB(2;0;0) et CA(l; l;O) on obtient CB A CA(O; 0;2) et if!+
1
d =-x2=1
2
a) Utilisons astucieusement la relation de Chasles :

u
if!i
0 a) Notons (E) : ll ü + Ü Il = ll ü - Ü Il.
C:
::J
0
Les normes étant positives : MÂ-oc +MË-û +Mê-AË
lfl
,-i 2 = MÂ-Bê +(MÂ+AË)-â+(MÂ +Aê)-Afi
0 (E) <=> llü + vll2 = llü - v ll
N
MA· (Bê+û +AË) +AË-û+Aê-AË
@ On obtient alors successivement :
...., MÂ-o +AË-CÂ.-CÂ.-AË
.c
Ol
ï:::: 0
>- 2 2
0
a. (E) llü ll + llvll2 +2ü. v = llü ll + llvll2 -2ü. v
u (E) 4Ü ·Û = O
(E) <=> Ü.L ü b) Utilisons la formule précédente en prenant M = H.

295

www.bibliomath.com
Chapitre 12 Géométrie

Notons ex l'angle géométrique formé par les droites ~ 1 et


- - ~~
Or HB.LCA donc HB ·CA= O.
~2-

De même HC.l.AB donc HC ·AB 0 = Les droites étant sécantes, elles ne sont pas parallèles.
On en déduit que HA·Ëê = 0 et donc que (HA) est la hau-
teur issue de A. D'après la représentation paramétrique de ~l donnée
Le point H appartient donc aux trois hauteurs du triangle v
dans l'énoncé, le vecteur de coordonnées (l;-1) est un
vecteur directeur de !?tl1 .

-
qui sont donc sécantes en ce point.
v
Or Ü · = 1 - 2 = - 1 i O (donc les droites ne sont pas or-
thogonales).
a) Commençons par déterminer une représentation paramé-
trique de !?tl2 :
De plus llü Il=V5 et llïi Il =Jz.
M(x,y)e: ~2 ~ 3ke:~.OM=kÛ On sait donc que :

" {:::k kcO cos(a) =


IÛ·vl 1
l üllllvll v'10
Un point M appartient à l'intersection des deux droites si,
et seulement si, il existe deux réels k et t tels que : Ainsi, l'angle formé par ces deux droites mesure
arccos( Jio).
"O J k=l+t b) Pour obtenir un point de la droite, on fixe une valeur quel-
0
C: 1 2k = 2 - t
conque de l'abscisse x et on calcule l'ordonnée y corres-
::J
0 pondante. Par exemple, pour x = 0 on obtient y = 2 donc
lfl
On obtient un unique couple solution : t =0 et k =1. Donc 92' passe par Je point A de coordonnées (O; 2).
,-i
0 9>1 n 91'2 = {M(l; 2)} Un vecteur directeur Ü a pour coordonnées (-1; l) et un
N
@
vecteur normal na
pour coordonnées (1; 1).
...., import numpy as np Un point M(x; y) appartient à 9> si, et seulement si, il existe
.c import matplotlib. pyplot as plt
Ol
·;:: un réel t tel que AM= tû, c'est-à-dire :
>-
a. t =np . linspace( - 5,5,21)
0 plt.plot(l+t,2 - t)
u plt.plot(t,2*t)
pH.show() J x-0=-t JX= -t
1 y-2=+t = 1 y=2+t
296

www.bibliomath.com
Géométrie Chapitre 12

c) En utilisant la formule de cours on obtient : m = 11 alors (E) est le point C de coordonnées


(2;-!);
l2x2-lxl+ll 4
d(Nl,6) = = -
J22+(-1)2 J5 m< 1 :alors (El est un cercle de centre C et de rayon
1

JIJ - m.
Dans ce dernier cas, (El est l'ensemble des points M de co-
ordonnées (x; y) vérifiant :

x= 2 + J J- m
1
x cos(t)
avec te:~
l y = - 2 + V~
1 . ()
4 - m x sm t

b) I.:équation cartésienne du cercle est :

X
2
cx - 3) + I =1

On cherche donc les solutions du système :

Un vecteur normal à 6 est ri de coordonnées (2; - 1).


Les vecteurs HM et ri sont colinéaires. Il existe donc À e: ~
tel que HM= Àri et donc HM a pour coordonnées (2À;-À).
Alors H a pour coordonnées (- 2À + 2; À+ 1). Il appartient à
6, donc ses coordonnées doivent vérifier l'équation carté-
(S)

l (X - 3) 2

y = x +l
+ y2

(x - 3)2 + (x+ 1)2


=1

=1
sienne de 6: CS) = {y=x+l
2x(-2À+2)-(À+l)+l =0

ce qui nous donne À = !.


Ainsi H a pour coordonnées ( g; g).
(S) =
l 2x2 - 4x + 9 = 0

y= x+ 1

"O
0

0
C:
::J

lfl
,-i
-
a) Réécrivons l'équation à l'aide des formes canoniques des
polynômes du second degré :
Or le discriminant de l'équation 2x2 - 4x + 9 = O vaut - 56
donc ce système n'admet pas de solutions réelles.
On en déduit que l'intersection du cercle et de la droite est
vide.
0
import numpy as np
N
@ (El = 2
(x - 2) -4+(y+~f-i+m=O import matplotlib.pyplot as plt
....,
.c 2

= 2 =4 - m
Ol (X - 2) + ( y + 1 )
2 17 t=np.linspace(-5,5,1001)
·;:: plt.plot(t,t+1)
>-
a. plt . plot(3+cos(t),sin(t))
0
u Trois cas se présentent : plt. show()

m> 1 :alors (E) est vide ;


1

297

www.bibliomath.com
Chapitre 12 Géométrie

l:tudions le signe de !:lm en fonction de m.


Notons Ôm le discriminant de 162 (-m2 + 52m - 576) = O.
On obtient Ôm =202 donc :
si m E ]O; 16[ u ]36; +oo[ alors l'intersection des deux
cercles est vide ;
si m E {16;36} alors les deux cercles sont tangents;
si m E )16;36[ alors les deux cercles sont sécants en
deux points P1 et P2.

Étudions plus précisément les deux derniers cas en déter-


minant les valeurs de x puis de y solutions du système.
On obtient pour m E ] 16; 36[ :

_ 3m+72-4J-m2+52m-576
l - 50
{ X _ 3m+72+4v'~--m~2-+-52_m
___5_7_6
X2 - 50

puis les valeurs des ordonnées correspondantes :


c) Cette fois on doit résoudre le système :
_ -4m+l04-3J-m2 +52m-576
Y1 - 50
x 2 +(y-4) 2 =m { _ - 4m+ l04+3J~
- -m~2-+-52-n-1--5-7-6
(S'): Y2-
{
(x - 3)2 + y2 =1

(S') <=>

1
l x2 +y2-8y+l6-m=O
2
x + y2 - 6x + 8 =0

{ -8y+6x+8-m =0 I
,/
,/
,/ ""
., ., .. --- ---- --- ... ........ .. ..
,,. .. --- --- ...
''
''
''
(S ) <=> I
"" '
x2 + y2 - 6x + 8 =0
I
I
I

I
,/
,/
' \
\
\

(S') <=> Jy = ix + 88m I' I


I
\
I
' \

lx 2
+ y2 - 6x + 8 =0 1
1
1
I

I
1
I
I \
\
1
1
2 B
(S') <=>
x 2 +{~x +
4
8
-m)
8
- 6x +B = O
1
1
1
1
1
\ 1
1
1

{ \ \ I
u
0
y= }x+ 8ït \
\
\
\ I
I
\
C: \
'' ' '
''
:J \
, { 10ox2 - (12m+288)x + (8 - ml2 +512 = O(64L1)
0
li)
,-i
(S ) <=>
y= 4X + -
3
8- 8- m
\
\
'' ', .. ..... 1
I
,/
I

0
N '' X
@ '', .. ....... .,
.....
.c
Ol
ï::::
Notons
ligne.
C:,.nz le discriminant de l'équation de la première
---- --- --- ., "

>-
a.
0
Pour m = 16, les deux cercles sont tangents en T 16 { 1J; ! ).
u C:,.m (12m + 288)2 -400(512 + (8- m) 2 ) Pour m = 36, les deux cercles sont tangents en
162 (-m 2 +52m -576) T36 ( 518.,-54) ·

298

www.bibliomath.com
Géométrie Chapitre 12

-
a) Le plan fYJ passe par exemple par le point A(O; 0; 2).
Un vecteur normal a pour coordonnées (1; l; 1).

-n
b) • Déterminons un vecteur normal à rY'1 en déterminant le
produit vectoriel de AB et AC.

- n1 - -
= ABAAC(- 4; - 2; - 2)
D'après la représentation paramétrique de rY'2, ce plan est
v
dirigé par Ü (l; 1;2) et (1;-1; 1). Un vecteurnormal de rY'2
estü Av:
V
u ÏÎ2 =UA Û(3;1; - 2)

Les vecteurs ïi 1 et ïi2 n'étant pas colinéaires, les plans ne


sont pas parallèles.
Formons leur produit scalaire :
Résoudre l'équation cartésienne va nous donner une re-
présentation paramétrique de fYJ et donc une famille gé-
nératrice:
ni· ni: 3 X (-4) + l X (-2) - 2 X (-2) = -10
Les vecteurs normaux n'étant pas orthogonaux, les plans
x = 2 - À- µ ne sont pas orthogonaux.
• On peut à présent déterminer une équation cartésienne
x+y +z = 2 <==> 3(À,µ)e:~ 2 / y=À de chaque plan.
{ Un point M(x, y, z) appartient à fYJ1 si, et seulement si, le
z=µ
produit scalaire de AM et ïi I est nul :
Et donc fYJ est engendré par les vecteurs Ü (- 1; l; 0) et
Me: rY'1 {;, -4x-2y-2z+6 = 0
ÏÎ(- 1;0; 1).
Pour calculer la distance, on utilise la formule du cours : Après simplification on obtient :

11 x 1 + 1 x 2 + 1 x 1- 21 2 Me:rY'1 ~2x+ y+z-3 =0


d(M,rY') = =-
v'l+l + l V3 D'après la représentation paramétrique de ~. ce plan
M passe par le point E(O;O; l).
Un point M(x, y, z) appartient à rY'2 si, et seulement si, le
produit scalaire de EM et ïi 2 est nul :

M e: rY'2 ~ 3x + y - 2z + 2 = 0
Les plans étant sécants, ils se coupent selon la droite défi-
nie pas le système d'équations cartésiennes :
A

2x+ y + z - 3 = 0
u Un vecteur normal à fYJ est ïi de coordonnées (l; l; 1). {
0 3x + y - 2z + 2 = 0
Les vecteurs HM et ïi sont colinéaires. Il existe donc À e: ~
C:
::J
• Notons o: l'angle géométrique formé par les plans rY'1 et
0 tel que HM = Àïi et donc HM a pour coordonnées (À; À; À).
lfl
,-i
~-
Alors Ha pour coordonnées (- À+ 1; - À+ 2; - À+ 1). Il appar-
0
N
tient à fYJ, donc ses coordonnées doivent vérifier l'équation
On a ni· ni= -10, llnill = v'24 = 2\1'6 et l nill = v'l.4.
@ cartésienne de fYJ : On sait donc que :
....,
.c
Ol
ï:::: (-À+l) +(-À +2) + (-À+ l) = 2 COS(O:) =
lni ·nil = -l- = - 5
>-
a. 11 ni 1111ni 11 20J84 v134
0
u ce qui nous donne À= i. Ainsi, l'angle formé par ces deux plans mesure
Ain s1"H a pour coor donnees
, (1 4 1)
3; 3 ; 3 . arccos( h)·
299

www.bibliomath.com
Chapitre 12 Géométrie

-
D éterminons des représentations paramétriques des droites.
Pour cela résolvons leur système d'équations cartésiennes:

X= at - 1
b)
en notant n le point de coordonnées (2; 1).
cercle de centre net de rayon l.
I.;équation réduite de ~m est

y= m(x+ 1)
~ est donc le

x-az+l =O
{
y-2z-3 = 0
= 3 tEPI./ {
y=2t+3
Le point M(x; y) appartient à l'intersection de ~m et de <~
z= t si, et seulement si, ses coordonnées vérifient le système :
Donc ~ est dirigée par le vecteur v (a; 2; 1) et passe par le
pointA(-1;3;0).
On procède de même pour~, :
(S)
x 2 + y 2 - 4x - 2y+4 =0
{
y = m(x+ 1)
x= t- 2
x - z +2 =0
<==>3tEPJ./ y=3t-l x2 + m 2 (x + 1)2 - 4x - 2m(x + 1) + 4 = 0
{ y-3z+ 1 = 0 { (Sl <>
{ y = m(x + l)
z= t

Donc~, est dirigée par le vecteur ïi'(1;3;1) et passe par le (m 2 + l)x2 + 2(m 2 - m - 2)x + m 2 - 2m + 4 = O
point A' (- 2; - l;O). (S) <>
{
Les vecteurs v
et ïi' n'étant pas colinéaires, les droites ne y= m(x+ 1)
sont pas parallèles. Par conséquent elles sont coplanaires si,
et seulement si, elJes sont sécantes. Comme m 2 + 1 ':/ 0, la première équation est du second de-
Étudions donc leur intersection. gré en x.
On peut calculer son discriminant :

x=-l+at =-2+ t'

M(x;y;z) € ~ n~' <> 3(t, t') € 2


~ / y = 3 + 2t =- 1 + 3t' 11 = 4(m 2 - m -2}2 -4(m 2 + I)(m 2 -2m + 4) = 8m(3 -4m)
{
Z=t=t1
Étudions son signe :
X= -1 +at = -2+ t'
1 3
Et y = 3 + 2t =- 1 + 3t' donne a=! ett= t =4,donc X -oo 0 4 +oo
{
Signe de
z = t = t'
8x
- 0 + +
X = 2, y = 11 et Z =4. Signe de
1 1

i
Réciproquement si a = alors le point n(2; 11; 4) est commun 3-4x + 1
+ 0
1
-
aux deux droites, et elles sont donc sécantes en ce point puis- Signe de
'O
- Q + Q -
qu'elles ne sont pas parallèles. Delta(x) 1 1
0
C:
::J Ainsi:
0 3
~et~' coplanaires <> a = - On en déduit que :
lfl
,-i
4
0
si m €] -oo;O[ u ] i;+oo[ alors l'équation n'a pas de
N
@
....,
ifJ!i solution réelle et la droite ne coupe pas le cercle ;
a) Notons (E) l'équation x 2 + /-4x-2y+4 = O.
.c
Ol si m € {o; i} alors l'équation admet une solution
·;::
double et la droite est tangente au cercle ;
0
>-
a. (El = (x - 2) 2 -4+ (y-1) 2 -1 +4 = 0
u (E) <=> (x - 2)2 + (y - 1)
2
=1 si m E ] O;} [, alors l'équation admet deux solutions dis-
(El = nM = l tinctes et la droite coupe le cercle en deux points.

300

www.bibliomath.com
Géométrie Chapitre 12

c) D'après ce qui précède, la droite est tangente au cercle si, d) Les vecteurs û et ri;;; appartiennent au plan Elm . De plus,
et seulement si, m = 0 ou m = i. étant non colinéaires, ils engendrent donc Elm :
Les abscisses des points d'intersection valent :
M(x;y;z) E: Elm ~ 3(À,µ) E: ~ 2;-
OM = À....
u + µnm --+

2(m 2 - m - 2) 7
XQ =- 2(m2 + 1) =2 X1 = 5 On en déduit une représentation paramétrique de Elè,m

ses ordonnées valent :


x = À+ µ
9 2
y 0 = ox (2 + 1) = O Y1 = M(x;y;z) E Elm ~ 3(À,µ) E ~ / y= >..-µm
5
I
{
I z = >.. + µm
I
I
I

I
I Étudions à présent l'intersection de 1!2 et f51Jm
I
I
I

~,:,. 1'
:} ,' l y+z = O
~ I M(x; y; z) E: 22 n !!l>m
I
I
{ x-my+mz=l
I
', I
' , , /i, I ...•..•.. • ...

X
z= k
To
M(x; y; z) E 22, n f51Jm 3ke:[ij/ y=-k

i
- x = l - 2mk

Ainsi 1!2 et f51Jm sont sécants selon une droite. Il ne reste


a) Vérifions tout d'abord que ces points sont non-alignés.
plus qu'à étudier l'intersection de cette droite avec §f,n.
AB(- 1;0;0) AC(O;l; - 1) donc.AB = - 7 et.AC = j - k. Ces
vecteurs étant non colinéaires, ils définissent un plan dont
un vecteur normal est :
l - 2mk = >.. + µ
- -
....n = AB /\ AC = - ....i /\ (....j - -k )= - -
k - ....
j M(x;y;z) e:22 n!!l>mn Elm ~ - k = >.. - µm
{
Une équation cartésienne de 22 est donc - y - z = d. Or k=>..+µm
A e: 22, donc ses coordonnées vérifient l'équation. On ob-
tient d = 0 donc l-2mk =À +µ

22:y+z = O
M(x; y;z) E 22 n!!l>m nElm ~ - k =>.. - µm
{
b) Un vecteur normal à f51Jm est ri;;; (1; -m; m). 0 =2>.. (L2 + L3 l

"O
0

0
C:
::J

lfl
cl • Si @ était orthogonale à !!l>m alors les vecteurs m et
û seraient colinéaires. Ayant la même abscisse, il faudrait
donc que m = - m = 1 ce qui est impossible.
Il n'existe donc pas de valeur de m telle que@ soit ortho-
n
M(x;y;z)e:22n!!l>mn Elm ~ {~::~: µ
À= O
,-i gonale à !!l>m.
0
N • La droite @ est parallèle à f51Jm si, et seulement si, les vec- l-2m 2 µ= µ
@ teurs û et ri;;; sont orthogonaux.
...., M(x;y;z) e:22n!!l>m nElm ~ k = µm
.c Calculons leur produit scalaire :
Ol {
ï:::: À= O
>-
a.
0
û-ii;;; = l - m + m = l
u µ= 1+~m2
Il n'existe donc pas de valeur de m telle que@ soit parallèle
à !!l>,n. M(x;y;z) E: 22n!!l>m nElm ~ k - _n_1_
- 1+2m2
{
À=O 301

www.bibliomath.com
Chapitre 12 Géométrie

On obtient: Soit Po(xo;yo;zo) un point de 6 1• Un point M(x;y;z) ap-


partient à 6 1 si, et seulement si, il existe un réel k tel que
PoM = k û. i.e. si, et seulement si, les coordonnées de M
- 1 vérifient Je système :
X - 1+2m2
M(x;y;z) E 22 n&il>m n&€m
y= -l+;m 2
{
z -- m
1+2m 2

l
x = xo +2k

Les trois plans se coupent donc en un unique point lm de 6.' : y= Yo+k ,ke:llll
coordonnées :
z =zo - 3k
1 m m )
lm ( 1 +2m2 ;- 1 + 2m 2 ; 1+ 2m 2

-
a) • Un vecteur normal à &il>1 est ni (1; l; 1). C'est aussi un vec-
teur normal à 8P{ car les deux plans sont parallèles.
Il faut aussi que Je point M appartienne à &il>1 or

M E 8P1 <=> x + y+ z =3
M(x;y;z)e:8P{ ç;, AM·ni=O = xo+Yo+zo+2k+k - 3k = 3

M(x;y;z) e: &il>{ ç> (x-1) + (y-2) + (z-3) =0 = xo+Yo+zo=3

M(x;y; z) E &il>{ ç> x+ y+ z = 6

• La distance de A à &il>1 est donnée par : De plus, il existe un des ces points M qui appartient à ~ -
Finalement :
11 x 1 + 1 x 2 + 1 x 3 - 31 r;:
d(A,8P1)= =v3
J12+12+12
b) Un point Tvl de coordonnées (x; y; z) appartient à l'intersec-
tion de &il>1 et ei1>2 si, et seulement si, ses coordonnées véri-
xo + 2k = 1 + 2À
fient Je système :
YO + k = 3À
M e: 6.' <=> 3Àe:illl/
zo - 3k =- l + À
(S) { x +y+ z =3
xo+Yo+zo = 3
2x - y+z =2
k = 3À - yo

~ 3kc.):::=3-k
u
0 x0 + 6À - 2y0 = 1 +n
C:
::J (S) Me:6.' <=> 3À e:illl/
0 zo - 9À + 3yo = - l + À
lfl 1-y + z = 2 - 2k
,-i
0 xo + Yo+zo = 3
N
@
...., k = 3À - YO
.c (S) ç> 3ke:Oll/ j :::~k
Ol À_ 3yo+zo+l
ï:::: - 5- 3k M e:6.' <=> 3ÀEilll/ - 10
>-
a.
Z - 2
0 xo-2Yo-l = -~(zo +3yo + 1)
u
c) La droite 6 admet pour vecteur directeur û (2; l; -3) donc X-O+Yo+zo=3
6 1 aussi.

302

www.bibliomath.com
Géomét rie Chapitre 12

k = 3.\ - YO
.\ _ 3yo+zo+1
MEt/
= 3.\E~/
- 10

-4y0 +2zo = 3-5x0 H


1
1
Yo+zo=3-xo 1
:::!...
k=3.\-yo
.\ _ 3yo+zo+l
- 10
MEt!..1
= 3.\E~/
Yo = xo;i
D'
Les vecteurs HD et ïi 1 sont colinéaires. Il existe donc
z _ -9xo+l5
o- 6 .\ E ~ tel que HD = .\ri 1 et donc HD a pour coordonnées
(2.\;3.\;-À).
Alors Ha pour coordonnées (- 2.\; - 3.\ - 3; .\ + 1). Il appar-
Choisissons arbitrairement xo = 0, alors Yo = ! et zo = ~. tient àf!J!, donc ses coordonnées doivent vérifier l'équation
Une représentation paramétrique de t:/ est donc : cartésienne de f!J! :

2x (-2.\) +3 X (-3.\-3)-(À+ 1) =4
x = 2k
ce qui nous donne.\ = - 1.
t!..': y= t +k ,kE~
Ainsi H a pour coordonnées (2; O; 0).
1z = ~ -3k 00' = 2DH donc D (4;3; - l). 1

-
Mais on a
d) Les vecteurs AB( -1; 1; 1) et AD( -1; -4; O) ne sont pas coli-
néaires donc A, B et D définissent un plan.
a) Les vecteurs AB(-1; l; 1) et AC(-2; l;-1) ne sont pas coli- Un vecteur normal à ce plan est ni = AB /\ AD donc
néaires donc A, B et C définissent un plan. ni(4; - 1;5).
Un vecteur normal à ce plan est ni = Aè /\ AB donc On a ni· ni = 2 x 4 + 3 x (- 1) + (- 1) x 5 = O. Donc ri 1 et ïi 2
ni (2; 3; -1). sont orthogonaux.
On en déduit que les plans f!JJ1 et f!JJ2 sont orthogonaux.
Donc l'angle formé par ces deux plans mesure ~.

-
Tvl (x; y; z) E (ABC) ç> AM· ni = 0
M(x;y;z) E (ABC) ç> 2(x- l) +3(y- l)-(z- l) = 0
M(x;y;z)E(ABC) ç> 2x+3y-z=4
a) Les vecteurs SJ\(3;0;-4) et SB(0;2;-4) ne sont pas coli-
néaires donc A, B et S définissent un plan.
"O
0 b) La droite t!.. est orthogonale à f!lJ si, et seulement si, ni est
C:
::J
un vecteur directeur de t!... On en déduit que : Un vecteur normal à ce plan est ri = SA /\ SB donc
0 ri(B; 12;6).

- -
lfl
,--i
0
M(x;y;z) E 6 ç;, 3.\E~, DM=.\n1
N M(x;y;z) E (SAB) ç> SM· ri= 0
@

l
...., x=2.\ M(x;y;z) E (SAB) ç> Bx+ 12y+6(z-4) = 0
.c
Ol M(x;y;z) E 6 ç;, 3.\E~/ y =-3+3.\ M(x;y;z) E (SAB) ç> 4x+6y+3z = 12
·;::
>-
a. b) On utilise la formule du cours :
0 z=l-.\
u
1- 121 12
c) Notons H le projeté orthogonal de D sur f!J!. d(O, (SAB)) = = 1r,
J42 +62 +32 61
VOl

303

www.bibliomath.com
Chapitre 12 Géométrie

c) Le plan (SAB) partage l'espace en deux demi-espaces. Cha- 0M ·n3 = Bx > 0 qui contient A;
cun est déterminé par une inéquation : SM ·ri 3 = Bx ~ 0 qui ne contient pas A.
SM· n = Bx+ 12y+6z-24 < 0 qui contientO;
SM· n = Bx+ l2y+6z-24;;,: 0 quinecontientpasO.
- -
(BAO) Les vecteurs OB(0;2;0) et OA(3;0;0) ne sont pas co-
linéaires donc A, 0 et B définissent un plan.

H M Un vecteur normal à ce plan est 74 = 6Â A OB donc


74(0; 0;6).

n
M(x; y; z) E (BAO) <=> 0M · 74 = 0
M(x;y;z) e: (BAO) ç;, 6z = 0
s M(x;y;z) e: (BAO) ç;, z=0

Le plan (BAO) partage l'espace en deux demi-


espaces. Chacun est déterminé par une inéquation :
- OM ·n4 =6z >OquicontientS;
H' M' - SM .n 4 = 6z ~ 0 qui ne contient pas S.
Finalement, l'intérieur du tétraèdre est défini par le sys-
On définit de même trois autres inéquations correspon- tème d'inéquations :
dant à chacun des trois autres plans définis par les autres
faces du tétraèdre.
I:intérieur du tétraèdre est l'intersection de ces quatre x>O
demi-espaces.
y>O
(SAO) Les vecteurs OS (O; O; 4) et OA(3; O; O) ne sont pas co-
z>O
linéaires donc A, 0 et S définissent un plan.
Un vecteur normal à ce plan est Ïiz = 6Â A OS donc Bx + l2y + 6z - 24 <0
ïiz (O; -12; 0).
itJFI
a) A1(3;-2;1), iij'(l;-1;1), A2(0;0;0). Les vecteurs (1;2;1) et
M(x;y;z) e: (SAO) <=> OM·ïiz=O (-1;2;-l) sont orthogonaux à @2, donc leur produit vecto-
M(x; y; z) e: (SAO) <=> - 12y = O riel ( - 4; 0; 4) est un vecteur directeur de @2. On choisit vi =
M(x; y; z) e: (SAO) <=> y=O (- 1;0;1).
b) On a: iii A vî = (-4;-8;-4). On choisit donc: Ü = (1; 2; 1).
Le plan (SAO) partage l'espace en deux demi- c) • Déterminons une équation cartésienne de 91'1.
espaces. Chacun est déterminé par une inéquation :
Le vecteur û A iii = (3;0; - 3) est orthogonal à .01\, donc .01\ a
0M ·n2 = l2y > 0 qui contient B; pour équation cartésienne: x - z = a où a e: ~- Mais A1 e: 91'1
- SM· n2 = 12y ~ 0 qui ne contient pas B. donne a = 2. Donc .01>1 : x - z = 2.
• Déterminons une équation cartésienne de .01>2.
'O (SBO) Les vecteurs OS (O; O; 4) et 08(0; 2; 0) ne sont pas co-
0 Le vecteur Ü A VÎ = (2; -2; 2) est orthogonal à .01>2, donc .01>2 a
C: linéaires donc B, 0 et S définissent un plan.
::J pour équation cartésienne: x- y+z = p où p e: ~- MaisA2 e: .01>2
0 Un vecteur normal à ce plan est nj = OB A OS donc
lfl
donne p = O. Donc .01>2 : x - y+ z = O.
,-i
Ïij(8;0;0).
0 2
. . ,,,. ,.,,. ms { X-Z
N • Ainsi : :v = .:r1 n .:r2 : est une droite car
@ 0
x-y+z
...., M(x;y;z) e: (SBO) <=> 0M ·nj = o intersection de deux plans non parallèles.
.c
Ol M(x; y; z) e: (SBO) ç;, Bx = 0
·;:: x-z 2
>-
a. M(x;y;z) e: (SBO) ç;, x=O • On ac M(x; y; z) , @, <> A,M A i,j = Ô <> {
0 y+z -l
u
Le plan (SBO) partage J'espace en deux demi- On se donne alors M(3 + t; - 2 - t; 1 + t) un point de @1. On a
espaces. Chacun est déterminé par une inéquation : M e:@<=> t = - 2. Ainsi@ et@1 sont sécantes au point (l; 0; - 1).

304

www.bibliomath.com
Géométrie Chapitre 12

En utilisant les équations cartésiennes de !?tJ et !?tl2, on obtient


D C
que:

M(x;y;z) ç>
{
x=1
y=O , /
z=-1

ce qui prouve que 9i) et 9i>2 sont sécantes. Q


• D'après les équations cartésiennes de 9i>, les vecteurs
(1; O; - 1) et (1; - 1; 1) sont orthogonaux à 9i>. On en déduit que p
A B
leur produit vectoriel ïi (-1;-2;-1) dirige !?tl. On vérifie alors
= =
que ïi .vi ïi .vi 0, ce qui prouve que 9i) est perpendiculaire
à@1 et @2 .

it!ii
c) Observons deux cas particuliers :
a) Dans le repère (A.AB.AD), notons t l'abscisse de M.
Comme M est sur la première bissectrice du repère, M a
pour coordonnées (t; t) avec te: [O; 1] .
On en déduit que P(t;O) et Q(l, t) donc

=! ;
1(:+~·!:)
2 2' 2
- si t = 0 alors 11t admet pour équation x

I appartient donc à la droite d'équation y = x - !- Or, - si t = 1 alors f1t admet pour équation y= !.
quand t parcowt [O; l]. x = f + ! parcourt [ ! ;1].
On en déduit que lorsque M décrit [AC]. I décrit le segment
de droite défini par :

Cela nous incite à vérifier que les médiatrices de [PQJ


passent toutes par Je point K ( ! ;!) :

b) Un point N(x; y) appartient à la médiatrice de 11c de [PQJ si,


et seulement si, les droites (PQ) et (IN) sont orthogonales.
OrPQ(l - t; t) etÏN(x- ttl ;y-!) donc
"O
0
C:
1
(t - 1)- -
2
-t+
1
2
-1
2
1 1
=- - +- =0
2 2
::J
0 Ne:11t ç> ÎN·PQ=O
lfl
,-i
0
N
ç> (1 - t)(x- t;l)+t(y-fl=o
@ 1
....,
2 =O
ç> (t-l)x-ty+
.c
Ol donc les coordonnées de K vérifient l'équation de 11t pour
ï:::: tout te: [O; 1].
>-
a. Une équation cartésienne de 11 est donc :
0
u Les médiatrices de [PQ] passent donc toutes par le point
1
(t - l)x - ty+ -= O, avec t e: [O;l]
2
K( 2'l. 21) ·
305

www.bibliomath.com
Chapitre 12 Géométrie

-a) La réponse est non comme le montre le contre-exemple


suivant:

7" (7 "1) = 7 "O = Oet (7 "1) "7 = k "7 = - 7.


- 1= 1Réciproquement, supposons que û et
linéaires.
wsont co-
nexiste donc o: -:JO tel que: ü = o:w ou w = o:ü .
Si û = o:w, alors:

ü" (ü "w) o:w" (ü "w)


b) On note Û = (u1 , u2,u3) , = (v1,v2,v3) et = v w - o:(v "w) "w
(w1, w2 , w3). Dans la base canonique @ = (7, k) de~3 • 7, o:(ui "v) "w
les coordonnées sont égales aux composantes. Les calculs
donnent: = (o:w/\v)Aw
(ü " ü ) "w

On obtient le même résultat si w= o:û.


111 " I + 112"2 + 113"3 0 En conclusion, on a donc :
Mat(g) = o 111111 +u2 v2+u3 l'3
~ (
Û A(v /\ w) = (u A v)Aw <==> Û et W sont colinéaires
0 0

et en remarquant que 8 = hu o hv on a:
-
a) Soit (To) la tangente à D au point Mo : tous les points du
disque sont dans le demi-plan délimité par (To) ne conte-
nant pas M.

Mat(8)
~

= (Mat(hul) x (Tvlat(hv))
~ ~

-u3v3-u2v2 u2v1
u1v2 -U3V3-U1V1
(
u1v3 u2v3

D'où 8 = hu o hv = f - g et donc pour tout x E IE:


- (- -) =
U /\VAX (- -)-
U.X V - (- -)-
U.V X.
0
u c) Raisonnons par analyse-synthèse.
0
C:
::J - 1 ==> 1 Supposons que :
0
ü " (v "w) = (ü "v) "w
lfl
,-i
0
N
On a alors:
@
....,
.c
Ol
·;::
-
( u .-)-
w v- (- -)-
u.v w=- (-
w .-)-
v u+ (- -)-
w.u v Soit M' un point de D. Notons M~ l'intersection de (M'M)
avec (To). Alors
>-
0
u
a. D'où:
-
( u. -v )-w= (-v.w
- )-u MM' =MM~ + M~M ;,:: MM~ ;,:: d(M, (To)) = Mlvlo

Donc û et wsont colinéaires. car Mo est le projeté orthogonal de M sur (To).

306

www.bibliomath.com
Géométrie Chapitre 12

On en déduit que d(M,D) = MMo. Siy<-2:

données de M.
J
Or MMo = OM - 1 = x 2 + y 2 - 1 en notant (x; y) les coor-
Dans ce cas y+d = -2 d'où d = -(y+2).
Or 2d = Tv!Tvlo = J x2 + y2 - 1.
Finalement, on obtient que : On en déduit que Jx2 + y2 =- 2y - 3.
d(M,D) = V + y2
x2 -1
Finalement x2 = 3y2 + 12y+9 i.e x = ±J3y2 + 12y+ 9.

Onvadoncétudierlafonctionf2+: y-. J3y2 +12y+9


b) Représentons la situation en distinguant deux cas. Notons et déduire le reste de la courbe par symétrie par rapport à
d la distance de M à 6 . (Oy).
Si y ç -2: Une rapide étude permet d'obtenir le tableau suivant :
Dans ce cas y-d = -2 d'où d = y+2.
X - oo -3 -2
Or 2d = MMo = J + y2 - 1.
x2
Signe de G) -
+
On en déduit que J + y2 =2y + 5.
x2 3x2 + 12x+9 1

Finalement x 2 = 3y2 +20y+ 25 i.e x = ±V3y2 +20y+25. On étudie donc h+ sur J-oo;-3)
y - oo -3
On va donc étudierla fonction fi+ : y ...... V3 y 2 + 20 y+ 25
et déduire le reste de la courbe par symétrie par rapport à Signe de
(Oy). -
1;+(y)
Une rapide étude permet d'obtenir le tableau suivant :
Variations de
-5 h+
X -2 3 +oo 00 0
1
Signe de G)
2
3x + 20x + 25
-
1
+

On étudie donc fi+ sur [-~;+oo[ X

y -5
3 +oo
Signe de
f{+(Y) +
+oo
Variations de
/1+ 0~

u
0
C:
::J
0
lfl
,--i
0
N
@
....,
.c
Ol
ï::::
>-
a.
0
u
Finalement, la courbe cherchée est la réunion des deux
courbes étudiées précédemment :

307

www.bibliomath.com
Chapitre 12 Géométrie

- -
ACAAD

0 D
-~ -------f-----;;J c

s; s;

-
On montre de même que SÎ · = s; · = O.
2 2
i 2
Finalementd2 = ills:;11 = (11sî 11 + lls;ll + l s;li2).
a) On rappelle que : On en déduit que Je carré del' aire du triangle BCD est égale
à la somme des carrés des aires des triangles ABC, ABD et
- uAv =-vAü; ACD.

- -
- ü A (v + wi =ü Aü + ü + ui ;
-uAu- = O.
On en déduit que :
c) On obtient successivement :

T = (M0 + ~)A(M0+,%)

= (- cxAB+pAê)A(- cxAB + yAD)

- - - -
ACABD+ADAAB
- ayAB AAD -apAê AAS+ pyAê AAD

Aê A BD+ (AB+ BD) AAS ayS3 + aps; + 13rSÎ


ay(S4 - 5Î - s;) + aps; + py5Î
AêABD+BDAAB
= (Aê - AB) ABD y (13-a) SÎ +a(f3 - y) s; +ayS4
- -
BCABD
S4 On en déduit, en utilisant l'inégalité triangulaire, que :
"O
0 b) On rappelJe que l'aire du triangle BCD vaut! li se Awll.
C:
::J
Notons d cette aire. llî Il :( ly (13-a) illsî Il + la (13-y) i11s; Il+ ay lls'.; Il
0
lfl On déduit de la question précédente que : :( y(f3-a) llsî ll +a(y-13) lls2 II + avlls:;11
,-i
0
N
2 :( (y(a+j3)-aj3) m~ l s;II
@
...., 4d 2 =ll sî112 + 11s;ll2 + l s;ll + 2 (sî. s; +SÎ . s; +s; . s;) 1$1$4

.c Or:
Ol
·;:: Or SÎ .s; = (Aê AAD) · (ABAAê). Le vecteur Aê AAD est y(a+l3)-al3 = y2 - (y - a)(y - (3) ~ 1
>-
a.
~~

0 orthogonal à Aê et à AD comme AB donc Aê AAD est coli- ~ 1 ~o ~o


u néaire à AB donc il est orthogonal à s; =AB AAê. Donc:
On en déduit que Je produit scalaire SÎ · s; est nul. llî ll :( max
1$1$4
ls;II
308

www.bibliomath.com
Géomét rie Chapitre 12

d) ! Il T Il représente l'aire du triangle M 1M2 M3. C


Donc, on a démontré que l'aire de toute section plane du D
tétraèdre est inférieure à l'aire de la plus grande des faces :

u
0
C:
::J
0
lfl
,-i
0
N
@
....,
.c
Ol
ï::::
>-
a.
0
u

309

www.bibliomath.com
CHAPITRE

Statistiques

'Tlièmes a6ordés dans {es e~ercices


- Statistique univariée
- Statistique bivariée

Points essentiefs du cours pour {a réso{ution


des e~ercices
- Description d'une série statistique (effectifs, fréquences, fréquences cumulées)
- Représentations graphiques (diagramme en bâtons, histogramme)
- Caractéristiques de position (moyenne, médiane, mode) et de dispersion (variance, écart-type, quartiles, dé-
ciles).
- Nuage de points associé à une série statistique double, point moyen
- Caractéristiques d'une série statistique double (covariance, coefficient de corrélation)
15 - Ajustement affine par la méthode des moindres carrés
C:
::J
0
lfl
,-i
0
N
@
....,
.c
Ol
·;::
>-
a.
0
u

310

www.bibliomath.com
Stat istiques Chapitre 13

Les méthodes à retenir

- On détermine les caractéristiques de la série


c........ Exercices 13.1,13.2, 13.3 et 13.4
Pour étudier une série statistique
univariée - On construit une représentation graphique de la série
c........ Exercices 13.l, 13.3 et 13.4

- On trace le nuage de points


c........ Exercices 13.4 et 13.5

- On applique la méthode des moindres carrés pour obtenir la


droite de régression linéaire, dans le cas où les variables semblent
liées par une relation affine
c........ Exercice 13.4
Pour étudier une série statistique
bivariée - Si les variables ne semblent pas liée par une relation affine, on
transforme les variables afin d'obtenir un modèle affine
c........ Exercice 13.5

- On calcule et on interp rète la valeur du coefficient de corrélation


linéaire
c........ Exercices 13.4 et 13.5

u
0
C:
::J
0
lfl
,-i
0
N
@
....,
.c
Ol
·;::
>-
a.
0
u

311

www.bibliomath.com
Chapitre 13 Statistiques

Énoncés des exercices

---- - La légende des épinards

Contrairement aux idées reçues, l'épinard n'est pas l'aliment le plus riche en fer. La
lentille, par exemple, en apporte davantage.
Pour vérifier ces propos, on a procédé à des analyses de fer sur 10 échantillons d'épi-
nard, et 10 échantiJlon de lentilles. Les résultats sont indiqués dans le tableau s11.1ivant
(la teneur est indiquée en mg pour 100g de produit frais) :

Échantillon 1 2 3 4 5 6 7 8 9 10
Épinard 2,64 2,75 2,82 2,72 2,66 2,59 2,83 2,70 2,67 2,62
Lentille 9,02 9,08 8,82 8,94 8,95 9,11 9,14 9,02 9,04 8,85

a) Calculer la teneur moyenne en fer des épinards, la valeur médiane et l'écart-type


associé.
b) Dresser la boîte à moustaches associée aux épinards.
c) Mêmes questions pour les lentilles.

---- - Croissance des haricots


On souhaite déterminer l'effet sur la croissance de plants de haricots d'apports en
nutriments. On prépare deux jeux de 100 pots, les premiers contenant un apport en
nutriment et les seconds n'en contenant pas (pots témoins). On plante un haricot
par pot, et on mesure, après 20 jours, la taille des plants obtenus. Le tableau s11.1ivant
u
0 présente les six premières mesures :
C:
::J
0
lfl
Échantillon 1 2 3 4 5 6
,-i
0 Avec apport 9,3 11,7 13,6 10,2 14,2 9,7
N
@ Sans apport 12,1 13,4 16,3 13,5 13,2 14,9
....,
.c
Ol
·;::
a) Calculer la moyenne Ta (reps. Ts) des tailles de ces 6 échantillons avec (resp. sans)
>-
a.
apport de nutriment.
0 b) Calculer la variance Vs et l'écart-type us du lot témoin.
u
Même question pour le lot avec apport.

3 12

www.bibliomath.com
Stat istiques Chapitre 13

---- - Varicelle au Canada


Le tableau suivant contient les données récentes sur le nombre de cas de varicelle
signalés au Canada de 2005 à 2008, par groupe d'âge (en années) :

Classes d'âge 10.11 11.41 [5,9] [10,14] [15,19] [20,24] [25,291 130,391 140,59) [60,100]
2008 75 254 455 139 43 16 35 41 58 35
2007 61 172 307 93 31 27 21 45 28 47
2006 72 226 352 100 39 25 36 63 43 47
2005 99 421 729 190 48 115 22 45 43 44

a) Quelle est le nombre moyen de cas déclarés par an ?


b) Dresser l'histogramme correspondant à l'année 2008.
c) Représenter les fréquences cumulées croissantes. En déduire graphiquement
une approximation des premier et troisième quartiles, et de la médiane.

---- - Volume d'importations de produits horticoles


Le tableau suivant présente le volume (en milliers de tonnes) d'importations de pro-
duits horticoles dans six pays de 2006 à 2010 :

~s États-Unis Allemagne Russie Chine France Canada


e
2006 13823 10141 6889 6454 6623 4180
2007 15083 10494 7471 6117 6920 4286
2008 15057 11708 8381 3836 7038 4W3
2009 14920 11500 7828 8599 6678 4342a
2010 16325 11555 8964 8669 6852 4566

a) Calculer le volume moyen d'importations de ces pays en 2010. Quel est l'écart-
u type correspondant ?
0
C: b) Dresser le diagramme en bâtons correspondant.
::J
0 c) On étudie les importations horticoles du Canada de 2006 à 2010.
lfl
,-i Établir le nuage de points exprimant le volume des importations en fonction de
0
N t, où l'on représente les années sous la forme 2006 + T, pour Te: [0,4].
@ En admettant que le volume des importations du Canada est une fonction af-
...., fine de T, ajustez à la série une droite d'équation y = ax + b par la méthode des
.c
Ol
·;::
moindres carrés.
>-
a. d) Calculer le coefficient de corrélation entre le volume des importations du Canada
0 etT.
u
Le modèle affine de la question précédente est-il pertinent ?

3 13

www.bibliomath.com
Chapitre 13 Statistiques

---- - LoiSPAR
Plus une région est vaste, plus le nombre d'espèces y vivant est grand. Pour modéliser
mathématiquement ce phénomène (et mesurer ce que l'on appelle la biodiversité),
les scientifiques utilisent régulièrement la loi SPAR ("species-area relationship"). Elle
stipule que si S représente la surface de la région étudiée et N le nombre d'espèce
présentes dans cette région, alors on a :

où C et z sont des constantes à ajuster selon la région étudiée. Le calcul (approché)


de la constante z est très primordial.
On demande à des élèves d'une classe de BCPST de vérifier cette loi pour les plantes
présentes dans une prairie. Les données récoltées sont résumées dans le tableau sui-
vant, exprimant le nombre de plantes différentes présentes N en fonction de la sur-
face S de la prairie étudiée :

Surface S (en m2 ) 1 2 4 8 16 32 64
Nombre d'espèces N 6 7 8 10 10 13 14

Afin de mettre en valeur la relation donnée par la loi SPAR, on décide de procéder à
une régression linéaire de ln N sur lnS.
a) Expliquer l'intérêt de cette régression linéaire.
b) Tracer le nuage de points correspondant aux variables lnS (en abscisses) et lnN,
ainsi que la droite de régression linéaire.
c) Combien vaut le coefficient de corrélation linéaire ? Que peut-on en déduire ?
d) Donner alors une approximation de C et z.

u
0
C:
::J
0
lfl
,-i
0
N
@
....,
.c
Ol
·;::
>-
a.
0
u

314

www.bibliomath.com
Stat istiq ues Chapitre 13

Du mal à démarrer ?

1111 a) Appli qu er les formu les du cours. et 0, 75.


b) et c) Calculer les q uartiles et la médiane. 1111 c) La pente de la droite de régression vaut a=C~ii/l. On

GD a) et b) Utiliser les formules du cours.


obtient l'ordonnée à l'origine en remarquant que la droite de ré-
g ression passe par le point-milieu du nuage de points.

GD c) On approche les fréquences cumulées par une courbe af- 11D =


a) Prendre le logarithme de la relation N CS 2 .
fine par morceaux. Les valeurs des quarti les et de la médiane sont d) Une app roximation de C et z est obtenue d 'après la re lat ion
approchées par les antécéden ts des fréquences cumulées 0,25, 0,5 trouvée en a).

u
0
C:
::J
0
lfl
,-i
0
N
@
....,
.c
Ol
·;::
>-
a.
0
u

3 15
Chapitre 13 Statistiques

Corrigés des exercices


- a) La teneur moyenne en fer des épinards est de - a) Il suffit de sommer tous les termes du tableau et
2, 7mg pour 100g. de diviser par le nombre d'années. Le nombre moyen ri de
La valeur médiane vaut 2,685 et l'écart-type u e = 0, 08. cas déclarés par année vaut donc :
b) Le premier et le troisième quartiles valent respective-
ment 2,62 et 2, 72, et la boîte à moustaches correspon- _ 75+254 + ··· +44 4742
n= =-- =1185,5.
dante figure ci-dessous : 4 4
b) {;histogramme demandé est représenté ci-dessous :

densités

100
2.59 2.62 2.72 2.83

c) La teneur moyenne en fer des épinards est de 8,997 =


9mg pour 100g.
La valeur médiane vaut 9, 02 et l'écart-type u e = 0, 10.
Les premier et troisième quartiles valent 8, 94 et 9, 08. On ré-
sume ces informations dans la boîte à moustaches suivante :

....._____,I 1
8.82 8.94 9.08 9.14

iifj a) Pour le lot témoin, on obtient : o~+-+-+-+-+-+-..-+-..-..-..--1a=..........................................,


0 5 JO )6 ~ ~ ~ ~ 10 IG 00 W 00 ~ mn 00 ~ 00 00 100

-T 9,3+ 11, 7+ 13,6+ 10,2+ 14,2+9,7 68,7


s=
6
=--
6
=11,45. c) Les fréquences cumulées croissantes (exprimées en pour-
centage) sont représentées ci-dessous, ainsi que les valeurs
Concernant le lot sans apport de nutriment, on a : approchées des quartiles et de la médiane :
-
Ta = 12,1+13,4+16,3+13,5+13,2+14,9
6
=-83,4
6
- =13, 9.

b) Les variances valent :


u 9,3 2 + 11, 7 2 + 13,62 + 10,22 + 14,22 +9, 72 - 2
0 Vs= - (Ts)
C: 6
:J
0 = 3,58
li)
,-i
0
12,1 2 + 13,42 + 16,32 + 13,52 + 13,22 + 14,92 - 2
N
Va= - (Ta)
6
@ =l,817
.....
..c:
Ol et les écarts-types correspondants :
·;::
>-
a. Us = ,/v; =1,89 00 100
0
u Ua =Fa= 1,35

3 16
Statistiques Chapitre 13

- a) Le volume moyen en 2010 vaut : où la dernière égalité exprime Je fait que la droite de régres-
sion passe par le point-milieu du nuage de points.
- 16325 + 11555 + 8964 + 8669 + 6852 + 4566 La figure suivante représente la nuage de points ainsi que la
V2010 = 6 droite de régression linéaire.
ce 9487 milliers de tonnes.

L'écart-type vaut : 4800

163252 + 115552 + · · · + 45662 - XM4


02010 = - - - - - - - - - - - - (V2010>2 4400 X
6 X
M3
M1 M2
ce3723
4000 Mo
b) On dessine Je diagramme en bâtons : 2006 2007 2008 2009 2010
d) L'écart-type de V vaut :
Volume (en milliers de tonnes)
2
93973365 - (21667) = J407936 ce 127 74
16000
16325 uv = 5 5 25 ' '

et celui de T vaut :
12000 11555

8964
Le coefficient de corrélation linéaire vaut donc :
8669
8000
6852 rv,T = Cov(V, T) ce o. 92
uvuT
4000
Étant proche de l, le coefficient de corrélation linéaire valide
'5i'>" .s,~"' d'~"' '<'§'""'
!?
~(!,

~~
,; !$'~
~c;
"<' cl' la pertinence du modèle de dépendance affine de V en fonc-
tion deT.
c) On note V Je volume, et les années sous la forme 2006 + T, - a) La fonction f: S - CS 2 n'est pas affine, alors que
avec Te: [0,4Il. l'on a bien une relation affine entre lnN et lnS
La covariance Cov(V, T) vaut :
lnN = lnC+zlnS.
Cov(V,T) =
1 - - Une méthode de régression linéaire sera donc plus adap-
- (4180 x O+ 4286 x 1 + 4293 x 2 + 4342 x 3 + 4566 x 4) - Vc T,
5 tée aux variables lnN et InS qu'aux variables Net S direc-
avec tement.
b) Le nuage de points et la droite de régression linéaire fi-
- 4180+4286+4293 +4342+4566 21667
Ve = = - - ===4333 gurent ci-dessous :
5 5
et
3 (~)
u î = 2.
0
C:
::J On obtientdoncCov(V,T) = 44 ~62 -2 21 ~67 = 8~8 = 165,6.
0 2
La variance de T vaut :
lfl
,-i
3
0
N V (T) = 0 - 22 = 2.
5 1
@
...., La droite d'ajustement affine~ donnée par la méthode des
.c
Ol moindres carrés a donc pour équation@: y = ax + b, avec : 0 +---+---+-----,f---+------l
ï:::: 2
>- 0 1 3 4 5
a.

l
a = Cov(V, T) = 828 =
0 82 8 La droite de régression linéaire a pour équation :
u V(T) 10 '
- - 20839
b=Vc-aT=-- =4167,8 ~: y= ax+b,
5

3 17
Chapitre 13 Statistiques

où a = 0.206 et b = 1.803. SPAR.


cl Le coefficient de corrélation linéaire vaut environ 0,989
très proche de 1, il semble valider le modèle donné par la loi dl D'après la question a), on a z =a= 0.206 et C =eb =6,07.

318

www.bibliomath.com

Vous aimerez peut-être aussi